4$. \nLet $k_n=\\prod_{i=1}^{x} p_i^{a_i}$. Let $p_x$ be the largest prime less than $n$. Then set all $a_i$ to $1$. This works as $k_n=\\prod_{i=1}^x p_i>n$. Now since $n>4$, we can also set $a_1=2$, which also works. $\\blacksquare$"
}
{
"Tag": [
"inequalities"
],
"Problem": "Let $a,b,c>0$. Prove or disprove that\r\n$a^6b^3+b^6c^3+c^6a^3 \\geq a^5b^2c^2+a^2b^5c^2+a^2b^2c^5$",
"Solution_1": "It's true by Muirhead ($(6,3,0) \\succeq (5,2,2)$) or more simply by rearrangement ?",
"Solution_2": "I'm wondering somewhat about that. Can Muirhead be used with non-symmetric sum?",
"Solution_3": "Indeed you're right, Muirhead can't be used here...",
"Solution_4": "Then can we use weight AM-GM to help?",
"Solution_5": "Rather use H\u00f6lder: \\[ \\begin{aligned} & \\left(a^6b^3 + b^6c^3 + c^6a^3\\right)^3 \\\\ = & \\left(a^6b^3 + b^6c^3 + c^6a^3\\right)\\left(a^6b^3 + b^6c^3 + c^6a^3\\right)\\left(c^6a^3 + a^6b^3 + b^6c^3\\right) \\\\ \\geq & \\left(\\sqrt[3]{a^6b^3a^6b^3c^6a^3} + \\sqrt[3]{b^6c^3b^6c^3a^6b^3} + \\sqrt[3]{c^6a^3c^6a^3b^6c^3}\\right)^3 \\\\ = & \\left(a^5b^2c^2 + b^5c^2a^2 + c^5a^2b^2\\right)^3. \\end{aligned} \\]"
}
{
"Tag": [
"number theory proposed",
"number theory"
],
"Problem": "determine if $ n!\\minus{}1$ is composite for infinitely many $ n$'s, where $ n$ is a positive integer",
"Solution_1": "$ (p \\minus{} 2)! \\equiv 1 \\mod{ p }$.\r\n\r\nIs it an open question if $ n! \\minus{} 1$ or $ n! \\plus{} 1$ is prime for infinitely many $ n$'s?"
}
{
"Tag": [
"inequalities",
"inequalities proposed"
],
"Problem": "Let $ a,b,c \\ge 0, ab \\plus{} bc \\plus{} ca \\equal{} 1$. Prove that:\r\n$ \\frac {ab \\plus{} 1}{a \\plus{} b} \\plus{} \\frac {bc \\plus{} 1}{b \\plus{} c} \\plus{} \\frac {ca \\plus{} 1}{c \\plus{} a} \\ge 3$\r\n\r\n[b]Vietnam Inequality Forum:[/b] http://www.batdangthuc.net/forum/index.php\r\n[b]Welcome to VI![/b]",
"Solution_1": "[quote=\"NguyenDungTN\"]Let $ a,b,c \\ge 0, ab \\plus{} bc \\plus{} ca \\equal{} 1$. Prove that:\n$ \\frac {ab \\plus{} 1}{a \\plus{} b} \\plus{} \\frac {bc \\plus{} 1}{b \\plus{} c} \\plus{} \\frac {ca \\plus{} 1}{c \\plus{} a} \\ge 3$\n\n[b]Vietnam Inequality Forum:[/b] http://www.batdangthuc.net/forum/index.php\n[b]Welcome to VI![/b][/quote]\r\nineq <=> $ p^2 \\plus{} (r \\minus{} 3)p \\plus{} 2r \\plus{} 3\\ge 0$\r\n$ \\delta \\equal{} r^2 \\minus{} 18r \\plus{} 1\\le 0$ (with $ r\\le \\frac {1}{3.\\sqrt {3}}$) \r\nso ineq was proved :D",
"Solution_2": "We also have the following result\r\n\\[ \\sum\\frac{a\\plus{}bc}{1\\plus{}ab} \\ge \\frac{3(\\sqrt{3}\\plus{}1)}{4}\r\n\\]\r\nfor any $ a,b,c \\ge 0,ab\\plus{}bc\\plus{}ca\\equal{}1$ :)",
"Solution_3": "[quote=\"vodanh_tieutot\"][quote=\"NguyenDungTN\"]Let $ a,b,c \\ge 0, ab \\plus{} bc \\plus{} ca \\equal{} 1$. Prove that:\n$ \\frac {ab \\plus{} 1}{a \\plus{} b} \\plus{} \\frac {bc \\plus{} 1}{b \\plus{} c} \\plus{} \\frac {ca \\plus{} 1}{c \\plus{} a} \\ge 3$\n\n[b]Vietnam Inequality Forum:[/b] http://www.batdangthuc.net/forum/index.php\n[b]Welcome to VI![/b][/quote]\nineq <=> $ p^2 \\plus{} (r \\minus{} 3)p \\plus{} 2r \\plus{} 3\\ge 0$\n$ \\delta \\equal{} r^2 \\minus{} 18r \\plus{} 1\\le 0$ (with $ r\\le \\frac {1}{3.\\sqrt {3}}$) \nso ineq was proved :D[/quote]\nWhen equality holds?\n[quote=\"can_hang2007\"]We also have the following result\n\\[ \\sum\\frac {a \\plus{} bc}{1 \\plus{} ab} \\ge \\frac {3(\\sqrt {3} \\plus{} 1)}{4}\n\\]\nfor any $ a,b,c \\ge 0,ab \\plus{} bc \\plus{} ca \\equal{} 1$ :)[/quote]\r\nNice problem, Vo Quoc Ba Can!",
"Solution_4": "[quote=\"vodanh_tieutot\"][quote=\"NguyenDungTN\"]Let $ a,b,c \\ge 0, ab \\plus{} bc \\plus{} ca \\equal{} 1$. Prove that:\n$ \\frac {ab \\plus{} 1}{a \\plus{} b} \\plus{} \\frac {bc \\plus{} 1}{b \\plus{} c} \\plus{} \\frac {ca \\plus{} 1}{c \\plus{} a} \\ge 3$\n\n[b]Vietnam Inequality Forum:[/b] http://www.batdangthuc.net/forum/index.php\n[b]Welcome to VI![/b][/quote]\nineq <=> $ p^2 \\plus{} (r \\minus{} 3)p \\plus{} 2r \\plus{} 3\\ge 0$\n$ \\delta \\equal{} r^2 \\minus{} 18r \\plus{} 1\\le 0$ (with $ r\\le \\frac {1}{3.\\sqrt {3}}$) \nso ineq was proved :D[/quote]\r\n\r\nWhy you can say that $ p^2 \\plus{} (r \\minus{} 3)p \\plus{} 2r \\plus{} 3\\ge 0$\r\n\r\nCan you please explain it to me?\r\n\r\nThank you very much.",
"Solution_5": "[quote=\"vodanh_tieutot\"]Let $ a,b,c \\ge 0, ab \\plus{} bc \\plus{} ca \\equal{} 1$. Prove that:\n$ \\frac {ab \\plus{} 1}{a \\plus{} b} \\plus{} \\frac {bc \\plus{} 1}{b \\plus{} c} \\plus{} \\frac {ca \\plus{} 1}{c \\plus{} a} \\ge 3$\n\n\nineq <=> $ p^2 \\plus{} (r \\minus{} 3)p \\plus{} 2r \\plus{} 3\\ge 0$\n$ \\delta \\equal{} r^2 \\minus{} 18r \\plus{} 1\\le 0$ (with $ r\\le \\frac {1}{3.\\sqrt {3}}$) \nso ineq was proved :D[/quote]\r\n\r\n\r\nWhy $ \\delta \\equal{} r^2 \\minus{} 18r \\plus{} 1\\le 0$ with $ r\\le \\frac {1}{3.\\sqrt {3}}$????. \r\n\r\nSorry, but I don't see it. :maybe: \r\n\r\nCan anyone,please, explain it to me. :wink: \r\n\r\nThank you very much. :)",
"Solution_6": "NguyenDungTN, can you please post your solution?\r\n\r\nI cannot solve it :( \r\n\r\nThank you very much.",
"Solution_7": "[quote=\"manlio\"]NguyenDungTN, can you please post your solution?\n\nI cannot solve it :( \n\nThank you very much.[/quote]\r\n[hide=\"Hint\"]We have:\n$ \\frac {ab \\plus{} 1}{a \\plus{} b} \\plus{} \\frac {bc \\plus{} 1}{b \\plus{} c} \\plus{} \\frac {ca \\plus{} 1}{c \\plus{} a} \\ge 3$\n$ \\Leftrightarrow \\sum_{cyc} (ab\\plus{}1)(c\\plus{}a)(c\\plus{}b) \\ge 3(a\\plus{}b)(b\\plus{}c)(c\\plus{}a)$\n$ \\Leftrightarrow \\sum_{cyc} (ab\\plus{}1)(c^2\\plus{}1) \\ge 3[(a\\plus{}b\\plus{}c)(ab\\plus{}bc\\plus{}ca)\\minus{}abc]$\n$ \\Leftrightarrow (a^2\\plus{}b^2\\plus{}c^2)\\plus{}ab\\plus{}bc\\plus{}ca\\plus{}abc(a\\plus{}b\\plus{}c)\\plus{}3\\plus{}3abc \\ge 3(a\\plus{}b\\plus{}c)$\n$ \\Leftrightarrow (a\\plus{}b\\plus{}c)^2\\plus{}abc(a\\plus{}b\\plus{}c\\plus{}3)\\plus{}2 \\ge 3(a\\plus{}b\\plus{}c)$\nNow let $ p\\equal{}a\\plus{}b\\plus{}c,q\\equal{}ab\\plus{}bc\\plus{}ca\\equal{}1,r\\equal{}abc$\nThe inequality becomes:\n$ p^2\\plus{}r(p\\plus{}3)\\minus{}3p\\plus{}2 \\ge 0$\n$ \\star p>2\\Rightarrow p^2\\minus{}3p\\plus{}2\\plus{}r(p\\plus{}3)\\equal{}(p\\minus{}1)(p\\minus{}2)\\plus{}r(p\\plus{}3)>0$\n$ \\star p \\le 2$\nBy Schur inequality, we have:\n$ p^3\\plus{}9r \\ge 4pq$ or $ r \\ge \\frac{4p\\minus{}p^3}{9}$\nSo we need to prove that:\n$ p^2\\minus{}3p\\plus{}2\\plus{}(p\\plus{}3)\\frac{4p\\minus{}p^3}{9} \\ge 0$\n$ \\Leftrightarrow p^4\\plus{}3p^3\\minus{}13p^2\\plus{}15p\\minus{}18 \\le 0$\n$ \\Leftrightarrow (p\\minus{}2)(p^3\\plus{}5p^2\\minus{}3p\\plus{}9)$\nBecause $ p \\le 2$ and $ p^3\\plus{}5p^2\\minus{}3p\\plus{}9\\equal{}p^3\\plus{}4p^2\\plus{}\\left(p\\minus{}\\frac{3}{2}\\right)^2\\plus{}\\frac{27}{4}>0$\nso we are done.\nEquality hold if and only if $ a\\equal{}b\\equal{}1,c\\equal{}0$ and permutations.[/hide]",
"Solution_8": "Dear NguyenDungTN, \r\n\r\nthank you very much for your nice solution and inequality. :)",
"Solution_9": "[quote=\"can_hang2007\"]We also have the following result\n\\[ \\sum\\frac {a \\plus{} bc}{1 \\plus{} ab} \\ge \\frac {3(\\sqrt {3} \\plus{} 1)}{4}\n\\]\nfor any $ a,b,c \\ge 0,ab \\plus{} bc \\plus{} ca \\equal{} 1$ :)[/quote]\r\n\r\n\r\nDear can_hang2007 have you a solution for this very hard inequality? I tried with full expansion by applying p,q,r method but it is almost impossible for me. :oops: \r\n\r\nThank you very much.",
"Solution_10": "Can't we see the solution of the first problem without using $ pqr$ method?",
"Solution_11": "[quote=\"can_hang2007\"]We also have the following result\n\\[ \\sum\\frac {a \\plus{} bc}{1 \\plus{} ab} \\ge \\frac {3(\\sqrt {3} \\plus{} 1)}{4}\n\\]\nfor any $ a,b,c \\ge 0,ab \\plus{} bc \\plus{} ca \\equal{} 1$ :)[/quote]\r\nMy urgly solution.",
"Solution_12": "Dear can_hang2007, \r\n\r\nthank you very much for your solution at this very hard inequality. :)"
}
{
"Tag": [
"LaTeX",
"function",
"USAMTS"
],
"Problem": "Hi, \r\nI looked on the latex guide and couldn't find much. \r\nIs there any way to page break?\r\nFore xample, say im doing texniccenter latex, and have 1.5 pages for problem 1. And i want problem 2 to be on the third page. How do i do that without doing \"\\linbreak\" the needed number of times?\r\n\r\nIs there a \\pagebreakuntilendofpage type function?\r\n\r\nThanks",
"Solution_1": "If you use the USAMTS template (which can be found on the USAMTS website), page breaks will be added automatically between solutions.",
"Solution_2": "If you don't want to do that, you can use \\newpage",
"Solution_3": "yep, thanks!",
"Solution_4": "Also, it says \"You only need to submit this file and the PDF file with your solutions (and please attach any images you include).\" \r\n\r\nWhy can't I just submit the pdf file?",
"Solution_5": "That's outdated...you can just upload the PDF file.",
"Solution_6": "Hmmm I've always used \\begin{newpage} to make a new page. It typically produces a few bad box errors and stuff though. Is there another \\lpagebreakish command?"
}
{
"Tag": [],
"Problem": "Given $ 2x^{2} \\plus{} y^{2} \\plus{}2xy \\minus{}12x \\minus{}8y \\minus{}5\\equal{}0$, find the maximum and minimum values of $ 7x\\plus{}4y$.",
"Solution_1": "hello, the maximum ist $ 47$ and will be attained for $ x\\equal{}5,y\\equal{}3$ , the minimum will reached for $ x\\equal{}\\minus{}1,y\\equal{}1$ and is $ \\minus{}3$.\r\nSonnhard.",
"Solution_2": "This problem is readily solved by geometric means after [hide=\"a bit of algebra\"]\nWrite as:\n[color=darkblue][i]Given $ x^2 \\plus{} (x \\plus{} y)^2 \\minus{} 4x \\minus{} 8(x \\plus{} y) \\plus{} 5 \\equal{} 0$, find the bounds on $ 3x \\plus{} 4(x \\plus{} y)$[/i][/color]\n\nPerform the change of variables $ z \\equal{} x \\plus{} y$ to rewrite as:\n[color=darkblue][i]Given $ x^2 \\plus{} z^2 \\minus{} 4x \\minus{} 8z \\plus{} 5 \\equal{} 0$, find the bounds on $ 3x \\plus{} 4z$[/i][/color]\n\nComplete squares and define an auxiliary variable $ M$ to rewrite as:\n[color=darkblue][i]Given $ (x \\minus{} 2)^2 \\plus{} (z \\minus{} 4)^2 \\equal{} 15$ and $ 3x \\plus{} 4y \\equal{} M$, find the bounds on $ M$.[/i][/color][/hide]",
"Solution_3": "Thanks Sonnhard and TZF."
}
{
"Tag": [
"superior algebra",
"superior algebra unsolved"
],
"Problem": "Let $ (G,.)$ is a group . \r\n\r\n$ \\exists n \\in N \\ ; \\ n >1 : (a.b)^n\\equal{}a^n.b^n \\ , \\forall a,b \\in G$ .\r\n\r\nProve that : \r\n\r\ni ) $ (a.b)^{n\\minus{}1}\\equal{}b^{n\\minus{}1}.a^{n\\minus{}1}$\r\n\r\nii) $ a^{n}.b^{n\\minus{}1}\\equal{}b^{n\\minus{}1}.a^{n}$\r\n\r\n$ \\forall a,b \\in G$ .",
"Solution_1": "1) $ a^nb^n\\equal{}(ab)^n\\equal{} a(ba)^{n\\minus{}1}b\\Rightarrow a^{n\\minus{}1}b^{n\\minus{}1}\\equal{}(ba)^{n\\minus{}1}$\r\n2) $ a^nb^{n\\minus{}1}\\equal{}aa^{n\\minus{}1}b^{n\\minus{}1}\\equal{}a(ba)^{n\\minus{}1}\\equal{}(ab)^{n\\minus{}1}a\\equal{}b^{n\\minus{}1}a^{n\\minus{}1}a\\equal{}b^{n\\minus{}1}a^n$"
}
{
"Tag": [
"algebra",
"polynomial",
"limit",
"Functional Analysis",
"advanced fields",
"advanced fields unsolved"
],
"Problem": "Let $A$ be a contraction ($\\|A\\|\\le 1$) on a complex Hilbert space, and let $D$ be the unit disc in the plane. Then, for every complex polynomial $p$, the following equality holds: $\\|p(A)\\|\\le\\sup_{z\\in D}|p(z)|$.",
"Solution_1": "I think this was used in my analysis class as a step in the proof that every linear constant coefficient PDE has a weak solution. However my professor went over the proof quite quickly and I didn't understand very well... I will think on it though :)",
"Solution_2": "Spectral radius $r(A)=\\lim\\|A^{n}\\|^{1/n}\\le1$ so $\\mathrm{Sp}\\,A\\subset D$ and thus $\\|p(A)\\|=\\max|p\\,(\\mathrm{Sp}\\,A)|\\le\\max|p\\,(D)|$.",
"Solution_3": "[quote=\"Vladimir Lubyshev\"]$\\|p(A)\\|=\\max|p\\,(\\mathrm{Sp}\\,A)|$[/quote]\r\n\r\nIn other words, the norm of $p(A)$ is equal to its spectral radius? This certainly isn't true for all operators.",
"Solution_4": "Oh... yes it is true for $A$ self-adjoint and $p$ having no poles inside $\\mathrm{Sp}\\,A$. I forgot that.\r\n\r\nI'll think on the problem when I have time. But do you know for sure the statement is true?",
"Solution_5": "I even have a proof. It's only here so that you guys can enjoy it :)."
}
{
"Tag": [
"pigeonhole principle",
"number theory solved",
"number theory"
],
"Problem": "The real number x between 0 and 1 has decimal representation .a1a2a3a4... with the following property: the number of distinct blocks of the form akak + 1ak + 2...ak + 2003 as k ranges through all positive integers, is less than or equal to 2004. Prove that x is rational.",
"Solution_1": "Did anyone think of this yet? Quite a nice question, but not so easy I think... I got somewhere using Pigeonhole Principle but I didn't manage to finish it off so far."
}
{
"Tag": [
"articles"
],
"Problem": "I do. I've found a lot of tension between a few countries. I've also found a few illegal things in there.",
"Solution_1": "What is it?\r\n\r\nA magazine?",
"Solution_2": "Yup, I do suscribe to it. How the hell do you find tension in a magazine? And what illegal stuff are you referring to?",
"Solution_3": "Using preformance enhancing drugs to improve your preformance. One guy hit farther than Babe Ruth, all thanks to drugs.",
"Solution_4": "[quote=\"nutz_for2.718281828\"]Using preformance enhancing drugs to improve your preformance. One guy hit farther than Babe Ruth, all thanks to drugs.[/quote]\r\nThat was an article a couple of issues back, wasn't it? A magazine reporting about illegal stuff, I don't see how that's particularly unusual...",
"Solution_5": "its ok, but it has too much stuff. i like The Week and Time better",
"Solution_6": "[quote=\"nutz_for2.718281828\"]I do. I've found a lot of tension between a few countries. I've also found a few illegal things in there.[/quote]\r\nWhat sort of tensions,and what sort of illegal things are we speaking about?",
"Solution_7": "[quote=\"star99\"]its ok, but it has too much stuff. i like The Week and Time better[/quote]\r\n\r\nI think it is a great magazine and extremely insightful when compared to those two magazines (the quality of Time has decreased dramatically, IMO, it is now nothing more than a pile of cultural biases plopped on top of several events and alleged \"trends\"). I do not usually read as much of it as I'd like though, just cause it's kinda dense."
}
{
"Tag": [
"geometry",
"Pythagorean Theorem"
],
"Problem": "The baseball catcher needs to stop the player on first base from stealing second base. Approximately how far must the catcher throw the ball to get to second base? A regular baseball diamond is a square 90 feet on each side.\r\n\r\nA) 90 ft B) 127 ft C) 180 ft D) 64 ft E)100 ft",
"Solution_1": "Off the top of my head, I know the answer is 127. The distance the catcher has to throw is the hypotenuse of the right triangle formed by the first and second base paths. So it's a triangle, 90 feet on two sides. The third side is the desired distance. The Pythagorean Theorem works."
}
{
"Tag": [
"group theory",
"superior algebra",
"superior algebra unsolved"
],
"Problem": "I just started my group theory class:\r\n\r\n$ \\mathbb{Q}'$ of $ \\mathbb{Q}$ is the collection of cauchy sequences in $ \\mathbb{Q}$ mod the equivalence relation of being co-Cauchy. The arithmetic is defined by:\r\n\r\n$ A\\plus{}B \\equal{} (a_{n} \\plus{}b_{n})$\r\n$ A\\minus{}B \\equal{} (a_{n} \\minus{}b_{n})$\r\n$ AB \\equal{} (a_{n} *b_{n})$\r\n$ A/B \\equal{} (a_{n} /b_{n})$\r\n\r\nHow do you check if this is a field? Can someone show an example? Thanks!",
"Solution_1": "Presumbably you have the definition of \"field\" ... check it.\r\nPart of this will be to check that that your operations are well-defined. For example, to check that + is well defined, you need: if $ (a_n)$ and $ (u_n)$ are co-Cauchy and $ (b_n)$ and $ (v_n)$ are co-Cauchy, then show $ (a_n\\plus{}b_n)$ and $ (u_n\\plus{}v_n)$ are co-Cauchy.",
"Solution_2": "How come \"well definedness\" is not included amongst the other definitions of the field that I see such as closure, associativity, distributivity, and commutativity?\r\n\r\nHow do you show co-Cauchy with the above? Do you just add each element together and just show that each resulting sum still forms a Cauchy sequence?",
"Solution_3": "Well-definedness is more likely a part of \"the addition/etc. can be defined that way\". That is, you need to show that the result is again a Cauchy sequence and that it doesn't depend on the equivalence class."
}
{
"Tag": [
"ratio",
"geometry",
"circumcircle",
"conics",
"geometric transformation",
"reflection",
"homothety"
],
"Problem": "Hi, everyone :roll: \r\n\r\nProve the following:\r\nif R be any point on a circle, A and B fixed points on a diameter and equidistant from the center,\r\nthe envelope of a line which cuts harmonically the two circles with A, B as centers and AR, BR as radii\r\nis independent of the position of R on the circle.\r\n\r\nThanks :)\r\nPlease your answers. I am studying materials related to this post nowadays.",
"Solution_1": "Let $ (O)$ be the circle with $ R$ on it. Let a line $ l$ cut the circle $ (A)$ at $ X, Y$ and the circle $ (B)$ at $ Z, T.$ Assume that the cross ratio $ \\frac {\\overline{XZ}}{\\overline{XT}} \\cdot \\frac {\\overline{YZ}}{\\overline{YT}} \\equal{} \\minus{} 1$ is harmonic. Let $ M, N$ be midpoints of $ XY, ZT$ and $ (M), (N)$ circles with diameters $ XY, ZT,$ intersecting at $ P, P'.$ Since $ PX \\perp PY,$ these lines bisect $ \\angle ZPT$ internally and externally, $ (M)$ is P-Apollonius circle of $ \\triangle PZT$ with circumcircle $ (N)$ $ \\Longrightarrow$ $ (M) \\perp (N).$ Inversion in $ (M)$ takes $ (N)$ to itself and $ Z$ to $ T$ $ \\Longrightarrow$ $ \\overline{MZ} \\cdot \\overline{MT} \\equal{} \\minus{} \\overline{MX} \\cdot \\overline{MY}$ $ \\Longrightarrow$ $ \\frac {p(M, (B))}{p(M, (A))} \\equal{} \\minus{} 1.$ In exactly the same way, $ \\frac {p(N, (B))}{p(N, (A))} \\equal{} \\minus{} 1.$ Since powers of $ M, N$ to $ (A), (B)$ are in the same ratio $ r \\equal{} \\minus{} 1,$ $ M, N$ are on a circle coaxal with $ (A), (B),$ centered on $ AB$ and passing through $ R.$ As $ \\frac {\\overline{AO}}{\\overline{BO}} \\equal{} r \\equal{} \\minus{} 1,$ $ \\odot(MNR) \\equiv (O).$ The feet $ M, N$ of perpendiculars $ AM \\perp l, BN \\perp l$ from the fixed points $ A, B$ to the arbitrary line $ l$ are therefore on the fixed circle $ (O)$ $ \\Longrightarrow$ $ l$ is tangent to a conic with foci $ A, B$ and pedal circle $ (O).$",
"Solution_2": "Suppose that $ A,B$ lie inside the circle $ (O)$ centered at the midpoint of $ AB$ with radius $ OR \\equal{} \\varrho.$ Let the variable line $ \\ell$ cut $ (A)$ at $ P,Q$ and $ (B)$ at $ X,Y.$ Let $ M$ denote the orthogonal projection of $ A$ on $ \\ell,$ which is the midpoint of the chord $ PQ.$ Since $ (P,Q,X,Y) \\equal{} \\minus{} 1,$ by Newton's theorem we have $ MP^2 \\equal{} MX \\cdot MY$ $ \\Longrightarrow$ $ p(M,(A)) \\equal{} p(M,(B)).$ Hence $ M$ lies on the circle centered at the midpoint of $ AB$ and coaxal with $ (A)$ and $ (B),$ i.e. $ M \\in (O).$ Let $ A'$ be the reflection of $ A$ across $ \\ell$ and let $ N \\equiv \\ell \\cap BA'.$ $ A'$ moves on the homothetic circle $ (B, 2\\varrho)$ of $ (O)$ through the homothety centered at $ A$ with coefficient $ 2.$ Therefore $ NA \\plus{} NB \\equal{} NA' \\plus{} NB \\equal{} 2\\varrho \\equal{} \\text{const}$ $ \\Longrightarrow$ Locus of $ N$ is the ellipse $ \\mathcal{E}$ with foci $ A,B$ and major axis $ 2\\varrho.$ Since $ \\ell$ is the external bisector of $ \\angle ANB,$ it follows that $ \\ell$ is tangent to $ \\mathcal{E}.$ When $ A,B$ lie outside $ (O),$ analogous reasoning yields that $ \\ell$ is tangent to the hyperbola with foci $ A,B$ and major axis $ 2\\varrho.$"
}
{
"Tag": [
"inequalities",
"inequalities unsolved"
],
"Problem": "$ a,b,c,p,q\\in R^\\plus{}$\r\n$ abc\\equal{}1$ and $ p\\ge q$\r\nfind minimum value of $ \\dfrac{a^p}{b^q\\plus{}c^q}\\plus{}\\dfrac{b^p}{c^q\\plus{}a^q}\\plus{}\\dfrac{c^p}{a^q\\plus{}b^q}$",
"Solution_1": "I think that p and q are in IN and $ p > q$\r\nChebychev:\r\n$ S \\equal{} \\dfrac{a^p}{b^q \\plus{} c^q} \\plus{} \\dfrac{b^p}{c^q \\plus{} a^q} \\plus{} \\dfrac{c^p}{a^q \\plus{} b^q}\\geq \\frac {1}{3}(a^p \\plus{} b^p \\plus{} c^p)\\left(\\sum \\frac {1}{a^q \\plus{} b^q}\\right)$\r\ncaushy:\r\n$ \\left(\\sum \\frac {1}{a^q \\plus{} b^q}\\right)(2(a^q \\plus{} b^q \\plus{} c^q))\\geq9$\r\n$ \\Leftrightarrow \\left(\\sum \\frac {1}{a^q \\plus{} b^q}\\right)\\geq\\frac {9}{2(a^q \\plus{} b^q \\plus{} c^q)}$\r\ntherefor:\r\n$ S \\geq\\frac {3}{2}\\times\\frac {a^p \\plus{} b^p \\plus{} c^p}{a^q \\plus{} b^q \\plus{} c^q}$\r\nwith Chebychev:\r\n$ S \\geq \\frac {3}{2}\\times\\frac {a^p \\plus{} b^p \\plus{} c^p}{a^q \\plus{} b^q \\plus{} c^q} \\geq \\frac {3}{2}\\times\\frac {1}{3}\\times\\frac {(a \\plus{} b \\plus{} c)(a^{p \\minus{} 1} \\plus{} b^{p \\minus{} 1} \\plus{} c^{p \\minus{} 1})}{a^q \\plus{} b^q \\plus{} c^q}\\geq \\frac {3}{2}$\r\nbecause with AM-GM:\r\n$ a \\plus{} b \\plus{} c \\geq3$\r\nthen:\r\n$ S\\geq \\frac {3}{2}$\r\nwith equality only if $ a \\equal{} b \\equal{} c$.",
"Solution_2": "thanks :)",
"Solution_3": "[quote=\"pqrs\"]$ a,b,c,p,q\\in R^ \\plus{}$\n$ abc \\equal{} 1$ and $ p\\ge q$\nfind minimum value of $ \\dfrac{a^p}{b^q \\plus{} c^q} \\plus{} \\dfrac{b^p}{c^q \\plus{} a^q} \\plus{} \\dfrac{c^p}{a^q \\plus{} b^q}$[/quote]\r\nLet $ r \\equal{} \\frac{p}{q}$. And $ x \\equal{} a^q$, $ y \\equal{} b^q$, $ z \\equal{} c^q$.\r\nThen we have to find the minimum of $ \\frac{x^r}{y\\plus{}z} \\plus{} \\frac{y^r}{z\\plus{}x} \\plus{} \\frac{z^r}{y\\plus{}x}$ when $ xyz\\equal{}1$.\r\nWe prove that $ \\frac{x^r}{y\\plus{}z} \\plus{} \\frac{y^r}{z\\plus{}x} \\plus{} \\frac{z^r}{y\\plus{}x} \\ge \\frac{3}{2}$ $ \\iff$\r\n$ \\sum_{cyc} 2x^r(x\\plus{}y)(x\\plus{}z) \\equal{} \\sum_{cyc} 2x^{r\\plus{}2} \\plus{} 2x^{r\\plus{}1}y \\plus{} 2x^{r\\plus{}1}z \\plus{} 2x^ryz \\ge 3(x\\plus{}y)(y\\plus{}z)(z\\plus{}x)$.\r\nWritten by muirhead means:\r\n$ 6[r\\plus{}2,0,0] \\plus{} 12[r\\plus{}1,1,0] \\plus{} 6[r,1,1] \\ge 12[1,1,1] \\plus{} 18[2,1,0]$. Which is obviously true when homogenizing since $ r \\ge 1$.",
"Solution_4": "More directly, by Chebyschev\r\n\r\n$ \\sum \\frac {a^{p \\minus{} q \\plus{} q}}{b^q \\plus{} c^q} \\ge (\\sum \\frac {a^{p \\minus{} q}}{3})(\\sum \\frac {a^q}{b^q \\plus{} c^q})$\r\n\r\nBy AM - GM,\r\n\r\n$ \\sum \\frac {a^{p \\minus{} q}}{3} \\ge 1$.\r\n\r\nBut $ \\sum \\frac {a^q}{b^q \\plus{} c^q} \\ge \\frac {3}{2}$ is Nesbitt."
}
{
"Tag": [],
"Problem": "$a,b,c \\in \\mathbb{R}$\r\nFor all $n \\in \\mathbb{N}$ , we have:\r\n$[na] + [nb] = [nc]$ \r\nProve that at least one of those numbers (a,b,c) is an integer.",
"Solution_1": "does anyone know the solution ?",
"Solution_2": "Hmm... I have solution but it is messy, I have another idea, so I'll try to solve it in easier way and then post the solution.",
"Solution_3": "Have a look here :\r\nhttp://www.mathlinks.ro/Forum/viewtopic.php?highlight=density&t=4568\r\n\r\nPierre."
}
{
"Tag": [
"complex analysis",
"function",
"integration",
"calculus",
"derivative",
"complex analysis unsolved"
],
"Problem": "Hello to all!\r\n\r\nI'm reading an article about series expansions and I found, what I think is a really strange result:\r\n\r\nWe have\r\n\r\n$ \\Theta(x)$ an holomorphic function inside a contour S, having only a root $ a$ inside S;\r\n\r\n$ x$ the affixe of a point inside S;\r\n\r\nFor every point $ z$ of the contour S:\r\n\r\n\\[ |\\Theta(x)|<|\\Theta(z)|\\]\r\n\r\nand the author states that the following:\r\n\r\n[i]The equation [/i]\r\n\r\n\\[ \\Theta(z)\\minus{}\\Theta(x)\\equal{}0\\]\r\n\r\n[i]has only one root $ z\\equal{}x$ inside $ S$, as we can see in this equality[/i]\r\n\\[ \\frac{1}{2i\\pi}\\int_{S}\\frac{\\Theta'(z)dz}{\\Theta (z)\\minus{}\\Theta(x)}\\equal{}\\frac{1}{2i\\pi}\\left[\\int_{S}\\frac{\\Theta'(z)dz}{\\Theta (z)}\\plus{}\\Theta(x)\\int_{S}\\frac{\\Theta'(z)dz}{\\Theta^{2}(z)}\\plus{}\\ldots\\right]\\equal{}\\frac{1}{2i\\pi}\\int_{S}\\frac{\\Theta'(z)dz}{\\Theta(z)}\\]\r\n\r\n[i]where the first and last members represent respectively, the number of roots of the considered equation and the number of roots of $ \\Theta(z)\\equal{}0$.[/i]\r\n\r\n\r\n\r\nI understand the first step of the equality (he used the same expansion as $ \\frac{1}{1\\minus{}x}$), but not the second... it looks as if he just made $ \\Theta(x)\\equal{}0$\r\n\r\nDoes anyone know what is happening there?\r\n\r\nThank you",
"Solution_1": "Key words to look for: [url=http://en.wikipedia.org/wiki/Rouch%C3%A9%27s_theorem]Rouche's theorem[/url], [url=http://en.wikipedia.org/wiki/Argument_principle]argument principle[/url].\r\n\r\nIn the second term, $ \\frac{\\Theta'}{\\Theta^{2}}$ is the derivative of $ \\minus{}\\frac{1}{\\Theta}$, and we're integrating around a closed curve. It evaluates to zero by the Fundamental Theorem.",
"Solution_2": "Thanks a lot!!! :) \r\n\r\nI thought I was never going to figure it out!! :oops:"
}
{
"Tag": [
"number theory",
"prime numbers"
],
"Problem": "The sequence of the Human Genome was published 2003. Some people are using this code to generate music or to cipher messages or use DNA to do computing.\r\n\r\nI was wondering if there is any correlation between the DNA code and the sequence of prime numbers. I am not a mathematician but generally interested in Science, so forgive me if this a \"weired\" idea.\r\nBut would it be not funny to be able to predict a DNA motif with number, especially prime number, theory?\r\n\r\nThanks!",
"Solution_1": "Why primes and not composite, or Fermat numbers ?\r\nDo you have any reasonable reason to believe there is any link with [b]any[/b] known number ?\r\nTo give my sceptical answer: You can \"find\" prime numbers anywhere in nature, but that's mostly (and most probably) numerology not mathematics.\r\n\r\nP.S: This post does not belong to the number theory section (tough it's \"related\" to prime) it should go in the Round Table section.\r\nP.S.S: Why do you need the human genome to generate music ? You could aswell (and with the same sucess) use the inscription on the butter you bought (let's suppose) today :)."
}
{
"Tag": [],
"Problem": "I was thinking to put this in SC competition but I decided not to because it might be hard for some beginners. Anyway, this is quite good thinking question so here it goes:\r\n\r\nd, a positive integer, is a nice number that can be resulted from either of expression a+b+c or a*b*c where a,b, and c are each distinct positive number. Prove that (a+b+c)*d equals to a*b*c*d by defining each value for a,b,c, and d (a,b,c, and d are each different numbers) if a,b,c are consecutive numbers while a,b,c,d aren't.\r\n\r\nWording might be pretty confusing but you will find this easy.",
"Solution_1": "But I thought:\n\n\n\n[hide]a+b+c = d\n\nd = a*b*c\n\n\n\nBy transitivity, \n\n\n\na+b+c = a*b*c\n\n\n\nMultiplying both sides by d,\n\n\n\n(a+b+c)*d = a*b*c*d[/hide]",
"Solution_2": "[quote]Wording might be pretty confusing but you will find this easy.[/quote]\r\n\r\nNo offense, but I'll say.",
"Solution_3": "Remember, you have to define the value for a,b,c, and d.",
"Solution_4": "Answer: [hide]{a, b, c, d} = {1, 2, 3, 6}[/hide]",
"Solution_5": "Excellent. Now, prove that it is only positive pair that's possible. Also, state the another pair that isn't all positive but satisfies the requirements I asked.",
"Solution_6": "[hide]{a,b,c,d)={-1,-2,-3,-6}\n\nor it also could be {-1,0,1,0}[/hide]"
}
{
"Tag": [
"inequalities unsolved",
"inequalities"
],
"Problem": "For $0< a,b,c < \\frac{1}{2}$ prove that\r\n\r\n$(\\frac{1}{a}-1)(\\frac{1}{b}-1)(\\frac{1}{c}-1)\\geq (\\frac{3}{a+b+c}-1)^3$",
"Solution_1": "Probably there is a more astute solution but log (1/x-1) is convex on ]0..1/2] + Jensen."
}
{
"Tag": [
"geometry"
],
"Problem": "Let $O =(0,0), A = (1,3), B = (4,1)$ $D$ is on $OB$ such that $AD$ is perpendicular to $OB$. Find the area of triangle $OAD$.",
"Solution_1": "The line through $O$ and $B$ is $y=\\frac{1}{4}x$\r\n\r\nSo let $D=(t, \\frac{1}{4})t$ $\\vec{AD}=[t-1 ; \\frac{1}{4}t-3]$ $\\vec{OB}=[4;1]$\r\n\r\n$AD$ is perpendicular to $OB$ so : $4t-4+\\frac{1}{4}t-3=0$ $t=\\frac{28}{17}$\r\n\r\nNow we know coordinations of $A,O,D$ so it's easy to calculate $AD$ and $DO$, \r\nArea of $\\triangle{ADO}= \\frac{1}{2}AD \\cdot DO$"
}
{
"Tag": [
"probability",
"parameterization",
"limit",
"integration",
"calculus",
"probability and stats"
],
"Problem": "$ X_1, ..., X_5$ are iid random variables having exponential distribution with parameter $ \\lambda$. Find $ P(X_1 \\geq 3|X_1 \\plus{} ... \\plus{} X_5\\equal{}5)$",
"Solution_1": "Can anyone help me:?:",
"Solution_2": "You may think about $ X_{1}$ as about minimum of five uniformly selected on $ [0;5]$ points.",
"Solution_3": "$ (1 \\minus{} e^{ \\minus{} 3\\lambda})^5$ ?",
"Solution_4": "[quote=\"roah\"]You may think about $ X_{1}$ as about minimum of five uniformly selected on $ [0;5]$ points.[/quote]\r\nCorrect answer is $ 0,0256$, but I don't know how to find it :wink: If we think like You said we get $ 0,01024$ which is wrong. When I think about $ X_{1}$ as about minimum of FOUR uniformly selected on $ [0;5]$ points we get $ 0,0256$. But, could You explain this thinking? Why should I think about $ X_1$ like You wrote? I would be appreciate :)",
"Solution_5": "it should be four, since the FIFTH point is AT 5 really, therefore we dont need to think about it.",
"Solution_6": "But I still don't understand why these point are uniformly selected from $ [0,5]$, I wrote in my first post that $ X_1,...,X_5$ are exponential distributed random variables :wink:. Any ideas to explain this fact? :blush:",
"Solution_7": "thinking about it today i came across the same issue you just asked...\r\n\r\nthe exponential thing doesnt seem to play a role if we can generalize like this.. as long as its a continuous distribution and $ X_n \\ge 0$ i think we can use the same argument..?",
"Solution_8": "why don't we try to compute for example the exact conditional law of $ X_1$, give that $ X_1 + \\ldots, X_n = A$:\r\n$ X = X_1$ is independent of $ Y = X_2 + \\ldots + X_n$, and $ Y$ has a Gamma probability distribution with parameter $ (\\lambda, n - 1)$:\r\n\\[ P(Y \\in [y; y + dy]) = G(y) dy = \\frac {\\lambda^{n - 1}}{\\Gamma(n - 1)} y^{n - 2} \\exp( - \\lambda y) dy\r\n\\]\r\nNow, what we are trying to compute is:\r\n\\[ F(b,a) = \\lim_{\\epsilon \\to 0} \\frac {P[ (X \\leq b) \\cap (X + Y \\in [a; a + \\epsilon])] }{ P[ (X + Y \\in [a; a + \\epsilon])] } = P(X \\leq b | X_1 + \\ldots + X_n = a)\r\n\\]\r\nwhere\r\n\\[ P[ (X \\leq b) \\cap (X + Y \\in [a; a + \\epsilon]) = A(b,a + \\epsilon) - A(b,A)\r\n\\]\r\n\r\n\\[ P[ (X + Y \\in [a; a + \\epsilon]) = A(\\infty,a + \\epsilon) - A(\\infty,A)\r\n\\]\r\n\r\n\\[ A(b,a) = P[(X \\leq b) \\cap (X + Y \\leq a)] = \\int_{x = 0}^b \\lambda \\exp( - \\lambda x) [\\int_{y = 0}^{a - x} G(y)dy] dx\r\n\\]\r\nhence\r\n\\[ F(b,a) = \\frac {\\partial_{a} A(b,a) }{\\partial_a A(\\infty, a)} = \\frac {\\int_{x = 0}^b \\lambda \\exp( - \\lambda x) G(a - x) dx }{\\int_{x = 0}^{a} \\lambda \\exp( - \\lambda x) G(a - x) dx } = \\frac {\\int_{x = 0}^b x(a - x)^{n - 2} dx }{\\int_{x = 0}^{a} x(a - x)^{n - 2} dx }\r\n\\]\r\nwhence $ P(X \\leq b | X_1 + \\ldots + X_n = a) = (\\frac {b}{a})^n \\frac {\\int_{u = 0}^1 u(\\frac {a}{b} - u)^{n - 2} du }{\\int_{u = 0}^{1} u(1 - u)^{n - 2} du } = \\phi(\\frac {b}{a})$.\r\nThis shows that the conditional density of $ X_1$ given that $ X_1 + \\ldots + X_n = a$ is something like $ Cx(a-x)^{n-2} dx$\r\n(which is a little weird for $ n=2$, so I may have made a computation mistake somewhere :blush: )",
"Solution_9": "Let me try to pick up some loose ends and give some more details to the above posts. \r\n\r\nLook at the more general problem $ P(X_1 \\le b| X_1 \\plus{} \\ldots \\plus{} X_n \\equal{} a) \\equal{} ?$, where $ n \\ge 2$, the $ X_i$ are iid exponential random variables (parameter $ \\lambda >0$) and $ 0 \\le b \\le a$ (as considered by alekk). \r\n\r\nThe connection to the uniform distribution is the following: Because of the exponential distribution (which is crucial for this) the points $ Y_n : \\equal{} X_1 \\plus{} \\ldots \\plus{} X_n$ form a Poisson point process on the positive part of the real line. The random set of points $ Y \\equal{} \\{ Y_n: n \\ge 1\\}$ has the following properties: \r\nFor every interval $ I$ of length $ L$ the number of points of $ Y$ in $ I$ is Poisson-distributed (with parameter $ \\lambda L$) and for a fixed number $ n$ of points the conditional distribution of the positions of these $ n$ points in $ I$ is: The positions are independent and uniformly distributed. The condition $ X_1\\plus{} \\ldots \\plus{} X_n \\equal{} a$ is the same as \"in the interval $ [0,a)$ there are exactly $ n\\minus{}1$ points of $ Y$\". The condition $ X_1\\plus{} \\ldots \\plus{} X_n \\equal{} a$ and $ X_1 \\le b$ is the same as \"in the interval $ [0,a)$ there are exactly $ n\\minus{}1$ points of $ Y$ and at least one of them is in $ [0,b]$\". (This elaborates the idea of roah, and the comment of kenn4000 that really only $ n\\minus{}1$ points are uniformly chosen, and $ X_1$ is indeed the minimum of $ n\\minus{}1$ independent uniformly distributed random variables.)\r\n\r\n\r\nTo apply this to the above problem: $ P(X_1 \\le b| X_2 \\plus{} \\ldots \\plus{} X_n \\equal{} a) \\equal{}$\r\n$ 1 \\minus{}P(Y$ has no point in $ [0,b]$ | $ Y$ has exactly $ n\\minus{}1$ points in $ [0,a)$) \r\n$ \\equal{} 1 \\minus{} P($of $ n\\minus{}1$ uniformly chosen independent points in $ [0,a)$ no point is in $ [0,b]) \\equal{} 1 \\minus{} (1 \\minus{} \\frac{b}{a})^{n\\minus{}1}$.\r\n\r\nThe calculation of alekk gives the same formula (small mistake in the last step of the calculation of $ F$):\r\n$ F(b,a) \\equal{} \\frac {\\partial_{a} A(b,a) }{\\partial_a A(\\infty, a)} \\equal{} \\frac {\\int_{x \\equal{} 0}^b (a \\minus{} x)^{n \\minus{} 2} dx }{\\int_{x \\equal{} 0}^{a} (a \\minus{} x)^{n \\minus{} 2} dx } \\equal{} 1 \\minus{} (1 \\minus{} \\frac{b}{a})^{n\\minus{}1}$,\r\n\r\nApplied to the original problem this gives $ P(X_1 \\geq 3|X_1 \\plus{} ... \\plus{} X_5\\equal{}5) \\equal{} (1 \\minus{} \\frac{3}{5})^4 \\equal{} 0.0256$.",
"Solution_10": "Thank You very much. I have only one question. [b]alekk[/b] wrote \r\n$ A(b,a) \\equal{} P[(X \\leq b) \\cap (X \\plus{} Y \\leq a)] \\equal{} \\int_{x \\equal{} 0}^b \\lambda \\exp( \\minus{} \\lambda x) [\\int_{y \\equal{} 0}^{a \\minus{} x} G(y)dy] dx$\r\nso, how should I understand $ A(\\infty ,a)$ ?\r\nI think it is $ P[(X \\leq \\infty) \\cap (X \\plus{} Y \\leq a)]\\equal{}P(X \\plus{} Y \\leq a)$? But in integral form? I guess it isn't \r\n$ \\int_{x \\equal{} 0}^\\infty \\lambda \\exp( \\minus{} \\lambda x) [\\int_{y \\equal{} 0}^{a \\minus{} x} G(y)dy] dx$ because this integral doesn't coverage on $ (0,\\infty)$. So my question is, how should I understand $ A(\\infty ,a)$? How to write this expresion using integrals?",
"Solution_11": "[quote=\"zuraw\"]\n$ \\int_{x \\equal{} 0}^\\infty \\lambda \\exp( \\minus{} \\lambda x) [\\int_{y \\equal{} 0}^{a \\minus{} x} G(y)dy] dx$ because this integral doesn't coverage on $ (0,\\infty)$ So my question is, how should I understand $ A(\\infty ,a)$? How to write this expresion using integrals?[/quote]\r\n\r\nyes, as you said:\r\n$ A(\\infty,a)\\equal{}P[X\\plus{}Y \\leq a]\\equal{}\\int_{x \\equal{} 0}^\\infty \\lambda \\exp( \\minus{} \\lambda x) [\\int_{y \\equal{} 0}^{a \\minus{} x} G(y)dy] dx$ \r\nwhich is equal to:\r\n$ \\int_{x \\equal{} 0}^a \\lambda \\exp( \\minus{} \\lambda x) [\\int_{y \\equal{} 0}^{a \\minus{} x} G(y)dy] dx$"
}
{
"Tag": [
"MIT",
"college",
"Harvard",
"Princeton",
"parameterization",
"topology",
"Yale"
],
"Problem": "Hi,\r\n\r\nWhat are the differences between top US Universities such as Harvard, Princeton, MIT with top UK Universities such as Cambridge and Oxford in terms of quality/character of students, environment, community, overall college experience (academically and non-academically), quality of teaching and X, \r\n\r\nwhere X is any parameter you may choose and any operation may be enclosed (If you know what I mean. I don't even sure whether I'm using the correct words! Haha! Never mind. You get the point. Just having a little fun here. Btw, it is not profane!)?\r\n\r\nPlease give your opinions. Thanks!",
"Solution_1": "Honestly, you can't go wrong choosing any of those. Do you want to have a chance to be around Stephen Hawking? Go to Cambridge. Do you want to be taught by the great Andrew Wiles? Go to Princeton. Do you want to say that you graduated from Harvard? Go to Harvard. You can't go wrong.",
"Solution_2": "Oxford isn't really advisable - it's a reasonable amount worse than Cambridge for maths and sciences in general. (This is pretty widely accepted; I'm not just being all anti-Oxford because I'm at Cambridge.)\r\nThere are loads of great mathematicians here - Tim Gowers, Alan Baker, Ben Green, Peter Swinnington Dyer etc. - I imagine this is a trait shared with all the major U.S. ones mentioned.\r\n(A nice thing about Oxbridge - supervisions/tutorials - you could reasonably end up having a Field's medallist go over your solutions to example sheets once a week.)\r\nIn truth I'm just saying all that famous people stuff just to wow you - it really matters what they're like as teachers and I can't really comment on that.\r\nOne more advantage about the U.K; you only have to do maths, so none of all those other classes you might end up going to at a U.S. school.\r\n(BTW - Cambridge and Oxford are stunning towns - I don't want to offend anyone but I'm willing to go ahead and say they are nicer places than the locations of the U.S. ones mentioned.)",
"Solution_3": "Don't make your decisions about what famous people are at the university. Take note of these key factors: \r\n\r\n1) Famous mathematicians and physicists might not actually have to teach undergrads anyway (Wiles rarely teaches undergrad classes here). If you're thinking grad school, it may be really competitive to get one of them for an advisor. Also some of the really famous people are also ancient and aren't doing much (like someone mentioned Swinnerton-Dyer before -- isn't he like 80 yrs old? and Stein at Princeton is almost that old, and he might stop teaching after this year.) \r\n\r\n2) Famous mathematicians and physicists aren't necessarily good teachers. For example, earlier this year I went to a lecture by Endre Szemeredi, who is one of the most brilliant combinatorialists alive. But his lecture was not good at all. And Chris Skinner (a great algebraic NT guy at princeton) is known for giving really tedious and painful and annoying assignments. And Stein is kinda boring and basically says exactly what's in the book (not surprising because he wrote it, but still). \r\n\r\nSo basically, my point is that you shouldn't base your decision on celebrities. That's not to say you won't benefit from them at all -- my friend at MIT has Munkres for topology and Artin for algebra and he says they're really good. And last year I did a reading course with Fefferman (fields medalist) and I got to have one-on-one discussions with him every week about geometric measure theory. \r\n\r\nNow I haven't been to Oxford or Cambridge and I don't know many people there, but I can say that the top USA universities are all pretty similar in overall quality, and you should choose between them based on your personal preferences. Do you want to live in a city? Then MIT or Harvard might be preferable over Yale and especially Princeton. If you prefer a more quiet setting with actual trees and green stuff and old buildings, reverse that order (and probably Oxbridge go ahead of all of the USA places). \r\n\r\nYou might also want to wait and find out what kind of financial aid you get (the USA places are all pretty generous but Harvard and Princeton are the best). \r\n\r\nOther factors might be the specific topics you want to study (for example, Princeton has a lot of people in analysis, number theory, and low dimensional topology, and I don't know the reputations of other schools' departments that well but you can find out easily enough), what options the schools offer (e.g. Princeton has a lot of interesting certificates you can get, but they won't let you double major), and what kind of classroom environment you want. Or extracurriculars -- do you want to do sports, join a performance group, join a math or physics club? Are you interested in frats and partying? You can ask a student at whichever university you're interested in to find out what the extracurriculars are like there. \r\n\r\nThere are tons of other factors too. But don't make 'what big name professor does the school have' one of them.",
"Solution_4": "Hello everyone!\r\n\r\n Im a student about 16 years old and im very interested about maths. What i mean is that i participate in mathematical competitions study hard maths especially number theory< i love Andreescu> but i think i dont have the brain to go to I.M.O. My dream is to join a high college like Princeton, M.I.T and others. Can i do that withoiut taking place in an I.M.O??",
"Solution_5": "You don't need to go to IMO in order to get into that kind of university. You just need to be a good student. In your application, you should stress the following points:\r\n\r\n(1) You have the necessary English skills and a good enough secondary school education to succeed in the university you're applying to. \r\n\r\n(2) You fall into one of the three major categories of students that are generally accepted to this type of university. They are: (i) Students who are amazingly good at one particular thing and not much else (you would probably need to go to IMO at least in order to qualify as this for math). (ii) Students who are well-rounded, i.e. good at a variety of things (and I don't mean maths, physics, and chemistry). If you do stuff like organizing a group, writing a lot, playing music, etc -- that will help in this category. (iii) Students who contribute significantly to the diversity of the community (you are Greek? Maybe you should emphasize Greek culture in your application. Do you play an instrument -- can you play Greek folk music? Would you want to get involved with (or create) a Greek culture student group at your university? etc...) \r\n\r\n(3) You have reasons for going to the university besides the name recognition. Learn some specific things about the university you're applying to and explain why those things make you want to go there. They don't want to be filled by a bunch of students who just want the name MIT or Princeton on their resumes (although it's inevitable that most people really are there for that reason). If you like number theory, in your Princeton application you could talk about the great number theorists in the Math Dept, like Sarnak and Skinner and Wiles and Katz.",
"Solution_6": "[quote=\"Xevarion\"] (i) Students who are amazingly good at one particular thing and not much else (you would probably need to go to IMO at least in order to qualify as this for math). [/quote]\r\n\r\nAren't there other ways of showing this? :(",
"Solution_7": "[quote=\"BanishedTraitor\"][quote=\"Xevarion\"] (i) Students who are amazingly good at one particular thing and not much else (you would probably need to go to IMO at least in order to qualify as this for math). [/quote]\n\nAren't there other ways of showing this? :([/quote]\r\nPublish something? My impression about math is that there are so many international students who are extremely good at math (but not particularly special in any other way) that you have to basically be one of the best in order to get in to a top university for that reason alone. Maybe if you don't go to IMO but show knowledge of advanced math and very high potential, that would be enough. Who knows, I'm not an admissions officer.",
"Solution_8": "Well, I don't really mean Mathematics (too many good people :D ) but what about Sciences?",
"Solution_9": "[quote=\"BanishedTraitor\"]Well, I don't really mean Mathematics (too many good people :D ) but what about Sciences?[/quote]\r\nI don't know enough about it. Sorry. I imagine anything beyond what a typical \"smart student\" would do in school in your country is at least a useful step in that direction."
}
{
"Tag": [],
"Problem": "Anyone interested in a chess tournament between mathlinks.ro members?",
"Solution_1": "I would be if I had a clue besides the basic rules of how to play that game :)",
"Solution_2": "the basic rules are everything you need :D",
"Solution_3": "I'm up for it, altough it has been a long time since I played \"seriously\" -- playing Go now instead.",
"Solution_4": "Maybe someone could put up a poll regarding this.",
"Solution_5": "Done.\r\nAlso,post which chess network would you like to play on.",
"Solution_6": "im up for it, but i dont have a mathlinks account, so if you go to funorb.com, my username is hyperhavoc5. funorb has many different games"
}
{
"Tag": [
"number theory unsolved",
"number theory"
],
"Problem": "Let $a,b,c,d > 1$ natural numbers which satisfy $a^{b^{c^d}}=d^{c^{b^a}}$ . Prove that $a = d$ and $b=c$.",
"Solution_1": "Who can post solution of this problem? Please post it!",
"Solution_2": "I'm not sure if I'm right anway I'll post my solution since nobody else does...\r\n\r\nSince LHS=RHS they must contain the same prime factors, especially being $ a,b,c,d \\in \\mathbb N$ then $ a | d , d| a$\r\n\r\nwhich means $ a=d$. With substitutions we get $ {a}^{b^{{c}^{a}}}=a^{c^{b^{a}}}$$ \\Rightarrow b^{c}=c^{b}$ $ \\Rightarrow b|c,c|b\\Rightarrow b=c$.\r\n\r\nPlease tell me if I'm wrong :)",
"Solution_3": "Your solution is based on the argument that, in general $ x^{y}=y^{x}\\Longleftrightarrow x|y\\wedge y|x\\Longleftrightarrow x=y$. The first equivalence is wrong. We have indeed $ 2^{4}=4^{2}$ and $ 4\\not=2$.",
"Solution_4": "I see.. :blush:",
"Solution_5": "however, i've been trying to find a second counter-example, but i can't find one. we can indeed use the following argument:\r\n\r\nIt is evident that (or quickly established), supposing $ x\\geq y$ by symetry, $ x^{y}=y^{x}\\Longleftrightarrow x=y^{k}$, $ k$ being a natural number. Now, we may say that the relation is equivalent to $ \\left(y^{k}\\right)^{y}=y^{y^{k}}\\Longleftrightarrow y^{k\\cdot y}=y^{y^{k}}\\Longleftrightarrow k\\cdot y=y^{k}$ which implies either $ k=1\\Longleftrightarrow x=y$ or $ y=2\\wedge x=4$. This lemma shows that, supposing we have established the relation $ a=d$, we also have the relation $ b=c$ (or the special case stated above)."
}
{
"Tag": [],
"Problem": "If $ 60$ miles per hour is $ 88$ feet per second, how many feet per second is $ 66$ miles per hour? Express your answer as a decimal to the nearest tenth.",
"Solution_1": "$ 88 \\times \\frac{11}{10}\\equal{}\\frac{88\\plus{}880}{10}\\equal{}\\boxed{96.8}$."
}
{
"Tag": [
"trigonometry"
],
"Problem": "1)Given sequence $ x_n$ such that:$ x_n \\in R;x_{n + 1} = x_n + \\frac{1}{2}(c{\\rm{os}}\\,x_n + \\sin \\,x_n )(\\forall n \\in N^* )$.Find limited of sequence depending on $ x_1$.\r\n2)Given p be a prime and $ p \\ge 5$.Prove that exist $ q_1$,$ q_2$ such that $ 1 < q_1 < q_2 < p$ simultaneously $ q_1 ^{p - 1} - 1$ and $ \\q_2 ^{p - 1} - 1$ non divisible $ p^2$",
"Solution_1": "some body help me !"
}
{
"Tag": [
"inequalities",
"inequalities unsolved"
],
"Problem": "If $a$, $b$, $c$ are positive real numbers, prove that\r\n$\\sqrt{\\frac{a}{b+c}}+\\sqrt{\\frac{b}{c+a}}+\\sqrt{\\frac{c}{a+b}}> 2$.",
"Solution_1": "Using Cauchy:\r\n$a+(b+c) \\geq 2\\sqrt{a(b+c))}$\r\n=> $\\sqrt{\\frac{a}{b+c}} \\geq \\frac{2a}{a+b+c}.$\r\nSimilar:\r\n $ \\sqrt{\\frac{b}{a+c}} \\geq \\frac{2b}{a+b+c}.$\r\n $ \\sqrt{\\frac{c}{a+b}} \\geq \\frac{2c}{a+b+c}.$\r\n=> $L.H.S > \\frac{2a+2b+2c}{a+b+c} = 2.$\r\n(Because a,b,c>0).\r\n\r\nThe inequality very easy.",
"Solution_2": "It was posted but i will give a solution\r\n$(a+b+c)^2\\geq4a(b+c)$ and equivalently $\\sqrt\\frac{a}{b+c}\\geq\\frac{2a}{a+b+c}$\r\nSame $\\sqrt\\frac{b}{a+c}\\geq\\frac{2b}{a+b+c}$ and\r\n$\\sqrt\\frac{c}{b+a}\\geq\\frac{2c}{a+b+c}$\r\nwe add the three ineqs and we have\r\n$\\sqrt\\frac{a}{b+c}+\\sqrt\\frac{b}{a+c}+\\sqrt\\frac{c}{b+a}\\geq2$\r\nthe equality holds when a=b+c ,b=c+a, c=a+b\r\nIf we add these we have that a+b+c=0 contradiction.\r\nSo $\\sqrt\\frac{a}{b+c}+\\sqrt\\frac{b}{a+c}+\\sqrt\\frac{c}{b+a}>2$",
"Solution_3": "Metru, excuse me for saying this, but this inequality has been posted for many times and solved as well. ;)",
"Solution_4": "Disprove or Prove that:\r\nIf $a,b,c>0$ and $k \\in N$ then:\r\n\r\n$ \\large \\sqrt[2k]{\\frac{a}{b+c}} + \\sqrt[2k]{\\frac{b}{a+c}} + \\sqrt[2k]{\\frac{c}{a+b}} > 2.$",
"Solution_5": "[quote=\"tranvinhphuctk14\"]Using Cauchy:\n$a+(b+c) \\geq 2\\sqrt{a(b+c))}$\n=> $\\sqrt{\\frac{a}{b+c}} \\geq \\frac{2a}{a+b+c}.$\n[/quote]\r\ncan you explain this step in more details ... i dont know this cauchy ineq ;<",
"Solution_6": "[quote=\"Gohan\"][quote=\"tranvinhphuctk14\"]Using Cauchy:\n$a+(b+c) \\geq 2\\sqrt{a(b+c))}$\n=> $\\sqrt{\\frac{a}{b+c}} \\geq \\frac{2a}{a+b+c}.$\n[/quote]\ncan you explain this step in more details ... i dont know this cauchy ineq ;<[/quote]\r\n\r\nyou can treat it as AM-GM by noticing that $x+y\\geq 2\\sqrt{xy}$",
"Solution_7": "in vietnam, amgm is called cauchy, cauchy is called bun???ski",
"Solution_8": "cezar lupu\r\n[url=http://kvant.mirror0.mccme.ru/pdf/2002/06/18.pdf]in another way:\r\n[/url]",
"Solution_9": "[quote=\"siuhochung\"]in vietnam, amgm is called cauchy, cauchy is called bun???ski[/quote]\r\nYes, i'am sorry for mistake."
}
{
"Tag": [
"function",
"inequalities",
"calculus",
"derivative",
"algebra",
"polynomial",
"inequalities unsolved"
],
"Problem": "Hi :D ! $ x,y,z$ are positive real numbers and $ 3x\\plus{}2y\\plus{}z\\equal{}6$ . prove that :\r\n$ x\\plus{}xy\\plus{}xyz\\leq 3$",
"Solution_1": "i have an ugly soloution for this one but it seems too easy !! plz help me ?!?!",
"Solution_2": "Let's even allow zero values, and try Lagrange multipliers. Build function $ L(x,y,z,\\lambda) \\equal{} x \\plus{} xy \\plus{} xyz \\minus{} \\lambda(3x\\plus{}2y\\plus{}z\\minus{}6)$. Then we get the system $ \\frac {\\textrm{d}} {\\textrm{d}x} L(x,y,z,\\lambda) \\equal{} 1\\plus{}y\\plus{}yz\\minus{}3\\lambda \\equal{} 0$, $ \\frac {\\textrm{d}} {\\textrm{d}y} L(x,y,z,\\lambda) \\equal{} x\\plus{}xz\\minus{}2\\lambda \\equal{} 0$, $ \\frac {\\textrm{d}} {\\textrm{d}z} L(x,y,z,\\lambda) \\equal{} xy\\minus{}\\lambda \\equal{} 0$. Plug $ \\lambda \\equal{} xy$ in the second equation to get $ x(1\\plus{}z\\minus{}2y) \\equal{} 0$, so $ z\\equal{}2y\\minus{}1$ ($ x\\equal{}0$ is on the contour, and provides a minimum, not a maximum). Plug again in the first equation to get $ 1\\plus{}y\\plus{}y(2y\\minus{}1)\\minus{}3xy \\equal{} 0$, so $ x \\equal{} \\frac {2y^2\\plus{}1} {3y}$. Back into the restriction, all these yield $ \\frac {2y^2\\plus{}1} {y} \\plus{} 2y \\plus{} (2y\\minus{}1) \\equal{} 6$, hence $ 6y^2 \\minus{}7y \\plus{} 1 \\equal{} 0$, so $ y\\equal{}1$, when $ x\\equal{}y\\equal{}1$, and $ y\\equal{} \\frac {1} {6}$, when $ z\\equal{}\\minus{}\\frac {2} {3}$ is not convenient. One can check that for $ x\\equal{}y\\equal{}z\\equal{}1$ the critical value of $ L \\equal{} 3$ is a maximum (on the contour, we have $ x\\equal{}0$, with $ L\\equal{}0$, or $ y\\equal{}0$, with $ L \\equal{} x \\leq 2 < 3$, or $ z\\equal{}0$, with $ L \\equal{} x\\plus{}xy \\leq \\frac {8} {3} < 3$).",
"Solution_3": "Thanks a lot mavropnevma !! but is there any easier soloution (by AM-GM , Couchy ,...) ?",
"Solution_4": "Ok, here is an elementary solution. Let's denote $ x \\equal{} a \\plus{} 1, y \\equal{} b \\plus{} 1, z \\equal{} c \\plus{} 1$. Then you can rewrite the condition of the problem as follows:\r\n$ 3a \\plus{} 2b \\plus{} c \\equal{} 0$ and you want to prove $ (a \\plus{} 1) \\plus{} (a \\plus{} 1)(b \\plus{} 1) \\plus{} (a \\plus{} 1)(b \\plus{} 1)(c \\plus{} 1)\\le 3$ or (after simplification) it's enough to verify\r\n$ 2ab \\plus{} bc \\plus{} ac\\plus{}abc\\le 0$. But that's almost obvious. Just write $ c \\equal{} \\minus{} (3a \\plus{} 2b)$ and you will get the inequality $ 3a^2 \\plus{} 2b^2\\plus{}ab(3a\\plus{}2b)\\plus{}3ab\\equal{}3a^2(b\\plus{}1)\\plus{}2b^2 (a\\plus{}1)\\plus{}3ab\\ge0$ for $ a,b,c\\ge \\minus{}1$ and $ 3a\\plus{}2b\\le 1$. Indeed, if $ ab\\ge 0$ we are done, and if not, then $ ab(3a\\plus{}2b)\\ge ab$ and we get that $ RHS\\ge 3a^2\\plus{}2b^2\\plus{}4ab\\ge 0$.",
"Solution_5": "More elementary solution (but, yes, ugly).\r\n\r\nKeep $ x$ constant, then $ L \\equal{} x(1\\plus{}y\\plus{}yz)$, with $ z \\equal{} 6\\minus{}3x\\minus{}2y$, so $ L\\equal{} x(1\\plus{}y\\plus{}6y\\minus{}3xy\\minus{}2y^2) \\equal{} \\minus{}x(2y^2 \\plus{}y(3x\\minus{}7)\\minus{}1)$. The maximum for this occurs for $ y \\equal{} \\frac {7\\minus{}3x} {4}$, and amounts to $ \\frac {1} {8} x(9x^2 \\minus{}42x \\plus{} 57)$. Now revert to considering $ x$ variable in the interval $ [0,2]$. The derivative of the above polynomial annuls in $ x\\equal{}1$ and $ x\\equal{}\\frac {57} {27} > 2$, so its maximum is obtained for $ x\\equal{}1$, when we get value $ 3$. Then $ y$ is also equal to $ 1$, and finally $ z\\equal{}1$.",
"Solution_6": "leshik, you wrote \"enough to verify $ 2ab \\plus{} bc \\plus{} ac\\le 0$\", but you forgot the term $ abc$, so one needs $ abc \\plus{} 2ab \\plus{} bc \\plus{} ac\\le 0$, with more work...",
"Solution_7": "Thanks for the corrections.I've edited :blush:"
}
{
"Tag": [],
"Problem": "I was wondering where everybody in AoPS lived. Because there are 195 countries in the world, I'm just going to do continents on the poll(but include a few big countries). Post where you live here and answer the poll.\r\n\r\nNote: I have to delete a few poll options because the settings won't allow it, so if your country doesn't get separately represented and you think it should, that's probably why.",
"Solution_1": "I live in a pineapple under the sea.",
"Solution_2": "Similarly, I live under a rock under the sea, and I am quite stupid when compared to mewto's spongy intellect.",
"Solution_3": "And I live in the middle of mewto and battered in a house that resembles my face. I am very grouchy compared to mewto and battered's two-legged kind....",
"Solution_4": "hee hee. THis isn't what I was thinking what your posts are like, but this is more fun than what I had in mind. of course, no being silly in the polls!",
"Solution_5": "If you would like me to be more formal...\r\nmy female clone lives in Washington... one of the states with the most rain. Sad part is we haven't had rain in... I don't know a month? And our temperatures are some what close to Nevada, :( I am NOT used to this type of weather.",
"Solution_6": "I live in an igloo, and I hunt caribou for dinner. It should be obvious what country that is.",
"Solution_7": "I live in an anchor under the sea, and I'm very cheap. I'm a big miser. I would even sell my soul for money :P . I make mewto work for me :P",
"Solution_8": "So far, 75% of the people here(6 out of 8) live in the U.S.A. I'm surprised there aren't any more from say, Eurpope and Asia.",
"Solution_9": "[quote=\"meewhee009\"]And I live in the middle of mewto and battered in a house that resembles my face. I am very grouchy compared to mewto and battered's two-legged kind....[/quote]\r\n\r\nI live in a dome and i am a scientist with a white suit.",
"Solution_10": "I live close to my computer.",
"Solution_11": "I live in a billion-dollar box....",
"Solution_12": "this is getting randomly SCARY.",
"Solution_13": "I live in your wildest dreams.\r\nI am stuck in a temporal tunnel with my computer.",
"Solution_14": "What the hell, am I the only Canadian here? (Besides dnkywin, of course. But then again, he's also American.)",
"Solution_15": "I am Canadian, but I don't live in Canada.",
"Solution_16": "[quote=\"PowerOfPi\"]I am Canadian, but I don't live in Canada.[/quote]\r\n\r\nSame deal with dnkywin. Why do people claim nationalities where they aren't a resident...? I mean, I'm Chinese both by origin and ethnicity, but I consider myself Canadian, because I've lived here for 12 years...",
"Solution_17": "Looking at the polls, the AoPS population is no where near as diverse as I thought(at least in the FF). I'm incredibly surprised I don't see any \"China's\" or \"India's\" considering the huge populations of the countries.",
"Solution_18": "Maybe it is because people in those countries study harder and don't have the time for this stuff.\r\n\r\n@sudoku dragon: I consider myself Canadian because I was born in Canada. I moved a year ago, and not very far either. I still visit every few months.",
"Solution_19": "[quote=\"meewhee009\"]And[b] I live in the middle of mewto[/b] and battered in a house that resembles my face. I am very grouchy compared to mewto and battered's two-legged kind....[/quote]\r\n\r\n*loud coughing*\r\n\r\nWell, maybe we have so many people living in the US because, hmm, well, this is a forum made for Americans and written in English.",
"Solution_20": "And those asians do study all day.\r\nThey don't have time for all this stuff.\r\n\r\nNo wonder the Chinese come first every year. :huh: \r\nDon't know about Indians, they rarely post outside their national forum on Aops :rotfl:\r\n\r\n\r\n\r\n[b][color=red]Edited to avoid any misunderstanding[/color][/b]\r\n :wink: :)",
"Solution_21": "[quote=\"Xaenir\"]And those asian nerds do study all day.[/quote]\r\nConsidering AoPS is a worldwide forum, and this forum is frequented by people internationally, that's not a very smart thing to say. :wink:",
"Solution_22": "Xaenir, that's the second time you insult somebody. Do it once more, this goes to mods.\r\n\r\nI am the only one voted to live in Europe...",
"Solution_23": "as you may have seen , I have long edited my post, so your post qualifies as spam :D\r\nBesides i didn't mean to insult anybody, just implied that asians work harder and are intellectually superior, you got it the wrong way. :wink:",
"Solution_24": "No, wasn't it ''asian nerds''? Maybach's post tells it all. And that ''girls are lazy, girls are dumb''? \r\n\r\nI say again, you can say anything without insults. If you want to continue the discussion, PM me. I'm not against you, just giving you a little piece of advice :)",
"Solution_25": "[b]nerd[/b] refers to a person who passionately pursues intellectual activities, esoteric knowledge, or other obscure interests rather than engaging in more social or popular activities.\r\n\r\nThat isn't an insult :!:",
"Solution_26": "um.........look at this(Source: Merriam Webster Dictionary&Thesaurus):\r\nnerd \\'n\u0259rd\\ n [perh. fr. nerd, a creature in the children's book If I Ran the Zoo (1950) by Dr. Seuss (Theodor Geisel)] (1951) : an unstylish, unattractive, or socially inept person ; esp: one slavishly devoted to intellectual or academic pursuits \u2039computer ~s\u203a\r\n\u2014 nerd\u00b7i\u00b7ness \\'n\u0259r-d\u0113-n\u0259s\\ n\r\n\u2014 nerd\u00b7ish \\'n\u0259r-dish\\ adj\r\n\u2014 nerdy \\-d\u0113\\ adj\r\n\r\nYour definition doesn't tell it all. I'm not saying \"nerd\" is a terrible word to say, but it usually has a very negative connotation, which is why you certainly shouldn't call someone that, let alone steryotype billions of people considred \"Asians\". Bottom line: This is your second strike. Don't do it again.",
"Solution_27": "Well I didn't mean it as an insult.I apologize if I offended someone, is it alright now?\r\nAnd I think I would be more careful in the future",
"Solution_28": "Japan!\r\n\r\n :lol:",
"Solution_29": "FANTASY LAND!!!! :D",
"Solution_30": "@Isabella,\r\nHa. Grammar gone wrong. I didn't think anyone would find that so I left it be. You proved me wrong :lol:",
"Solution_31": "to me, nerds are a type of candy\r\ni like nerds\r\nthey do taste quite sweet\r\ni live in a Gullible"
}
{
"Tag": [],
"Problem": "I want a recent national competition.\r\nI don't have anything to trade.\r\nWell, my teacher has 2005 (maybe?) state competition. \r\nAlso A few of the recent work books.\r\nPlease(If there is any chance of avoiding the actual trade, and I could just get the competition, It would be highly appreciated. I don't know if just giving away a nat competition is legal, though.) :help:",
"Solution_1": "I have 2008 or 2006 nats, if you could trade anything it would be most helpful."
}
{
"Tag": [
"algebra",
"polynomial",
"factorial",
"function",
"domain",
"Ring Theory",
"superior algebra"
],
"Problem": "Let $ f_1 ,f_2 \\in k[x_1,...,x_n]$ ($ k$ is a field) be polynomials with no common irreducible factors. Show That\r\n$ Syz(f_1 ,f_2)\\equal{}(f_2 ,\\minus{}f_1)k[x_1,...,x_n] \\subset k[x_1,...,x_n]^2$.\r\nWhat happens if $ f_1$ and $ f_2$ have common irreducible factors?",
"Solution_1": "This sounds very much like homework, so I'd just give a few hints, but there is no much to hint here - if you can't solve the problem, you have either not understood the meaning of \"syzygy\", or you don't know some basic facts about factorial domains (also called unique factorization domains, UFDs, ...). If any of these is not familiar to you, read them up first!\r\n\r\nDo you agree that $ \\left(g_1,g_2,...,g_n\\right)\\in\\text{Syz}\\left(f_1,f_2,...,f_n\\right)$ if and only if $ g_1f_1 \\plus{} g_2f_2 \\plus{} ... \\plus{} g_nf_n \\equal{} 0$ ? Now think about what this means for $ n \\equal{} 2$. That is, $ g_1f_1 \\plus{} g_2f_2 \\equal{} 0$. That is, $ g_1f_1 \\equal{} \\minus{} g_2f_2$. If $ f_1$, $ f_2$, $ g_1$, $ g_2$ were integers, this would yield some divisibility... now, $ f_1$, $ f_2$, $ g_1$, $ g_2$ are not integers, but polynomials. What can you say about divisibility among polynomials?\r\n\r\nIf $ f_1$ and $ f_2$ have common irreducible factors - worst case: $ f_1 \\equal{} f_2$, there are many more syzygies. Do you see a very obvious one?\r\n\r\n darij"
}
{
"Tag": [
"inequalities",
"induction",
"number theory unsolved",
"number theory"
],
"Problem": "Show that:\r\n$F_{2n}\\leq \\frac{(2n+2)^n}{n!}$\r\nDenote :$F_{2n}$ is $2n$-th Fibolnaxi number\r\nIt's itshelf Mathlinks Contest Round 4 ,after some conclusions ,of course ! \r\nBut I have no final solution.Anybody has one??",
"Solution_1": "It is trivial. Let Z(n) is right expression. We have Z(1)=4, Z(n)/Z(n-1)=2*(1+1/n)^n>4 if n>=2. Therefore Z(n)>=4^n.\r\nIf F(0)=a>=0,F(1)=b>=0,F(n+1)=F(n)+F(n-1) we have F(2n+2)=3F(2n)-F(2n-2)<=3F(2n+2). Therefore F(2n)<=F(2)*3^(n-1)<4^n if F(2)=a+b<4.",
"Solution_2": "[quote=\"Rust\"]It is trivial. Let Z(n) is right expression. We have Z(1)=4, Z(n)/Z(n-1)=2*(1+1/n)^n>4 if n>=2. Therefore Z(n)>=4^n.\nIf F(0)=a>=0,F(1)=b>=0,F(n+1)=F(n)+F(n-1) we have F(2n+2)=3F(2n)-F(2n-2)<=3F(2n+2). Therefore F(2n)<=F(2)*3^(n-1)<4^n if F(2)=a+b<4.[/quote]\r\nPlease, use LaTeX...",
"Solution_3": "It's easy to know $F_m\\geq F_{m-1}(m=1,2,\\ldots)$.So $F_{m+1}=F_m+F_{m-1}\\leq2F_m$.\r\nSo $F_{2n}=F_{2n-1}+F_{2n-2}\\leq2F_{2n-2}$.\r\nFirst,$F_2<4$,so the inequality is correct for $n=1$.\r\nBy induction,suppose $F_{2n}\\leq\\frac{(2n+2)^n}{n!}$\r\nSo $F_{2n+2}\\leq3F_{2n}\\leq3\\frac{(2n+2)^n}{n!}$.\r\nWe only need to show $3\\frac{(2n+2)^n}{n!}\\leq\\frac{(2n+4)^{n+1}}{(n+1)!}$.\r\nIt can be writed like this:$3(n+1)\\leq(2n+2)(1+\\frac{1}{n+1})^{n+1}$.\r\n$\\frac{3}{2}\\leq(1+\\frac{1}{n+1})^{n+1}$.\r\nAnd we can easily know $(1+\\frac{1}{n+1})^{n+1}\\geq1+(n+1)\\frac{1}{n+1}=2>\\frac{3}{2}$.\r\nSo the original inequality is correct. ;)"
}
{
"Tag": [],
"Problem": "There are 20 people in my club. 8 of them are left-handed. 15 of them like jazz music. 2 of them are right-handed and dislike jazz music. How many club members are left-handed and like jazz music?",
"Solution_1": "Draw a Venn diagram: one category is for left-handed, the other for likes jazz. We are told 2 are right-handed and don't like jazz, so they are outside, meaning there are 18 inside. Let $ x\\equal{}\\text{number of left\\minus{}handed people}$, $ y\\equal{}\\text{left\\minus{}handed and like jazz}$ and $ z\\equal{}\\text{like jazz}$. Then $ x\\plus{}y\\plus{}z\\equal{}18$, $ x\\plus{}y\\equal{}8$, and $ y\\plus{}z\\equal{}15$. Then $ y\\equal{}15\\minus{}z\\Rightarrow x\\plus{}15\\minus{}z\\equal{}8\\Rightarrow x\\equal{}z\\minus{}7$. This means that $ z\\minus{}7\\plus{}15\\minus{}z\\plus{}z\\equal{}18\\Rightarrow z\\equal{}10\\Rightarrow x\\equal{}3,y\\equal{}5$. So there are 5 people who like jazz and are left-handed.",
"Solution_2": "Or for a faster way:\r\n\r\nWe want the lefties and those who like jazz, so we can disregard those 2 righties who don't like jazz. That leaves 18. Since 15 like jazz and 8 are leftie, our answer is simply $ (15 \\plus{} 8) \\minus{} 18 \\equal{} \\boxed{5}$.\r\n\r\nEDIT: Err...I knew that... :P Thanks, Coptie.",
"Solution_3": "Uh izzy...$ (15\\plus{}8)\\minus{}18\\equal{}5\\neq3$"
}
{
"Tag": [],
"Problem": "The fraction $ \\frac{x}{5}$ does not change its value when 3 is added to both its numerator and its denominator. What is the value of $ x$?",
"Solution_1": "hello \r\nu can simply say it should be $ 5$ \r\n$ \\frac{x\\plus{}3}{8 }\\equal{} \\frac{x}{5}$ \r\n$ x \\equal{}5$\r\nthank u",
"Solution_2": "[hide=\"Clearer solution\"]\nWe have the equation \\[ \\frac{x}{5}\\equal{}\\frac{x\\plus{}3}{5\\plus{}3}.\\] Cross multiplying gives \\[ 8x\\equal{}5x\\plus{}15\\Rightarrow3x\\equal{}15\\Rightarrow x\\equal{}\\boxed{5}.\\][/hide]"
}
{
"Tag": [
"geometry"
],
"Problem": "The circumference of a circle is $ 24\\pi$ cm. What is the number of square centimeters in the area of the largest square that can fit inside the circle?",
"Solution_1": "[asy]draw((1,0)--(0,1)--(-1,0)--(0,-1)--cycle);\ndraw((1,0)..(0,1)..(-1,0)..(0,-1)..cycle);\n\ndraw((1,0)--(-1,0));\ndraw((0,1)--(0,-1));\n\nlabel(\"$r$\",(0.5,0),S);\nlabel(\"$r$\",(0,0.5),W);\nlabel(\"$\\sqrt{2}r$\",(0.5,0.5),N);[/asy]\r\n\r\n$ 2\\pi r\\equal{}24\\pi \\iff r \\equal{} 12$.\r\nThus, $ (12\\sqrt{2})^2 \\equal{} \\boxed{288}$."
}
{
"Tag": [
"AMC"
],
"Problem": "I need help on this problem. \r\n\r\nLet $a_1 = 1$ and for $n>1$, $a_n = 2a_{n-1}$ if $a_{n-1}$ is odd and equals $a_{n-1}+1$ if it's otherwise. Find $a_{2004}$.\r\n\r\nThe problem I'm facing is writing $a_n$ without using $a_{n-1}$. Like I see that:\r\n\r\n$a_4 = 2(2a_1+1)$\r\n$a_6 = 2[2(2a_1+1)+1]$\r\n\r\nand so forth but I'm having trouble writing $a_n$. Can anyone help me on how to do this? \r\n\r\nI posted in AMC forum because this type of problem appears quite often in AMC.",
"Solution_1": "[hide=\"Hint\"]\\[a_2=2a_1=2\\]\n\\[a_4=2(a_2+1)=4+2\\]\n\\[a_6=2(a_4+1)=8+4+2\\]\n\\[a_8=2(a_6+1)=16+8+4+2\\]\nSee it now? ;) [/hide]",
"Solution_2": "Oh..\r\n\r\nI was writing $a_1 = 1, a_2 = 2, a_3 = 3, a_4 = 6, a_5 = 7, a_6 = 14, a_7 = 15, a_8 = 30$ and so forth. I didn't see it that way.\r\n\r\nSo $a_{2004}$ would be:\r\n\r\n[hide=\"Then..\"]\n\n$a_{2004} = 2^1+2^2+2^3+2^4+ \\cdots 2^{1002}$? [/hide]",
"Solution_3": "Yup.\r\n\r\n[hide=\"That expression collapses...\"]\\[\\sum_{n=1}^{1002}2^n=\\sum_{n=0}^{1002}2^n-1=2^{1003}-2\\][/hide]"
}
{
"Tag": [
"trigonometry"
],
"Problem": "Inscribed in a circle is a quadrilateral having side lengths 25,39,52 and 60 taken consecutively. What is the diameter of this circle?",
"Solution_1": "[hide]just by luck, if you draw the diagonal, they're right triangles (and Pythagorean tripleets at that!) so the hypotenuse (and the diameter) is [b]65.[/b][/hide]",
"Solution_2": "Or...\r\nThe magic of....\r\n[hide=\"Super Awesome Theorem of pwnage\"]\nPtolemy's Theorem!!!\n[/hide]\r\n\r\nOf course, it takes some more manipulation, because the diagonals of the quad aren't actually the diameters...",
"Solution_3": "How would you use Ptolemy's Theorem on this problem?",
"Solution_4": "[hide]Bhramagupta's formula says that the answer is $\\sqrt{63 \\cdot 49 \\cdot 36 \\cdot 28}$\n$84 \\cdot 21$\n$1764$[/hide]",
"Solution_5": "Random trivia:\r\nWe can generalize Brhamagupta's Formula (or however you spell his name) to any quadrilateral, cyclic or non-cyclic. This formula is sometimes called Bretschneider's formula:\r\n$\\sqrt{(s-a)(s-b)(s-c)(s-d)-abcd\\cos^{2}{\\frac{A+C}{2}}}$, where $s$ is the semiperimeter, $a, b, c, d$ are the sides of the quadrilateral, and $A, C$ are opposite angles. In fact, any set of opposite angles will do."
}
{
"Tag": [
"algebra",
"polynomial",
"search",
"algebra unsolved"
],
"Problem": "Hi all!!!!!!!!! :wink: \r\nHere is a problem from VIETNAM 2003 olimpiad.\r\n$ B2.$ Define $ p(x) = 4x^{3}-2x^{2}-15x+9, q(x) = 12x^{3}+6x^{2}-7x+1$. Show that each polynomial has just three distinct real roots. Let $ A$ be the largest root of $ p(x)$ and $ B$ the largest root of $ q(x)$. Show that $ A^{2}+3B^{2}= 4$.",
"Solution_1": "Please search it on this forum. It is posted before!"
}
{
"Tag": [
"function",
"inequalities",
"integration",
"triangle inequality",
"real analysis",
"real analysis unsolved"
],
"Problem": "let $ f_n$ be a sequence of non-negative measurable functions in $ L^p(\\Bbb R)$ for some $ 1 0,$ we can let $ x \\equal{} \\frac ab \\minus{} 1$ and rearrange this to \r\n\r\n$ \\left(\\frac ab\\right)^p \\minus{} 1\\ge\\left(\\frac1b \\minus{} 1\\right)^p$ or $ a^p \\minus{} b^p\\ge(a \\minus{} b)^p$\r\n\r\nSo we have that $ |f_n \\minus{} f|^p\\le |f_n^p \\minus{} f^p|$ and hence $ \\|f_n \\minus{} f\\|_p^p\\le \\|f_n^p \\minus{} f^p\\|_1.$\r\n\r\nSo this gives us $ f_n^p\\to f^p$ in $ L^1$ implies $ f_n\\to f$ in $ L^p.$",
"Solution_2": "Here is a somewhat weaker (I think, I very well could be wrong) version of the converse:\r\n\r\nSince $ f_{n}\\rightarrow f$ in $ L^{p}$, then $ \\left\\|f_{n}\\minus{}f\\right\\|_{p}\\rightarrow 0$ as $ n\\rightarrow\\infty$. By the triangle inequality, $ \\left\\|f_{n}\\right\\|_{p}\\leq\\left\\|f_{n}\\minus{}f\\right\\|_{p}\\plus{}\\left\\|f\\right\\|_{p}\\Rightarrow\\left\\|f_{n}\\right\\|_{p}\\minus{}\\left\\|f\\right\\|_{p}\\leq\\left\\|f_{n}\\minus{}f\\right\\|_{p}$ and $ \\left\\|f\\right\\|_{p}\\leq\\left\\|f_{n}\\minus{}f\\right\\|_{p}\\plus{}\\left\\|f_{n}\\right\\|_{p}\\Rightarrow\\left\\|f\\right\\|_{p}\\minus{}\\left\\|f_{n}\\right\\|_{p}\\leq\\left\\|f_{n}\\minus{}f\\right\\|_{p}$. Hence $ (\\left|f_{n}\\right|^{p})$ converges to $ \\left|f\\right|^{p}$ in $ L^{1}$.",
"Solution_3": "[quote=\"JRav\"]Hence $ (\\left|f_{n}\\right|^{p})$ converges to $ \\left|f\\right|^{p}$ in $ L^{1}$.[/quote]\r\n\r\nare you sure that this is what you've shown?",
"Solution_4": "By Minkowski's we have:\r\n$ \\left\\parallel{}f^{p}_{n}\\right\\parallel{}_{1} \\equal{} \\parallel{}f^{p}_{n} \\minus{} f^{p} \\plus{} f^{p}\\parallel{}_{1} \\leq \\parallel{}f^{p}_{n} \\minus{} f^{p}\\parallel{}_{1} \\plus{} \\parallel{}f^{p}\\parallel{}_{1}$\r\n\r\nThen we get:\r\n$ \\left| \\parallel{}f^{p}_{n}\\parallel{}_{1} \\minus{} \\parallel{}f^{p}\\parallel{}_{1} \\right| \\leq \\parallel{}f^{p}_{n} \\minus{} f^{p}\\parallel{}_{1} \\equal{} \\int \\left|\\left(f^{p}_{n} \\minus{} f^{p}\\right)^{\\frac{1}{p}}\\right|^{p} \\leq \\left\\parallel{}f_{n} \\minus{} f \\right\\parallel{}^{p}_{p} \\plus{} \\parallel{}f_{n}\\parallel{}^{p}_{p}$",
"Solution_5": "for other direction : let $ M \\equal{} max\\{\\parallel{}f\\parallel{}_{p}, \\parallel{}f_{n}\\parallel{}_{p}: n\\in\\mathbb{N}\\}$, then $ \\int\\parallel{}f|^{p} \\minus{} |f_{n}|^{p}|d\\nu\\leq 2pM^{p \\minus{} 1}\\parallel{}f \\minus{} f_{n}\\parallel{}_{p}$\r\n\r\nProf. Kent proposed very nice solution but I think we can apply Fatou's lemma : \r\n\r\nwe have $ |f \\minus{} f_{n}|^{p}\\leq 2^{p}(|f|^{p} \\plus{} |f|_{n}^{p})$, so consider : $ 2^{p}(|f|^{p} \\plus{} |f|_{n}^{p}) \\minus{} |f \\minus{} f_{n}|^{p}\\to 2^{p \\plus{} 1}|f|^{p}$\r\n\r\nand apply the Fatou's lemma we have $ \\overline{lim}\\int|f \\minus{} f_{n}|^{p}\\leq 0$ .",
"Solution_6": "1234567a: You have to pass to a subsequence first, though. So you get that every subsequence of $ f_n$ has a further subsequence that converges to $ f$ in $ L^p$, which is good enough.",
"Solution_7": "[quote=\"blahblahblah\"]are you sure that this is what you've shown?[/quote]\r\nI showed that $ \\left|\\left\\|f\\right\\|_{p}\\minus{}\\left\\|f_{n}\\right\\|_{p}\\right|\\leq\\left\\|f_{n}\\minus{}f\\right\\|_{p}$, which means that $ \\left|\\int\\left|f_{n}\\right|^{p}\\minus{}\\int\\left|f\\right|^{p}\\right|\\rightarrow 0$, which I thought meant that $ \\left|f_{n}\\right|^{p}$ converged to $ \\left|f\\right|^{p}$ in the $ L^{1}$ norm but apparently I am mistaken. Could you elaborate?",
"Solution_8": "[quote=\"ficsur\"]let $ f_n$ be a sequence of non-negative measurable functions in $ L^p(\\Bbb R)$ for some $ 1 < p < \\infty$. show that $ f_n \\to f$ in $ L^p$ iff $ f_n^p \\to f^p$ in $ L^1(\\Bbb R)$.[/quote]\r\n\r\nI will prove that \" if $ f_n\\to f$ in $ L^p$, then $ f_n^p\\to f^p$ in $ L^1$\".\r\nIndeed, bacause the function $ \\varphi(x)\\equal{}x^p$ is convex with $ p>1$. By the convexity we have \r\n$ |x^p\\minus{}y^p|\\leq p(\\max\\{x,y\\})^{p\\minus{}1}|x\\minus{}y|$ with $ x,y\\geq 0$. \r\nFrom inequality $ \\max\\{x,y\\}\\leq x\\plus{}y$, we have $ |x^p\\minus{}y^p|\\leq p(x\\plus{}y)^{p\\minus{}1}|x\\minus{}y|$ (by $ p\\minus{}1>0$). Apply this inequality for $ f_n, f$, we obtain \r\n$ |f_n^p\\minus{}f^p|\\leq p(f_n\\plus{}f)^{p\\minus{}1}|f_n\\minus{}f|$\r\nHence, by using the Holder's inequality, we have \r\n$ \\parallel{}f_n^p\\minus{}f^p\\parallel{}_1\\leq p\\parallel{}(f_n\\plus{}f)^{p\\minus{}1}\\parallel{}_q\\parallel{}f_n\\minus{}f\\parallel{}_p(*)$\r\nhere, $ 1/p \\plus{} 1/q\\equal{}1$ therefor $ q(p\\minus{}1)\\equal{}p$.\r\n Thus, $ \\parallel{}(f_n\\plus{}f)^{p\\minus{}1}\\parallel{}_q\\equal{}\\parallel{}f_n\\plus{}f\\parallel{}_p^{p/q}\\leq 2^{p/q}(\\parallel{}f_n\\parallel{}_p\\plus{}\\parallel{}f\\parallel{}_p)^{p/q}$, this term is bounded, by the hypothesis $ f_n\\to f$ in $ L_p$\r\nThe proof is completed by the inequality (*).",
"Solution_9": "[quote=\"JRav\"][quote=\"blahblahblah\"]are you sure that this is what you've shown?[/quote]\nI showed that $ \\left|\\left\\|f\\right\\|_{p} \\minus{} \\left\\|f_{n}\\right\\|_{p}\\right|\\leq\\left\\|f_{n} \\minus{} f\\right\\|_{p}$, which means that $ \\left|\\int\\left|f_{n}\\right|^{p} \\minus{} \\int\\left|f\\right|^{p}\\right|\\rightarrow 0$, which I thought meant that $ \\left|f_{n}\\right|^{p}$ converged to $ \\left|f\\right|^{p}$ in the $ L^{1}$ norm but apparently I am mistaken. Could you elaborate?[/quote]\r\n\r\nwell, first of all, you've left out the $ 1/p \\minus{}$th roots. Second of all, showing that $ |f_n|^p\\to |f|^p$ in $ L^1$ is the same as showing\r\n\\[ \\int \\parallel{}f_n|^p \\minus{} |f|^p|dx\\to 0,\\]\r\nwhich is harder, since $ \\int |u_n|dx \\to 0$ is stronger than $ |\\int u_ndx|\\to 0$.",
"Solution_10": "Indeed it does. Stupid mistake. :)"
}
{
"Tag": [
"trigonometry",
"trig identities",
"Law of Cosines",
"geometry proposed",
"geometry"
],
"Problem": "Let \u03b1+\u03b2= Fixed Value\r\na , b is Fixed Value respectively\r\n\r\nFind R , \u03b1 , \u03b2",
"Solution_1": "Let $O$ be the center of the circle, and the segments $b=BC,a=CA$.\r\nAlso let the given angle $x=\\angle AOB$\r\n\r\nAt first, we can notice that we can easy construct this figure geometricaly.\r\n\r\nThe angle $\\phi=\\angle ACB$ is such that $\\phi+\\frac{x}{2}=\\pi\\Rightarrow \\phi = \\pi -\\frac{x}{2}$\r\n\r\nfrom the triangle $ABC$ we can get that\r\n\r\n$AB = 2Rsin \\phi\\Rightarrow AB^2 = 4R^2 sin^2 \\phi$\r\n\r\nLaw of cosines in $\\triangle ABC: AB^2 = a^2+b^2-2ab\\cdot cos \\phi$\r\n\r\nSo we have \r\n\r\n$4R^2 sin^2 \\phi = a^2+b^2-2ab\\cdot cos \\phi \\Rightarrow$\r\n\r\n$R^2 = \\frac{a^2+b^2-2ab\\cdot cos \\phi}{4sin^2 \\phi}$\r\n\r\n$R = \\frac{\\sqrt{a^2+b^2-2ab\\cdot cos \\phi}}{2sin \\phi}$\r\n\r\nOnce we have found $R$, we can find the angles on the center using the formulas\r\n\r\n$\\boxed{ sin \\frac{\\widehat{\\alpha}}{2} = \\frac{a}{2R} } \\boxed{sin \\frac{\\widehat{\\beta}}{2} = \\frac{b}{2R}}$\r\n\r\nWe can use sine and cosine of $\\ \\frac{x}{2}$ instead of $\\phi$:\r\n\r\n$\\phi = \\pi -\\frac{x}{2} \\Rightarrow$\r\n\r\n$sin \\phi = sin \\frac{x}{2}$\r\n\r\n$cos \\phi = -cos \\frac{x}{2}$"
}
{
"Tag": [
"linear algebra",
"matrix",
"linear algebra unsolved"
],
"Problem": "Let an invertible square complex matrix A have Jordan form X. Find the Jordan form of the inverse of A.",
"Solution_1": "This comes down to this question: can you find the inverse of one Jordan block? \r\n\r\nLet's do this in a slightly more general way. Suppose $ A\\equal{}\\lambda I\\plus{}N$ where $ N$ is nilpotent and $ \\lambda\\ne 0.$ What is $ A^{\\minus{}1}?$\r\n\r\nWrite $ A\\equal{}\\lambda(I\\plus{}\\lambda^{\\minus{}1}N).$ I claim that\r\n\\[ A^{\\minus{}1}\\equal{}\\lambda^{\\minus{}1}\\sum_{k\\equal{}0}^{\\infty}(\\minus{}1)^k\\lambda^{\\minus{}k}N^k\\equal{}\\sum_{k\\equal{}0}^{\\infty}(\\minus{}1)^k\\lambda^{\\minus{}k\\minus{}1}N^k.\\]\r\nNote that this apparent infinite series is in fact a finite sum, since $ N$ is nilpotent.\r\n\r\nAn example of this theorem:\r\n\\[ \\begin{bmatrix}\\lambda&1&0&0\\\\0&\\lambda&1&0\\\\0&0&\\lambda&1\\\\0&0&0&\\lambda\\end{bmatrix}^{\\minus{}1}\\equal{}\\begin{bmatrix}\\lambda^{\\minus{}1}&\\minus{}\\lambda^{\\minus{}2}&\\lambda^{\\minus{}3}&\\minus{}\\lambda^{\\minus{}4}\\\\0&\\lambda^{\\minus{}1}&\\minus{}\\lambda^{\\minus{}2}&\\lambda^{\\minus{}3}\\\\0&0&\\lambda^{\\minus{}1}&\\minus{}\\lambda^{\\minus{}2}\\\\0&0&0&\\lambda^{\\minus{}1}\\end{bmatrix}\\]\r\nOne you figure out how to do that to any Jordan block, you should be able to figure out how to handle several blocks put together.",
"Solution_2": "Hi Kent. Your answer is not complete.\r\n1) $ \\lambda,\\mu\\not\\equal{}0,\\lambda\\not\\equal{}\\mu$ imply $ 1/\\lambda\\not\\equal{}1/\\mu$.\r\n 2) Let $ A\\equal{}\\lambda{I}\\plus{}J$ where $ J$ is a nilpotent Jordan block ($ J^n\\equal{}0,J^{n\\minus{}1}\\not\\equal{}0$). $ A^{\\minus{}1}\\equal{}(1/\\lambda)I\\plus{}N$ where $ N^n\\equal{}0,N^{n\\minus{}1}\\not\\equal{}0$. Then $ A^{\\minus{}1}$ is similar to $ (1/\\lambda)I\\plus{}J$, its Jordan form.\r\nConclusion: Let $ A\\in\\mathcal{M}_n(\\mathbb{C})$. If in the Jordan form of $ A$ , we change, for all $ i$, $ \\lambda_i$ with $ 1/\\lambda_i$ then we obtain the Jordan form of $ A^{\\minus{}1}$."
}
{
"Tag": [
"geometry",
"ratio"
],
"Problem": "Sorry this is a pretty standard question but when trying to come up with the formula for the area ratio asked in the following:\r\n\r\nSuppose $ ABC$ is a triangle and let $ D$, $ E$, $ F$ are on $ BC$, $ CA$, and $ AB$ respectively s.t.\r\n$ \\frac {BD}{DC} \\equal{} \\frac {1}{x}$\r\n\r\n$ \\frac {CE}{EA} \\equal{} \\frac {y}{1}$\r\n\r\n$ \\frac {AF}{FB} \\equal{} \\frac {1}{z}$\r\n\r\nIf $ AD$, $ BE$, $ CF$ are drawn then you get a a smaller triangle $ PQR$ where $ P$ is intersection of $ AD$ and $ BE$, $ Q$ is intersection of $ CF$ and $ BE$, and $ R$ is intersection of $ AD$ and $ CF$.\r\n\r\nCompute $ \\frac {[PQR]}{[ABC]}$\r\n\r\nI know this uses mass-point and is not pretty,as it looks at only two cevians at a time, etc (i've done it before, but can't seem to remember the sequence of steps i used) could somebody help?",
"Solution_1": "Check out [url=http://www.cut-the-knot.org/triangle/RouthTheorem.shtml]Routh's Theorem[/url]."
}
{
"Tag": [
"inequalities"
],
"Problem": "Given $\\sum^{n}_{i=1} x_i=n$ for $x_i\\in \\mathbb{R}$ and\r\n\r\n$\\sum^{n}_{i=1}x_i^4=\\sum^{n}_{i=1}x_i^3$\r\n\r\nSolve the system of equation for $x_i$ .\r\n\r\n :)",
"Solution_1": "Two of the power means turn out to be equal, so we must have equality everywhere, and hence $x_1 = x_2 = \\dots = x_n = 1$.\r\n\r\nYou could also notice that $\\sum_{i = 1}^n\\left(x_i - 1\\right) = 0$ and $\\sum_{i = 1}^nx_i^3\\left(x_i - 1\\right) = 0$.\r\n\r\nSubtract the first equation from the second to get $\\sum_{i = 1}^n\\left(x_i - 1\\right)^2\\left(x_i^2 + x_i + 1\\right) = 0$. Hence $x_i = 1$ for all $i$.",
"Solution_2": "Arne , I dont undestand what you mean by\r\n\r\n[quote]\nTwo of the power means turn out to be equal, so we must have equality everywhere, \n[/quote]\r\n\r\nCan you explain more detail ?\r\n :)",
"Solution_3": "Let's see... We have \\[\\sqrt[4]{\\frac{\\sum_{i = 1}^nx_i^4}{n}} \\geq \\sqrt[3]{\\frac{\\sum_{i = 1}^nx_i^3}{n}}\\] by Power means, which gives \\[{{n \\cdot (\\sum_{i = 1}^nx_i^4})^3 \\geq (\\sum_{i = 1}^nx_i^3})^4\\] or \\[n \\geq \\sum_{i = 1}^nx_i^3.\\] So, \\[1 = \\frac{\\sum_{i = 1}^nx_i}{n} \\geq \\sqrt[3]{\\frac{\\sum_{i = 1}^nx_i^3}{n}}\\] which is the power mean inequality \"the wrong way around\".\r\n\r\nSo, equality must hold, and all variables must be equal.",
"Solution_4": "Thats great ! Arne , thank you . :)"
}
{
"Tag": [
"inequalities unsolved",
"inequalities"
],
"Problem": "Prove that:$\\frac{1}{2+a}+\\frac{1}{2+b}+\\frac{1}{2+c}\\leq 1$\r\nwhere $a;b;c$ are non-negative and satisfying $abc=1$",
"Solution_1": "[quote=\"chien than\"]Prove that:$\\frac{1}{2+a}+\\frac{1}{2+b}+\\frac{1}{2+c}\\leq 1$\nwhere $a;b;c$ are non-negative and satisfying $abc=1$[/quote]\r\n\r\nThis is a particular case (n = 3) of the following fact:\r\n\r\nIf $x_{1}$, $x_{2}$, ..., $x_{n}$ are n nonnegative reals that satisfy $x_{1}x_{2}...x_{n}=1$, then $\\sum_{i=1}^{n}\\frac{1}{n-1+x_{i}}\\leq 1$.\r\n\r\nSee\r\nhttp://www.mathlinks.ro/Forum/viewtopic.php?t=4951\r\nhttp://www.mathlinks.ro/Forum/viewtopic.php?t=27900\r\nhttp://www.mathlinks.ro/Forum/viewtopic.php?t=164\r\nhttp://www.mathlinks.ro/Forum/viewtopic.php?t=64349\r\nhttp://www.mathlinks.ro/Forum/viewtopic.php?t=116016\r\n\r\n(Someone should really moderate this forum...)\r\n\r\n darij",
"Solution_2": "It was also discussed here http://www.mathlinks.ro/Forum/viewtopic.php?t=131882"
}
{
"Tag": [
"logarithms"
],
"Problem": "Source: MAML 2006 States!!\r\n\r\nIf $\\log_4{36}=a$ and $\\log_4{9}=b$, find $\\log_8{\\frac{216}{81}}$ in terms of $a$ and $b$.\r\n\r\nOh yea, you should have 2.5 minutes to do this question.",
"Solution_1": "[hide=\"Took me around 60 - 80 seconds.\"]Break up the original $\\log_8 \\frac{216}{81} = 3 \\log_8 6 - 4 \\log_8 3$. Now change the a and b expressions to base 8, so $a= \\log_4 36 = \\frac{\\log_8 36}{\\log_8 4} = \\frac{\\log_8 36}{\\frac{2}{3}} = \\frac{3}{2} 2 \\log_8 6 = 3 \\log_8 6$ then doing the same for b yields $b=3 \\log_8 3$. Plugging back in appropriate a and b values into the simplified original, $3 \\log_8 6 - 4 \\log_8 3 = a- \\frac43 b$.[/hide]",
"Solution_2": "[quote=\"weatherbottle\"]Source: MAML 2006 States!!\n\nIf $\\log_4{36}=a$ and $\\log_4{9}=b$, find $\\log_8{\\frac{216}{81}}$ in terms of $a$ and $b$.\n\nOh yea, you should have 2.5 minutes to do this question.[/quote]\r\n[hide]$\\log_8{\\frac{216}{81}}=\\log_2{\\frac{6}{3^{\\frac{4}{3}}}=\\log_2{6}-\\log_2{3^{\\frac{4}{3}}}=\\log_2{6}-\\frac{4}{3}\\log_2{3}}$.\n$a=\\log_2{6}$, $b=\\log_2{3}$, so $\\log_2{6}-\\frac{4}{3}\\log_2{3}=\\boxed{a-\\frac{4}{3}b}$.[/hide]",
"Solution_3": "[quote=\"weatherbottle\"]Source: MAML 2006 States!!\n\nIf $\\log_4{36}=a$ and $\\log_4{9}=b$, find $\\log_8{\\frac{216}{81}}$ in terms of $a$ and $b$.\n\nOh yea, you should have 2.5 minutes to do this question.[/quote]\r\n[hide=\"logs are fun\"]\nLets start with $\\log_4{36}$\n$\\log_4 {36}=\\log_4 {4\\cdot9}=\\log_4 {4}+\\log_4 {9}$\n$a=1+\\log_4{9}$\n$\\log_4 {9}=a-1$\n$a-1=b$\nSo now we have to break down $\\log_8{\\frac{216}{81}}$.\nImmediately we see that that equals $\\log_8{216}-\\log_8 {81}$\nUsing the exponent rules, we get $3\\log_8 {2}-4\\log_8 {2}$\nAfter converting to base 4, we get $a-4b/3$\n[/hide]"
}
{
"Tag": [
"trigonometry",
"algebra unsolved",
"algebra"
],
"Problem": "[b](1) Is there is any Geometrical method to find the value of cos(18 degree), cos(36 degree) ,cos(54 degree).[/b]",
"Solution_1": "hello, maybe this will help you\r\nhttp://2000clicks.com/mathhelp/GeometryTrigSpecialAngles.htm\r\nSonnhard.",
"Solution_2": "Sir have found no. idea how to find the value of sin(18 degree) or tan(18degree).\r\nso sir plz explain me.",
"Solution_3": "we have $ \\sin 36^o \\equal{} \\cos 54^o$\r\nuse Triple Angle Formula on $ RHS$ and Double Angle Formula on $ LHS$\r\nand it will solve the problem"
}
{
"Tag": [
"geometry",
"circumcircle",
"incenter",
"angle bisector"
],
"Problem": "Right triangle $ ABC,$ with $ \\angle A\\equal{}90^{\\circ},$ is inscribed in circle $ \\Gamma.$ Point $ E$ lies on the interior of arc $ {BC}$ (not containing $ A$) with $ EA>EC.$ Point $ F$ lies on ray $ EC$ with $ \\angle EAC \\equal{} \\angle CAF.$ Segment $ BF$ meets $ \\Gamma$ again at $ D$ (other than $ B$). Let $ O$ denote the circumcenter of triangle $ DEF.$ Prove that $ A,C,O$ are collinear.",
"Solution_1": "[hide=\"click text\"]Let I be the incenter of AEF. you should prove EC.$ Point $ F$ lies on ray $ EC$ with $ \\angle EAC \\equal{} \\angle CAF.$ Segment $ BF$ meets $ \\Gamma$ again at $ D$ (other than $ B$). Let $ O$ denote the circumcenter of triangle $ DEF.$ Prove that $ A,C,O$ are collinear.[/quote]\r\nHere is my solution\r\n\r\nSince $ \\angle OED$ $ \\equal{}90^{\\circ}\\minus{}$ $ \\frac {1}{2}$ $ \\angle EOD$ $ \\equal{}90^{\\circ}\\minus{}$ $ \\angle DFE$ $ \\equal{}\\angle DCF$ $ \\equal{}\\angle DBE$, implying $ OE$ is a tangent from $ O$ to $ (\\Gamma)$. The same argument holds for $ OD$, which yields that $ (O)$ and $ (\\Gamma)$ are orthogonal. But in the other hand, due to the fact that $ (AB,AC,AE,AS)\\equal{}\\minus{}1$, then if $ W$ is the intersection of $ AB$ with $ EF$ then $ (EFCW)\\equal{}\\minus{}1$, which implies $ (BE,BF,BC,BW)\\equal{}\\minus{}1$ or we can say $ (BE,BD,BC,BA)\\equal{}\\minus{}1$ $ \\Longrightarrow$ $ ADCE$ is a [i]harmonic quadrilateral[/i]. As the result, $ AC$ with then tangents at $ D$ and $ E$ will concur, hence, $ AC,$ $ OD,$ $ OE$ will concur at $ O$, which implies $ A,$ $ C,$ $ O$ are collinear. $ \\square$",
"Solution_8": "Notice $$-1=(E,F;C,\\overline{AB}\\cap\\overline{EF})\\stackrel{B}=(E,D;C,A)$$ by the Right Angles and Bisectors Lemma. Since $$\\angle CEO=90-\\tfrac{1}{2}\\angle FOE=90-\\angle EDB=90-\\angle EAB=\\angle CAE,$$ we know $\\overline{OE}$ is tangent to $\\Gamma$ at $E.$ Similarly, $O=\\overline{DD}\\cap\\overline{EE}$ so if $A'=\\overline{OC}\\cap\\Gamma,$ we see $(A',C;DE)=-1.$ Hence, $A=A'.$ $\\square$"
}
{
"Tag": [
"trigonometry",
"real analysis",
"real analysis unsolved"
],
"Problem": "Here is, I think, a nice product.\r\n\r\n\r\n$\\prod_{n=0}^{\\infty}\\frac{(2n+1)^{4}}{(2n+1)^{4}-(2/\\pi)^{4}}=\\frac{2e\\sec(1)}{e^{2}+1}$.",
"Solution_1": "[quote=\"didilica\"]Here is, I think, a nice product.\n\n\n$\\prod_{n=0}^{\\infty}\\frac{(2n+1)^{4}}{(2n+1)^{4}-(2/\\pi)^{4}}=\\frac{2e\\sec(1)}{e^{2}+1}$.[/quote]\r\n\r\nLet consider a generalization: for $z\\in\\mathbb{C},$ put $f(z)=\\prod_{n=0}^{\\infty}\\frac{(2n+1)^{4}}{(2n+1)^{4}-(2z/\\pi)^{4}}$. Then,\r\n\r\n\\[f(z)=\\prod_{n=0}^{\\infty}\\left[\\frac{(2n+1)^{2}}{(2n+1)^{2}-(2z/\\pi)^{2}}\\cdot\\frac{(2n+1)^{2}}{(2n+1)^{2}+(2z/\\pi)^{2}}\\right] =\\left[\\prod_{n=0}^{\\infty}\\left(1-\\frac{4z^{2}}{(2n+1)^{2}\\pi^{2}}\\right)\\cdot\\prod_{k=0}^{\\infty}\\left(1+\\frac{4z^{2}}{(2k+1)^{2}\\pi^{2}}\\right)\\right]^{-1}\\]\r\n\r\nbut since we know that for all $z\\in\\mathbb{C},$\r\n\r\n\\[\\cos z =\\prod_{n=0}^{\\infty}\\left(1-\\frac{4z^{2}}{(2n+1)^{2}\\pi^{2}}\\right)\\ ,\\qquad\\qquad \\mbox{ and }\\qquad\\qquad\\mbox{cosh}z =\\prod_{k=0}^{\\infty}\\left(1+\\frac{4z^{2}}{(2k+1)^{2}\\pi^{2}}\\right)\\ ,\\] \r\n\r\nwe may write\r\n\r\n\\[f(z)=\\left(\\cos z \\cdot\\mbox{cosh}z\\right)^{-1}= \\sec z \\cdot\\mbox{sech}z =\\frac{2\\sec z}{e^{z}+e^{-z}}= \\frac{2e^{z}\\sec z}{e^{z}+1}\\]\r\n\r\nClearly the given product is then $f(1) = \\frac{2e\\sec(1)}{e^{2}+1}.$"
}
{
"Tag": [
"inequalities",
"logarithms",
"calculus",
"calculus computations"
],
"Problem": "Let $a,\\ b$ be real numbers such that $\\log_{a}8=\\log_{\\sqrt{b}}2.$\r\n\r\n(1) Find the values of $\\log_{ab}a+\\log_{b}{\\frac{1}{\\sqrt{a}}}+(\\log_{a}b)^{2}.$\r\n\r\n(2) Solve the following inequality with respect to $x.$ \\[a^{3x}+a^{x+2}+a^{x}+a^{x-2} 3$, we have $ t > 1$. Then, as $ 2^{p \\minus{} 1} \\equiv 1 mod p$, $ 2^t|p \\minus{} 1 \\Rightarrow 4|p \\minus{} 1$, q.e.d."
}
{
"Tag": [
"greatest common divisor",
"number theory",
"least common multiple"
],
"Problem": "the least common multiple of two nmbers is $2^{2}*3^{4}*5*7$. The greatest common divisor of the same two numbers is $2*3*5$. One of the numbers is 210. What's the other?[color=cyan]\n\n[size=50][hide=\"answer\"]3240[/hide][/size][/color]",
"Solution_1": "[hide] The product of the GCF and LCM is the product of the two numbers. When you multiply the GCF and LCM, you get $2^{3}*3^{5}*5^{2}*7$. Call the other number n. This means that 210n=8*243*25*7, so n=1620. [/hide]",
"Solution_2": "[quote=\"espark52\"]the least common multiple of two nmbers is $2^{2}*3^{4}*5*7$. The greatest common divisor of the same two numbers is $2*3*5$. One of the numbers is 210. What's the other?[color=cyan]\n\n[size=50][hide=\"answer\"]3240[/hide][/size][/color][/quote]\n[hide]\nLCM*GCF=Product\nLCM*GCF/1number=theother number\n$210=2*3*5*7$\nLCM*GCF$=2^{3}*3^{5}*5^{2}*7$\nLCM*GCF/210$=2^{2}*3^{4}*5$\n$=1620$\n[/hide]",
"Solution_3": "[quote=\"espark52\"]the least common multiple of two nmbers is $2^{2}*3^{4}*5*7$. The greatest common divisor of the same two numbers is $2*3*5$. One of the numbers is 210. What's the other?[color=cyan]\n\n[size=50][hide=\"answer\"]3240[/hide][/size][/color][/quote]\r\n\r\nuhh... espark, i think the answer is 1620, not 3240 as you have posted.",
"Solution_4": "[quote=\"davidlizeng\"][quote=\"espark52\"]the least common multiple of two nmbers is $2^{2}*3^{4}*5*7$. The greatest common divisor of the same two numbers is $2*3*5$. One of the numbers is 210. What's the other?[color=cyan]\n\n[size=50][hide=\"answer\"]3240[/hide][/size][/color][/quote]\n\nuhh... espark, i think the answer is 1620, not 3240 as you have posted.[/quote]\r\n\r\noh woops, ur right :blush: the reason i put 3240 is cause i mis-typed the problem it's supposed to be $2^{3}*3^{4}*5*7$"
}
{
"Tag": [
"geometry",
"circumcircle",
"inradius",
"incenter",
"geometric transformation",
"reflection",
"homothety"
],
"Problem": "[color=darkblue]Given are two parallel lines $d_{1}$ , $d_{2}$ and the circle $w$ which touches the line $d_{1}$ in the point $A$ and $d_{2}\\cap w=\\emptyset\\ .$\nFor a mobile point $M\\in d_{2}$ denote the intersections $X$ , $Y$ between the line $d_{1}$ and the tangents from the point $M$ to the circle $w\\ .$ \nProve that exist two constants $k$ , $l$ so that the product $AX\\cdot AY\\in \\{k,l\\}\\ .$\n\nSee http://www.mathlinks.ro/Forum/viewtopic.php?t=115606 (seek and will find ... the hint !).[/color]",
"Solution_1": "[hide][url]http://www.mathlinks.ro/Forum/viewtopic.php?t=44854[/url][/hide]",
"Solution_2": "The problem was also posted here: [url]http://www.mathlinks.ro/Forum/viewtopic.php?t=85336[/url].\r\n\r\nAssume the circle center [color=blue][b]is[/b][/color] between the lines $d_{1},\\ d_{2}.$ Then $w$ is the incircle of $\\triangle MXY,$ so we denote its center I. Let (O) be the circumcircle of this triangle, $(I_{m})$ its M-excircle, T the tangency point of $(I_{m})$ with $d_{1}\\equiv XY,$ and let $r,\\ r_{m},\\ h_{m}$ be its inradius, M-exradius, and M-altitude. Since $\\frac{r_{m}}{r}= \\frac{h_{m}}{h_{m}-2r}$ and since $r,\\ h_{m}$ are constant, $r_{m}$ is also constant. The bisector $MI \\equiv MI_{m}$ of $\\angle XMY$ meets (O) again at the midpoint P of the arc XY of (O) opposite to M. The incenter I and the M-excenter $I_{m}$ are both on a circle (P) centered at P and passing through the triangle vertices X, Y. Since this circle is centered on the perpendicular bisector p of XY and $I_{m}\\in (P),$ the reflection $I_{m}'$ of $I_{m}$ in p is also on (P). In addition, the reflection of T in p is A, the incircle tangency point with XY. Thus $I_{m}'A \\perp XY,\\ I_{m}'A = r_{m},$ and the points $I,\\ I_{m}' \\in P$ are both fixed. Hence the power of A to (P) is $AX \\cdot AY = AI \\cdot AI_{m}' = rr_{m}= \\frac{h_{m}r^{2}}{h_{m}-2r}$ is constant. If we substitute the distance $d = h_{m}-r$ of the line $d_{2}$ from the center I of the circle $w \\equiv (I),$ we get\r\n\r\n$\\frac{AX \\cdot AY}{r^{2}}= \\frac{d+r}{d-r}$\r\n\r\nSimilarly, assuming the circle center [color=blue][b]is not[/b][/color] between the lines $d_{1},\\ d_{2},$ $w$ is the M-excircle of $\\triangle MXY$ and can get \r\n\r\n$\\frac{AX \\cdot AY}{r^{2}}= \\frac{d-r}{d+r}$\r\n\r\nThis proof is easily modified for the case $w \\cap d_{2}\\neq \\emptyset$ and M is still outside of $w.$ WLOG assuming AX > AY, $w$ is the X-excircle of $\\triangle MXY.$ For the proof, we use the Y-excircle, the radius of which is fixed. For example, if $r_{x}> h_{m}$ are fixed, since M is the internal homothety center of $(I_{x}),\\ (I_{y})$, $\\frac{r_{x}-h_{m}}{r_{x}}= \\frac{h_{m}-r_{y}}{r_{y}}$ and $r_{y}= \\frac{r_{x}h_{m}}{2r_{x}-h_{m}}$ is also fixed. The X, Y-excenters are both on a circle (P) centered at the midpoint P of the arc XY of (O) containing M. Similarly as before, (P) passes through the reflection of the Y-excenter in the perpendicular bisector of XY, which is fixed, etc.",
"Solution_3": "[quote=\"Virgil Nicula\"][color=darkred]Given are two parallel lines $d_{1}$ , $d_{2}$ and the circle $w$ which touches the line $d_{1}$ in the point $T$ and $d_{2}\\cap w=\\emptyset\\ .$\nFor a mobile point $A\\in d_{2}$ denote the intersections $B$ , $C$ between the line $d_{1}$ and the tangents from the point $A$ to the circle $w\\ .$ \nProve that exist two constants $k$ , $l$ so that the product $TB\\cdot TC\\in \\{k,l\\}\\ .$[/color][/quote]\r\n[color=darkblue][b]Proof.[/b] I\"ll use the standard notations for the triangle $ABC\\ .$ Prove easily that the relations $(p-b)(p-c)=\\frac{r^{2}h_{a}}{h_{a}-2r}=\\frac{r_{a}^{2}h_{a}}{h_{a}+2r_{a}}\\ .$\n[hide=\"Here is prove.\"]\n$\\{\\begin{array}{cc}|\\begin{array}{c}h_{a}=\\frac{2pr}{a}\\\\\\\\ (p-a)(p-b)(p-c)=pr^{2}\\end{array}& (p-b)(p-c)(h_{a}-2r)=(p-b)(p-c)(\\frac{2pr}{a}-2r)=(p-a)(p-b)(p-c)\\cdot \\frac{2r}{a}=r^{2}h_{a}\\\\\\\\ |\\begin{array}{c}h_{a}=\\frac{2(p-a)r_{a}}{a}\\\\\\\\ p(p-b)(p-c)=(p-a)r^{2}_{a}\\end{array}& (p-b)(p-c)(h_{a}+2r_{a})=(p-b)(p-c)[\\frac{2(p-a)r_{a}}{a}+2r_{a}]=p(p-b)(p-c)\\cdot\\frac{2r_{a}}{a}=r_{a}^{2}h_{a}\\end{array}$[/hide] \n $TB\\cdot TC=(p-b)(p-c)\\Longrightarrow$ $\\{\\begin{array}{cccccc}\\mathrm{Case\\ 1.\\ : \\ }& w=C(I,r) & \\Longrightarrow & (p-b)(p-c)=\\frac{r^{2}h_{a}}{h_{a}-2r}& \\Longrightarrow &{ k=\\frac{r^{2}h_{a}}{h_{a}-2r}\\ }\\\\\\\\ \\mathrm{Case\\ 2.\\ : \\ }& w=C(I_{a},r_{a}) & \\Longrightarrow & (p-b)(p-c)=\\frac{r_{a}^{2}h_{a}}{h_{a}+2r_{a}}& \\Longrightarrow & l=\\frac{r_{a}^{2}h_{a}}{h_{a}+2r_{a}}\\ . \\end{array}$[/color]",
"Solution_4": "that's a nice problem I see follow other way\r\nGiven $(I)$ touch line $d$ at $T$, find locus $A$ such that $(I)$ is incenter of $\\triangle ABC$ with $B,C$ move on $d$ and $TB.TC=const$.\r\n[hide=\"Solution\"]\na line $// BC$:D :D\n[/hide]"
}
{
"Tag": [
"MATHCOUNTS",
"geometry",
"algebra",
"polynomial"
],
"Problem": "I'll post COPSD 3rd B soon to encourage more middle schoolers to attend my competition.\r\n\r\nAnyone interested? (Even you qualified on COPSD 3rd A, you're welcome to just take it again for fun --> no prize for double qualifying though :P )",
"Solution_1": "When do you get when you qualify for the A one? (I'll do this one too :D )",
"Solution_2": "What's COPSD 3rd A? :huh:",
"Solution_3": "I'm interested, but could you please explain what COPSD 3rd A is? :? And what COSPD 3rd B is?",
"Solution_4": "COPSD 3rd A was this thread 236factorial and Drunken_Math did:\r\n\r\nhttp://www.artofproblemsolving.com/Forum/viewtopic.php?t=50030\r\n\r\nCOPSD 3rd B is just like COPSD 3rd A but with different set of problems that are in same level. The reason that I have A and B is because on the last time, some of people couldn't compete because of AP tests.\r\n\r\nAnyway, if you guys know AMC, this is same. Same level problems, just in different date.\r\n\r\n(Although you can do both :P )\r\n\r\nTo qualify, you must send at least 7 SOLUTIONS (answers + explanation) to the problems that are correct. In another word, if you get 7 right, you're qualified. If you get more, good to you. If you get less, :( .\r\n\r\nAnyone who qualifies from COPSD will go to COP Round 2, along with people from COP Round 1 (who are mostly high schoolers ---> This means you can go against the high school students!!).\r\n\r\nIf you guys have any more questions, please post!!",
"Solution_5": "Oh, so now I get to participate in COP 5th now? :D And can I do the COPSD B round at the same time?",
"Solution_6": "I guess I will do it, it sounds fun. Are the problems like AMC 10 level? When are you going to post the questions?",
"Solution_7": "[quote=\"236factorial\"]Oh, so now I get to participate in COP 5th now? :D And can I do the COPSD B round at the same time?[/quote] \n\nYes. You're ALREADY qualified to COP Round 2 which is eh.. about level of later AMC-12 and possibly easy AIME.\n\n[quote=\"b-flat\"]Are the problems like AMC 10 level?[/quote]\n\nNo. They're level of mathcounts varying from chapter to national level.\n\n[quote=\"b-flat]When are you going to post the questions?[/quote]\r\n\r\nSoon. But I can't give you an exact date since I'm very busy this year (about 3 times more than last year). I'll announce it when it's ready. \r\n\r\nWe need more people to sign up!! Come on, guys! This is a VERY good practice for mathcounts!",
"Solution_8": "I'm signing up! :)",
"Solution_9": "sign me up",
"Solution_10": "me 2",
"Solution_11": "Alright I think I'll have time to participate in this one. I'm sorry i didn't do 3rd A... I didn't have the time.",
"Solution_12": "Here are the problems, folks! Enjoy!\r\n\r\nNote: These are VERY good practices for local, state, national mathcounts. Everybody who visit MC forum should give a shot. Also, some problems may require brute forcing and use of calculator so don't be afraid on this fact.\r\n\r\n[b]Problems[/b]\r\n\r\n1. Find the maximum or minimum height defined in equation $y = x^2-12x+2$.\r\n\r\n2. Find $a+b+c$.\r\n\r\n\\[2a+b+c = 10\\\\\r\na+2b+c = 12\\\\\r\na+b+2c = 14\\]\r\n\r\n3. In a 10-questioned test, the student can answer either true or false but must answer all of the 10 questions (no omitting!). How many ways can the student do this?\r\n\r\n4. Find $a$ and $b$ in:\r\n\r\n$\\sqrt{4 + \\sqrt{4 + \\sqrt{4 + \\sqrt{4 + \\cdots}}}} = \\frac{a+ \\sqrt b}{2}$\r\n\r\n5. The equilateral triangle $ABC$ has side length of 10. Connecting triangle $ABC$'s midpoints, we form triangle $A'B'C'$ with side length 5. In same manner, we form $A''B''C'', A'''B'''C'''$, and so on. Find the area of all equilateral triangles formed this way (including triangle $ABC$). In another word, what is $[ABC]+[A'B'C']+[A''B''C'']+[A'''B'''C''']+[A''''B''''+C''''] + \\cdots $? ($[z]$ denotes the area)\r\n\r\n6. Find the EXACT value of\r\n\r\n\\[\\sqrt{4 + 2 \\sqrt 3} + \\sqrt{4 - 2 \\sqrt 3}\\]\r\n\r\n7. The polynomial below has four roots. One of them is $4i$. Find the three other roots.\r\n\r\n\\[x^4-5x^3+22x^2-80x+96\\]\r\n\r\nNote on #7. This type of problem won't appear on Mathcounts history because $i$ indicates $\\sqrt{-1}$ which is a complex number. But here is a big hint on this problem. If a polynomial contains the $a+bi$ as one of root, it also contains $a-bi$ as another root.\r\n\r\n8. For non-real number $r,s$, we have:\r\n\r\n\\[r+s = 5\\\\\r\nrs = 10\\]\r\n\r\nFind $\\frac{1}{r^2} + \\frac{1}{s^2}$.\r\n\r\n9. How many number less than 500 satisfy:\r\n\r\n(i) has perfect square as one of its factors\r\n(ii) divisible by 7\r\n(iii) sum of its digit is 18\r\n\r\n10. In how many ways can you pick two numbers in a set $\\{1,2,3,4,5 \\}$ such that their sum is even?",
"Solution_13": "All SOLUTIONS must be turned to ME by PM today (11:59:59).\r\n\r\nThank you and good luck!",
"Solution_14": "I'm too late :(",
"Solution_15": "[quote=\"nonie\"]I'm too late :([/quote]\r\n\r\nHere's a very good news for you.\r\n\r\nI decided to extend the deadline since I only received solution from only one contestant (can't believe only one in COP and only one in COPSD :( ).\r\n\r\nSo, the new deadline is 10/3/05. Enjoy problem solving till then!\r\n\r\nRemember: I need the solutions to at least 7 of the problems."
}
{
"Tag": [
"function",
"calculus",
"calculus computations"
],
"Problem": "li m x----8 sin{ x-10}/{10-x}\r\n\r\n{} denotes fractional part function\r\npl also explain little bit about fractional part function.i dont know any thing about it\r\n\r\nQ2 lim X---0 Sec-(x/sin x)=l and limx---0 sec-(x/tan x)=m then which limit exist and which will not pl explain",
"Solution_1": "The [url=http://mathworld.wolfram.com/FractionalPart.html]fractional part of a function[/url]."
}
{
"Tag": [
"number theory proposed",
"number theory"
],
"Problem": "This Problem is marvelous ,i shall not say the source ,cause then you will not think on it ,but here it is [hide=\"Dont look\"]IMO 1991,Problem 6[/hide]\n\n[color=blue]Problem[/color]\nThe real number $a>1$ is given .\nA sequence like \\[ \\{x_i \\}_{i=0}^{ \\infty} \\]\nis called $LHTB(\\ little \\ hard \\ to \\ build )$ if for every two not-negative integers like $i,j,that, i \\neq j$ \\[ |x_i-x_j| \\geq \\frac{1}{|i-j|^a} \\] \n\n1.Prove that there exists a[b] bounded[/b] $LHTB$ sequence.\n\n2.[hide=\"The Original problem,\"]IMO1991,6[/hide] Built a[b] bounded[/b] $LHTB$ sequence.",
"Solution_1": "$x_i=i$.",
"Solution_2": "@lomos_lupin: Isn't this sequence supposed to be bounded in the original statement ?",
"Solution_3": "If you want bounded sequence, then it can be build:\r\nLet $n=a_0+a_1*2+\\dots+a_k*2^k, a_i\\in \\{0,1\\}$.\r\n$x_n=2(a_0+a_1*2^{-1}+\\dots+a_k*2^{-k}).$",
"Solution_4": ":blush: Yes its bounded ,All of its fun was for being bounded.My bad\r\n\r\nA prove for existance of such a sequence:\r\n\r\n[color=blue]Solution[/color]Assume that we have built $x_0,...,x_{n-1}$ and we want to built the $x_n$ . Let $I$ be a bound for the sequence .for satisfying the condition of the problem ,we should have :\r\n\r\n \\[ |x_n-x_i| \\geq \\frac{1}{|n-i|^a} ,0 \\leq i \\leq n-1 \\]\r\n\r\nSo for existance of $x_n$ we should have :\r\n\r\n\\[ \\sum_{i=0}^{n-1} \\frac{1}{|n-i|^a} < I \\]\r\n\r\nbut we know that :\r\n\\[ \\sum_{i=1}^{n} \\frac{1}{i^a} < M= \\sum_{i=1}^{\\infty} \\frac{1}{i^a} < \\infty \\]\r\n(Actually $M$ is a rational product of $\\pi^{a})$\r\nBy choosing $I=2M$ ,such a $x_n$ satisfying the condition of the problem exists.",
"Solution_5": "To Rust : NO that sequance doesnt work ,Your sequance is not related to $a$,but it should be related.\r\nA book that i have present this sequence :\r\n\r\nLet the $x_n$ in the Rust sequence be $y_n$ then define your sequence as following \r\n\\[ x_n=\\frac{2^a-2}{2^a-1}y_n \\]"
}
{
"Tag": [
"calculus",
"integration",
"trigonometry",
"complex analysis"
],
"Problem": "How would I use Residue Theorem and Laurent expansions to do the following problem?\r\n\r\n[img]http://i3.photobucket.com/albums/y57/lilshantypanty/prob1-1.jpg[/img]\r\n\r\nThanks!",
"Solution_1": "For the first, how about first just expanding $ e^{1/z}$ in a Laurent expansion:\r\n\r\n$ e^{1/z}\\equal{}1\\plus{}1/z\\plus{}1/2(1/z)^2\\plus{}1/6(1/z)^3\\plus{}...$\r\n\r\nAlright, now expand $ Exp[1\\plus{}1/z\\plus{}1/2(1/z)^2\\plus{}1/6(1/z)^3\\plus{}...]$\r\n\r\nCan you show that I get for the residue:\r\n\r\n$ \\mathop\\text{Res}\\limits_{z\\equal{}0}\\left\\{e^{e^{1/z}}\\right\\}\\equal{}\r\n\\sum_{n\\equal{}1}^{\\infty}\\frac{n}{n!}$",
"Solution_2": "[quote=\"sailorsenshi\"]How would I use Residue Theorem and Laurent expansions to do the following problem?\n\n[img]http://i3.photobucket.com/albums/y57/lilshantypanty/prob1-1.jpg[/img]\n\nThanks![/quote]\r\nI don't known. \r\nIs it right?\r\n${ e^{ix}=\\cos{x}+i\\sin{x}}$\r\n${ e^{x}=\\cos{\\frac{x}{i}}+i\\sin{\\frac{x}{i}}}$\r\nUse it , we will ..."
}
{
"Tag": [
"linear algebra",
"matrix"
],
"Problem": "Hi,\r\nI need help in solving the equation.\r\n\r\n5x-7y=31\r\n-4x+2y=-14\r\n\r\nPlease solve by substitution and show steps. Thanks",
"Solution_1": "[quote=\"Trisha\"]Hi,\nI need help in solving the equation.\n\n5x-7y=31\n-4x+2y=-14\n\nPlease solve by substitution and show steps. Thanks[/quote]\r\n\r\n[hide=\"hint\"]\n\nsolve for either variable in either equation.\n\nthen substitute that equation into the corresponding variable in the other equation\n\nsolve for the other variable\n\nplug the integer into an equation and solve for the original variable. \n[/hide]",
"Solution_2": "[hide]\n1.$5x-7y=31$\n2.$-4x+2y=-14\\implies-2x+y=-7\\implies y=2x-7$\n\nSubstitute into 1\n\n$\\Rightarrow 5x-7(2x-7)=31\\implies x=2$\n\nsubstitute $x=2$ into 2\n\n$\\Rightarrow y=2(2)-7=-3$\n\n$\\boxed{x=2,y=-3}$\n[/hide]",
"Solution_3": "Thanks SM4RT this is exactly what I wanted to see. :lol:",
"Solution_4": "Another option is to convert into the coefficient matrix into reduced row echelon form...\r\n\r\nOr use cramer's rule, but both of these solutions are overkill on a two equation system.\r\n\r\nThe substitution method in general is the fastest way of solving small (2x2 or 3x3) systems.\r\n\r\nAnything larger, and matrix transforms are your friend."
}
{
"Tag": [
"real analysis",
"topology",
"real analysis unsolved"
],
"Problem": "This question is from royden's real analysis from the chapter on lebesgue measure. Should be quite easy to solve. But i cant :( \r\n\r\nLet $E$ be a set. Denote $m(E)$ to be the outer measure of $E$. Assume further that $m(E)$ is finite. Prove that the following are equivalent.\r\n\r\n(i) For all $\\epsilon>0$, there exists an open set $O\\supset E$ s.t. $m(O\\backslash E)<\\epsilon$.\r\n(ii) For all $\\epsilon>0$, there exists a finite union of open intervals $U$ s.t. $m(U\\Delta E)<\\epsilon$.\r\n\r\nNB: $U\\Delta E= (U\\backslash E) \\cup (E\\backslash U)$.",
"Solution_1": "You missed a condition in (i). In royden's book, we also have $O\\supseteq E$.\r\n\r\n(i) => (ii)\r\n\r\nLet $\\epsilon>0$, by (i) there exists $O=\\bigcup I_{n}$ such that $m^\\ast(O\\setminus E)<\\epsilon/2$, where $I_{n}$ are open intervals.\r\nThen\r\n$\\sum|I_{n}| = m(O) \\leq m^\\ast(O\\setminus E)+m^\\ast(E) < \\frac \\epsilon2+m^\\ast E$ is finite.\r\n\r\nHence $\\sum |I_{n}|$ converges, so there exist $N$ such that $\\sum_{n=N}^\\infty |I_{n}| < \\frac\\epsilon 2$. Let $U = \\sum_{i=1}^{N}I_{n}$ then $m^\\ast(U\\Delta E) \\leq m^\\ast(U\\setminus E)+m^\\ast(E\\setminus U)<\\frac \\epsilon 2+\\frac\\epsilon 2 = \\epsilon$. (Note that $E\\setminus U$ is a subset of $\\sum_{i=N}^\\infty I_{n}$)\r\n\r\n(ii) => (i)\r\n\r\nLet $\\epsilon>0$, by (ii) we have a finite union of open intervals $U$ such that $m^\\ast(U\\Delta E)<\\frac\\epsilon 3$. By definite of outer measure, we have an open covering of $U\\Delta E$ thus an open set $G\\supseteq G\\Delta E$ such that $m(G)0$, by (i) there exists $O=\\bigcup I_{n}$ such that $m^\\ast(O\\setminus E)<\\epsilon/2$, where $I_{n}$ are open intervals.\n[/quote]\r\n\r\nthink i'm forgetting some facts. But is it always true that any open set $O$ can be written as a countable disjoint union of open intervals? (i think you made use of the fact that it's disjoint when you assumed $\\sum|I_{n}|=m^{*}(O)$.)",
"Solution_3": "Consider an open subset of the reals. Its connected components must be open, as given any element of a particular connected component, there exists a neighborhood of that element contained in the set, and the union of this neighborhood with the connected component is readily seen to still be connected. Thus, as open, connected subsets of the reals are simply open intervals, any open subset is a union of open intervals; they are disjoint as connected components are disjoint and there are a countable number as we can choose a rational in each of the open intervals.",
"Solution_4": "[quote]think i'm forgetting some facts. But is it always true that any open set $O$ can be written as a countable disjoint union of open intervals? (i think you made use of the fact that it's disjoint when you assumed $\\sum|I_{n}|=m^{*}(O)$.)[/quote]\r\nYes, i think you can find the standard proof in Royden's book. But it is for sets on the real line only.",
"Solution_5": "Ok. QED. Thanks for the help!"
}
{
"Tag": [
"inequalities"
],
"Problem": "a,b,c>0\r\n ab+bc+ac = k\r\n\r\n ????\r\nprove a+b+c>=k",
"Solution_1": "a=1, b=1, c=3\r\nAnd I have proved it wrong.\r\n\r\nFor your information, \r\n$(a+b+c)^2\\ge 3(ab+bc+ca)$\r\nThis can be proved using AM-GM",
"Solution_2": "k=3\r\n :ninja:",
"Solution_3": "See my above post, I've solved it.\r\n\r\n[hide=\"Solution\"]$(a+b+c)^2\\ge 3(ab+bc+ca)$\n$\\Rightarrow (a+b+c)^2 \\ge 9$\nThus proved.[/hide]\n\nYou may like to prove the inequality yourself. \n[hide=\"Big Hint\"]\nUse AM-GM[/hide]",
"Solution_4": "[hide=\"Is this it?\"]Expaninding gives us \\[ a^2+b^2+c^2+2ab+2ac+2bc \\geq 3ab+3ac+3bc \\] So we need to proof that \\[ a^2+b^2+c^2 \\geq ab+ac+bc \\] Apply AM-GM on $\\{a^2,b^2\\}$, $\\{a^2,c^2\\}$ and $\\{b^2,c^2\\}$, getting $\\frac{a^2+b^2}{2} \\geq ab$, $\\frac{a^2+c^2}{2} \\geq ac$ and $\\frac{b^2+c^2}{2} \\geq bc$. Adding those three: \\[ \\frac{a^2+b^2+a^2+c^2+b^2+c^2}{2} = a^2+b^2+c^2 \\geq ab+ac+bc\\ \\ \\square \\] [/hide]",
"Solution_5": "Yes, it is \"this\" :)",
"Solution_6": "A simpler way to prove $a^2 + b^2 + c^2 \\geq ab + bc + ac$ is to re-arrange and factor it as\r\n\r\n$1/2 (a - b)^2 + 1/2 (b-c)^2 + 1/2(c -a)^2 \\geq 0$, which is always true."
}
{
"Tag": [],
"Problem": "i)x,y mh midenikoi pragmatikoi ari8moi kai (1-x^2)(1-y^2)=1 Na upologistei to A=(1+|x|)*(1+|y|)!\r\n\r\nii) x^2+y^2+z^2=x+y+z+1 na apodeixth oti dn exei lush stous rhtous\r\n\r\niii)P(x^2)=x^2(x^2+1)P(x) , P(2)=12 na bre8ei to poluwnhmo (P(x) me pragmatikous sidelestes..)",
"Solution_1": "\u0393\u03b9\u03b1 \u03c4\u03bf \u03c4\u03c1\u03af\u03c4\u03bf \u03b5\u03af\u03bd\u03b1\u03b9 $ p(x)\\equal{}x^4\\minus{}x^2.$ \u03b1\u03bd \u03b4\u03b5\u03bd \u03ad\u03c7\u03c9 \u03ba\u03ac\u03bd\u03b5\u03b9 \u03ba\u03ac\u03c0\u03bf\u03c5 \u03bb\u03ac\u03b8\u03bf\u03c2 \u03c3\u03c4\u03b9\u03c2 \u03c0\u03c1\u03ac\u03be\u03b5\u03b9\u03c2....",
"Solution_2": "swsta tis ekanes..! to prwto einai to pio wraio try it",
"Solution_3": "\u03c4\u03bf \u03b4\u03b5\u03c5\u03c4\u03b5\u03c1\u03bf \u03bc\u03b5 \u03bc\u03b9\u03b1 \u03c0\u03c1\u03bf\u03c7\u03b5\u03b9\u03c1\u03b7 \u03bc\u03b1\u03c4\u03b9\u03b1 \u03b2\u03b3\u03b1\u03b9\u03bd\u03b5\u03b9 \u03bc\u03b5 \u03b1\u03c0\u03b5\u03b9\u03c1\u03b7 \u03ba\u03b1\u03b8\u03bf\u03b4\u03bf.....\u0395\u03c7\u03c9 \u03ba\u03b1\u03c0\u03bf\u03b9\u03b5\u03c2 \u03b1\u03bc\u03c6\u03b9\u03b2\u03bf\u03bb\u03b9\u03b5\u03c2 \u03b3\u03b9\u03b1 \u03c4\u03bf \u03c0\u03c1\u03c9\u03c4\u03bf \u03bf\u03bc\u03c9\u03c2...\u0391\u03bd \u03b8\u03b5\u03c3\u03bf\u03c5\u03bc\u03b5 \u03ba\u03b1\u03c0\u03bf\u03b9\u03b5\u03c2 \u03c4\u03b9\u03bc\u03b5\u03c2 \u03c3\u03c4\u03bf \u03c7 \u03ba\u03b1\u03b9 \u03bb\u03c5\u03c3\u03bf\u03c5\u03bc\u03b5 \u03c9\u03c2 \u03c0\u03c1\u03bf\u03c3 y \u03b4\u03b5\u03bd \u03b8\u03b1 \u03b5\u03c7\u03b5\u03b9 \u03c4\u03b7\u03bd \u03b9\u03b4\u03b9\u03b1 \u03c4\u03b9\u03bc\u03b7 \u03c4\u03bf \u0391.... :( :maybe:\u03a4\u03b9 \u03b5\u03bd\u03bd\u03bf\u03b5\u03b9\u03c2 \u03bd\u03b1 \u03c5\u03c0\u03bf\u03bb\u03bf\u03b3\u03b9\u03c3\u03c4\u03b5\u03b9????\u0391\u03bd \u03bc\u03c0\u03bf\u03c1\u03b5\u03b9\u03c2 \u03b5\u03be\u03b7\u03b3\u03b7\u03c3\u03b5 \u03c4\u03bf \u03bc\u03bf\u03c5 \u03bb\u03b9\u03b3\u03bf \u03c3\u03b5 \u03c0\u03b1\u03c1\u03b1\u03ba\u03b1\u03bb\u03c9....... :)",
"Solution_4": "\u03a4\u03bf 3\u03bf \u03b5\u03b9\u03bd\u03b1\u03b9 \u03c0\u03c1\u03c9\u03c4\u03bf \u03b8\u03b5\u03bc\u03b1 \u03b1\u03c0\u03bf \u03c0\u03b1\u03bb\u03b9\u03bf \u0391\u03c1\u03c7\u03b9\u03bc\u03b7\u03b4\u03b7, \u03b5\u03c5\u03ba\u03bf\u03bb\u03bf\u03c5\u03c4\u03c3\u03b9\u03ba\u03bf\r\n\r\n\u03a3\u03c4\u03bf 1\u03bf \u03bf\u03b9 \u03b1\u03c1\u03b9\u03b8\u03bc\u03bf\u03b9 $ |x|^2, |y|^2$ \u03b5\u03b9\u03bd\u03b1\u03b9 \u03c1\u03b9\u03b6\u03b5\u03c2 \u03ba\u03b1\u03c0\u03bf\u03b9\u03b1\u03c2 \u03b5\u03be\u03b9\u03c3\u03c9\u03c3\u03b7\u03c2 $ x^2 \\minus{} ax \\plus{} a$ \u03bc\u03b5 $ a \\geq 4$, \u03b3\u03b9\u03b1 \u03ba\u03b1\u03b8\u03b5 \u03c4\u03b5\u03c4\u03bf\u03b9\u03bf $ a$ \u03b7 \u03c0\u03b1\u03c1\u03b1\u03c3\u03c4\u03b1\u03c3\u03b7 \u03b9\u03c3\u03bf\u03c5\u03c4\u03b5 \u03bc\u03b5 $ f(a) \\equal{} \\sqrt {a \\plus{} 2\\sqrt {a}} \\plus{} \\sqrt {a} \\plus{} 1$ (\u03b1\u03bd \u03b4\u03b5\u03bd \u03bc\u03bf\u03c5 \u03be\u03b5\u03c6\u03c5\u03b3\u03b5 \u03ba\u03b1\u03c4\u03b9). \u0397 f \u03c4\u03c9\u03c1\u03b1 \u03c9\u03c2 \u03c3\u03c5\u03bd\u03b5\u03c7\u03b7\u03c2 \u03ba\u03b1\u03b9 \u03b3\u03bd\u03b7\u03c3\u03b9\u03c9\u03c2 \u03b1\u03c5\u03be\u03bf\u03c5\u03c3\u03b1 \u03c3\u03c5\u03bd\u03b1\u03c1\u03c4\u03b7\u03c3\u03b7 \u03c4\u03bf\u03c5 $ a$, \u03c3\u03b1\u03c1\u03c9\u03bd\u03b5\u03b9 \u03bf\u03bb\u03bf \u03c4\u03bf \u03b4\u03b9\u03b1\u03c3\u03c4\u03b7\u03bc\u03b1 $ [2\\sqrt {2} \\plus{} 3, \\plus{} \\infty)$, \u03bf\u03c4\u03b1\u03bd \u03bf $ a$ \u03c3\u03b1\u03c1\u03c9\u03bd\u03b5\u03b9 \u03c4\u03bf \u03b4\u03b9\u03b1\u03c3\u03c4\u03b7\u03bc\u03b1 $ [4, \\plus{} \\infty)$. \u0394\u03b5\u03bd \u03ba\u03b1\u03c4\u03b1\u03bb\u03b1\u03b2\u03b1\u03b9\u03bd\u03c9 \u03c4\u03b9 \u03b1\u03bb\u03bb\u03bf \u03bc\u03c0\u03bf\u03c1\u03b5\u03b9 \u03bd\u03b1 \u03b6\u03b7\u03c4\u03b1\u03b5\u03b9 \u03c4\u03bf \u03c0\u03c1\u03bf\u03b2\u03bb\u03b7\u03bc\u03b1...\r\n\r\nEdit: \u039f\u03c0\u03bf\u03c4\u03b5 \u03b7 \u03b5\u03bb\u03b1\u03c7\u03b9\u03c3\u03c4\u03b7 \u03c4\u03b9\u03bc\u03b7 \u03b5\u03b9\u03bd\u03b1\u03b9 $ 2\\sqrt {2} \\plus{} 3$\r\n\r\n\u03a4\u03bf \u03b4\u03b5\u03c5\u03c4\u03b5\u03c1\u03bf \u03b5\u03c5\u03ba\u03bf\u03bb\u03b1 \u03b3\u03b9\u03bd\u03b5\u03c4\u03b5 \u03b9\u03c3\u03bf\u03b4\u03c5\u03bd\u03b1\u03bc\u03bf \u03bc\u03b5 \u03c4\u03bf \u03bd\u03b1 \u03b4\u03b5\u03b9\u03c7\u03c4\u03b5\u03b9 \u03bf\u03c4\u03b9 \u03c0\u03b1\u03bd\u03c9 \u03c3\u03c4\u03b7 \u03c3\u03c6\u03b1\u03b9\u03c1\u03b1 \u03bc\u03b5 \u03ba\u03b5\u03bd\u03c4\u03c1\u03bf \u03c4\u03b7\u03bd \u03b1\u03c1\u03c7\u03b7 \u03c4\u03c9\u03bd \u03b1\u03be\u03bf\u03bd\u03c9\u03bd \u03ba\u03b1\u03b9 \u03b1\u03ba\u03c4\u03b9\u03bd\u03b1 $ \\sqrt {7}$ \u03c3\u03c4\u03bf\u03bd $ R^3$, \u03b4\u03b5\u03bd \u03c5\u03c0\u03b1\u03c1\u03c7\u03b5\u03b9 \u03c3\u03b7\u03bc\u03b5\u03b9\u03bf \u03bc\u03b5 \u03c1\u03b7\u03c4\u03b5\u03c2 \u03c3\u03c5\u03bd\u03c4\u03b5\u03c4\u03b1\u03b3\u03bc\u03b5\u03bd\u03b5\u03c2, \u03c0\u03c1\u03bf\u03b2\u03bb\u03b7\u03bc\u03b1 \u03c4\u03bf \u03bf\u03c0\u03bf\u03b9\u03bf \u03b5\u03c7\u03c9 \u03c4\u03b7\u03bd \u03b5\u03bd\u03c4\u03c5\u03c0\u03c9\u03c3\u03b7 \u03c0\u03c9\u03c2 \u03b5\u03b9\u03bd\u03b1\u03b9 \u03b1\u03c1\u03ba\u03b5\u03c4\u03b1 \u03b3\u03bd\u03c9\u03c3\u03c4\u03bf \u03b1\u03bb\u03bb\u03b1 \u03c0\u03bf\u03c4\u03b5 \u03b4\u03b5\u03bd \u03c4\u03bf \u03b5\u03c7\u03c9 \u03bc\u03b5\u03bb\u03b5\u03c4\u03b7\u03c3\u03b5\u03b9 \u03ba\u03b1\u03b9 \u03b3\u03b9\u03b1 \u03c4\u03b7\u03bd \u03c9\u03c1\u03b1 \u03b4\u03b5\u03bd \u03bc\u03c0\u03bf\u03c1\u03c9 \u03bd\u03b1 \u03c6\u03b1\u03bd\u03c4\u03b1\u03c3\u03c4\u03c9 \u03c0\u03c9\u03c2 \u03bc\u03c0\u03bf\u03c1\u03b5\u03b9 \u03bd\u03b1 \u03b1\u03bd\u03c4\u03b9\u03bc\u03b5\u03c4\u03bf\u03c0\u03b9\u03c3\u03c4\u03b5\u03b9, \u03bc\u03b5 \u03b1\u03bd\u03c4\u03b9\u03ba\u03b1\u03c4\u03b1\u03c3\u03c4\u03b1\u03c3\u03b5\u03b9\u03c2 \u03c0\u03b1\u03bd\u03c4\u03c9\u03c2 \u03c4\u03c9\u03bd \u03c1\u03b7\u03c4\u03c9\u03bd \u03bc\u03b5 \u03c0\u03b7\u03bb\u03b9\u03ba\u03b1 \u03b1\u03ba\u03b5\u03c1\u03b1\u03b9\u03c9\u03bd \u03b4\u03b5\u03bd \u03bd\u03bf\u03bc\u03b9\u03b6\u03c9 \u03bd\u03b1 \u03b2\u03b3\u03b1\u03b9\u03bd\u03b5\u03b9 \u03b4\u03b9\u03bf\u03c4\u03b9 \u03b3\u03b9\u03bd\u03b5\u03c4\u03b1\u03b9 \u03c7\u03b1\u03bc\u03bf\u03c2 \u03b1\u03c0\u03bf \u03bc\u03b5\u03c4\u03b1\u03b2\u03bb\u03b7\u03c4\u03b5\u03c2 \u03b5\u03bd\u03c9 \u03bf\u03b9 \u03b4\u03b9\u03b1\u03b9\u03c1\u03b5\u03c4\u03bf\u03c4\u03b7\u03c4\u03b5\u03c2 \u03b4\u03b5\u03bd \u03bf\u03b4\u03b7\u03b3\u03bf\u03c5\u03bd\u03b5 \u03c0\u03bf\u03c5\u03b8\u03b5\u03bd\u03b1, \u03b9\u03c3\u03c9\u03c2 \u03bd\u03b1 \u03b8\u03b5\u03bb\u03b5\u03b9 infinite descent, \u03c4\u03bf \u03bc\u03bf\u03bd\u03bf \u03c3\u03b9\u03b3\u03bf\u03c5\u03c1\u03bf \u03c0\u03b1\u03bd\u03c4\u03bf\u03c2 \u03b5\u03b9\u03bd\u03b1\u03b9 \u03bf\u03c4\u03b9 \u03b4\u03b5\u03bd \u03c0\u03c1\u03bf\u03ba\u03b5\u03b9\u03c4\u03b5 \u03c0\u03b5\u03c1\u03b9 \u03c3\u03c7\u03bf\u03bb\u03b9\u03ba\u03b7\u03c2 \u03b1\u03c3\u03ba\u03b7\u03c3\u03b7\u03c2....",
"Solution_5": "\u039a\u03b1\u03bb\u03b1 \u03c3\u03c7\u03bf\u03bb\u03b9\u03ba\u03bf \u03b4\u03b5\u03bd \u03c4\u03bf \u03bb\u03b5\u03c2 \u03b1\u03bb\u03bb\u03b1 \u03b4\u03b5\u03bd \u03b5\u03b9\u03bd\u03b1\u03b9 \u03b9\u03b4\u03b9\u03b1\u03b9\u03c4\u03b5\u03c1\u03b1 \u03b4\u03c5\u03c3\u03ba\u03bf\u03bb\u03bf \u03c4\u03bf 2\u03bf \u03b1\u03bd \u03ba\u03b1\u03b9 \u03b8\u03b5\u03bb\u03b5\u03b9 \u03c0\u03c1\u03bf\u03c3\u03bf\u03c7\u03b7.\r\n\u039f\u03c5\u03c3\u03b9\u03b1\u03c3\u03c4\u03b9\u03ba\u03b1 \u03b5\u03b9\u03bd\u03b1\u03b9 \u03b9\u03c3\u03bf\u03b4\u03c5\u03bd\u03b1\u03bc\u03bf \u03bc\u03b5 \u03c4\u03b7\u03bd \u03b5\u03be\u03b9\u03c3\u03c9\u03c3\u03b7 $ (2x \\minus{} 1)^2 \\plus{} (2y \\minus{} 1)^2 \\plus{} (2z \\minus{} 1)^2 \\equal{} 7$ $ x,y,z$ \u03c3\u03c4\u03bf Q.\r\n\u0395\u03c4\u03c3\u03b9 \u03c4\u03bf \u03c0\u03c1\u03bf\u03b2\u03bb\u03b7\u03bc\u03b1 \u03b5\u03b9\u03bd\u03b1\u03b9 \u03c3\u03b1\u03bd \u03bd\u03b1 \u03b8\u03b5\u03bb\u03bf\u03c5\u03bc\u03b5 \u03bd\u03b1 \u03b4\u03b5\u03b9\u03be\u03bf\u03c5\u03bc\u03b5 \u03bf\u03c4\u03b9 \u03b4\u03b5\u03bd \u03b5\u03c7\u03b5\u03b9 \u03b1\u03ba\u03b5\u03c1\u03b1\u03b9\u03b5\u03c2 \u03bb\u03c5\u03c3\u03b5\u03b9\u03c2 \u03b7 \u03b5\u03be\u03b9\u03c3\u03c9\u03c3\u03b7 $ a^2 \\plus{} b^2 \\plus{} c^2 \\equal{} 7s^2$ \u03c9\u03c2 \u03c0\u03c1\u03bf\u03c2 $ a,b,c,s$ (\u03b4\u03b5\u03bd \u03b5\u03b9\u03bd\u03b1\u03b9 \u03b9\u03c3\u03bf\u03b4\u03c5\u03bd\u03b1\u03bc\u03bf \u03b1\u03bb\u03bb\u03b1 \u03b5\u03c0\u03b1\u03c1\u03ba\u03b5\u03c2 \u03c3\u03b9\u03b3\u03bf\u03c5\u03c1\u03b1)\r\n\u0395\u03c5\u03ba\u03bf\u03bb\u03b1 \u03b2\u03bb\u03b5\u03c0\u03bf\u03c5\u03bc\u03b5 \u03bf\u03c4\u03b9 \u03bc\u03c0\u03bf\u03c1\u03bf\u03c5\u03bc\u03b5 \u03bd\u03b1 \u03ba\u03b1\u03bd\u03bf\u03c5\u03bc\u03b5 \u03c4\u03b7\u03bd \u03c4\u03b5\u03c4\u03c1\u03b1\u03b4\u03b1 \u03c0\u03b5\u03c1\u03b9\u03c4\u03c4\u03bf\u03c5\u03c2($ \\mod 4$) \u03ba\u03b1\u03b9 \u03b1\u03c1\u03b1 \u03bc\u03b5\u03c4\u03b1 $ \\mod 8$ \u03b5\u03c7\u03bf\u03c5\u03bc\u03b5 \u03b1\u03c4\u03bf\u03c0\u03bf.\r\n\u0393\u03b9\u03b1 \u03c4\u03b7\u03bd \u03c3\u03c6\u03b1\u03b9\u03c1\u03b1 \u03c0\u03bf\u03c5 \u03bb\u03b5\u03c2 nick \u03bc\u03c0\u03bf\u03c1\u03b5\u03b9 \u03ba\u03b1\u03c0\u03bf\u03b9\u03bf\u03c2 \u03bd\u03b1 \u03c0\u03b1\u03c1\u03b1\u03c4\u03b7\u03c1\u03b7\u03c3\u03b5\u03b9 \u03bf\u03c4\u03b9 \u03b7 \u03c4\u03bf \u03c0\u03c1\u03bf\u03b2\u03bb\u03b7\u03bc\u03b1 2 \u03b3\u03b5\u03bd\u03b9\u03ba\u03b5\u03c5\u03b5\u03c4\u03b1\u03b9 \u03b3\u03b9\u03b1 \u03bf\u03c0\u03bf\u03b9\u03bf\u03b4\u03b7\u03c0\u03bf\u03c4\u03b5 \u03b1\u03c1\u03b9\u03b8\u03bc\u03bf $ 8k \\plus{} 1$ \u03c3\u03c4\u03bf RHS.\r\n\r\nbtw leftmast \u03ba\u03b1\u03bb\u03bf \u03b5\u03b9\u03bd\u03b1\u03b9 \u03b3\u03b5\u03bd\u03b9\u03ba\u03b1 \u03bf\u03c4\u03b1\u03bd \u03b5\u03c7\u03b5\u03b9\u03c2 \u03bb\u03c5\u03c3\u03b5\u03b9 \u03ba\u03b1\u03c4\u03b9 \u03bd\u03b1 \u03b2\u03b1\u03b6\u03b5\u03b9\u03c2 \u03c0\u03bf\u03c5 \u03ba\u03b1\u03b9 \u03c0\u03bf\u03c5 \u03b5\u03c3\u03c4\u03c9, \u03bc\u03b9\u03b1 \u03c0\u03b5\u03c1\u03b9\u03c0\u03bf\u03c5 \u03bf\u03bb\u03bf\u03ba\u03bb\u03b7\u03c1\u03c9\u03bc\u03b5\u03bd\u03b7 \u03bb\u03c5\u03c3\u03b7. \r\n\u0397 \u03b9\u03b4\u03b5\u03b1 \u03b5\u03b9\u03bd\u03b1\u03b9 \u03c3\u03b7\u03bc\u03b1\u03bd\u03c4\u03b9\u03ba\u03b7 \u03b1\u03bb\u03bb\u03b1 \u03ba\u03b1\u03b9 \u03b7 \u03b4\u03b9\u03b1\u03c4\u03c5\u03c0\u03c9\u03c3\u03b7 \u03bf\u03c0\u03c9\u03c2 \u03ba\u03b1\u03b9 \u03bf\u03b9 \u03bb\u03b5\u03c0\u03c4\u03bf\u03bc\u03b5\u03c1\u03b5\u03b9\u03b5\u03c2 \u03b5\u03c7\u03bf\u03c5\u03bd \u03c3\u03b7\u03bc\u03b1\u03c3\u03b9\u03b1 \u03b3\u03b9\u03b1 \u03c4\u03b7\u03bd \u03bb\u03c5\u03c3\u03b7 \u03b5\u03bd\u03bf\u03c2 \u03c0\u03c1\u03bf\u03b2\u03bb\u03b7\u03bc\u03b1\u03c4\u03bf\u03c2.",
"Solution_6": "[quote=\"Athinaios\"]\n\u0395\u03c5\u03ba\u03bf\u03bb\u03b1 \u03b2\u03bb\u03b5\u03c0\u03bf\u03c5\u03bc\u03b5 \u03bf\u03c4\u03b9 \u03bc\u03c0\u03bf\u03c1\u03bf\u03c5\u03bc\u03b5 \u03bd\u03b1 \u03ba\u03b1\u03bd\u03bf\u03c5\u03bc\u03b5 \u03c4\u03b7\u03bd \u03c4\u03b5\u03c4\u03c1\u03b1\u03b4\u03b1 \u03c0\u03b5\u03c1\u03b9\u03c4\u03c4\u03bf\u03c5\u03c2($ \\mod 4$) \u03ba\u03b1\u03b9 \u03b1\u03c1\u03b1 \u03bc\u03b5\u03c4\u03b1 $ \\mod 8$ \u03b5\u03c7\u03bf\u03c5\u03bc\u03b5 \u03b1\u03c4\u03bf\u03c0\u03bf.\n[/quote]\r\n\r\n\u03a0\u03bb\u03b1\u03ba\u03b1 \u03ba\u03b1\u03bd\u03b5\u03b9\u03c2 \u03c4\u03c9\u03c1\u03b1.... \u03ba\u03b1\u03b9 \u03c0\u03b1\u03bb\u03b5\u03c5\u03b1 5 \u03c9\u03c1\u03b5\u03c2 \u03c4\u03c9\u03c1\u03b1 \u03bc\u03b5 6 \u03bc\u03b5\u03c4\u03b1\u03b2\u03bb\u03b7\u03c4\u03b5\u03c2 \u03ba\u03b1\u03b9 \u03b4\u03b9\u03b1\u03b9\u03c1\u03b5\u03c4\u03bf\u03c4\u03b7\u03c4\u03b5\u03c2 \u03ba\u03b1\u03b9 \u03b7\u03c4\u03b1\u03bd \u03c4\u03bf\u03c3\u03bf \u03b1\u03c0\u03bb\u03bf... \u03b5\u03c7\u03c9 \u03b1\u03c1\u03c7\u03b9\u03c3\u03b5\u03b9 \u03ba\u03b1\u03b9 \u03be\u03b5\u03c7\u03bd\u03b1\u03c9 \u03b2\u03b1\u03c3\u03b9\u03ba\u03b1 \u03c0\u03c1\u03b1\u03b3\u03bc\u03b1\u03c4\u03b1 \u03ba\u03b1\u03c4\u03b9 \u03bc\u03bf\u03c5 \u03c6\u03b1\u03b9\u03bd\u03b5\u03c4\u03b1\u03b9....",
"Solution_7": "Z\u03b7\u03c4\u03c9 \u03c3\u03c5\u03b3\u03b3\u03bd\u03c9\u03bc\u03b7 .....! :blush: \u03a4\u03bf \u03b5\u03ba\u03b1\u03bd\u03b1 \u03b1\u03c3\u03c5\u03bd\u03b1\u03b9\u03c3\u03b8\u03b7\u03c4\u03b1!!!\u03a0\u03b1\u03bd\u03c4\u03c9\u03c2 \u03b3\u03b9\u03b1 \u03c4\u03b7\u03bd \u03b4\u03b5\u03c5\u03c4\u03b5\u03c1\u03b7 \u03b1\u03c3\u03ba\u03b7\u03c3\u03b7, \u03b1\u03c6\u03bf\u03c5 \u03ba\u03b1\u03bd\u03bf\u03c5\u03bc\u03b5 \u03c4\u03b9\u03c3 \u03b1\u03bd\u03c4\u03b9\u03ba\u03b1\u03c4\u03b1\u03c3\u03c4\u03b1\u03c3\u03b5\u03b9\u03c2 \u03b1\u03bd \u03c4\u03bf \u03c0\u03b1\u03bc\u03b5 mod3 \u03ba\u03b1\u03b9 \u03bc\u03b5\u03c4\u03b1 mod 9 \u03b8\u03b1 \u03b2\u03b3\u03b5\u03b9,\u03b1\u03bd \u03ba\u03b1\u03b9 \u03bc\u03bf\u03c5 \u03b5\u03c0\u03b9\u03b1\u03c3\u03b5 2 \u03c3\u03b5\u03bb\u03b9\u03b4\u03b5\u03c2 \u03c0\u03b5\u03c1\u03b9\u03c0\u03bf\u03c5...\u039c\u03bf\u03bb\u03b9\u03c2 \u03b2\u03c1\u03c9 \u03c7\u03c1\u03bf\u03bd\u03bf \u03b8\u03b1 \u03b2\u03b1\u03bb\u03c9 \u03ba\u03b1\u03b9 \u03c4\u03b7\u03bd \u03bb\u03c5\u03c3\u03b7 \u03bc\u03bf\u03c5!!!!\u03a0\u03bf\u03bb\u03c5 \u03ba\u03b1\u03bb\u03b7 \u03b7 \u03bb\u03c5\u03c3\u03b7 \u03c3\u03bf\u03c5 Athinaios \u03c0\u03b1\u03bd\u03c4\u03c9\u03c2!!! :)",
"Solution_8": "[quote=\"lefmast\"]\u03c4\u03bf \u03b4\u03b5\u03c5\u03c4\u03b5\u03c1\u03bf \u03bc\u03b5 \u03bc\u03b9\u03b1 \u03c0\u03c1\u03bf\u03c7\u03b5\u03b9\u03c1\u03b7 \u03bc\u03b1\u03c4\u03b9\u03b1 \u03b2\u03b3\u03b1\u03b9\u03bd\u03b5\u03b9 \u03bc\u03b5 \u03b1\u03c0\u03b5\u03b9\u03c1\u03b7 \u03ba\u03b1\u03b8\u03bf\u03b4\u03bf.....\u0395\u03c7\u03c9 \u03ba\u03b1\u03c0\u03bf\u03b9\u03b5\u03c2 \u03b1\u03bc\u03c6\u03b9\u03b2\u03bf\u03bb\u03b9\u03b5\u03c2 \u03b3\u03b9\u03b1 \u03c4\u03bf \u03c0\u03c1\u03c9\u03c4\u03bf \u03bf\u03bc\u03c9\u03c2...\u0391\u03bd \u03b8\u03b5\u03c3\u03bf\u03c5\u03bc\u03b5 \u03ba\u03b1\u03c0\u03bf\u03b9\u03b5\u03c2 \u03c4\u03b9\u03bc\u03b5\u03c2 \u03c3\u03c4\u03bf \u03c7 \u03ba\u03b1\u03b9 \u03bb\u03c5\u03c3\u03bf\u03c5\u03bc\u03b5 \u03c9\u03c2 \u03c0\u03c1\u03bf\u03c3 y \u03b4\u03b5\u03bd \u03b8\u03b1 \u03b5\u03c7\u03b5\u03b9 \u03c4\u03b7\u03bd \u03b9\u03b4\u03b9\u03b1 \u03c4\u03b9\u03bc\u03b7 \u03c4\u03bf \u0391.... :( :maybe:\u03a4\u03b9 \u03b5\u03bd\u03bd\u03bf\u03b5\u03b9\u03c2 \u03bd\u03b1 \u03c5\u03c0\u03bf\u03bb\u03bf\u03b3\u03b9\u03c3\u03c4\u03b5\u03b9????\u0391\u03bd \u03bc\u03c0\u03bf\u03c1\u03b5\u03b9\u03c2 \u03b5\u03be\u03b7\u03b3\u03b7\u03c3\u03b5 \u03c4\u03bf \u03bc\u03bf\u03c5 \u03bb\u03b9\u03b3\u03bf \u03c3\u03b5 \u03c0\u03b1\u03c1\u03b1\u03ba\u03b1\u03bb\u03c9....... :)[/quote]\r\n\r\nlipon katarxas sorry eprepe na diefkrinisw oti zitw thn elaxisth timh tou A , kai episis ola afta ta 8emata lunode polu wraia me gnwseis lukeiou (ulh panelinion) ara kai xwris mod hlia :P",
"Solution_9": "[quote=\"KapioPulsar\"][quote=\"lefmast\"]\u03c4\u03bf \u03b4\u03b5\u03c5\u03c4\u03b5\u03c1\u03bf \u03bc\u03b5 \u03bc\u03b9\u03b1 \u03c0\u03c1\u03bf\u03c7\u03b5\u03b9\u03c1\u03b7 \u03bc\u03b1\u03c4\u03b9\u03b1 \u03b2\u03b3\u03b1\u03b9\u03bd\u03b5\u03b9 \u03bc\u03b5 \u03b1\u03c0\u03b5\u03b9\u03c1\u03b7 \u03ba\u03b1\u03b8\u03bf\u03b4\u03bf.....\u0395\u03c7\u03c9 \u03ba\u03b1\u03c0\u03bf\u03b9\u03b5\u03c2 \u03b1\u03bc\u03c6\u03b9\u03b2\u03bf\u03bb\u03b9\u03b5\u03c2 \u03b3\u03b9\u03b1 \u03c4\u03bf \u03c0\u03c1\u03c9\u03c4\u03bf \u03bf\u03bc\u03c9\u03c2...\u0391\u03bd \u03b8\u03b5\u03c3\u03bf\u03c5\u03bc\u03b5 \u03ba\u03b1\u03c0\u03bf\u03b9\u03b5\u03c2 \u03c4\u03b9\u03bc\u03b5\u03c2 \u03c3\u03c4\u03bf \u03c7 \u03ba\u03b1\u03b9 \u03bb\u03c5\u03c3\u03bf\u03c5\u03bc\u03b5 \u03c9\u03c2 \u03c0\u03c1\u03bf\u03c3 y \u03b4\u03b5\u03bd \u03b8\u03b1 \u03b5\u03c7\u03b5\u03b9 \u03c4\u03b7\u03bd \u03b9\u03b4\u03b9\u03b1 \u03c4\u03b9\u03bc\u03b7 \u03c4\u03bf \u0391.... :( :maybe:\u03a4\u03b9 \u03b5\u03bd\u03bd\u03bf\u03b5\u03b9\u03c2 \u03bd\u03b1 \u03c5\u03c0\u03bf\u03bb\u03bf\u03b3\u03b9\u03c3\u03c4\u03b5\u03b9????\u0391\u03bd \u03bc\u03c0\u03bf\u03c1\u03b5\u03b9\u03c2 \u03b5\u03be\u03b7\u03b3\u03b7\u03c3\u03b5 \u03c4\u03bf \u03bc\u03bf\u03c5 \u03bb\u03b9\u03b3\u03bf \u03c3\u03b5 \u03c0\u03b1\u03c1\u03b1\u03ba\u03b1\u03bb\u03c9....... :)[/quote]\n\nlipon katarxas sorry eprepe na diefkrinisw oti zitw thn elaxisth timh tou A , kai episis ola afta ta 8emata lunode polu wraia me gnwseis lukeiou (ulh panelinion) ara kai xwris mod hlia :P[/quote]\r\n\r\n\u03a4\u03b1 \u03c0\u03b1\u03c1\u03b1\u03bb\u03b5\u03c2 \u03c4\u03c9\u03c1\u03b1... \u03c4\u03bf \u03c0\u03c1\u03c9\u03c4\u03bf \u03ba\u03b1\u03b9 \u03c4\u03bf \u03c4\u03c1\u03b9\u03c4\u03bf \u03c0\u03c1\u03bf\u03b2\u03bb\u03b7\u03bc\u03b1 \u03b5\u03b9\u03bd\u03b1\u03b9 \u03bf\u03bd\u03c4\u03c9\u03c2 \u03c3\u03c7\u03bf\u03bb\u03b9\u03ba\u03b1, \u03b5\u03bd\u03c9 \u03c4\u03bf \u03c4\u03c1\u03b9\u03c4\u03bf \u03bc\u03c0\u03bf\u03c1\u03b5\u03b9 \u03bd\u03b1 \u03bc\u03c0\u03b5\u03b9 \u03ba\u03b1\u03b9 \u03c3\u03b5 \u03b5\u03bd\u03b1\u03bd \u03b4\u03b9\u03b1\u03b3\u03c9\u03bd\u03b9\u03c3\u03bc\u03bf \u03bc\u03b9\u03ba\u03c1\u03bf\u03c5 \u03b5\u03c0\u03b9\u03c0\u03b5\u03b4\u03bf\u03c5 (\u03bf\u03c0\u03c9\u03c2 \u03b5\u03b9\u03c0\u03b1 \u03ba\u03b1\u03b9 \u03c0\u03b1\u03c1\u03b1\u03c0\u03b1\u03bd\u03c9, \u03b5\u03b9\u03c7\u03b5 \u03bc\u03c0\u03b5\u03b9 \u03c3\u03b1\u03bd \u03b5\u03c5\u03ba\u03bf\u03bb\u03bf \u03b8\u03b5\u03bc\u03b1 \u03c3\u03b5 \u0391\u03c1\u03c7\u03b9\u03bc\u03b7\u03b4\u03b7 \u03c0\u03b1\u03bb\u03b9\u03b1). \u03a4\u03bf \u03b4\u03b5\u03c5\u03c4\u03b5\u03c1\u03bf \u03bf\u03bc\u03c9\u03c2 \u03b4\u03b5\u03bd \u03b5\u03c7\u03b5\u03b9 \u03ba\u03b1\u03bc\u03b9\u03b1 \u03c3\u03c7\u03b5\u03c3\u03b7 \u03bc\u03b5 \u03c4\u03b7\u03bd \u03c3\u03c7\u03bf\u03bb\u03b9\u03ba\u03b7 \u03c5\u03bb\u03b7, \u03b5\u03b9\u03b4\u03b9\u03ba\u03b1 \u03bc\u03b5 \u03b1\u03c5\u03c4\u03b7 \u03c4\u03c9\u03bd \u03c0\u03b1\u03bd\u03b5\u03bb\u03bb\u03b7\u03bd\u03b9\u03c9\u03bd, \u03b1\u03c6\u03bf\u03c5 \u03bc\u03b5\u03c4\u03b1 \u03c4\u03bf \u03bc\u03b5\u03c4\u03b1\u03c3\u03c7\u03b7\u03bc\u03b1\u03c4\u03b9\u03c3\u03bc\u03bf \u03b1\u03bd\u03b1\u03b3\u03b5\u03c4\u03b1\u03b9 \u03c3\u03b5 \u03bc\u03b9\u03b1 \u03c0\u03bf\u03bb\u03c5\u03c0\u03bb\u03bf\u03ba\u03b7 \u03b4\u03b9\u03bf\u03c6\u03b1\u03bd\u03c4\u03b9\u03ba\u03b7 \u03b5\u03be\u03b9\u03c3\u03c9\u03c3\u03b7 \u03bc\u03b5 \u03c4\u03b1 \u03c4\u03b5\u03c4\u03c1\u03b1\u03b3\u03c9\u03bd\u03b1 6 \u03b1\u03ba\u03b5\u03c1\u03b1\u03b9\u03c9\u03bd \u03bc\u03b5\u03c4\u03b1\u03b2\u03bb\u03b7\u03c4\u03c9\u03bd, \u03bf\u03c0\u03bf\u03c4\u03b5 \u03bf\u03c0\u03c9\u03c2 \u03ba\u03b1\u03b9 \u03bd\u03b1 \u03b3\u03b9\u03bd\u03b5\u03b9 \u03c7\u03c1\u03b5\u03b9\u03b1\u03b6\u03bf\u03bd\u03c4\u03b1\u03b9 \u03b3\u03bd\u03c9\u03c3\u03b5\u03b9\u03c2 \u03ba\u03b1\u03b9 \u03c4\u03b5\u03c7\u03bd\u03b1\u03c3\u03bc\u03b1\u03c4\u03b1 \u03b1\u03c0\u03bf \u03c4\u03b7 \u03b8\u03b5\u03c9\u03c1\u03b9\u03b1 \u03c4\u03c9\u03bd \u03b1\u03c1\u03b9\u03b8\u03bc\u03c9\u03bd, \u03b5\u03b3\u03c9 \u03bf\u03c4\u03b1\u03bd \u03b5\u03b9\u03bc\u03bf\u03c5\u03bd \u03c3\u03c4\u03bf \u03bb\u03c5\u03ba\u03b5\u03b9\u03bf (\u03c3\u03c4\u03bf \u03b4\u03b9\u03ba\u03bf \u03bc\u03bf\u03c5 \u03bb\u03c5\u03ba\u03b5\u03b9\u03bf \u03c4\u03bf\u03c5\u03bb\u03b1\u03c7\u03b9\u03c3\u03c4\u03bf\u03bd) \u03b4\u03b5\u03bd \u03b8\u03c5\u03bc\u03b1\u03bc\u03b5 \u03bd\u03b1 \u03b1\u03bd\u03b1\u03c6\u03b5\u03c1\u03b8\u03b7\u03ba\u03b5 \u03c0\u03bf\u03c4\u03b5 \u03bf\u03c5\u03c4\u03b5 \u03ba\u03b1\u03bd \u03bf \u03bf\u03c1\u03bf\u03c2 \"\u03b4\u03b9\u03b1\u03b9\u03c1\u03b5\u03c4\u03bf\u03c4\u03b7\u03c4\u03b1\" \u03c3\u03c4\u03b7\u03bd \u03c4\u03b1\u03be\u03b7...",
"Solution_10": "[quote=\"NickNafplio\"][quote=\"KapioPulsar\"][quote=\"lefmast\"]\u03c4\u03bf \u03b4\u03b5\u03c5\u03c4\u03b5\u03c1\u03bf \u03bc\u03b5 \u03bc\u03b9\u03b1 \u03c0\u03c1\u03bf\u03c7\u03b5\u03b9\u03c1\u03b7 \u03bc\u03b1\u03c4\u03b9\u03b1 \u03b2\u03b3\u03b1\u03b9\u03bd\u03b5\u03b9 \u03bc\u03b5 \u03b1\u03c0\u03b5\u03b9\u03c1\u03b7 \u03ba\u03b1\u03b8\u03bf\u03b4\u03bf.....\u0395\u03c7\u03c9 \u03ba\u03b1\u03c0\u03bf\u03b9\u03b5\u03c2 \u03b1\u03bc\u03c6\u03b9\u03b2\u03bf\u03bb\u03b9\u03b5\u03c2 \u03b3\u03b9\u03b1 \u03c4\u03bf \u03c0\u03c1\u03c9\u03c4\u03bf \u03bf\u03bc\u03c9\u03c2...\u0391\u03bd \u03b8\u03b5\u03c3\u03bf\u03c5\u03bc\u03b5 \u03ba\u03b1\u03c0\u03bf\u03b9\u03b5\u03c2 \u03c4\u03b9\u03bc\u03b5\u03c2 \u03c3\u03c4\u03bf \u03c7 \u03ba\u03b1\u03b9 \u03bb\u03c5\u03c3\u03bf\u03c5\u03bc\u03b5 \u03c9\u03c2 \u03c0\u03c1\u03bf\u03c3 y \u03b4\u03b5\u03bd \u03b8\u03b1 \u03b5\u03c7\u03b5\u03b9 \u03c4\u03b7\u03bd \u03b9\u03b4\u03b9\u03b1 \u03c4\u03b9\u03bc\u03b7 \u03c4\u03bf \u0391.... :( :maybe:\u03a4\u03b9 \u03b5\u03bd\u03bd\u03bf\u03b5\u03b9\u03c2 \u03bd\u03b1 \u03c5\u03c0\u03bf\u03bb\u03bf\u03b3\u03b9\u03c3\u03c4\u03b5\u03b9????\u0391\u03bd \u03bc\u03c0\u03bf\u03c1\u03b5\u03b9\u03c2 \u03b5\u03be\u03b7\u03b3\u03b7\u03c3\u03b5 \u03c4\u03bf \u03bc\u03bf\u03c5 \u03bb\u03b9\u03b3\u03bf \u03c3\u03b5 \u03c0\u03b1\u03c1\u03b1\u03ba\u03b1\u03bb\u03c9....... :)[/quote]\n\nlipon katarxas sorry eprepe na diefkrinisw oti zitw thn elaxisth timh tou A , kai episis ola afta ta 8emata lunode polu wraia me gnwseis lukeiou (ulh panelinion) ara kai xwris mod hlia :P[/quote]\n\n\u03a4\u03b1 \u03c0\u03b1\u03c1\u03b1\u03bb\u03b5\u03c2 \u03c4\u03c9\u03c1\u03b1... \u03c4\u03bf \u03c0\u03c1\u03c9\u03c4\u03bf \u03ba\u03b1\u03b9 \u03c4\u03bf \u03c4\u03c1\u03b9\u03c4\u03bf \u03c0\u03c1\u03bf\u03b2\u03bb\u03b7\u03bc\u03b1 \u03b5\u03b9\u03bd\u03b1\u03b9 \u03bf\u03bd\u03c4\u03c9\u03c2 \u03c3\u03c7\u03bf\u03bb\u03b9\u03ba\u03b1, \u03b5\u03bd\u03c9 \u03c4\u03bf \u03c4\u03c1\u03b9\u03c4\u03bf \u03bc\u03c0\u03bf\u03c1\u03b5\u03b9 \u03bd\u03b1 \u03bc\u03c0\u03b5\u03b9 \u03ba\u03b1\u03b9 \u03c3\u03b5 \u03b5\u03bd\u03b1\u03bd \u03b4\u03b9\u03b1\u03b3\u03c9\u03bd\u03b9\u03c3\u03bc\u03bf \u03bc\u03b9\u03ba\u03c1\u03bf\u03c5 \u03b5\u03c0\u03b9\u03c0\u03b5\u03b4\u03bf\u03c5 (\u03bf\u03c0\u03c9\u03c2 \u03b5\u03b9\u03c0\u03b1 \u03ba\u03b1\u03b9 \u03c0\u03b1\u03c1\u03b1\u03c0\u03b1\u03bd\u03c9, \u03b5\u03b9\u03c7\u03b5 \u03bc\u03c0\u03b5\u03b9 \u03c3\u03b1\u03bd \u03b5\u03c5\u03ba\u03bf\u03bb\u03bf \u03b8\u03b5\u03bc\u03b1 \u03c3\u03b5 \u0391\u03c1\u03c7\u03b9\u03bc\u03b7\u03b4\u03b7 \u03c0\u03b1\u03bb\u03b9\u03b1). \u03a4\u03bf \u03b4\u03b5\u03c5\u03c4\u03b5\u03c1\u03bf \u03bf\u03bc\u03c9\u03c2 \u03b4\u03b5\u03bd \u03b5\u03c7\u03b5\u03b9 \u03ba\u03b1\u03bc\u03b9\u03b1 \u03c3\u03c7\u03b5\u03c3\u03b7 \u03bc\u03b5 \u03c4\u03b7\u03bd \u03c3\u03c7\u03bf\u03bb\u03b9\u03ba\u03b7 \u03c5\u03bb\u03b7, \u03b5\u03b9\u03b4\u03b9\u03ba\u03b1 \u03bc\u03b5 \u03b1\u03c5\u03c4\u03b7 \u03c4\u03c9\u03bd \u03c0\u03b1\u03bd\u03b5\u03bb\u03bb\u03b7\u03bd\u03b9\u03c9\u03bd, \u03b1\u03c6\u03bf\u03c5 \u03bc\u03b5\u03c4\u03b1 \u03c4\u03bf \u03bc\u03b5\u03c4\u03b1\u03c3\u03c7\u03b7\u03bc\u03b1\u03c4\u03b9\u03c3\u03bc\u03bf \u03b1\u03bd\u03b1\u03b3\u03b5\u03c4\u03b1\u03b9 \u03c3\u03b5 \u03bc\u03b9\u03b1 \u03c0\u03bf\u03bb\u03c5\u03c0\u03bb\u03bf\u03ba\u03b7 \u03b4\u03b9\u03bf\u03c6\u03b1\u03bd\u03c4\u03b9\u03ba\u03b7 \u03b5\u03be\u03b9\u03c3\u03c9\u03c3\u03b7 \u03bc\u03b5 \u03c4\u03b1 \u03c4\u03b5\u03c4\u03c1\u03b1\u03b3\u03c9\u03bd\u03b1 6 \u03b1\u03ba\u03b5\u03c1\u03b1\u03b9\u03c9\u03bd \u03bc\u03b5\u03c4\u03b1\u03b2\u03bb\u03b7\u03c4\u03c9\u03bd, \u03bf\u03c0\u03bf\u03c4\u03b5 \u03bf\u03c0\u03c9\u03c2 \u03ba\u03b1\u03b9 \u03bd\u03b1 \u03b3\u03b9\u03bd\u03b5\u03b9 \u03c7\u03c1\u03b5\u03b9\u03b1\u03b6\u03bf\u03bd\u03c4\u03b1\u03b9 \u03b3\u03bd\u03c9\u03c3\u03b5\u03b9\u03c2 \u03ba\u03b1\u03b9 \u03c4\u03b5\u03c7\u03bd\u03b1\u03c3\u03bc\u03b1\u03c4\u03b1 \u03b1\u03c0\u03bf \u03c4\u03b7 \u03b8\u03b5\u03c9\u03c1\u03b9\u03b1 \u03c4\u03c9\u03bd \u03b1\u03c1\u03b9\u03b8\u03bc\u03c9\u03bd, \u03b5\u03b3\u03c9 \u03bf\u03c4\u03b1\u03bd \u03b5\u03b9\u03bc\u03bf\u03c5\u03bd \u03c3\u03c4\u03bf \u03bb\u03c5\u03ba\u03b5\u03b9\u03bf (\u03c3\u03c4\u03bf \u03b4\u03b9\u03ba\u03bf \u03bc\u03bf\u03c5 \u03bb\u03c5\u03ba\u03b5\u03b9\u03bf \u03c4\u03bf\u03c5\u03bb\u03b1\u03c7\u03b9\u03c3\u03c4\u03bf\u03bd) \u03b4\u03b5\u03bd \u03b8\u03c5\u03bc\u03b1\u03bc\u03b5 \u03bd\u03b1 \u03b1\u03bd\u03b1\u03c6\u03b5\u03c1\u03b8\u03b7\u03ba\u03b5 \u03c0\u03bf\u03c4\u03b5 \u03bf\u03c5\u03c4\u03b5 \u03ba\u03b1\u03bd \u03bf \u03bf\u03c1\u03bf\u03c2 \"\u03b4\u03b9\u03b1\u03b9\u03c1\u03b5\u03c4\u03bf\u03c4\u03b7\u03c4\u03b1\" \u03c3\u03c4\u03b7\u03bd \u03c4\u03b1\u03be\u03b7...[/quote] emeis ton anaferame persi sthn 2 lykeiou kai malista kaname kai eisagwgi sto mod alla mexri kai tis diofadikes kaname akoma kai duskoles",
"Solution_11": "gia thn prwth askhsh exw sto mualo mou mia polu wraia trigwnometrikh lush! :D",
"Solution_12": "[quote=\"KapioPulsar\"][/quote][quote=\"NickNafplio\"][quote=\"KapioPulsar\"][quote=\"lefmast\"]\u03c4\u03bf \u03b4\u03b5\u03c5\u03c4\u03b5\u03c1\u03bf \u03bc\u03b5 \u03bc\u03b9\u03b1 \u03c0\u03c1\u03bf\u03c7\u03b5\u03b9\u03c1\u03b7 \u03bc\u03b1\u03c4\u03b9\u03b1 \u03b2\u03b3\u03b1\u03b9\u03bd\u03b5\u03b9 \u03bc\u03b5 \u03b1\u03c0\u03b5\u03b9\u03c1\u03b7 \u03ba\u03b1\u03b8\u03bf\u03b4\u03bf.....\u0395\u03c7\u03c9 \u03ba\u03b1\u03c0\u03bf\u03b9\u03b5\u03c2 \u03b1\u03bc\u03c6\u03b9\u03b2\u03bf\u03bb\u03b9\u03b5\u03c2 \u03b3\u03b9\u03b1 \u03c4\u03bf \u03c0\u03c1\u03c9\u03c4\u03bf \u03bf\u03bc\u03c9\u03c2...\u0391\u03bd \u03b8\u03b5\u03c3\u03bf\u03c5\u03bc\u03b5 \u03ba\u03b1\u03c0\u03bf\u03b9\u03b5\u03c2 \u03c4\u03b9\u03bc\u03b5\u03c2 \u03c3\u03c4\u03bf \u03c7 \u03ba\u03b1\u03b9 \u03bb\u03c5\u03c3\u03bf\u03c5\u03bc\u03b5 \u03c9\u03c2 \u03c0\u03c1\u03bf\u03c3 y \u03b4\u03b5\u03bd \u03b8\u03b1 \u03b5\u03c7\u03b5\u03b9 \u03c4\u03b7\u03bd \u03b9\u03b4\u03b9\u03b1 \u03c4\u03b9\u03bc\u03b7 \u03c4\u03bf \u0391.... :( :maybe:\u03a4\u03b9 \u03b5\u03bd\u03bd\u03bf\u03b5\u03b9\u03c2 \u03bd\u03b1 \u03c5\u03c0\u03bf\u03bb\u03bf\u03b3\u03b9\u03c3\u03c4\u03b5\u03b9????\u0391\u03bd \u03bc\u03c0\u03bf\u03c1\u03b5\u03b9\u03c2 \u03b5\u03be\u03b7\u03b3\u03b7\u03c3\u03b5 \u03c4\u03bf \u03bc\u03bf\u03c5 \u03bb\u03b9\u03b3\u03bf \u03c3\u03b5 \u03c0\u03b1\u03c1\u03b1\u03ba\u03b1\u03bb\u03c9....... :)[/quote]\n\nlipon katarxas sorry eprepe na diefkrinisw oti zitw thn elaxisth timh tou A , kai episis ola afta ta 8emata lunode polu wraia me gnwseis lukeiou (ulh panelinion) ara kai xwris mod hlia :P[/quote]\n\n\u03a4\u03b1 \u03c0\u03b1\u03c1\u03b1\u03bb\u03b5\u03c2 \u03c4\u03c9\u03c1\u03b1... \u03c4\u03bf \u03c0\u03c1\u03c9\u03c4\u03bf \u03ba\u03b1\u03b9 \u03c4\u03bf \u03c4\u03c1\u03b9\u03c4\u03bf \u03c0\u03c1\u03bf\u03b2\u03bb\u03b7\u03bc\u03b1 \u03b5\u03b9\u03bd\u03b1\u03b9 \u03bf\u03bd\u03c4\u03c9\u03c2 \u03c3\u03c7\u03bf\u03bb\u03b9\u03ba\u03b1, \u03b5\u03bd\u03c9 \u03c4\u03bf \u03c4\u03c1\u03b9\u03c4\u03bf \u03bc\u03c0\u03bf\u03c1\u03b5\u03b9 \u03bd\u03b1 \u03bc\u03c0\u03b5\u03b9 \u03ba\u03b1\u03b9 \u03c3\u03b5 \u03b5\u03bd\u03b1\u03bd \u03b4\u03b9\u03b1\u03b3\u03c9\u03bd\u03b9\u03c3\u03bc\u03bf \u03bc\u03b9\u03ba\u03c1\u03bf\u03c5 \u03b5\u03c0\u03b9\u03c0\u03b5\u03b4\u03bf\u03c5 (\u03bf\u03c0\u03c9\u03c2 \u03b5\u03b9\u03c0\u03b1 \u03ba\u03b1\u03b9 \u03c0\u03b1\u03c1\u03b1\u03c0\u03b1\u03bd\u03c9, \u03b5\u03b9\u03c7\u03b5 \u03bc\u03c0\u03b5\u03b9 \u03c3\u03b1\u03bd \u03b5\u03c5\u03ba\u03bf\u03bb\u03bf \u03b8\u03b5\u03bc\u03b1 \u03c3\u03b5 \u0391\u03c1\u03c7\u03b9\u03bc\u03b7\u03b4\u03b7 \u03c0\u03b1\u03bb\u03b9\u03b1). \u03a4\u03bf \u03b4\u03b5\u03c5\u03c4\u03b5\u03c1\u03bf \u03bf\u03bc\u03c9\u03c2 \u03b4\u03b5\u03bd \u03b5\u03c7\u03b5\u03b9 \u03ba\u03b1\u03bc\u03b9\u03b1 \u03c3\u03c7\u03b5\u03c3\u03b7 \u03bc\u03b5 \u03c4\u03b7\u03bd \u03c3\u03c7\u03bf\u03bb\u03b9\u03ba\u03b7 \u03c5\u03bb\u03b7, \u03b5\u03b9\u03b4\u03b9\u03ba\u03b1 \u03bc\u03b5 \u03b1\u03c5\u03c4\u03b7 \u03c4\u03c9\u03bd \u03c0\u03b1\u03bd\u03b5\u03bb\u03bb\u03b7\u03bd\u03b9\u03c9\u03bd, \u03b1\u03c6\u03bf\u03c5 \u03bc\u03b5\u03c4\u03b1 \u03c4\u03bf \u03bc\u03b5\u03c4\u03b1\u03c3\u03c7\u03b7\u03bc\u03b1\u03c4\u03b9\u03c3\u03bc\u03bf \u03b1\u03bd\u03b1\u03b3\u03b5\u03c4\u03b1\u03b9 \u03c3\u03b5 \u03bc\u03b9\u03b1 \u03c0\u03bf\u03bb\u03c5\u03c0\u03bb\u03bf\u03ba\u03b7 \u03b4\u03b9\u03bf\u03c6\u03b1\u03bd\u03c4\u03b9\u03ba\u03b7 \u03b5\u03be\u03b9\u03c3\u03c9\u03c3\u03b7 \u03bc\u03b5 \u03c4\u03b1 \u03c4\u03b5\u03c4\u03c1\u03b1\u03b3\u03c9\u03bd\u03b1 6 \u03b1\u03ba\u03b5\u03c1\u03b1\u03b9\u03c9\u03bd \u03bc\u03b5\u03c4\u03b1\u03b2\u03bb\u03b7\u03c4\u03c9\u03bd, \u03bf\u03c0\u03bf\u03c4\u03b5 \u03bf\u03c0\u03c9\u03c2 \u03ba\u03b1\u03b9 \u03bd\u03b1 \u03b3\u03b9\u03bd\u03b5\u03b9 \u03c7\u03c1\u03b5\u03b9\u03b1\u03b6\u03bf\u03bd\u03c4\u03b1\u03b9 \u03b3\u03bd\u03c9\u03c3\u03b5\u03b9\u03c2 \u03ba\u03b1\u03b9 \u03c4\u03b5\u03c7\u03bd\u03b1\u03c3\u03bc\u03b1\u03c4\u03b1 \u03b1\u03c0\u03bf \u03c4\u03b7 \u03b8\u03b5\u03c9\u03c1\u03b9\u03b1 \u03c4\u03c9\u03bd \u03b1\u03c1\u03b9\u03b8\u03bc\u03c9\u03bd, \u03b5\u03b3\u03c9 \u03bf\u03c4\u03b1\u03bd \u03b5\u03b9\u03bc\u03bf\u03c5\u03bd \u03c3\u03c4\u03bf \u03bb\u03c5\u03ba\u03b5\u03b9\u03bf (\u03c3\u03c4\u03bf \u03b4\u03b9\u03ba\u03bf \u03bc\u03bf\u03c5 \u03bb\u03c5\u03ba\u03b5\u03b9\u03bf \u03c4\u03bf\u03c5\u03bb\u03b1\u03c7\u03b9\u03c3\u03c4\u03bf\u03bd) \u03b4\u03b5\u03bd \u03b8\u03c5\u03bc\u03b1\u03bc\u03b5 \u03bd\u03b1 \u03b1\u03bd\u03b1\u03c6\u03b5\u03c1\u03b8\u03b7\u03ba\u03b5 \u03c0\u03bf\u03c4\u03b5 \u03bf\u03c5\u03c4\u03b5 \u03ba\u03b1\u03bd \u03bf \u03bf\u03c1\u03bf\u03c2 \"\u03b4\u03b9\u03b1\u03b9\u03c1\u03b5\u03c4\u03bf\u03c4\u03b7\u03c4\u03b1\" \u03c3\u03c4\u03b7\u03bd \u03c4\u03b1\u03be\u03b7...[/quote][quote=\"KapioPulsar\"] emeis ton anaferame persi sthn 2 lykeiou kai malista kaname kai eisagwgi sto mod alla mexri kai tis diofadikes kaname akoma kai duskoles[/quote]\r\n\r\n\u0397 \u03b8\u03b5\u03c9\u03c1\u03b9\u03b1 \u03b1\u03c1\u03b9\u03b8\u03bc\u03c9\u03bd \u03c0\u03bf\u03c5 \u03c5\u03c0\u03b1\u03c1\u03c7\u03b5\u03b9 \u03c3\u03c4\u03bf \u03b2\u03b9\u03b2\u03bb\u03b9\u03bf \u03c4\u03b7\u03c2 \u0392 \u039b\u03c5\u03ba\u03b5\u03b9\u03bf\u03c5 \u03b9\u03c3\u03c9\u03c2 \u03bd\u03b1 \u03ba\u03b1\u03bb\u03c5\u03c0\u03c4\u03b5\u03b9 2-3 \u03c3\u03b5\u03bb\u03b9\u03b4\u03b5\u03c2 \u03b1\u03c0\u03bf \u03b5\u03bd\u03b1 \u03b2\u03b9\u03b2\u03bb\u03b9\u03bf \u03b8\u03b5\u03c9\u03c1\u03b9\u03b1\u03c2 \u03b1\u03c1\u03b9\u03b8\u03bc\u03c9\u03bd \u03b3\u03b9\u03b1 \u03b4\u03b9\u03b1\u03b3\u03c9\u03bd\u03b9\u03c3\u03bc\u03bf\u03c5\u03c2, \u03bf\u03c0\u03bf\u03c4\u03b5 \u03b1\u03bd \u03ba\u03b1\u03bd\u03b1\u03c4\u03b5 \u03b4\u03c5\u03c3\u03ba\u03bf\u03bb\u03b1 \u03c0\u03c1\u03bf\u03b2\u03bb\u03b7\u03bc\u03b1\u03c4\u03b1 \u03b1\u03c1\u03b9\u03b8\u03bc\u03bf\u03b8\u03b5\u03c9\u03c1\u03b9\u03b1\u03c2 \u03bf\u03c0\u03c9\u03c2 \u03bb\u03b5\u03c2 \u03c4\u03bf\u03c4\u03b5 \u03c3\u03b9\u03b3\u03bf\u03c5\u03c1\u03b1 \u03b2\u03b3\u03b7\u03ba\u03b1\u03c4\u03b5 \u03c0\u03bf\u03bb\u03c5 \u03b5\u03be\u03c9 \u03b1\u03c0\u03bf \u03c4\u03b7\u03bd \u03c5\u03bb\u03b7 \u03c4\u03bf\u03c5 \u03c3\u03c7\u03bf\u03bb\u03b5\u03b9\u03bf\u03c5, \u03b1\u03bd \u03b5\u03b9\u03bd\u03b1\u03b9 \u03b5\u03c4\u03c3\u03b9 \u03c4\u03b1 \u03c0\u03c1\u03b1\u03b3\u03bc\u03b1\u03c4\u03b1 \u03c4\u03bf\u03c4\u03b5 \u03b1\u03be\u03b9\u03b6\u03b5\u03b9 \u03c3\u03c5\u03b3\u03c7\u03b1\u03c1\u03b7\u03c4\u03b7\u03c1\u03b9\u03b1 \u03bf \u03ba\u03b1\u03b8\u03b7\u03b3\u03b7\u03c4\u03b7\u03c2 \u03c3\u03b1\u03c2 \u03b4\u03b9\u03bf\u03c4\u03b9 \u03c3\u03c0\u03b1\u03bd\u03b9\u03b1 \u03ba\u03b1\u03b8\u03bf\u03bd\u03c4\u03b1\u03b9 \u03bf\u03b9 \u03ba\u03b1\u03b8\u03b7\u03b3\u03b7\u03c4\u03b5\u03c2 \u03bd\u03b1 \u03b4\u03b9\u03b4\u03b1\u03be\u03bf\u03c5\u03bd \u03c0\u03c1\u03b1\u03b3\u03bc\u03b1\u03c4\u03b1 \u03c0\u03bf\u03c5 \u03b4\u03b5\u03bd \u03c7\u03c1\u03b5\u03b9\u03b1\u03b6\u03bf\u03bd\u03c4\u03b1\u03b9 \u03c3\u03c4\u03b9\u03c2 \u03b5\u03be\u03b5\u03c4\u03b1\u03c3\u03b5\u03b9\u03c2... \u03c0\u03b1\u03bd\u03c4\u03bf\u03c2 \u03b8\u03b1 \u03b7\u03b8\u03b5\u03bb\u03b1 \u03c0\u03bf\u03bb\u03c5 \u03bd\u03b1 \u03b4\u03c9 \u03c4\u03b7 \u03bb\u03c5\u03c3\u03b7 \u03c7\u03c9\u03c1\u03b9\u03c2 \u03c4\u03b5\u03c4\u03c1\u03b1\u03b3\u03c9\u03bd\u03b9\u03ba\u03b1 \u03c5\u03c0\u03bf\u03bb\u03bf\u03b9\u03c0\u03b1 \u03c0\u03bf\u03c5 \u03bb\u03b5\u03c2 \u03bf\u03c4\u03b9 \u03b5\u03c7\u03b5\u03b9\u03c2 \u03b2\u03c1\u03b5\u03b9.",
"Solution_13": "[quote=\"NickNafplio\"][/quote][quote=\"KapioPulsar\"][/quote][quote=\"NickNafplio\"][quote=\"KapioPulsar\"][quote=\"lefmast\"]\u03c4\u03bf \u03b4\u03b5\u03c5\u03c4\u03b5\u03c1\u03bf \u03bc\u03b5 \u03bc\u03b9\u03b1 \u03c0\u03c1\u03bf\u03c7\u03b5\u03b9\u03c1\u03b7 \u03bc\u03b1\u03c4\u03b9\u03b1 \u03b2\u03b3\u03b1\u03b9\u03bd\u03b5\u03b9 \u03bc\u03b5 \u03b1\u03c0\u03b5\u03b9\u03c1\u03b7 \u03ba\u03b1\u03b8\u03bf\u03b4\u03bf.....\u0395\u03c7\u03c9 \u03ba\u03b1\u03c0\u03bf\u03b9\u03b5\u03c2 \u03b1\u03bc\u03c6\u03b9\u03b2\u03bf\u03bb\u03b9\u03b5\u03c2 \u03b3\u03b9\u03b1 \u03c4\u03bf \u03c0\u03c1\u03c9\u03c4\u03bf \u03bf\u03bc\u03c9\u03c2...\u0391\u03bd \u03b8\u03b5\u03c3\u03bf\u03c5\u03bc\u03b5 \u03ba\u03b1\u03c0\u03bf\u03b9\u03b5\u03c2 \u03c4\u03b9\u03bc\u03b5\u03c2 \u03c3\u03c4\u03bf \u03c7 \u03ba\u03b1\u03b9 \u03bb\u03c5\u03c3\u03bf\u03c5\u03bc\u03b5 \u03c9\u03c2 \u03c0\u03c1\u03bf\u03c3 y \u03b4\u03b5\u03bd \u03b8\u03b1 \u03b5\u03c7\u03b5\u03b9 \u03c4\u03b7\u03bd \u03b9\u03b4\u03b9\u03b1 \u03c4\u03b9\u03bc\u03b7 \u03c4\u03bf \u0391.... :( :maybe:\u03a4\u03b9 \u03b5\u03bd\u03bd\u03bf\u03b5\u03b9\u03c2 \u03bd\u03b1 \u03c5\u03c0\u03bf\u03bb\u03bf\u03b3\u03b9\u03c3\u03c4\u03b5\u03b9????\u0391\u03bd \u03bc\u03c0\u03bf\u03c1\u03b5\u03b9\u03c2 \u03b5\u03be\u03b7\u03b3\u03b7\u03c3\u03b5 \u03c4\u03bf \u03bc\u03bf\u03c5 \u03bb\u03b9\u03b3\u03bf \u03c3\u03b5 \u03c0\u03b1\u03c1\u03b1\u03ba\u03b1\u03bb\u03c9....... :)[/quote]\n\nlipon katarxas sorry eprepe na diefkrinisw oti zitw thn elaxisth timh tou A , kai episis ola afta ta 8emata lunode polu wraia me gnwseis lukeiou (ulh panelinion) ara kai xwris mod hlia :P[/quote]\n\n\u03a4\u03b1 \u03c0\u03b1\u03c1\u03b1\u03bb\u03b5\u03c2 \u03c4\u03c9\u03c1\u03b1... \u03c4\u03bf \u03c0\u03c1\u03c9\u03c4\u03bf \u03ba\u03b1\u03b9 \u03c4\u03bf \u03c4\u03c1\u03b9\u03c4\u03bf \u03c0\u03c1\u03bf\u03b2\u03bb\u03b7\u03bc\u03b1 \u03b5\u03b9\u03bd\u03b1\u03b9 \u03bf\u03bd\u03c4\u03c9\u03c2 \u03c3\u03c7\u03bf\u03bb\u03b9\u03ba\u03b1, \u03b5\u03bd\u03c9 \u03c4\u03bf \u03c4\u03c1\u03b9\u03c4\u03bf \u03bc\u03c0\u03bf\u03c1\u03b5\u03b9 \u03bd\u03b1 \u03bc\u03c0\u03b5\u03b9 \u03ba\u03b1\u03b9 \u03c3\u03b5 \u03b5\u03bd\u03b1\u03bd \u03b4\u03b9\u03b1\u03b3\u03c9\u03bd\u03b9\u03c3\u03bc\u03bf \u03bc\u03b9\u03ba\u03c1\u03bf\u03c5 \u03b5\u03c0\u03b9\u03c0\u03b5\u03b4\u03bf\u03c5 (\u03bf\u03c0\u03c9\u03c2 \u03b5\u03b9\u03c0\u03b1 \u03ba\u03b1\u03b9 \u03c0\u03b1\u03c1\u03b1\u03c0\u03b1\u03bd\u03c9, \u03b5\u03b9\u03c7\u03b5 \u03bc\u03c0\u03b5\u03b9 \u03c3\u03b1\u03bd \u03b5\u03c5\u03ba\u03bf\u03bb\u03bf \u03b8\u03b5\u03bc\u03b1 \u03c3\u03b5 \u0391\u03c1\u03c7\u03b9\u03bc\u03b7\u03b4\u03b7 \u03c0\u03b1\u03bb\u03b9\u03b1). \u03a4\u03bf \u03b4\u03b5\u03c5\u03c4\u03b5\u03c1\u03bf \u03bf\u03bc\u03c9\u03c2 \u03b4\u03b5\u03bd \u03b5\u03c7\u03b5\u03b9 \u03ba\u03b1\u03bc\u03b9\u03b1 \u03c3\u03c7\u03b5\u03c3\u03b7 \u03bc\u03b5 \u03c4\u03b7\u03bd \u03c3\u03c7\u03bf\u03bb\u03b9\u03ba\u03b7 \u03c5\u03bb\u03b7, \u03b5\u03b9\u03b4\u03b9\u03ba\u03b1 \u03bc\u03b5 \u03b1\u03c5\u03c4\u03b7 \u03c4\u03c9\u03bd \u03c0\u03b1\u03bd\u03b5\u03bb\u03bb\u03b7\u03bd\u03b9\u03c9\u03bd, \u03b1\u03c6\u03bf\u03c5 \u03bc\u03b5\u03c4\u03b1 \u03c4\u03bf \u03bc\u03b5\u03c4\u03b1\u03c3\u03c7\u03b7\u03bc\u03b1\u03c4\u03b9\u03c3\u03bc\u03bf \u03b1\u03bd\u03b1\u03b3\u03b5\u03c4\u03b1\u03b9 \u03c3\u03b5 \u03bc\u03b9\u03b1 \u03c0\u03bf\u03bb\u03c5\u03c0\u03bb\u03bf\u03ba\u03b7 \u03b4\u03b9\u03bf\u03c6\u03b1\u03bd\u03c4\u03b9\u03ba\u03b7 \u03b5\u03be\u03b9\u03c3\u03c9\u03c3\u03b7 \u03bc\u03b5 \u03c4\u03b1 \u03c4\u03b5\u03c4\u03c1\u03b1\u03b3\u03c9\u03bd\u03b1 6 \u03b1\u03ba\u03b5\u03c1\u03b1\u03b9\u03c9\u03bd \u03bc\u03b5\u03c4\u03b1\u03b2\u03bb\u03b7\u03c4\u03c9\u03bd, \u03bf\u03c0\u03bf\u03c4\u03b5 \u03bf\u03c0\u03c9\u03c2 \u03ba\u03b1\u03b9 \u03bd\u03b1 \u03b3\u03b9\u03bd\u03b5\u03b9 \u03c7\u03c1\u03b5\u03b9\u03b1\u03b6\u03bf\u03bd\u03c4\u03b1\u03b9 \u03b3\u03bd\u03c9\u03c3\u03b5\u03b9\u03c2 \u03ba\u03b1\u03b9 \u03c4\u03b5\u03c7\u03bd\u03b1\u03c3\u03bc\u03b1\u03c4\u03b1 \u03b1\u03c0\u03bf \u03c4\u03b7 \u03b8\u03b5\u03c9\u03c1\u03b9\u03b1 \u03c4\u03c9\u03bd \u03b1\u03c1\u03b9\u03b8\u03bc\u03c9\u03bd, \u03b5\u03b3\u03c9 \u03bf\u03c4\u03b1\u03bd \u03b5\u03b9\u03bc\u03bf\u03c5\u03bd \u03c3\u03c4\u03bf \u03bb\u03c5\u03ba\u03b5\u03b9\u03bf (\u03c3\u03c4\u03bf \u03b4\u03b9\u03ba\u03bf \u03bc\u03bf\u03c5 \u03bb\u03c5\u03ba\u03b5\u03b9\u03bf \u03c4\u03bf\u03c5\u03bb\u03b1\u03c7\u03b9\u03c3\u03c4\u03bf\u03bd) \u03b4\u03b5\u03bd \u03b8\u03c5\u03bc\u03b1\u03bc\u03b5 \u03bd\u03b1 \u03b1\u03bd\u03b1\u03c6\u03b5\u03c1\u03b8\u03b7\u03ba\u03b5 \u03c0\u03bf\u03c4\u03b5 \u03bf\u03c5\u03c4\u03b5 \u03ba\u03b1\u03bd \u03bf \u03bf\u03c1\u03bf\u03c2 \"\u03b4\u03b9\u03b1\u03b9\u03c1\u03b5\u03c4\u03bf\u03c4\u03b7\u03c4\u03b1\" \u03c3\u03c4\u03b7\u03bd \u03c4\u03b1\u03be\u03b7...[/quote][quote=\"KapioPulsar\"] emeis ton anaferame persi sthn 2 lykeiou kai malista kaname kai eisagwgi sto mod alla mexri kai tis diofadikes kaname akoma kai duskoles[/quote][quote=\"NickNafplio\"]\n\n\u0397 \u03b8\u03b5\u03c9\u03c1\u03b9\u03b1 \u03b1\u03c1\u03b9\u03b8\u03bc\u03c9\u03bd \u03c0\u03bf\u03c5 \u03c5\u03c0\u03b1\u03c1\u03c7\u03b5\u03b9 \u03c3\u03c4\u03bf \u03b2\u03b9\u03b2\u03bb\u03b9\u03bf \u03c4\u03b7\u03c2 \u0392 \u039b\u03c5\u03ba\u03b5\u03b9\u03bf\u03c5 \u03b9\u03c3\u03c9\u03c2 \u03bd\u03b1 \u03ba\u03b1\u03bb\u03c5\u03c0\u03c4\u03b5\u03b9 2-3 \u03c3\u03b5\u03bb\u03b9\u03b4\u03b5\u03c2 \u03b1\u03c0\u03bf \u03b5\u03bd\u03b1 \u03b2\u03b9\u03b2\u03bb\u03b9\u03bf \u03b8\u03b5\u03c9\u03c1\u03b9\u03b1\u03c2 \u03b1\u03c1\u03b9\u03b8\u03bc\u03c9\u03bd \u03b3\u03b9\u03b1 \u03b4\u03b9\u03b1\u03b3\u03c9\u03bd\u03b9\u03c3\u03bc\u03bf\u03c5\u03c2, \u03bf\u03c0\u03bf\u03c4\u03b5 \u03b1\u03bd \u03ba\u03b1\u03bd\u03b1\u03c4\u03b5 \u03b4\u03c5\u03c3\u03ba\u03bf\u03bb\u03b1 \u03c0\u03c1\u03bf\u03b2\u03bb\u03b7\u03bc\u03b1\u03c4\u03b1 \u03b1\u03c1\u03b9\u03b8\u03bc\u03bf\u03b8\u03b5\u03c9\u03c1\u03b9\u03b1\u03c2 \u03bf\u03c0\u03c9\u03c2 \u03bb\u03b5\u03c2 \u03c4\u03bf\u03c4\u03b5 \u03c3\u03b9\u03b3\u03bf\u03c5\u03c1\u03b1 \u03b2\u03b3\u03b7\u03ba\u03b1\u03c4\u03b5 \u03c0\u03bf\u03bb\u03c5 \u03b5\u03be\u03c9 \u03b1\u03c0\u03bf \u03c4\u03b7\u03bd \u03c5\u03bb\u03b7 \u03c4\u03bf\u03c5 \u03c3\u03c7\u03bf\u03bb\u03b5\u03b9\u03bf\u03c5, \u03b1\u03bd \u03b5\u03b9\u03bd\u03b1\u03b9 \u03b5\u03c4\u03c3\u03b9 \u03c4\u03b1 \u03c0\u03c1\u03b1\u03b3\u03bc\u03b1\u03c4\u03b1 \u03c4\u03bf\u03c4\u03b5 \u03b1\u03be\u03b9\u03b6\u03b5\u03b9 \u03c3\u03c5\u03b3\u03c7\u03b1\u03c1\u03b7\u03c4\u03b7\u03c1\u03b9\u03b1 \u03bf \u03ba\u03b1\u03b8\u03b7\u03b3\u03b7\u03c4\u03b7\u03c2 \u03c3\u03b1\u03c2 \u03b4\u03b9\u03bf\u03c4\u03b9 \u03c3\u03c0\u03b1\u03bd\u03b9\u03b1 \u03ba\u03b1\u03b8\u03bf\u03bd\u03c4\u03b1\u03b9 \u03bf\u03b9 \u03ba\u03b1\u03b8\u03b7\u03b3\u03b7\u03c4\u03b5\u03c2 \u03bd\u03b1 \u03b4\u03b9\u03b4\u03b1\u03be\u03bf\u03c5\u03bd \u03c0\u03c1\u03b1\u03b3\u03bc\u03b1\u03c4\u03b1 \u03c0\u03bf\u03c5 \u03b4\u03b5\u03bd \u03c7\u03c1\u03b5\u03b9\u03b1\u03b6\u03bf\u03bd\u03c4\u03b1\u03b9 \u03c3\u03c4\u03b9\u03c2 \u03b5\u03be\u03b5\u03c4\u03b1\u03c3\u03b5\u03b9\u03c2... \u03c0\u03b1\u03bd\u03c4\u03bf\u03c2 \u03b8\u03b1 \u03b7\u03b8\u03b5\u03bb\u03b1 \u03c0\u03bf\u03bb\u03c5 \u03bd\u03b1 \u03b4\u03c9 \u03c4\u03b7 \u03bb\u03c5\u03c3\u03b7 \u03c7\u03c9\u03c1\u03b9\u03c2 \u03c4\u03b5\u03c4\u03c1\u03b1\u03b3\u03c9\u03bd\u03b9\u03ba\u03b1 \u03c5\u03c0\u03bf\u03bb\u03bf\u03b9\u03c0\u03b1 \u03c0\u03bf\u03c5 \u03bb\u03b5\u03c2 \u03bf\u03c4\u03b9 \u03b5\u03c7\u03b5\u03b9\u03c2 \u03b2\u03c1\u03b5\u03b9.[/quote]\r\n\r\nloipon gia to deftero kai egw me mod to elusa alla mou eipe o ka8igiths mou (pou mou thn eixe pei o idos ) oti lunete kai me aples gnwseis giafto kai thn anarthsa bas kai thn brei kapios afthn thn polupo8eth lush :S\r\n\r\ngia to prwto twra h lush mou einai trigwnometrikh :to arxiko ginete $ \\frac {1}{x^2} + \\frac {1}{y^2} = 1$ara yparxei gwnia b anoikoi (0,\u03c0/2) = >1/|x|=sin(b) 1/|y|=cos(b) ara A=(1+1/sin(b))(1+1/cos(b))= 1+(sin(b)+cos(b)+1)/(sin(b)cos(b))=1+2(2^(1/2)*sin(\u03c0/4+b)+1)/(sin(2b))=1+2^(3/2)*(sin(\u03c0/4+b))/sin(2b))+2/sin(2b)\r\npou pernei thn elaxisth timh otan to sin2b einai megisto dld gia b=\u03c0/4 opou $ A=3+2\\sqrt{2}$",
"Solution_14": "\u03a4\u03bf \u03b4\u03b5\u03cd\u03c4\u03b5\u03c1\u03bf \u03b8\u03ad\u03bc\u03b1 \u03b5\u03af\u03bd\u03b1\u03b9 \u03b1\u03c0\u03cc \u03c4\u03b7 \u039c\u03b5\u03c3\u03bf\u03b3\u03b5\u03b9\u03ac\u03b4\u03b1 \u03c4\u03bf\u03c5 2003 (\u03c0\u03c1\u03ce\u03c4\u03bf \u03b8\u03ad\u03bc\u03b1). \u03a4\u03bf \u03c4\u03c1\u03af\u03c4\u03bf \u03b8\u03ad\u03bc\u03b1 \u03b5\u03af\u03bd\u03b1\u03b9 \u03cc\u03bd\u03c4\u03c9\u03c2 \u03b1\u03c0\u03cc \u03c4\u03bf\u03bd \u03b1\u03c1\u03c7\u03b9\u03bc\u03ae\u03b4\u03b7 \u03c4\u03bf\u03c5 2005 \u03c0\u03c1\u03ce\u03c4\u03bf \u03b8\u03ad\u03bc\u03b1 \u03ba\u03b1\u03b9 \u03c0\u03ac\u03bb\u03b9. \u0391\u03bb\u03bb\u03ac \u03bc\u03b7\u03bd \u03c4\u03c1\u03b5\u03bb\u03b1\u03b8\u03bf\u03cd\u03bc\u03b5, \u03c4\u03bf \u03cc\u03c4\u03b9 \u03bb\u03cd\u03bd\u03bf\u03bd\u03c4\u03b1\u03b9 \u03bc\u03b5 \u03c3\u03c7\u03bf\u03bb\u03b9\u03ba\u03ad\u03c2 \u03b3\u03bd\u03ce\u03c3\u03b5\u03b9\u03c2 \u03b4\u03b5\u03bd \u03c3\u03b7\u03bc\u03b1\u03af\u03bd\u03b5\u03b9 \u03cc\u03c4\u03b9 \u03b5\u03af\u03bd\u03b1\u03b9 \u03c3\u03c7\u03bf\u03bb\u03b9\u03ba\u03ac ! \u03a0\u03c7 \u03c4\u03bf 6\u03bf \u03b8\u03ad\u03bc\u03b1 \u03c4\u03b7\u03c2 \u03c6\u03b5\u03c4\u03b9\u03bd\u03ae\u03c2 \u0399\u039c\u039f \u03b5\u03c0\u03b1\u03b3\u03c9\u03b3\u03ae \u03ae\u03b8\u03b5\u03bb\u03b5, \u03c0\u03bf\u03c5 \u03b4\u03b9\u03b4\u03ac\u03c3\u03ba\u03b5\u03c4\u03b1\u03b9 \u03c3\u03c4\u03bf \u03bb\u03cd\u03ba\u03b5\u03b9\u03bf ... \u03ac\u03c1\u03b1 \u03ae\u03c4\u03b1\u03bd \u03c3\u03c7\u03bf\u03bb\u03b9\u03ba\u03cc ? \r\n\u039a\u03b1\u03b9 \u03bc\u03b9\u03b1 \u03c0\u03b1\u03c1\u03b1\u03c4\u03ae\u03c1\u03b7\u03c3\u03b7 \u03b1\u03ba\u03cc\u03bc\u03b1... \u03a0\u03bf\u03c3\u03c4 \u03c4\u03bf\u03c5 style \"\u03c0\u03b9\u03c3\u03c4\u03b5\u03cd\u03c9 \u03cc\u03c4\u03b9 \u03bb\u03cd\u03bd\u03b5\u03c4\u03b1\u03b9 \u03bc\u03b5 cauchy-swartz \u03b5\u03cd\u03ba\u03bf\u03bb\u03b1\", \u03c4\u03b1 \u03b8\u03b5\u03c9\u03c1\u03ce \u03b1\u03bd\u03bf\u03cd\u03c3\u03b9\u03b1 \u03ba\u03b1\u03b9 spam",
"Solution_15": "[quote=\"silouan\"]\u03a4\u03bf \u03b4\u03b5\u03cd\u03c4\u03b5\u03c1\u03bf \u03b8\u03ad\u03bc\u03b1 \u03b5\u03af\u03bd\u03b1\u03b9 \u03b1\u03c0\u03cc \u03c4\u03b7 \u039c\u03b5\u03c3\u03bf\u03b3\u03b5\u03b9\u03ac\u03b4\u03b1 \u03c4\u03bf\u03c5 2003 (\u03c0\u03c1\u03ce\u03c4\u03bf \u03b8\u03ad\u03bc\u03b1). \u03a4\u03bf \u03c4\u03c1\u03af\u03c4\u03bf \u03b8\u03ad\u03bc\u03b1 \u03b5\u03af\u03bd\u03b1\u03b9 \u03cc\u03bd\u03c4\u03c9\u03c2 \u03b1\u03c0\u03cc \u03c4\u03bf\u03bd \u03b1\u03c1\u03c7\u03b9\u03bc\u03ae\u03b4\u03b7 \u03c4\u03bf\u03c5 2005 \u03c0\u03c1\u03ce\u03c4\u03bf \u03b8\u03ad\u03bc\u03b1 \u03ba\u03b1\u03b9 \u03c0\u03ac\u03bb\u03b9. \u0391\u03bb\u03bb\u03ac \u03bc\u03b7\u03bd \u03c4\u03c1\u03b5\u03bb\u03b1\u03b8\u03bf\u03cd\u03bc\u03b5, \u03c4\u03bf \u03cc\u03c4\u03b9 \u03bb\u03cd\u03bd\u03bf\u03bd\u03c4\u03b1\u03b9 \u03bc\u03b5 \u03c3\u03c7\u03bf\u03bb\u03b9\u03ba\u03ad\u03c2 \u03b3\u03bd\u03ce\u03c3\u03b5\u03b9\u03c2 \u03b4\u03b5\u03bd \u03c3\u03b7\u03bc\u03b1\u03af\u03bd\u03b5\u03b9 \u03cc\u03c4\u03b9 \u03b5\u03af\u03bd\u03b1\u03b9 \u03c3\u03c7\u03bf\u03bb\u03b9\u03ba\u03ac ! \u03a0\u03c7 \u03c4\u03bf 6\u03bf \u03b8\u03ad\u03bc\u03b1 \u03c4\u03b7\u03c2 \u03c6\u03b5\u03c4\u03b9\u03bd\u03ae\u03c2 \u0399\u039c\u039f \u03b5\u03c0\u03b1\u03b3\u03c9\u03b3\u03ae \u03ae\u03b8\u03b5\u03bb\u03b5, \u03c0\u03bf\u03c5 \u03b4\u03b9\u03b4\u03ac\u03c3\u03ba\u03b5\u03c4\u03b1\u03b9 \u03c3\u03c4\u03bf \u03bb\u03cd\u03ba\u03b5\u03b9\u03bf ... \u03ac\u03c1\u03b1 \u03ae\u03c4\u03b1\u03bd \u03c3\u03c7\u03bf\u03bb\u03b9\u03ba\u03cc ? \n\u039a\u03b1\u03b9 \u03bc\u03b9\u03b1 \u03c0\u03b1\u03c1\u03b1\u03c4\u03ae\u03c1\u03b7\u03c3\u03b7 \u03b1\u03ba\u03cc\u03bc\u03b1... \u03a0\u03bf\u03c3\u03c4 \u03c4\u03bf\u03c5 style \"\u03c0\u03b9\u03c3\u03c4\u03b5\u03cd\u03c9 \u03cc\u03c4\u03b9 \u03bb\u03cd\u03bd\u03b5\u03c4\u03b1\u03b9 \u03bc\u03b5 cauchy-swartz \u03b5\u03cd\u03ba\u03bf\u03bb\u03b1\", \u03c4\u03b1 \u03b8\u03b5\u03c9\u03c1\u03ce \u03b1\u03bd\u03bf\u03cd\u03c3\u03b9\u03b1 \u03ba\u03b1\u03b9 spam[/quote] oxi da3i , egw eipa apla oti 8elei mexri gnwseis sxoleiou ade kai tetragwnika upolipa afto apla",
"Solution_16": "[quote=\"KapioPulsar\"] oxi da3i , egw eipa apla oti 8elei mexri gnwseis sxoleiou ade kai tetragwnika upolipa afto apla[/quote]\r\n\r\nO\u03c0\u03c9\u03c2 \u03c0\u03bf\u03bb\u03cd \u03b5\u03cd\u03c3\u03c4\u03bf\u03c7\u03b1 \u03c0\u03b1\u03c1\u03b1\u03c4\u03ae\u03c1\u03b7\u03c3\u03b5 \u03bf \u03a3\u03b9\u03bb\u03bf\u03c5\u03b1\u03bd\u03cc\u03c2 \u03c0\u03b1\u03c1\u03b1\u03c0\u03ac\u03bd\u03c9 \u03c4\u03bf \u03b3\u03b5\u03b3\u03bf\u03bd\u03ce\u03c2 \u03cc\u03c4\u03b9 \u03c3\u03c4\u03b7 \u03bb\u03cd\u03c3\u03b7 \u03bc\u03b9\u03b1\u03c2 \u03ac\u03c3\u03ba\u03b7\u03c3\u03b7\u03c2 ''\u03c7\u03c1\u03b5\u03b9\u03ac\u03b6\u03bf\u03bd\u03c4\u03b1\u03b9'' \u03b3\u03bd\u03ce\u03c3\u03b5\u03b9\u03c2 \u03bb\u03c5\u03ba\u03b5\u03af\u03bf\u03c5 \u03b4\u03b5\u03bd \u03c3\u03b7\u03bc\u03ad\u03bd\u03b5\u03b9 \u03cc\u03c4\u03b9 \u03b5\u03af\u03bd\u03b1\u03b9 \u03ba\u03b1\u03b9 \u03bb\u03c5\u03ba\u03b5\u03b9\u03b1\u03ba\u03bf\u03cd \u03b5\u03c0\u03b9\u03c0\u03ad\u03b4\u03bf\u03c5.....\u03a0.\u03c7. \u03c3\u03c4\u03b7 \u03b3\u03b5\u03c9\u03bc\u03b5\u03c4\u03c1\u03af\u03b1 \u03b1\u03bd \u03ad\u03c7\u03b5\u03b9\u03c2 \u03c0\u03b1\u03c1\u03b1\u03c4\u03b7\u03c1\u03ae\u03c3\u03b5\u03b9 \u03bf\u03b9 \u03c0\u03b9\u03bf \u03c0\u03bf\u03bb\u03bb\u03ad\u03c2 \u03b1\u03c3\u03ba\u03ae\u03c3\u03b5\u03b9\u03c2 \u03c3\u03c4\u03bf\u03c5\u03c2 \u03b4\u03b9\u03b1\u03b3\u03c9\u03bd\u03b9\u03c3\u03bc\u03bf\u03cd\u03c2 \u03c3\u03c4\u03c5\u03c1\u03af\u03b6\u03bf\u03bd\u03c4\u03b1\u03b9 \u03c3\u03c4\u03b1 ''\u03c3\u03c7\u03bf\u03bb\u03b9\u03ba\u03bf\u03cd \u03b5\u03c0\u03b9\u03c0\u03ad\u03b4\u03bf\u03c5''...(\u03c0\u03cc\u03c3\u03bf \u03bc\u03ac\u03bb\u03bf\u03bd \u03b1\u03bd \u03b1\u03bd\u03b1\u03bb\u03bf\u03b3\u03b9\u03c3\u03c4\u03b5\u03af\u03c2 \u03cc\u03c4\u03b9 \u03bc\u03ad\u03c3\u03b1 \u03c3\u03c4\u03bf \u03c3\u03c7\u03bf\u03bb\u03b9\u03ba\u03cc \u03b2\u03b9\u03b2\u03bb\u03af\u03bf \u03b1\u03bd\u03b1\u03c6\u03ad\u03c1\u03bf\u03bd\u03c4\u03b1\u03b9 \u03c4\u03b1 \u03b8\u03b5\u03c9\u03c1\u03ae\u03bc\u03b1\u03c4\u03b1 stewart,ceva \u03ba\u03bb\u03c0).\u0391\u03c5\u03c4\u03cc \u03cc\u03bc\u03c9\u03c2 \u03b4\u03b5\u03bd \u03c3\u03b7\u03bc\u03ad\u03bd\u03b5\u03b9 \u03cc\u03c4\u03b9 \u03bf\u03b9 \u03b1\u03c3\u03ba\u03ae\u03c3\u03b5\u03b9\u03c2 \u03c0\u03bf\u03c5 \u03bc\u03c0\u03ad\u03bd\u03bf\u03c5\u03bd \u03c3\u03c4\u03bf\u03c5\u03c2 \u03b4\u03b9\u03b1\u03b3\u03c9\u03bd\u03b9\u03c3\u03bc\u03bf\u03cd\u03c2 \u03b5\u03af\u03bd\u03b1\u03b9 ''\u03c3\u03c7\u03bf\u03bb\u03b9\u03ba\u03bf\u03cd'' \u03b5\u03c0\u03b9\u03c0\u03ad\u03b4\u03bf\u03c5....\r\n\r\n\u0397 \u03bc\u03bf\u03bd\u03b1\u03b4\u03b9\u03ba\u03ae \u03ac\u03c3\u03ba\u03b7\u03c3\u03b7 \u03b1\u03c0\u03cc \u03b1\u03c5\u03c4\u03ad\u03c2 \u03c0\u03bf\u03c5 \u03b1\u03bd\u03ad\u03b2\u03b1\u03c3\u03b5\u03c2 \u03c0\u03bf\u03c5 \u03b8\u03b1 \u03bc\u03c0\u03bf\u03c1\u03bf\u03cd\u03c3\u03b5 \u03bd\u03b1 \u03c7\u03b1\u03c1\u03b1\u03ba\u03c4\u03b9\u03c1\u03b9\u03c3\u03c4\u03b5\u03af \u03ad\u03c4\u03c3\u03b9 \u03b5\u03af\u03bd\u03b1\u03b9 \u03b7 \u03c4\u03c1\u03af\u03c4\u03b7 :) \r\n\r\n\u03a6\u03b9\u03bb\u03b9\u03ba\u03ac\r\n\u0394\u03b7\u03bc\u03ae\u03c4\u03c1\u03b7\u03c2"
}
{
"Tag": [
"trigonometry",
"function",
"calculus",
"calculus computations"
],
"Problem": "All I know is arcsin(sin x)=x. How to calculate arcsin(sin x/2), arctan(tan(15*t), etc.?\r\nThe result should not contain trigonometrical functions.\r\n\r\nAnd I've learned calculus.",
"Solution_1": "If you know that $ \\arcsin\\left(\\sin\\left(x\\right)\\right)\\equal{}x$, then how is $ \\arcsin\\left(\\sin\\left(\\frac{x}{2}\\right)\\right)$ different? Think of $ \\frac{x}{2}$ as a single unit. So...$ \\arcsin\\left(\\sin\\left(\\frac{x}{2}\\right)\\right)\\equal{}\\frac{x}{2}$.",
"Solution_2": "But be careful: these things are only true within certain limited domains. $ \\arcsin\\left(\\sin\\left(\\frac{x}{2}\\right)\\right)\\equal{}\\frac{x}{2}$ is only true for $ \\minus{}\\pi\\le x\\le\\pi,$ for instance. That function is defined for all real $ x,$ and gives a periodic ziz-zag line graph.",
"Solution_3": "sorry I meant to say arcsin(cos x/2), things like that\r\nhelp",
"Solution_4": "use sinx=cos(pi-x/2)"
}
{
"Tag": [
"calculus",
"integration",
"algebra",
"polynomial",
"real analysis",
"real analysis unsolved"
],
"Problem": "$f(x)$ is a polynomial with the degree of $n$,such that $\\int_0^1 x^kf(x)dx=0(k=1,2,\\cdot,n)$\r\nprove:$\\int_0^1f^2(x)dx=(n+1)^2(\\int_0^1f(x)dx)^2$",
"Solution_1": "hmm. Let $f(x)=a_nx^n + a_{n-1}x^{n-1} + \\cdots + a_1 x + a_0$\r\n\r\nThus\r\n$\\int_0^1 x^k f(x)dx = \\frac{a_n}{n+k+1} +\\cdots+ \\frac{a_1}{k+2}+\\frac{a_0}{k+1} = \\frac{Q(x)}{(k+1)(k+2)\\cdots(n+k+1)}$ (1)\r\n, where $Q(k)$ is a polynomial of degree $n$.\r\n\r\nAnd we have $Q(k)=0$ for $k=1,2,\\dots,n$. Hence\r\n\r\n$Q(k)=c(k-1)(k-2)\\cdots(k-n)$ (2)\r\n\r\nSubstitute (2) into (1), and multiply $k+1$ in both sides. Let $k=-1$. We obtain that\r\n$a_0 = (-1)^n(n+1)c$ (3)\r\nSubstitute (2) into (1) and ket $k=0$ then we have $\\int_0^1 f(x)dx = \\frac{(-1)^n}{n+1}c$ (4)\r\n\r\nCombine (3) and (4), eliminating $c$, we get $a_0=(n+1)^2\\int_0^1 f(x)dx$.\r\n\r\nNow we are going to prove that $\\int_0^1 f^2(x)dx = a_0\\int_0^1 f(x)dx$. This is quite straight-forward.\r\n$\\int_0^1 f^2(x)dx = \\int_0^1 (a_nx^n +\\cdots +a_1x + a_0)f(x)dx = a_0\\int_0^1f(x)dx$.",
"Solution_2": "very nice,thank you.liyi"
}
{
"Tag": [
"real analysis",
"real analysis solved"
],
"Problem": "Compute the limit of x_n, where \r\nx_n=([sqrt(1)]+[sqrt(3)]+[sqrt(5)]+..+[sqrt(4n^2+4n-1)])/n^3.\r\n\r\nwhere [x]=x-{x}.\r\n\r\ncheers! :D",
"Solution_1": "Take a_n= \\sum (k=1, 2n^2+2n) [ \\sqrt 2k-1]. We would like to apply Cesaro-Stolz. But a_(n+1)-a_n= \\sum (k=2n^2+2n+1, 2n^2+6n+4) [ \\sqrt 2k-1]. Observe that each such integer part is 2n+1 or 2n+2. Thus, a_(n+1)-a_n is between (4n+4)(2n+1) and (4n+4)(2n+2). Thus, a_(n+1)-a_n divided by n^2 is about 8. Thus, a_n/n^3 is 8/3 when n is large.",
"Solution_2": "suppa duppa! :D:D"
}
{
"Tag": [
"geometry",
"analytic geometry",
"circumcircle",
"symmetry",
"parameterization",
"2009 USAMO",
"USAMO"
],
"Problem": "Given circles $ \\omega_1$ and $ \\omega_2$ intersecting at points $ X$ and $ Y$, let $ \\ell_1$ be a line through the center of $ \\omega_1$ intersecting $ \\omega_2$ at points $ P$ and $ Q$ and let $ \\ell_2$ be a line through the center of $ \\omega_2$ intersecting $ \\omega_1$ at points $ R$ and $ S$. Prove that if $ P, Q, R$ and $ S$ lie on a circle then the center of this circle lies on line $ XY$.",
"Solution_1": "[hide=\"Solution\"]Let $ PQ$ and $ RS$ meet at $ Z$, let the centers of $ \\omega_{1}$ and $ \\omega_{2}$ be $ O_{1}$ and $ O_{2}$, respectively, and let the center of the circle containing $ P,Q,R,S$ be $ O$.\n\nFirst, observe that by power of a point, $ (PZ)(ZQ)\\equal{}(RZ)(ZS)$. But the LHS is the power of $ Z$ with respect to $ \\omega_{2}$, while the RHS is the power of $ Z$ with respect to $ \\omega_{1}$. Since these are equal, $ Z$ is on the radical axis of $ \\omega_{1}$ and $ \\omega_{2}$, that is, $ X,Y,Z$ are collinear.\n\nIt is well known that $ XY\\perp O_{1}O_{2}$ (this is easy to prove with congruent triangles). Note that $ O$ and $ O_{1}$ are both on the perpendicular bisector of $ RS$, so $ OO_{1}\\perp RS$ and $ OO_{1}\\perp O_{2}S$. Similarly, $ OO_{2}\\perp O_{1}Q$. Thus, $ O_{1}Q\\cap O_{2}S$ is the orthocenter of $ \\Delta OO_{1}O_{2}$, but this is just $ Z$. Thus, $ OZ\\perp O_{1}O_{2}$. But we have $ O_{1}O_{2}\\perp XY$ and $ Z$ is on $ XY$, so thus $ O$ is also on $ XY$, as desired.[/hide]",
"Solution_2": "Let w be circle passing through PQRS. Let radius of $ w_1,w_2,w$ be $ r_1,r_2,r$ respectively, center be $ O_1,O_2,O$ respectively. Now power of $ O_1$ according to circle w is $ O_1P$*$ O_1Q$=$ O_1O^2\\minus{}r^2$. But $ O_1P$*$ O_1Q$=$ O_1O_2^2\\minus{}{r_2}^2$, so $ O_1O^2\\equal{}r^2\\minus{}{r_2}^2\\plus{}O_1O_2^2$. So the power of O according to $ w_1$ is $ O_1O^2\\minus{}{r_1}^2\\equal{}r^2\\minus{}{r_2}^2\\plus{}O_1O_2^2\\minus{}{r_1}^2$. Similarly, O has same power to $ w_2$. So O lies on radical axis XY.",
"Solution_3": "let the centres of the circles be O1,O2,O3. (O3 being the circle around PQRS)\r\n\r\nlet O1 be (x1,y1), O2 be (x2,y2), and O3 (x3,y3)\r\nlet the radiuses of the three circles be r1,r2,r3.\r\n\r\nwe construct 3 equations, for each circle.\r\n\r\nby subtracting the equation of O3 from O1, we get the equation of line RS. -(1)\r\nby subtracting the equation of O3 from O2, we get the equation of line PQ. -(2)\r\nby subtracting the equation of O2 from O1, we get the equation of line XY. -(3)\r\n\r\nbut we have another condition: O1 is on PQ and O2 is on RS.\r\n\r\nwe put in the coordinates of O1 into (1), and the coordinated of O2 into (2). then we add them (or subtract them)\r\n\r\nwe get an equation that says O3 must be on line (3), which is XY.\r\n\r\nCAN SOMEBODY VERIFY MY SOLUTION?",
"Solution_4": "[quote= 'CatalystOfNostalgia']But the LHS is the power of Z with respect to w_2, while the RHS is the power of Z with respect to w_1. Since these are equal, Z is on the radical axis of w_1 and w_2. \r\n[/quote]\r\n\r\n\r\n[quote= 'nine34']we construct 3 equations, for each circle. \r\n\r\nby subtracting the equation of O3 from O1, we get the equation of line RS[/quote]\r\n\r\nHow would you prove either of these assertions? Thanks for the help.",
"Solution_5": "hmm I thought this was an odd choice for a #1 .. my writeup was done within 15 minutes of the start, but I'm pretty sure I would not have been able to solve this last year.",
"Solution_6": "My general method:\r\nSuppose $ Z$ was the intersection of $ PQ$ and $ RS$, and $ D$ was the intersection of $ XY$ and $ O_1O_2$, where $ O_1$ and $ O_2$ are the centers of $ \\omega_1$ and $ \\omega_2$ respectively. Also suppose $ C$ is the center of the circle through $ PQRS$.\r\n\r\nStep 1: Prove $ C,Z,D$ are collinear. Check.\r\nStep 2: Prove $ Z$ is on $ XY$. Uhhhhhh..... Nope, I had no idea what a radical axis was during the test :( Too bad, it would've been trivial otherwise.\r\n\r\nAnd plus I missed the parallel case like so many other people T-T. So at most a 2 for me on problem 1.",
"Solution_7": "yeah, a easy one, just a radical axis and an orthocentre, solved within 40 minutes.",
"Solution_8": "[quote=\"bzauzmer\"]\n\n[quote= 'nine34']we construct 3 equations, for each circle. \n\nby subtracting the equation of O3 from O1, we get the equation of line RS[/quote]\n\nHow would you prove either of these assertions? Thanks for the help.[/quote]\r\n\r\nWell, since R and S are both on O3 and O1, by subtracting the equation of O3 from O1, we get a equation with both R and S on them.\r\n\r\nbut this equation is linear, so it is a line, and there is only one line that can go thru R and S. (which is line RS)",
"Solution_9": "[quote=\"Yongyi781\"]My general method:\nAnd plus I missed the parallel case like so many other people T-T. So at most a 2 for me on problem 1.[/quote]\r\n\r\nWhat was the parallel case? :o",
"Solution_10": "Let the center of $\\omega_1$ be $ A$, and the center of $\\omega_2$ be $ B$. Let the circumcenter of $ PQRS$ be $ C$. Let $ r_1, r_2, r_3$ be the radii of $ A, B, C$ respectively. $ PQ$ is the radical axis of circles $ A$ and $ C$, $ RS$ is the radical axis of circles $ C$ and $ B$, and $ XY$ is the radical axis of circles $ A$ and $ B$, because the circles intersect and it is a well known fact that the radical axis of intersecting circles goes through the intersection points.\nBy definition of radical axis, the power of $ A$ with respect to circles $ B$ and $ C$, and similarly for the points $ B$ and $ C$ with respect to the other 2 circles, we get:\n$ AC^2 \\minus{} r_3^2 \\equal{} AB^2 \\minus{} r_1^2$\n$ AB^2 \\minus{} r_2^2 \\equal{} AC^2 \\minus{} r_3^2$\nAdding the 2 equations gives us:\n$ AC^2 \\minus{} r_3^2 \\plus{} AB^2 \\minus{} r_2^2 \\equal{} AB^2 \\minus{} r_1^2 \\plus{} AC^2 \\minus{} r_3^2$\n$ BC^2 \\minus{} r_2^2 \\equal{} AC^2 \\minus{} r_1^2$\nThen by defintion,$ C$ has equal power with respect to circles $ A$ and $ B$, so $ C$ lies on the radical axis of circles $ A$ and $ B$, which is $ XY$, so we are done.\n\nBasically what greentreeroad did, use the powers being equal to find the third circle center having the same power with respect to the other 2 circles.\n\nThat takes care of the parallel case since the lengths aren't affected if triangle ABC is degenerate, and the powers being equal still holds.",
"Solution_11": "Letting the circle containing $ P,Q,R,S$ have center $ C$, I proved that $ C$ had a power of a point equal with respect to $ \\omega_1$ and $ \\omega_2$. But instead of citing radical axes (I only have a vague idea about them, mostly that it is the line $ XY$, so I was careful), I said to consider the extension of line $ CX$ and its two intersections with $ \\omega_1$ and $ \\omega_2$ be $ X_1, X_2$, respectively. Proving the power of a point is equal for both circles proves that $ CX_1\\equal{}CX_2$, implying that they are the same point, i.e. $ Y$. Then this implies that $ C,X,Y$ are collinear.\r\nDoes this logic work?",
"Solution_12": "Yeah that looks like it works (and it's pretty much how you prove the existence of radical axes [in the case of intersecting circles]).",
"Solution_13": "a problem very easily, I do not think this is the problem USAMO.",
"Solution_14": "[quote=\"math10\"]a problem very easily, I do not think this is the problem USAMO.[/quote]\r\n\r\nWhat do you mean? This was problem #1 on the 2009 USAMO.",
"Solution_15": "Here I noticed that the intersection of $C_2S$ and $C_1Q$ is the radical axis of $\\omega_{1,2,3}$, but here I don't think the Radical Axis Theorem says that the center of $\\omega_3$ lies on $XY$ :wacko: ",
"Solution_16": "Let $T = PQ \\cap RS$, $(PRQS) = \\omega_3$, the center of $\\omega_3$ be $O_3$, and $K = XY \\cap O_1O_2$. We first assume $O_1$, $O_2$, and $O_3$ are non-collinear. Notice $$Pow_{\\omega_1}(T) = TR \\cdot TS = Pow_{\\omega_3}(T) = TP \\cdot TQ = Pow_{\\omega_2}(T) \\implies T \\in XY$$ (the Radical Axis of $\\omega_1$ and $\\omega_2$). Since $RS$ is the Radical Axis of $\\omega_1$ and $\\omega_3$, and $PQ$ is the Radical Axis of $\\omega_2$ and $\\omega_3$, we know the intersection of the perpendicular from $O_1$ to $RS$ and the perpendicular from $O_2$ to $PQ$ is $O_3$.\n\nLooking at $\\triangle O_1O_2O_3$, it's obvious that $T$ is its orthocenter. Now, let the foot of the altitude from $O_3$ to $O_1O_2$ be $K'$. Then, $\\angle TK'O_1 = 90^{\\circ}$. Using Radical Axis properties, however, we know $\\angle TKO_1 = 90^{\\circ}$. Thus, $K = K'$ because they both lie on $O_1O_2$. Hence, $O_3, T, K$ are collinear or $O_3 \\in XY$ as desired. \n\nIf $O_1$, $O_2$, and $O_3$ are collinear, then we know $PQ \\parallel RS \\parallel XY$. \n\n[b]Claim:[/b] If $O_1$, $O_2$, and $O_3$ are collinear, then $O_3 = K = XY \\cap O_1O_2$.\n\n[i]Proof.[/i] The claim is equivalent to showing $KP = KQ = KR = KS$ (in this configuration). This is trivial via the Pythagorean Theorem, however, so we're done. $\\square$\n$\\blacksquare$\n\n\n[b]Remark:[/b] In order for the problem to hold when the centers are collinear, $O_3 \\in O_1O_2$ and $O_3 \\in XY$ has to hold, implying $O_3 = XY \\cap O_1O_2 = K$ must be true, motivating the last claim.",
"Solution_17": "[hide=Solution]Clearly $XY$ is the radical axis of $\\omega_1, \\omega_2$. Define the circumcircle of $PQRS$ to be $\\omega$. Similarly, $SR$ is the radical axis of $\\omega, \\omega_1$. Let $O$ be the center of this circle. Since $O_2$ lies on $RS$, we have $\\text{Pow}_{\\omega} O_{2} = \\text{Pow}_{\\omega_1} O_2$, or $OO_2^2-OP^2=O_1O_2^2-O_1P^2$. Also, we get $OO_1^2-OP^2=O_1O_2^2-O_2Q^2$ from similar methods. Subtracting the first equation from the second gives $OO_1^2-OO_2^2=O_1P^2-O_2Q^2$, or $OO_1^2-O_1P^2=OO_2-O_2Q^2$. This is all that's necessary to finish and we are done.[/hide]",
"Solution_18": "Let the center of the circle $\\omega$ passing through $P,Q,R,S$ be $O$, and let the length of its radius be $r$. Let the center of $\\omega_1$ be $O_1$ and let the center of $\\omega_2$ be $O_2$, and let the lengths of their radii be $r_1,r_2$ respectively. It suffices to show that $O_1O^2-r_1^2=O_2O^2-r_2^2$. But\n\\[O_1O^2-r^2=O_1P\\cdot O_1Q=O_1O_2^2-r_2^2\\]\n\\[O_2O^2-r^2=O_2R\\cdot O_2S=O_1O_2^2-r_1^2\\]\nso we are done.",
"Solution_19": "By Radical Center theorem we know O1Q and O2S and XY meet at single point Z.\nLet O3 be center of SPRQ. we have O1O3 \u22a5 SR and O2O3 \u22a5 PQ and O1O2 \u22a5 XY so Z is orthocenter of O1O2O3 and as it lies on XY we have O3 lies on XY as well.\nwe're Done.",
"Solution_20": "Let $\\omega_3$ and $O_3$ be $(PQRS)$ and its center, respectively. Notice $$O_1O_2^2-r_2^2=\\textit{pow}_{\\omega_2}(O_1)=\\textit{pow}_{\\omega_3}(O_1)=O_1O_3^2-r_3^2$$ and $$O_2O_3^2-r_3^2=\\textit{pow}_{\\omega_3}(O_2)=\\textit{pow}_{\\omega_1}(O_2)=O_2O_1^2-r_1^2.$$ Adding yields $$\\textit{pow}_{\\omega_2}(O_3)=O_2O_3^2-r_2^2=O_1O_3^2-r_1^2=\\textit{pow}_{\\omega_1}(O_3)$$ so $O_3$ lies on $\\overline{XY}.$ $\\square$",
"Solution_21": "[hide=for fun.]\nLet $O$ be the center of $(PQRS)$, let $O_1,O_2$ are centers of $\\omega_1,\\omega_2$, respectively. By radical axis theorem, $Z=\\overline{PQ}\\cap\\overline{SR}$ lies on $\\overline{XY}\\perp \\overline{O_1O_2}$. Observe that by perpendicular bisectors, $Z$ is the orthocenter of $\\triangle OO_1O_2$. Hence, $\\overline{OZ}\\perp\\overline{O_1O_2}\\perp\\overline{AX}\\implies Z\\in \\overline{XY}$, as desired.\n[/hide]",
"Solution_22": "imagine doing amo problems \"for fun\" orz",
"Solution_23": "Let $O_1, r_1$ and $O_2, r_2$ be the centers and radii of $w_1$ and $w_2$, respectively. Also, let $w_3$ be the circle containing $P, Q, R, S$ with a center at $O_3$ and a radius of $r_3$. Since $O_1$ is on the radical axis of $w_2$ and $w_3$, we have:\n$$Pow_{w_2}(O_1)=Pow_{w_3}(O_1)$$\n$$O_1O_2^2-r_2^2=O_1O_3^2-r_3^2$$\nSimilarly, since $O_2$ lies on the radical axis of $w_1$ and $w_3$:\n$$Pow_{w_1}(O_2)=Pow_{w_3}(O_2)$$\n$$O_1O_2^2-r_1^2=O_2O_3^2-r_3^2$$\nSubtracting these two equations gives the following equation, which we continue to simplify.\n$$(O_1O_2^2-r_2^2)-(O_1O_2^2-r_1^2)=(O_1O_3^2-r_3^2)-(O_2O_3^2-r_3^2)$$\n$$r_1^2-r_2^2=O_1O_3^2-O_2O_3^2$$\n$$O_2O_3^2-r_2^2=O_1O_3^2-r_1^2$$\n$$Pow_{w_2}(O_3)=Pow_{w_1}(O_3)$$\nTherefore, $O_3$ lies on the radical axis (line $XY$) of $w_1$ and $w_2$. $\\square$",
"Solution_24": "Let $O_1, O_2$ be the centers of the circles $w_1,$ and $w_2$ with radius $r_1$ and $r_2$ respectively. Furthermore, let $O_3$ be the center of the circle $w_3$ passing through $P, Q, R,$ and $S$ with radius $r_3$. Then, the problem is equivalent to proving that $P(O_3, w_1) = P(O_3, w_2) \\Leftrightarrow O_3O_1^2 - r_1^2 = O_3O_2^2 - r_2^2$.\n\nNote that $P(O_1, w_2) = P(O_1, w_3) \\implies O_1O_3^2 -r_3^2 = O_1O_2^2 - r_2^2$ and $P(O_2, w_1) = P(O_2, w_3) \\implies O_2O_3^2 - r_3^2 = O_2O_2^2 - r_1^2$. Subtracting the two equations gives $O_1O_3^2 - O_2O_3^2 = r_1^2 - r_2^2 \\implies O_3O_1^2 - r_1^2 = O_3O_2^2 - r_2^2$ which is what we wanted to prove.",
"Solution_25": "Let $(PQRS)=\\omega_3$ and let $r_i$ and $O_i$ denote the radius and center of circle $\\omega_i$ for $i\\in[1,2,3]$. Since $O_1$ is on the radical axis of $O_2$ and $O_3$ we have $$\\text{Pow}_{\\omega_2}(O_1)=\\text{Pow}_{\\omega_3}(O_1) \\iff O_1O_2^2-r_2^2=O_1O_3^2-r_3^2.$$ Since $O_2$ is on the radical axis of $O_1$ and $O_3$ we have $$\\text{Pow}_{\\omega_1}(O_2)=\\text{Pow}_{\\omega_3}(O_2) \\iff O_2O_1^2-r_1^2=O_2O_3^2-r_3^2.$$ Subtracting yields $$r_1^2-r_2^2=O_1O_3^2-O_2O_3^2 \\iff \\text{Pow}_{\\omega_1}(O_3)=\\text{Pow}_{\\omega_2}(O_3)$$ and we are done.",
"Solution_26": "Let $O_1,O_2,O_3$ be the centers of $\\omega_1,\\omega_2$, and $(PQRS)$. Let $C$ be the intersection of $PQ$ and $RS$. Note that $C$ also lies on $XY$ since the pairwise radical axes of the three circles are concurrent. Now, note that $O_3$ lies on the perpendicular bisector of both $PQ$ and $RS$. Thus, we have that $O_1O_3$ is perpendicular to $RS$, and $O_2O_3$ is perpendicular to $PQ$. Therefore, $C$ is the orthocenter of $\\triangle O_1O_2O_3$. This means that $O_3C$ is perpendicular to $O_1O_2$. Since $C$ lies on $XY$, we are done.",
"Solution_27": ":unamused:\n\nDenote $\\omega_3:=(PQRS)$. Because $O_1\\in PQ$,\n\\[\\text{Pow}_{\\omega_2}(O_1)=\\text{Pow}_{\\omega_3}(O_1)\\implies O_1O_2^2-r_2^2=O_1O_3^2-r_3^2;\\]\nand since $O_2\\in RS$,\n\\[\\text{Pow}_{\\omega_1}(O_2)=\\text{Pow}_{\\omega_3}(O_2)\\implies O_1O_2^2-r_1^2=O_2O_3^2-r_3^2.\\]\nSubtracting,\n\\[r_1^2-r_2^2=O_1O_3^2-O_2O_3^2\\implies O_1O_3^2-r_1^2=O_2O_3^2-r_2^2,\\]\ni.e., $\\text{Pow}_{\\omega_1}(O_3)=\\text{Pow}_{\\omega_2}(O_3)$ or $O_3\\in XY$, as desired. $\\square$",
"Solution_28": " Let $M=\\overline{SR} \\cap \\overline{PQ}$. Let $O_1,O_2,O_3$ be the centers of $\\omega_1,\\omega_3$ and $(SPRQ)$. Now, it is well known that the radical axis of two circles is perpendicular to the line joining their centers. Then, $SR \\perp O_1,O_2$ and $PQ \\perp O_2O_3$. This means that $M$ must be the orthocenter of $\\triangle O_1O_2O_3$. This means, $O_3M\\perp O_1O_2$ as well. Clearly $M$ lies on $XY$ ($MP\\cdot MQ = MR\\cdot MS$ so it lies on the radical axis of $\\omega_1$ and $\\omega_2$). But then, $XY$ is the line passing through $M$ and perpendicular to $O_1O_2$. This means that indeed $O_3$ lies on the line $XY$ and we are done.",
"Solution_29": "Let $O_1$ be the center of $\\omega_1$ and let $O_2$ be the center of $\\omega_3$. Denote the center of $(PRQS)$ to be $O_3$ and let the intersection of $PQ$ and $SR$ be $C$. Now note that $SR$, $PQ$, and $XY$ are the three pairwise radical axes of $\\omega_1$, $\\omega_2$, and $(PRQS)$. \n\nDenote the length of $O_1O_2$ to be $a$ and let the radii of $\\omega_1$, $\\omega_2$, and $(PRQS)$ be $r_1$, $r_2$, and $r_3$, respectively. Since $O_2$ is on the line $PQ$, by the Power of a Point definition of the radical axis, we have that \n\\[a^2-r_1^2=O_2O_1^2-r_1^2=Pow_{\\omega_1}(O_2)=Pow_{\\omega_3}(O_1)=O_2O_3^2-r_3^2,\\]\nwhich gives that $O_2O_3=\\sqrt{a^2-r_1^2+r_3^2}$. Similarly, since $O_1$ is on $RS$, using Power of a Point on $O_1$ with respect to $O_2$ and $O_3$, we get that $O_1O_3=\\sqrt{a^2-r_2^2+r_3^2}$. Finally, using these lengths and Power of a Point one last time, \n\\[Pow_{\\omega_1}(O_3)=O_1O_3^2-r_1^2=a^2+r_3^2-(r_1^2+r_2^2)=O_1O_2^2-r_2^2=Pow_{\\omega_2}(O_3),\\]\nwhich implies that $O_3$ is on the radical axis of $\\omega_1$ and $\\omega_2$, which is $XY$, finishing the problem."
}
{
"Tag": [
"\\/closed"
],
"Problem": "Do we have a new administrator nsato, or has he always been here? I understand he's an instructor for some classes.",
"Solution_1": "Naoki Sato joined the staff of Art of Problem Solving as of yesterday. He will be a regular instructor. He was an IMO medalist and worked with the Canadian IMO team for several years, so he is an experienced teacher of top students."
}
{
"Tag": [
"function",
"limit",
"algebra",
"domain",
"calculus",
"derivative",
"analytic geometry"
],
"Problem": "Prove that the function $ \\sqrt{x}$ is uniformly continuous for $ x\\in[1,\\infty)$.\r\n\r\nThe hint given was to use the facts that the function continuous on a segment is also uniformly continuous on it, and the fact if the function is continuous for $ x\\in[a,\\infty)$, and the $ \\lim_{x\\to\\infty}$ exists, then it's uniformly continuous. But still I do believe that the word \"finite\" (limit) was dropped out of this last theorem, and that it cannot be applied here.\r\n\r\nIt's an unbounded function on an unbounded domain-it's quite surprising for me that it's uniformly convergent... (but again I have to say that I lack knowledge in more than few topics in analysis...)",
"Solution_1": "That's the wrong hint at $ \\infty$; what's going on here is that the derivative is bounded for large $ x$. On the other side, you only need a second argument if you extend all the way to zero.",
"Solution_2": "Oh, I didn't think of the fact that if the derivative is bounded on the interval (and the function is continuous), then the continuity is uniform (that's \"sufficient Lipschitz\", right?)-the theorem is not in the curriculum I have to work by as a UTA (and it would surely be fun to prove it to my students using MVT they've never heard of-oh well, they don't know derivatives yet, so the argument will be a bit misty for them).\r\n\r\nSo I'll try an intuitive-slope-approach, and hope they'll understand :)\r\n\r\nI just wonder is there a nice $ \\epsilon \\minus{} \\delta$ proof? It seems to me that $ \\epsilon\\equal{}\\delta$ should do the trick, as $ x_1\\minus{}x_2>\\sqrt{x_1}\\minus{}\\sqrt{x_2}$ for $ x_1>x_2$.",
"Solution_3": "If $ |f'(x)|\\le k$ for all $ x$, $ f$ satisfies the Lipschitz condition $ |f(x)-f(y)|\\le k|x-y|$ by the Mean Value Theorem. That implies uniform continuity; if $ |x-y|<\\frac{\\epsilon}{k}$, $ |f(x)-f(y)|<\\epsilon$.\r\n\r\nIn this case, the derivative is no more than $ \\frac12$, so we use $ \\delta=2\\epsilon$ or smaller.\r\n\r\nIntroducing technical notions like uniform continuity before we know about derivatives and the MVT sounds very weird to me.",
"Solution_4": "Similarly, the function $ \\sqrt{|x|}$ is uniformly continuous.\r\n$ \\left|\\sqrt{|x|}-\\sqrt{|y|}\\right|\\leq \\sqrt\\left\\parallel{}x|-|y|\\right|}\\leq\\sqrt{|x-y|}$ $ \\forall x,y\\in\\mathbb{R}$.\r\nGiven $ \\varepsilon>0$, we can be sure that $ |f(x)-f(y)|<\\varepsilon$ as long as $ |x-y|<\\varepsilon^{2}$.",
"Solution_5": "[quote=\"jmerry\"]\nIntroducing technical notions like uniform continuity before we know about derivatives and the MVT sounds very weird to me.[/quote]\r\n\r\nThis Mathematics course is extremely wide, so some things are put upside down /but de facto they're there just pro formae/. First semester has to cover single-variable Calculus and the Analysis basics while in the other semester: multivariable Calculus, DE, Laplace & Fourier, Field theory... Third semester: Complex Analysis, Fourier, Laplace (again)... Quite a blend for young engineers :)",
"Solution_6": "[quote=\"hsiljak\"]\nI just wonder is there a nice $ \\epsilon \\minus{} \\delta$ proof? It seems to me that $ \\epsilon \\equal{} \\delta$ should do the trick, as $ x_1 \\minus{} x_2 > \\sqrt {x_1} \\minus{} \\sqrt {x_2}$ for $ x_1 > x_2$.[/quote]\r\n\r\nThis proof is correct on the given interval. Of course, it won't work for $ x \\leq 1$ because then the above inequality won't hold."
}
{
"Tag": [
"function",
"real analysis",
"real analysis theorems"
],
"Problem": "I have always wondered whether there is a geometric interpretation for a function which is uniform continuity. For instance, the geometric interpretation for a continuous function from $ R$ to $ R$ is the classic \"a function is continuous if its graph can be drawn without lifting the pencil from the paper\". Another geometric interpretation of a definition is the one for uniform convergence (I like that because it is very helpful) that says that a sequence of functions converges uniformly to a function $ f$ if I can draw a \"tube\" ,of size epsilon, around $ f$ such that in some moment the tail of the sequence get inside this tube (I hope I explained this well :P).\r\n\r\nI appreciate you can help me with my question and if you want you can add another nice geometric interpretation of some definition.\r\n\r\nThank you!",
"Solution_1": "My friend and I are in an advanced analysis course and when we first got to uniform continuity and tried to do some proofs, we concluded that a uniformly continuous function can't go \"Blaaah!\" Now that phrase probably doesn't mean anything to you, but when my friend attempted to draw a function that was not uniformly continuous, he basically drew a line shooting upwards. In other words, the function can't blow up without control. For example, $ f(x)=x^{2}$ can't be uniformly continuous because it's getting steeper and steeper so we can't pick a $ \\delta$ that always works. On the other hand, $ f(x)=\\sqrt{x}$ (and even $ \\sqrt{|x|}$) are both uniformly continuous because with the exception of the cusp at the origin, it's very easy to control the function.\r\n\r\nDid that help at all?",
"Solution_2": "I guess an o.k. geometric analogy is that a function is uniformly continuous on an interval $ I$ if when you pick a horizontal strip of size $ \\epsilon$, there exists a $ \\delta$ such that no matter where you slide your $ \\epsilon$ strip across the function (as long as it stays in $ I$) any points that are $ \\delta$ units away from each other or less will have images that fit into that epsilon strip. \r\n\r\n\r\nIt's easier to visualize if you draw it out :P.",
"Solution_3": "I agree with [b]max_tm[/b]...and with a little searching look what I found...[youtube]oGkgIzSppik[/youtube]",
"Solution_4": "That's basically the idea behind what I wrote in the first post: the function never blows up without control."
}
{
"Tag": [
"topology",
"real analysis",
"real analysis unsolved"
],
"Problem": "Need some help. Consider the lower limit topology defined on $ \\mathbb{R}$. That is, the topology generated by the basis, $ \\{[a,b): a,b\\in\\mathbb{R}\\}$.\r\n\r\nGive a covering of $ [0.1]$ to show that $ [0,1]$ is not compact.",
"Solution_1": "The obvious covering to try is something about upper limits; how about $ V_n \\equal{} \\left[ 0, 1 \\minus{} \\frac {1}{n} \\right)$? A finite subcover of this cover is of the form $ \\left[ 0, 1 \\minus{} \\frac {1}{N} \\right)$ and hence cannot cover $ [0, 1]$.",
"Solution_2": "[quote=\"t0rajir0u\"]$ V_n \\equal{} \\left[ 0, 1 \\minus{} \\frac {1}{n} \\right)$.[/quote]\r\n\r\nBut it is not an open cover of $ [0, 1]$, neither. You may add $ [1, 2)$ to get around this problem.",
"Solution_3": "Ah... :lol: Thanks sos440.",
"Solution_4": "Is the space locally compact?",
"Solution_5": "Nope. All compact subsets are at most countable, but all open sets are uncountable (as they are intervals), so no compact set can contain an open set."
}
{
"Tag": [
"geometry"
],
"Problem": "The lengths of the tangents from vertices A, B, C to the incircle are x, y and z respectively. If $a\\geq b\\geq c$, prove that $az+by+cx \\geq \\frac{a^2+b^2+c^2}{2} \\geq ax+by+cz$",
"Solution_1": "WE have obviously $x+y=c$\r\n $x+z=b$ and\r\n $y+z=a$\r\nSolving this little system we determine $x,y,z$ and the ineq is then easily proved!\r\nDo not forget the triangle ineq!!!! :) :D :D"
}
{
"Tag": [
"search",
"geometry",
"cyclic quadrilateral",
"geometry open"
],
"Problem": "[b]\nLet $ ABCD$ be a cyclic quadrilateral and $ I$ denote the point of intersection of the diagonals \n$ AC$ and $ BD$, $ M$ is the midpoint of side $ BC$ and $ N$ is the midpoint of side $ AD$. \nIf $ P \\in AB$ such that $ IP \\bot AB$ and $ R \\in CD$ such that $ IR \\bot CD$ prove that $ MN \\bot PR$.\n[/b]\r\n------------------------------------------------------------------------------------------------------------------\r\n[img]http://img.photobucket.com/albums/v218/hudrea/metrutest.jpg[/img]",
"Solution_1": "I have seen this problem here ---> http://www.mathlinks.ro/Forum/viewtopic.php?search_id=1549164493&t=55836\r\nyetti's very nice answer :)",
"Solution_2": "Thank you very much for your valuable opinion.",
"Solution_3": "this problem will be solve very easy by the down lemma and congruity triangles. :wink: \r\n\r\n\r\nlemma: if $ ABC$ is a triangle and $ x ,y$ are the points that $ ABX , ACY$ are rectangular triangles . ($ BXA \\equal{} CYA \\equal{} 90$) then if $ M$ is the midpoint of $ BC$ and $ XBA \\equal{}YCA$ then we have \r\n\r\n[size=150]$ XMY \\equal{} 90$ and $ mx \\equal{} my$[/size]",
"Solution_4": "It seems this problem is very well known and many people tried it in advance and there are lots of skilled people in mathlinks.\r\nDo you know what is the source of the problem?\r\n\r\nI like finding interesting math problems and I post most many of them here.",
"Solution_5": "no. i don't know . i solve this 8month ago when i was 16. :o :roll:",
"Solution_6": ":) it means you have good teacher(s).",
"Solution_7": "I cant understand u. please speak more cleraly :wink: :arrow:",
"Solution_8": "I want to say - if people solve such problems when they are 16 years old - they have good teachers that propose such problems and prepare them to solve these problems.",
"Solution_9": "YOU ARE SO $ benignity$ :)",
"Solution_10": "Thank you very much. I said that because when I was 16 old a had no good teacher. But she said me that the Bulgarian Math Olympiad exist and there was good problems in the competitions in their earlier stages. It is the reason for me to invent math problems and to try to find interesting problems and to propose them in forums, magazines and comeptitions."
}
{
"Tag": [
"number theory unsolved",
"number theory"
],
"Problem": "For $n>1$ , prove that $\\sum_{j=1}^{n}\\frac{1}{2j-1}$ can not be integer.",
"Solution_1": "Analogously with a similar proof for the harmonic series, we can show that the greatest power of $3$ that divides the common denominator is greater than the greatest power of $3$ that divides the common numerator.\r\n\r\nThere is a formulation with $p$-adic numbers, but so far I have been unable to tease it out. Could somebody who knows about such things sketch it out for me?",
"Solution_2": "t0rajir0u, Thanks. I saw.",
"Solution_3": "I wonder that are there other solution except this classical solution? Can anybody help me?",
"Solution_4": "From Bertrand's Postulate, there exists $k\\le n$ such that $n\\le 2k-1=p\\le 2n-1$ for some prime $p$. Then the fraction (the sum) in lowest terms has denominator divisible by $p$. on the other hand the numerator is equal to $\\text{(some terms divisible by }p)+\\text{(a term not divisible by }p)$. thus it cannot be an integer.",
"Solution_5": "We can prove the following more general result. \r\n\r\nLet $a_{1},a_{2},...,a_{n}(n>1)\\in\\mathbb{Z}-\\{0\\}$ . Suppose there is a prime $p$ and positive integer $h$ such that $p^{h}|a_{i}$ for some $i$ and $p^{h}\\not |a_{j}\\forall j\\not =i$. Then show that $\\frac{1}{a_{1}}+\\frac{1}{a_{2}}+...+\\frac{1}{a_{n}}\\not\\in\\mathbb{Z}$."
}
{
"Tag": [],
"Problem": "What is the value of $ 501^2 \\minus{} 499^2$?",
"Solution_1": "In general, $ x^2-y^2=(x+y)(x-y)$. In this case, $ x=501$ and $ y=499$. Thus\r\n\\begin{align*}501^2-499^2&=(501+499)(501-499)\\\\\r\n&=(1000)(2)\\\\\r\n&=\\boxed{2000}\\end{align*}"
}
{
"Tag": [
"number theory",
"prime numbers",
"arithmetic sequence",
"algebra unsolved",
"algebra"
],
"Problem": "proof that $\\xi=0,p_{1}p_{2}p_{3}....$is irrational \r\n$p_{n}$is the $n$ th prime number.",
"Solution_1": "Same reasoning than in http://www.mathlinks.ro/Forum/viewtopic.php?t=104311 , noting that there are for example abitrary long sequences of zero's in the (decimal) representation because there are primes $\\equiv 1 \\mod 10^{k}$ for all $k$.",
"Solution_2": "i want a proof and i don't that it's so simple.\r\nmay be the first is simple but this no :D",
"Solution_3": "Well, if you don't ask for a proof of Dirichlet's theorem (even in the special case mentioned by ZetaX), I don't see what you are wanting...\r\n\r\nPierre.",
"Solution_4": "I didn't just say it is simple but gave a sketch of a proof... :maybe: \r\n\r\nA bit more explicit:\r\nAssume it is rational. Then it's (decimal) representation is periodic. By that, there can't be arbitrary long sequences of zeros in it (this could only happen for a periodic representation only consisting of zeros from some point, but this is not the case).\r\nSo if we proof that there are arbitrary long sequences of zeros in it, we are done. So we look for prime numbers of type $a\\cdot 10^{k}+1$ since they have (at least) $k-1$ consecutive zeros in them (they look like $...x000...0001$).\r\nSo it suffices to prove that we can find such a prime for all given $k$, meaing to prove there is a prime $\\equiv 1 \\mod 10^{k}$. But such prime exists by Dirichlets theorem on primes in arithmetic progression, so we are done (note that this case of Dirichlet can be done on an elementary way; for example all prime divisors $\\neq 2,5$ of $x^{4 \\cdot 10^{k-1}}-x^{3 \\cdot 10^{k-1}}+x^{2 \\cdot 10^{k-1}}-x^{10^{k-1}}+1$ work).\r\n\r\nwell, I can post (or link since it surely is on MathLinks) the proof of Dirichlet's theorem for that case if you want..."
}
{
"Tag": [
"LaTeX",
"combinatorics open",
"combinatorics"
],
"Problem": "$n$ is integer $>0$.let $A={1,2,...n}$.find $f(n)_{max}$ such that when $A$ is divised to $f(n)$ any subset $A_1,A_2,...A_{f(n)}$st : \r\n1)$A_{i}\\bigcap\\(A_{j}=\\emptyset$ for all $i,j$\r\n2)$A_{1}\\cup\\(A_{2}\\cup\\...\\cup\\(A_{n}=A$\r\n there exist $A_{k}$ include three elements $x=2 there exists at most n-2 divisors of 0.Show that the ring is a field.",
"Solution_1": "Unfortunately, I published this problem in Recreatii Matematice when I was in the 12-th form and I didn't know allmost anything about superiour algebra. So, I didn't know it is well-known and I didn't know it is easy. I just had an idea I thought is nice. But I was wrong. So, the source should be: well-known.",
"Solution_2": "Does this work?\r\n\r\nWe know that in a finite ring an element is not invertible iff it's a divisor of 0 (even if the ring isn't commutative, but that's just a little bit harder to prove), so in our ring there are at least n2-n+1 invertible elements. Let's look at the set x*U(A), where x is a divisor of 0. All the elements of the set are divisors of 0, so |x*U(A)|<=n-2. This means that there is a set M included in U(A) s.t. |M|>=(n2-n+1)/(n-2)>n+1 (so |M|>=n+2) and x*M has only one element. Let the elements of M be ai with i from 1 to |M|. The elements ai-a1 with i from 2 to |M| are distinct divisors of 0, but there are at least n+1 such elements, so we have a contradiction (because there are at most n-2 a divisor of 0 is a bit trickier. \r\n\r\nWe consider the function f:A->A, f(x)=ax. Let's assume a is not a divisor of 0. This means that f is injective, but f is injective iff it?s surjective, so there is an x\\in A s.t. ax=1. Let M be the set of all x\\in A s.t. ax=1. If x\\in M then x(a+1)-1\\in M. Since f is injective, x=x(a+1)-1, so xa=1=ax, so a is invertible. Conversely, if a is invertible, then it's clearly not a divisor of 0.\r\n\r\nThis shows that the proof works fine for any ring with n2 elements."
}
{
"Tag": [
"inequalities",
"inequalities unsolved"
],
"Problem": "For every n positive reals $a_i (i=1,..,n)$ such that $a_1^2+a_2^2+..+a_n^2=1$ prove that\r\n\r\n$a_1+a_2+...+a_n +\\frac{1}{a_1a_2....a_n} \\geq \\sqrt{n} +(\\sqrt{n})^n$",
"Solution_1": "First of all I will prove a beautiful lemma (which I cheated from Romanian magazine):\r\n\r\nWe have e \u2264 a \u2264 b \u2264 f and we want to prove e+f \u2265 a+b.\r\n\r\n(e-a)(f-a) \u2264 0 => a2+ef \u2264 a(e+f)\r\n(e-b)(f-b) \u2264 0 => b2+ef \u2264 b(e+f)\r\n\r\nSumming : a2 + b2 + 2ef \u2264 (e+f)(a+b)\r\n\r\nHence if we prove ef \u2265 ab then (e+f) \u2265 (a+b) \r\n and if we prove e+f \u2264 a+b then ef \u2264 ab.\r\n\r\nSo we have by AM-GM inequality we have:\r\n(a1*a2 an) \u2264 (\u2211ai)^n / n^n \u2264 1/ (\u221an)^n\r\n\r\n(\u2211ai) \u2264 (\u221an) \u2264 (\u221an)^n \u2264 1/(a1*a2 an)\r\n\r\nBy our lemma it is enough to prove:\r\n(\u2211ai)/(a1*a2 an) \u2265 (\u221an)^(n+1)\r\n\r\nBut (\u2211ai)/(a1*a2 an) \u2265 n/(a1*a2 an)^[(n-1)/n] \u2265 (\u221an)^(n+1)\r\n\r\nOr again (\u221an) \u2264 1/(a1*a2 an)^n q.e.d",
"Solution_2": "Very nice proof.",
"Solution_3": "I have another proof\r\nI will use this Theorem \r\n${p_1}{a_1}+{p_2}{a_2}+..+{p_n}{a_n} \\ge (\\sum p_i)({a_1}^{p_1}+{a_2}^{p_2}+..+{a_n}^{p_n})^\\frac{1}{\\sum p_i}$ for positive reals $a_i,p_i$\r\nSo rewrite the inequality \r\n$a_1+a_2+..+a_n+\\sqrt{n}^{n+1}\\frac{1}{a_1a_2..a_n\\sqrt{n}^{n+1}} \\ge \\sqrt{n}+\\sqrt{n}^n$\r\nUsing the Theorem in the left hand side of the inequality we have\r\n$a_1+a_2+..+a_n+\\sqrt{n}^{n+1}\\frac{1}{a_1a_2..a_n\\sqrt{n}^{n+1}} \\ge (n+\\sqrt{n}^{n+1})\\left(\\frac{\\prod a_i}{(\\prod a_i)^{\\sqrt{n}^{n+1}}\\sqrt{n}^{(n+1)\\sqrt{n}^{(n+1)}}}\\right)^\\frac{1}{n+\\sqrt{n}^{(n+1)}}$\r\nNow we have to prove that \r\n$\\left(\\frac{\\prod a_i}{(\\prod a_i)^{\\sqrt{n}^{n+1}}\\sqrt{n}^{(n+1)\\sqrt{n}^{(n+1)}}}\\right)^\\frac{1}{n+\\sqrt{n}^{(n+1)}}$ $\\ge\\left(\\frac{1}{\\sqrt{n}}\\right)$\r\nUsing the fact that $\\left(\\frac{1}{\\sqrt{n}}\\right)^n \\ge \\prod a_i $ which holds right away from the condition of the problem one can easily prove the remaining part of the inequality. Of course Prowler's solution is much more easier but i really hope anybody reads mine ;)",
"Solution_4": "I hope first inequality is \r\n p1*x1 + p2*x2 +...+pn*xn \u2265 (\u2211pi)*(x1^p1 +x2^p2+...+xn^pn)\r\nSolution is very beautiful, my congratulations.",
"Solution_5": "Andrei proof is a very nice application of AM-GM generalized inequality.\r\n\r\nSorry, Prowler but I don't understand your post.\r\n\r\nI think that Andrei initial theorem is correct.",
"Solution_6": "I am sleeping :blush: \r\nI meant p1*x1 + p2*x2 +...+pn*xn \u2265 (\u2211pi)*(x1^p1*x2^p2*...*xn^pn)^1/(\u2211pi)\r\nbecause \r\n[quote=\"Andrei\"]I have another proof\n\n${p_1}{a_1}+{p_2}{a_2}+..+{p_n}{a_n} \\ge (\\sum p_i)({a_1}^{p_1}+{a_2}^{p_2}+..+{a_n}^{p_n})^\\frac{1}{\\sum p_i}$ for positive reals $a_i,p_i$\n\nand uses it in a such way:\n\n$a_1+a_2+..+a_n+\\sqrt{n}^{n+1}\\frac{1}{a_1a_2..a_n\\sqrt{n}^{n+1}} \\ge (n+\\sqrt{n}^{n+1})\\left(\\frac{\\prod a_i}{(\\prod a_i)^{\\sqrt{n}^{n+1}}\\sqrt{n}^{(n+1)\\sqrt{n}^{(n+1)}}}\\right)^\\frac{1}{n+\\sqrt{n}^{(n+1)}}$\n[/quote]\r\n Or I misunderstand solution.",
"Solution_7": "[quote=\"prowler\"]First of all I will prove a beautiful lemma (which I cheated from Romanian magazine):\n\nWe have e \u2264 a \u2264 b \u2264 f and we want to prove e+f \u2265 a+b.[/quote]\r\nIf $ef \\geq ab$, of course."
}
{
"Tag": [
"calculus",
"integration",
"inequalities",
"real analysis",
"real analysis theorems"
],
"Problem": "I am looking for a simple proof of the following inequality:\r\n\r\n${ \\left\\lgroup\\int_{x=a}^{b}\\left\\lgroup \\int_{y=c}^{d}f(x,y)\\,dy\\right\\rgroup^{p}\\,dx\\right\\rgroup^{1/p}\\leq \\int_{y=c}^{d}\\left\\lgroup \\int_{x=a}^{b}f(x,y)^{p}\\,dx\\right\\rgroup^{1/p}\\,dy}$\r\n\r\nwhere $p>1$. If summation sign is used instead of integral sign, this is Minkowski Inequality. Where can I find a proof? :oops:",
"Solution_1": "I had this on a problem set in analysis -- the proof there went via Holder, although of course then you would have to prove the Holder integral inequality first. (All together, than took about 2 pages of official solution from the professor.)",
"Solution_2": "It goes by the name \"Minkowski integral inequality.\"\r\n\r\nOne way to prove it: \"linearize\" it by making usue of the following fact:\r\n\r\n$\\|f\\|_{p}=\\sup_{\\|g\\|_{p'}=1}\\int fg$\r\n\r\nwhere $p'$ is the dual exponent, $\\frac1p+\\frac1{p'}=1.$",
"Solution_3": "What is the equality condition for the inequality?\r\nThe following link gives the prove without the equality condition:\r\n\r\nhttp://calclab.math.tamu.edu/~sivan/math663_06a/appendix2.pdf\r\n :roll:",
"Solution_4": "Here's the complete result:\r\n\r\n$(a)~~{ \\left\\lgroup\\int_{x=a}^{b}\\left\\lgroup \\int_{y=c}^{d}f(x,y)\\,dy\\right\\rgroup^{p}\\, dx\\right\\rgroup^{1/p}\\leq \\int_{y=c}^{d}\\left\\lgroup \\int_{x=a}^{b}f(x,y)^{p}\\,dx\\right\\rgroup^{1/p}\\,dy }$\r\n\r\n$(b)~~{ \\left\\lgroup\\int_{x=a}^{b}\\left\\lgroup \\int_{y=c}^{d}f(x,y)\\,dy\\right\\rgroup^{p}\\, dx\\right\\rgroup^{1/p}= \\int_{y=c}^{d}\\left\\lgroup \\int_{x=a}^{b}f(x,y)^{p}\\,dx\\right\\rgroup^{1/p}\\,dy ~~\\Leftrightarrow~~\\frac{f(x,y)}{f(x,z)}=\\frac{f(u,y)}{f(u,y)}}$\r\n\r\n :P"
}
{
"Tag": [
"search",
"algebra solved",
"algebra"
],
"Problem": "Find $ f:R--->R $ satisfying :\r\n $ f(f(x-y)) = f(x)-f(y)+f(x)f(y)-xy $",
"Solution_1": "f(x)=x or sqrt(x^2+2)?",
"Solution_2": "Can you post your solution ?",
"Solution_3": "Put x=y, \r\nf(x)^2 - x^2 = f(f(0)) = C (a constant)\r\nThen f(x) = sqrt(x^2+C) or -sqrt(x^2+C)\r\nThen substitute some values of x,y and it can be found that C=0 or 2",
"Solution_4": "Who are you , Romano ?\r\n This problem is rather easy : \r\n Take $ x=y --> f(f(0)) = f^2(x)-x^2 $\r\n $ ----> f(f(0))=f^2(0) $\r\n Take $ x=f(0) ---> f(0)=0 $ or $ f(0)=\\sqrt2 $\r\n After that , this problem will be easy !!!!",
"Solution_5": "But if for all $x$, we have $f(x) = \\sqrt {x^2 + 2}$ or $f(x) = x$, it does not mean that we have \r\n$f(x) = x$ for all $x$\r\nor\r\n$f(x) = \\sqrt {x^2 + 2}$ for all $x$.\r\n\r\nPierre.",
"Solution_6": "So is the problem completely solved? ;) :D",
"Solution_7": "No.\r\n\r\nPierre.",
"Solution_8": "[quote=\"pbornsztein\"]No.\n\nPierre.[/quote]\r\n\r\ndo u mind posting ur solution..?",
"Solution_9": "I didn't search this problem. I was only reading the proposed answers.\r\n\r\nPierre.",
"Solution_10": "I hope this is ok.\r\nAssume f(0)=0, then\r\nf(x)^2=x^2\r\nPutting y=0 we have\r\nf(f(x))=f(x)\r\nso f(x-y)=f(x)-f(y)+f(x)f(y)-xy\r\nPut x=0 we have f(-y)=-f(y)\r\nSo f(x+y)=f(x)+f(y)-f(x)f(y)+xy\r\nSum them up and squaring\r\nf(x-y)^2+f(x+y)^2+2f(x-y)f(x+y)=4x^2\r\nf(x-y)/(x-y) * f(x+y)/(x+y)=1\r\nIf we take f(x-y)=x-y, then f(x+y)=x+y\r\nSo f(x)=x for all reals, when we take y tends to 0.\r\nSimilarly we can have f(x)=-x for all reals too.\r\n\r\nfor this case is it ok..?",
"Solution_11": "Case 1: Suppose $f(x) = 0$. It's easy to check that $f(x) = -x \\forall x \\in \\mathbb{R}$ is not a solution, and that $f(x) = x \\forall x \\in \\mathbb{R}$ is. Select $a \\neq 0$ such that $f(a) = a$, and suppose there is $b \\neq 0$ such that $f(b) = -b$. Let $x = a, y = b$ in the original equation, then we find that $a=1$. This means that $f(x) = -x \\forall x \\neq 0, 1$. But now, setting $x \\neq 0, 1, y = 1$, it's easy to check that this does not work.\r\n\r\nThe other case looks...ugly :( .",
"Solution_12": "Case 2: $f^2(x) = x^2+2$.\r\n\r\nSuppose that there exists an $a > 0$ such that $f(a) = f(-a)$. Substituting $x = -y = a$ into the original equation, we obtain $\\sqrt{4a^2 + 4} = 2a^2+2$ (observe that the LHS cannot be $- \\sqrt{4a^2 + 4}$. But this implies that $a = 0$. Hence $f(-x) = -f(x) \\forall x \\neq 0$.\r\n\r\nCase 2.1: Suppose that $f(a) = \\sqrt{a^2+2}$ for some $a \\neq 0$. Let $x = -y = a$, then we get $\\pm \\sqrt{a^2+1} = \\sqrt{a^2+2} - 1$. It's easy to verify that there are no solutions for both $+$ and $-$.\r\n\r\nCase 2.2: Suppose that $f(a) = -\\sqrt{a^2+2}$ for some $a \\neq 0$. Using the same method, we get $\\sqrt{a^2+1} = \\sqrt{a^2+2} + 1$, which is even more impossible.",
"Solution_13": "this case is really ugly..... anyway u solved it :)"
}
{
"Tag": [
"geometry",
"geometric transformation",
"reflection"
],
"Problem": "particle man is located at $ (2,1)$ and wants to get to the lines $ x\\equal{}3, x\\equal{}0,$ and $ y\\equal{}0$. what is the length of the shortest path he can take?",
"Solution_1": "Graph and it will seem much, much easeier.",
"Solution_2": "ive tried reflections",
"Solution_3": "[hide]Since the lines $ x \\equal{} 0$ and $ x \\equal{} 3$ are $ 3$ units apart and particle man is located $ 1$ unit away from the nearest of these lines, he must move a total of at least $ 4$ units in the $ x$-direction. \nSince he is located $ 1$ unit away from $ y \\equal{} 0$, he must move a total of at least $ 1$ unit in the $ y$-direction. \nThus, the path must be at least $ \\sqrt {4^2 \\plus{} 1^2} \\equal{} \\sqrt {17}$ units long. \n\nReflect $ (2,1)$ over $ x \\equal{} 3$ to $ (4,1)$. Connecting $ (0,0)$ to $ (4,1)$ and reflecting the part such that $ x > 3$ over the line $ x \\equal{} 3$. \nThis gives $ (0,0)$ to $ (3,\\frac {3}{4})$ to $ (2,1)$ which has a total length of $ \\sqrt {17}$.[/hide]"
}
{
"Tag": [
"probability",
"counting",
"derangement",
"search",
"function",
"probability and stats"
],
"Problem": "A person writes $ n$ mails and he also writes addresses on $ n$ envelopes. What is the probability that neither one mail will reach its destination? What becomes this probability when $ n \\to \\infty$.",
"Solution_1": "Read [url=http://en.wikipedia.org/wiki/Derangement]here[/url]. You can find also useful links at the end.",
"Solution_2": "Or put the word \"derangement\" into the search function of this site to find many past discussions."
}
{
"Tag": [
"Ramsey Theory",
"combinatorics unsolved",
"combinatorics"
],
"Problem": "There are $ n$ points on the plane, no three of which are collinear. Each pair of points is joined by a red, yellow or green line. For any three points, the sides of the triangle they form consist of exactly two colours. Show that $ n<13$.",
"Solution_1": "[quote=\"KenHungKK\"]There are $ n$ points on the plane, no three of which are collinear. Each pair of points is joined by a red, yellow or green line. For any three points, the sides of the triangle they form consist of exactly two colours. Show that $ n < 13$.[/quote]\r\n\r\nI can certainly show that $ n < 17$. Suppose, to the contrary, that there exists a set of $ n \\equal{} 17$ points such that any triangle is colored with exactly $ 2$ colors. Consider a given point $ A$, and assume without loss of generality that the color most commonly used in lines connected to $ A$ is red. Then at least $ 6$ points must be joined with a red line to $ A$. Any two of these points must be joined with yellow or green (lest we have a triangle with all red lines). Consider a given point in this subset, $ B$. Assume without loss of generality that the color most commonly used in lines, within this subset, connected to $ B$ is yellow. Then at least $ 3$ points must be joined with a yellow segment to $ B$. Any two of these points must be joined with green (lest we have a triangle with all yellow lines). This gives us a triangle with all green lines, which provides a contradiction.",
"Solution_2": "NoYoo-hooForMe, what you have proved is essentially the Ramsey number $ N(3,3,3,2) \\le 17$. (But here is it is equal to 17 due to the additional condition that any triangle will have two same colored edges.\r\n\r\nI think this problem is quite hard(atleast for me). Can someone give a hint?"
}
{
"Tag": [
"geometry",
"3D geometry",
"sphere",
"ratio"
],
"Problem": "A right circular cone whose base has a radius of 4 cm and whose height is greater than 2 cm is inscribed in a sphere with a radius of 5 cm, as shown. What is the ratio of the volume of the cone to the volume of the sphere? Express your answer as a common fraction",
"Solution_1": "Draw the radius from the center of the sphere to a point on the circumference of the base of the cone. Then, the distance from the center of the sphere to the center of the base of the cone is $ \\sqrt {5^2 \\minus{} 4^2} \\equal{} 3$, so the height of the cone is 3+5=8. The volume of the sphere is $ \\frac {4}{3}5^3\\pi \\equal{} \\frac {500\\pi}{3}$, and the volume of the cone is $ \\frac {1}{3}4^2\\times 8\\pi \\equal{} \\frac {128\\pi}{3}$, so the desired ratio is $ \\frac {\\frac {128\\pi}{3}}{\\frac {500\\pi}{3}} \\equal{} \\frac {32}{125}$",
"Solution_2": "Isnt this question from the handbook?"
}
{
"Tag": [
"number theory proposed",
"number theory"
],
"Problem": "Find all natural numbers $n$ less than 200 such that $n^2+(n+1)^2$ is a perfect square.",
"Solution_1": "$n=k^2-m^2$ and $n+1=2km$\r\nor\r\n$n=2km$ and $n+1=k^2-m^2$ \r\n\r\nand in each of these cases solving it like the Pell's equation."
}
{
"Tag": [
"induction",
"LaTeX",
"strong induction",
"combinatorics unsolved",
"combinatorics"
],
"Problem": "There are two strips. On the first is written letter $ A$, and on the second letter $ B$. In each minute, John writes all the letters presented on one strip on another (in the same sequence), either to the right or to the left of the existing letters. (For example, if at a moment the two strips are $ BAB$ and $ ABB$, John can write to the right of the first strip to make the two strips $ BABABB$ and $ ABB$.) Prove that at any moment, the letters on each strip form two palindromes, one of which could be empty. (For example, $ BBA$ is the palindrome $ BB$ and the palindrome $ A$.)",
"Solution_1": "[quote=\"hollandman\"]There are two strips. On the first is written letter $ A$, and on the second letter $ B$. In each minute, John writes all the letters presented on one strip on another (in the same sequence), either to the right or to the left of the existing letters. (For example, if at a moment the two strips are $ BAB$ and $ ABB$, John can write to the right of the first strip to make the two strips $ BABABB$ and $ ABB$.) Prove that at any moment, the letters on each strip form two palindromes, one of which could be empty. (For example, $ BBA$ is the palindrome $ BB$ and the palindrome $ A$.)[/quote]\r\n\r\nPosted before, but nobody wrote a solution.\r\n\r\nTo make the problem statement clearer, John doesn't need to copy the two strips one after another. That is, he can copy the second one to right or left of the first strips multiple times successively.\r\n\r\nThis problem is VERY VERY tricky and hard. I had it solved but can only supply some basic ideas. Several suggestions.\r\n\r\n1.Use strong induction on number of steps\r\n2.At the beginning, Let's say John copy B to the first strip p time left and q time right. If p=q, then the thing on the first strip is still a palindrome and we can look at it as a whole. (i.e. BBABB can be simply denoted as C) If p>q, we have on the first strip like BBBAB. Make it into a palindrome and several B's either to left or right. So BBBAB becomes BBC, or BBABBBB becomes CBB. After that, John should start to copy the thing on the first strip to the other one at least once. At this point, things are getting complex. However, the main goal is to show that several steps later, you can always reach a situation that can be achieved using less steps if you change A and B at the beginning to some other palindromes. This is when the strong induction applies.",
"Solution_2": "Hmm.. I would appreciate it if you could write a complete solution. I'm trying to understand your basic ideas, but they seem complicated.",
"Solution_3": "Sorry, I will try to explain some detail when I have time. However, since I have few knowledge regarding Latex, it will be very hard for me to draw a diagram.\r\n\r\nHave you at least understand all the thing before the \"After that\" in my second point? If so, just start copy things on the first strip to the second strip and you might see some magic happening."
}
{
"Tag": [
"modular arithmetic",
"algebra",
"function",
"domain",
"quadratics",
"Ring Theory",
"algorithm"
],
"Problem": "Let $p$ be a prime number, and suppose that $x$ is an integer such that $x^2 \\equiv -2 \\ mod p$.By cnsidering $u+xv$ for various pairs of $(u,v)$.show that at least one of the equations $a^2+2b^2=p, a^2+2b^2=2p$has a solution.",
"Solution_1": "First, note that $x^2 + 2\\cdot 1^2 = mp$ for some positive integer $m$. \r\n\r\nChoose $u,v$ from the interval $[-m/2,m/2]$ so that $u\\equiv x \\pmod m$ and $v\\equiv 1\\pmod m$. Then $mr=u^2+2v^2$ where \\[r=\\frac{1}{m}(u^2+2v^2)\\leq \\frac{1}{m}(\\frac{m^2}{4}+\\frac{m^2}{2})1$. Show that there exists a positive integer $ k$ for which $ u_k\\equal{}0$.",
"Solution_1": "[quote=\"moldovan\"]Suppose that $ u$ is a real parameter with $ 0 < u < 1$. Define $ f(x) \\equal{} 0$ if $ 0 \\le x \\le u$, and $ f(x) \\equal{} 1 \\minus{} (\\sqrt {ux} \\plus{} \\sqrt {(1 \\minus{} u)(1 \\minus{} x)})^2$ if $ u \\le x \\le 1$, and define the sequence $ u_n$ recursively by $ u_1 \\equal{} f(1)$ and $ u_n \\equal{} f(u_{n \\minus{} 1})$ for all $ n > 1$. Show that there exists a positive integer $ k$ for which $ u_k \\equal{} 0$.[/quote]\r\n\r\nThe key here is to see that $ f(x)0$. So $ u_n$ is a positive non increasing sequence, so converging towards the only fixed point of $ f(x)$, so $ 0$. \r\nSo, for $ n$ great enough, $ u_n\\leq u$ and $ u_{n\\plus{}1}\\equal{}0$"
}
{
"Tag": [],
"Problem": "[b]In a class 54 % of the student are girls and 16 % are boys.If the number of girls is 27 , find the total number of student ?[/b]",
"Solution_1": "where did the other 30% go?\r\nAs if there is none, this is a FAILED question :(",
"Solution_2": "The (fixed) problem (I'm guessing) is here http://www.artofproblemsolving.com/Forum/viewtopic.php?t=302062",
"Solution_3": "Didn't you make this thread twice ra ny?\r\n\r\nUh 30% are neither girls nor boys??"
}
{
"Tag": [
"number theory unsolved",
"number theory"
],
"Problem": "Prove : equation $ x^2\\minus{}2y^2\\plus{}8z^3\\equal{}3$ don't have solution $ x,y,z\\in N$",
"Solution_1": "Check both sides $ mod \\;\\; 4$ we have $ x\\equal{}2a\\plus{}1, y\\equal{}2b\\plus{}1$ are odd.\r\nSo the equation becomes $ 4a^2\\plus{}4a\\minus{}8y^2\\minus{}8y\\equal{}4 \\Leftrightarrow a(a\\plus{}1)\\minus{}2y^2\\minus{}2y\\equal{}1$\r\n$ L.H.S.$ is even but $ R.H.S.$ is odd. So there is no solution.",
"Solution_2": "please post this in the pre olympiad section"
}
{
"Tag": [
"linear algebra",
"matrix",
"linear algebra unsolved"
],
"Problem": "Solve the following matrix system: $\\{\\begin{array}{cc}AX-BY=C \\\\ BX-AY=C \\end{array}$ where $A,B,C,X,Y\\in\\mathcal l M_{n}(R)$ and $A+B,A-B$ are not singular matrix.",
"Solution_1": "No one? C`mon guys :D !",
"Solution_2": "So let`s see .... by adding the two equations we get $(A+B)(X-Y)=2C \\rightarrow X-Y=2C(A+B)^{-1} \\rightarrow X=Y+2C(A+B)^{-1}$ and by substracting one from another we get:\r\n$(A-B)(X+Y)=O_n\\rightarrow (A-B)[2Y+2C(A+B)^{-1}]=O_n$ \r\n$\\rightarrow (A-B)Y=-(A-B)C(A+B)^{-1}$\r\n$\\rightarrow Y=-C(A+B)^{-1} \\rightarrow X=C(A+B)^{-1}$",
"Solution_3": "Well done ;)!"
}
{
"Tag": [],
"Problem": "How many factors of 1800 are multiples of 10?",
"Solution_1": "I'm not so smart, so I did it in probably the longest way possible. :D \r\n\r\n[hide]Anyway, I listed all of the factors and came out with these:\n\n1800: 1, [color=red]1800[/color], 2, [color=red]900[/color], 3, [color=red]600[/color], 5, [color=red]360[/color], 6, [color=red]300[/color], 9, [color=red]200[/color], [color=red]10[/color], [color=red]180[/color], 18, [color=red]100[/color], [color=red]20, 90, 30, 60, 50[/color], 36, [color=red]60, 30, 90, 20, 100,[/color] 18, [color=red]200[/color], 9, [color=red]300[/color], 6, [color=red]600[/color], 3, [color=red]900[/color], 2. That was all I came up with. :( \n\nThen I counted up the factors:\n1800, 900, 600, 360, etc.\n\nFinal Answer: 23 factors are multiples of ten.\n\nPlease tell me if this is wrong, thank you...also tell me any shortcut way[/hide]",
"Solution_2": "YOu counted numbers like 600 and 900 twice, and there's definetly a better solution (should just take 35 seconds max) with prime factorization.",
"Solution_3": "Oops...sorry. :blush: \r\n\r\n[hide]Here's my answer now:\n\nAfter a close examination of my solution, I found out I had repeated a LOT of numbers.\n\nNow my answer is 14 factors.\n\nAnd also, I seriously need to know the 35 seconds way.[/hide]",
"Solution_4": "No, it is incorrect. This is one of two ways to solve this that I know (without listing). Contribute other ways. \r\n\r\n1800 = 180*10. \r\nSo any factor of 180 times 10 is a factor of 1800 (and a multiple of 10)!. \r\nWe need to find the number of factors in 180. \r\n\r\n180 = $2^2\\cdot3^2\\cdot5$. That means 3*3*2 = 18 factors.",
"Solution_5": "[hide=\"woot!!!\"]\ni found all the factors of 180 and then multiplied them by 10. There are 18 factors. ( i don't know how to do this by prime factorization). So the final answer is [b]28[/b][/hide]",
"Solution_6": "[quote=\"236factorial\"]How many factors of 1800 are multiples of 10?[/quote]\r\n\r\nUsing prime factorizaton, i get 18",
"Solution_7": "[hide]Well $1800=2^3\\cdot3^2\\cdot5^2$ We have to have at least one $2$ and one $5$, so from the exponents we have $3\\cdot3\\cdot2=18$[/hide]",
"Solution_8": "[quote=\"math92\"][hide=\"woot!!!\"]\ni found all the factors of 180 and then multiplied them by 10. There are 18 factors. ( i don't know how to do this by prime factorization). So the final answer is [b]28[/b][/hide][/quote]\r\n\r\nDid you make a typo? I can't see where 28 came from.",
"Solution_9": "[quote=\"236factorial\"][quote=\"math92\"][hide=\"woot!!!\"]\ni found all the factors of 180 and then multiplied them by 10. There are 18 factors. ( i don't know how to do this by prime factorization). So the final answer is [b]28[/b][/hide][/quote]\n\nDid you make a typo? I can't see where 28 came from.[/quote]\noops, :blush: \nthat was a typo\n[hide]18 is the real answer[/hide]",
"Solution_10": "[quote=\"math92\"][quote=\"236factorial\"][quote=\"math92\"][hide=\"woot!!!\"]\ni found all the factors of 180 and then multiplied them by 10. There are 18 factors. ( i don't know how to do this by prime factorization). So the final answer is [b]28[/b][/hide][/quote]\n\nDid you make a typo? I can't see where 28 came from.[/quote]\noops, :blush: \nthat was a typo\n[hide]18 is the real answer[/hide][/quote]\r\n\r\ni dont think so, 1800 has 18 different factors , but the factors thats a multiple of 10 is not 18 ( 6 for example isnt a multiple of 10 :s)",
"Solution_11": "[quote=\"Mo\u00b2\"]i dont think so, 1800 has 18 different factors , but the factors thats a multiple of 10 is not 18 ( 6 for example isnt a multiple of 10 :s)[/quote]\r\nNo, $1800=2^33^25^2$ $4\\cdot3\\cdot3=36$ There has to be a $2$ and a $5$ to make a $10$, so you can have $1,2,3,$ 2's, $0,1,2$ 3's, and $1,2$ 5's. $3\\cdot3\\cdot2=18$",
"Solution_12": "I got it!:)",
"Solution_13": "[quote=\"loohcsnuf\"]I got it!:)[/quote]\r\nthen what did you get?",
"Solution_14": "[quote=\"loohcsnuf\"]I got it!:)[/quote]\r\nwhat did you get?"
}
{
"Tag": [
"algebra",
"polynomial",
"trigonometry",
"algebra solved"
],
"Problem": "$ f(x)=a_1cos x + b_1sinx + a_2 cos (2x) + b_2 sin (2x) $\r\n Prove that : If $ f(x) \\leq \\sqrt{a_2^2+b_2^2} $ then $ a_1=b_1=0 $",
"Solution_1": "Let $\\theta$ satisfies $a_{2}\\cos 2\\theta+b_{2}\\sin 2\\theta=\\sqrt{a_{2} ^2+b_{2} ^2}$.\r\nSubstitute $x=\\theta$ ,and $x=\\theta+\\pi$ and you will get it.\r\n\r\nNo war\r\n\r\n[i]Edited by Myth[/i]",
"Solution_2": ":D Sorry, the proof is not enough :D\r\n :cool: Try again ! :cool:",
"Solution_3": "Exist $a,b$ st:\r\n$f(x)=usin(x+a)+vcos(2x+b)$\r\nwith $u=\\sqrt{a_1^2+b_1^2}$\r\n $v=\\sqrt{a_2^2+b_2^2}$\r\n$f(x)\\le v<->v(1-cos(2x+b))-usin(x+a) \\ge 0$ for all $x$\r\n$<->2vsin^2(x+\\frac{b}{2})+usin(x+a+pi) \\ge 0$ for all $x$\r\nthen exist $c$:\r\n$2vsin^2(x-c)+usin(x) \\ge 0$ for all $x$.\r\nwe can assume that $-pi \\le c \\le pi$\r\nFor $x=c->sinc \\ge 0 ->0\\le c \\le pi$\r\nFor $x=c-pi->sin(c-pi)\\ge0->0\\le c-pi\\le pi$\r\nThen $c=0,pi,-pi$\r\n$->2vsin^2(x)+usinx \\ge 0 for all x$\r\n$->u=0$"
}
{
"Tag": [
"function"
],
"Problem": "prove that for prime $ p > 2$ , $ p|[(2 \\plus{} \\sqrt {5})^p] \\minus{} 2^p *2^1$",
"Solution_1": "We have the question \r\n$ \\Leftrightarrow p|(2\\plus{}\\sqrt{5})^p\\plus{}(2\\minus{}\\sqrt{5})^p\\minus{}2^p \\times 2$\r\n$ \\equal{}2(_p C_2 2^{p\\minus{}2}(5)\\plus{}_p C_4 2^{p\\minus{}4}(5^2)\\plus{}...\\plus{}_p C_{p\\minus{}1} 2^{1}(5^{\\frac{p\\minus{}1}{2}})$\r\n\r\nObviously, $ _p C_k\\equal{}\\frac{p!}{(p\\minus{}k)!(k)!}$, \r\nso for $ 1 \\le k \\le p\\minus{}1$, $ p|nominator$ but $ p \\nmid denominator$ $ \\Rightarrow p|_p C_k$, \r\nSo $ p|[(2 \\plus{} \\sqrt {5})^p] \\minus{} 2^p *2^1$, done.",
"Solution_2": "stephencheng i think this is what you meant\r\n$ (2 \\plus{} \\sqrt{5})^p \\equal{}$ integer say $ I$ + some number $ k$ where $ k$ is a number $ >0,<1$\r\nso gif of this $ (2 \\plus{} \\sqrt{5})^p$ is $ I$ and \r\n$ (2 \\minus{} \\sqrt{5})^p$ is a negative fraction say $ h$\r\nadding both we get \r\n$ (2 \\plus{} \\sqrt{5})^p \\minus{} (2 \\minus{} \\sqrt{5})^p \\equal{}I \\plus{} k \\plus{} h$ but\r\n$ (2 \\plus{} \\sqrt{5})^p \\minus{} (2 \\minus{} \\sqrt{5})^p$ is an integer (by the binomial expansion we get so)\r\nso $ I\\plus{}k \\plus{}h$ is an integer which implies $ k \\plus{} h$ is an integer \r\nwe have $ 1>k>0 ,\\minus{}1 \\frac{n\\minus{}1}{n\\plus{}2}.\\]",
"Solution_1": "By Cauchy-Schwarz in Engel Form, we have\r\n\\begin{align*}\r\n\\sum^{n-1}_{i=1} \\frac{1}{P_i + P_{i+1}}&>\\frac{(n-1)^2}{2(P_1+P_2+\\ldots+P_n)-P_1-P_n}\r\n\\\\&=\\frac{(n-1)^2}{2(1+2+\\ldots+n)-P_1-P_n}\r\n\\\\&=\\frac{(n-1)^2}{n(n+1)-P_1-P_n}\r\n\\\\&\\ge\\frac{(n-1)^2}{n(n+1)-(n-1)-n}\r\n\\\\&=\\frac{(n-1)^2}{n^2-n+1}\r\n\\\\&>\\frac{(n-1)^2}{n^2+n-2}\r\n\\\\&=\\frac{n-1}{n+2}.\\end{align*}",
"Solution_2": "So cute :) First, the statement is altogether wrong; there are [b]not[/b] $n$[b] permutations [/b]$P_1,P_2,\\ldots,P_n$, each being an arbitrary permutation of $\\{1,2,\\ldots,n\\}$, since $\\dfrac {1} {P_i + P_{i+1}}$ makes no sense as a number, but rather $n$ [b]numbers[/b] $P_1,P_2,\\ldots,P_n$, being an arbitrary[b] permutation [/b]of $\\{1,2,\\ldots,n\\}$. Second, the proof is quite nice, up to a patently reversed inequality; the right form is\n\\begin{align*}\n\\sum^{n-1}_{i=1} \\frac{1}{P_i + P_{i+1}}&>\\frac{(n-1)^2}{2(P_1+P_2+\\cdots+P_n)-P_1-P_n}\n\\\\&=\\frac{(n-1)^2}{2(1+2+\\cdots+n)-P_1-P_n}\n\\\\&=\\frac{(n-1)^2}{n(n+1)-P_1-P_n}\n\\\\&\\ge\\frac{(n-1)^2}{n(n+1)-1-2}\n\\\\&=\\frac{(n-1)^2}{n^2+n-3}\n\\\\&>\\frac{(n-1)^2}{n^2+n-2}\n\\\\&=\\frac{n-1}{n+2}.\\end{align*}\n\nMost likely the best such permutation is $1,n,2,n-1,\\ldots, \\lfloor n/2\\rfloor +1$.",
"Solution_3": "Could you use a smoothing argument to turn all the fractions into 1/(n+1)? I turned $p_i < p_j < p_k < p_l $ into $ \\frac{1}{p_i+p_j}+\\frac{1}{p_k+p_l} > \\frac{1}{p_i+p_l} + \\frac{1}{p_j+p_k}$ but I'm not sure if one can treat this as an invariant and solve the problem.\n\n*Also Cauchy-Schwarz in Engel-Form = Titu's Lemma for those who read the solutions above and were a bit confused on what that was. (#me)",
"Solution_4": "You can see also here for a very similar problem. http://artofproblemsolving.com/community/c6h17331p118699",
"Solution_5": "$\\noindent \\textit {Sketch}$: use the HM-AM inequality, in which your terms are $P_i+P_{i+1}$, for $i\\in \\{1, 2\\ldots n-1\\}$ (a total of $n-1$ terms)."
}
{
"Tag": [
"videos"
],
"Problem": "How will the twin impacts of the Lunar Crater Observation and Sensing Satellite, or LCROSS conducted by NASA affect science?\r\n\r\nOnly five and a half hours remain.Check this out and do express your views on the subject!\r\n\r\n[hide][url]http://www.nasa.gov/mission_pages/LCROSS/main/index.html[/url][/hide]",
"Solution_1": "The mission seems to have been successfull. Now we just have to wait for the results.",
"Solution_2": "I totally want to see that being played on the news, and then have someone hack in and edit the video so the moon blows up at the end. \"NASA invented a weapon capable of destroying THE MOON!\""
}
{
"Tag": [
"function",
"trigonometry",
"logarithms",
"integration",
"calculus",
"calculus computations"
],
"Problem": "(1) Find the general solution of the following differential equation.\r\n\r\n$ \\frac {dy}{dx} \\plus{} P(x)y \\equal{} Q(x)$ where $ P(x),\\ Q(x)$ are functions of $ x$.\r\n\r\n(2) Solve the following differential equation.\r\n\r\n$ x\\frac {dy}{dx} \\plus{} 2y \\equal{} \\sin x$",
"Solution_1": "For later easier notation, suppose we have:\r\n\r\n$ \\frac {dy}{dx} + y p(x) = q(x)$\r\n\r\nThen\r\n\r\n$ \\frac {dy_h}{dx} + y_h p(x) = 0$\r\n\r\n$ \\frac {dy_h}{ y_h} = - p(x) dx$\r\n\r\n$ \\ln y_h = - \\int p(x) \\,dx + C = - P(x) + C$\r\n\r\n$ y_h = C e^{ - P(x)} \\Rightarrow y = C(x) e ^{ - P(x)}$\r\n\r\n$ \\frac {dy}{dx} = C'(x) e ^{ - P(x)} - C(x) p(x) e ^{ - P(x)}$\r\n\r\nTherefor:\r\n\r\n$ C'(x) e ^{ - P(x)} - C(x) p(x) e ^{ - P(x)} + C(x) e ^{ - P(x)} p(x) = q(x)$\r\n\r\n$ C'(x) e ^{ - P(x)} = q(x)$\r\n\r\n$ C'(x) = q(x) e ^{P(x)}$\r\n\r\n$ C(x) = \\int q(x) e ^{P(x)} \\,dx + K$\r\n\r\nAnd finally:\r\n\r\n$ y = e^{ - P(x)} (\\int q(x) e ^{P(x)} \\,dx +K)$ with $ P(x) = \\int p(x) \\,dx$\r\n\r\nSolving the second problem is easy. We have $ p(x) = \\frac {2}{x}$ and $ q(x) = \\frac {\\sin x}{x}$. This gives us\r\n\r\n$ P(x) = \\int \\frac {2}{x} \\,dx = 2 \\ln x$\r\n\r\nand with this:\r\n\r\n${ y = \\frac {1}{x^2} (\\int \\frac {\\sin x}{x} x^2} \\,dx + C) = \\frac {1}{x^2} (\\sin x - x \\cos x + C) = \\frac {\\sin x}{x^2} - \\frac {\\cos x}{x} + \\frac {C}{x^2}$",
"Solution_2": "That's correct.\r\n\r\nAlternative Solution for (1):\r\n\r\nMultiply by integerating factor $ e^{\\int P(x)dx}$ both sides of the differential equation, we have \r\n\r\n$ \\frac {d}{dx} e^{\\int P(x)dx}y \\equal{} e^{\\int P(x)dx}Q(x)$\r\n\r\n$ \\therefore y \\equal{} e^{ \\minus{} \\int P(x)dx}\\left(e^{\\int P(x)dx}Q(x)dx \\plus{} C\\right)$.",
"Solution_3": "I have a feeling these are going to get harder real quick.",
"Solution_4": "HINT for 2::\r\ndivide the eq. by x.Find Integrating factor which is equal to x^2.\r\n\r\nAnswer of integral ::\r\ny*(x^2)=sin x - x cos (x) + C",
"Solution_5": "[quote=\"kabi\"]HINT for 2::\ndivide the eq. by x.Find Integrating factor which is equal to x^2.\n\nAnswer of integral ::\ny*(x^2)=sin x - x cos (x) + C[/quote]\r\n\r\nThat's correct. :)",
"Solution_6": "[quote=\"JRav\"]$ \\boxed{1}$\nSet $ \\mu(x) \\equal{} \\exp\\left(\\int P(x)\\ dx\\right)$. Then some algebra reveals that\n$ y(x) \\equal{} \\frac {1}{\\mu(x)}\\int\\mu(x)Q(x)\\ dx \\equal{} \\exp\\left( \\minus{} \\int P(x)\\ dx\\right)\\int\\left(\\exp\\left(\\int P(x)\\ dx\\right)Q(x)\\ dx\\right)$\n\nDid I handle this correctly?[/quote]\r\n\r\nYes, you did."
}
{
"Tag": [
"group theory",
"abstract algebra",
"superior algebra",
"superior algebra solved"
],
"Problem": "I like this one:\r\n\r\nFor any finite group G define n(G) as the number of elements of G and s(G) as the number of subgroups of G. Determine whether the following sentences are true or false:\r\n\r\na) For any a>0 there is a finite group G s.t. n(G)/s(G)0 there is a finite group G s.t. n(G)/s(G)>a.\r\n\r\n[i]Romanian NMO/final round/Alba Iulia 1999/2'nd problem/12'th grade/proposed by Barbu Berceanu[/i]",
"Solution_1": "For any finite group G define n(G) as the number of elements of G and s(G) as the number of subgroups of G. Determine whether the following sentences are true or false: \r\n\r\na) Take a commutative G where the order of every element is 2 (such groups exist: Z2*Z2*...*Z2 is one example).\r\nlet k=n(G)\r\nFor any x<>y in G-{e} we have that e,x,y,xy is a subgroup of G. There are (k-1)(k-2)/6 such subgroups.\r\nSo s(G)>=(k-1)(k-2)/6 -> n(G)/s(G) <= k/( (k-1)(k-2)/6 ) which goes to 0 as n goes to infinity.\r\n\r\nb) Take (Zp,+) for p prime and get n(G)/s(G) = p/2>a",
"Solution_2": "Thx! This was the official solution as well. There's a similar problem, only using the number of elements (n) and the number of multiplicative operations which can be defined on Zn s.t. it's a ring. Try it (it's also composed by Barbu Berceanu).",
"Solution_3": "Can there be defined more than one multiplicative operation on Zn?",
"Solution_4": "Of course not :D. Here's the real problem:\r\n\r\n(G,+) is a commutative group with n(G) elements. i(G) is the number of laws (G,*) s.t. (G,+,*) is a ring. Find out if there exists a sequence of groups Hk (commutative and finite groups) s.t. \r\n\r\n(a) n(Hk)/i(Hk)->0 as k->00;\r\n(b) n(Hk)/i(Hk)->00 as k->00.",
"Solution_5": "b) This is the easy part. you only need to take Zn and use the fact that only one multiplicative law makes it a ring and get n/1->00 :) \r\n\r\na) I will consider the group Gn=(Z2,+)x(Z2,+)x...x(Z2,+).\r\nDenote:\r\n e1=(1,0,...,0)\r\n e2=(0,1,...,0)\r\n ...\r\n en=(0,0,...,1)\r\nEvery element of Gn can be written uniquely as a sum of some of this elements. So it is enough to define the multiplicative law for this elements.\r\nIn order to have a unit element, define e1ei=eie1=ei\r\nThe other n2-2n+1=(n-1)2 products may take any of the 2n values in Gn.\r\nSo n(Gn)/i(Gn)=2n/((n-1)2*2n)=1/(n-1)2->0 :)",
"Solution_6": "I guess it's Ok. This is the 4'th problem from the final round of the RMO in 2003, 12'th grade",
"Solution_7": "There is something wrong in my proof: for Zn we may choose fi(n) multiplicative laws, because the unit element is not necesarily 1. (fi(n) is Euler's function).\r\n\r\nBut the result is still OK as pn/fi(pn)=p and the set of primes goes to 00. :D",
"Solution_8": "This is easier than what I tried. I used the fact that \\pi{i=1->n}pi/phi(\\pi{i=1->n}pi)->00 as n->00, but it's a bit harder to prove (not really hard, but your idea is definitely nicer)."
}
{
"Tag": [
"calculus",
"integration",
"logarithms",
"function",
"calculus computations"
],
"Problem": "Let $ f(x)\\equal{}[x(1\\plus{}(\\ln x)^2)]^{\\minus{}q/p}$.\r\nFix $ p \\in (0,\\infty)$.\r\nFor what $ q \\in (0,\\infty)$ will the integral of $ f(x)$ over $ (1,\\infty)$ exsists(finite)?\r\nFor what $ q \\in (0,\\infty)$ will the integral of $ f(x)$ over $ (0,1)$ exsists(finite)?\r\nThanks for any help.",
"Solution_1": "Hint: $ \\minus{} \\frac {d}{dx}\\frac {1}{\\log x} \\equal{} \\frac {1}{x(\\log x)^2}$, and the presence of the $ 1$ has negligible effect on the function near $ \\infty$ and near $ 0$ (which is where convergence or divergence of this integral is determined).",
"Solution_2": "I still don't know how to do the second part.\r\nCould someone explain more?",
"Solution_3": "This is simply a matter of comparison tests for the integrability of a positive function.\r\n\r\nFor large $ x,$ $ f(x)\\approx \\frac1{x^{q/p}(\\ln x)^{2q/p}}.$\r\n\r\nFor $ \\frac{q}{p}\\ne 1,$ it's the power of $ x$ that matters, and we have convergence for $ \\frac{q}p>1$ and divergence for $ \\frac qp<1.$\r\n\r\nThe borderline case is $ \\frac qp\\equal{}1,$ in which case we have essentially $ \\int_?^{\\infty}\\frac1{x(\\ln x)^2}\\,dx.$ That's a convergent integral (which is what JoeBlow was saying).\r\n\r\nFor $ x$ near zero, we also have $ f(x)\\approx \\frac1{x^{q/p}(\\ln x)^{2q/p}}.$\r\n\r\nThis time, we have convergence for $ \\frac qp<1$ and divergence for $ \\frac qp>1.$\r\n\r\nLooking at the borderline case again, and let $ u\\equal{}\\minus{}\\ln x$ :\r\n\r\n$ \\int_0^?\\frac1{x(\\ln x)^2}\\,dx\\equal{}\\int_?^{\\infty}\\frac{du}{u^2},$ which converges.\r\n\r\n(Note: the use of a question mark as a limit of integration simply means some value which doesn't cause us any unnecessary problems.)",
"Solution_4": "[color=green][Unneeded quote removed by moderator.][/color]\r\n\r\nWhy we can neglect the effect of $ (\\ln x)^{2q/p}$ near 0?\r\n\r\nOk,I see.\r\nThanks for your help.",
"Solution_5": "If $ x$ has any exponent other than $ \\minus{}1,$ than we can peel of a sliver of a power of $ x$ to overwhelm any logarithm. That's why we have divergence near zero for $ \\frac qp>1.$ On the other hand, if $ \\frac qp\\equal{}1,$ then we can't neglect the logarithm at all; it really matters. That last line of mine is a change of variables.\r\n\r\nAlso: please don't quote the entire previous post; that's what you're assumed to be replying to anyway. Use the \"reply\" button in most cases; only use \"quote\" to quote a specific portion of a post not the whole thing."
}
{
"Tag": [],
"Problem": "$a,b\\in\\mathbb{N}$ and $p=\\frac{b}{4}\\sqrt{\\frac{2a-b}{2a+b}}$ is a prime,\r\nfind the maximum value of $p$(with proof! :) )",
"Solution_1": "[hide=\" \"]$p=\\frac{b}{4}\\sqrt{\\frac{2a-b}{2a+b}}$ is prime.\n$p=\\frac{b}{8a+4b}\\sqrt{4a^2-b^2}$ is prime. Make c=2a:\n$p=\\frac{b}{4c+4b}\\sqrt{c^2-b^2}$ is prime for $b\\in\\mathbb{N}$ and c is even.\nMultiply by 4 and square:\n$16p^2=\\frac{b^2(c+b)(c-b)}{(b+c)^2}=\\frac{b^2(c-b)}{c+b}$.\nSince $16p^2$ has 15 factors, so does $\\frac{b^2(c-b)}{c+b}$...[/hide]\r\n\r\nNevermind. Just being my ordinary stupid self. I ought to get better at olympiad problems before I use this forum...",
"Solution_2": "hmm, i don't know where you're going with this, but i'm interested... :)",
"Solution_3": "it is also on the forum .thats is for iran olympiads.",
"Solution_4": "yes it is correct, it is from the iran's 16th, (i think 1998) olympiad, second round",
"Solution_5": "[hide=\"hint\"]\n$p^2=\\frac{b^2}{16}\\frac{2a-b}{2a+b}$\n$b$ must be an even number[/hide]",
"Solution_6": "I was able to do a little simplification, but not much more.\r\n\r\n[hide]\nb obviously must be even. Let $b = 2b_0$. Then it simplifies to \n\\[\np = \\frac{b_0}{2} \\sqrt{\\frac{a-b_0}{a+b_0}}\n\\]\nNow because $\\frac{a-b_0}{a+b_0} = \\frac{a^2 - b_0^2}{(a+b_0)^2}$, we can rewrite it as\n\\[\np = \\frac{b_0}{2} \\cdot \\frac{\\sqrt{a^2 - b_0^2}}{a+b_0}\n\\]\n$(\\sqrt{a^2-b_0^2},b_0, a)$ must be a Pythagorean Triple, so let $a = r^2+s^2$ and $b_0$ can be either $2rs$ or $r^2-s^2$. No matter which $b_0$ we chose, the equation for $p$ simplifies to the same thing, so for now let $b_0 = 2rs$. ($r$ and $s$ are integers.) We have $a^2-b_0^2 = (r^2-s^2)^2$ and $a + b_0 = (r+s)^2$, so it simplifies to\n\\[\np = rs \\cdot \\frac{r^2-s^2}{(r+s)^2} = rs \\cdot \\frac{r-s}{r+s}.\n\\]\n(note how b_0/2 = 2rs/2 = rs) This looks much nicer, but I have no clue what to do with it :P\n[/hide]\r\n\r\nI didn't get any farther than that...",
"Solution_7": "i dont think that would help you, don't do the $r$ and $s$ thing... ;)",
"Solution_8": "you guys, want me to post the proof?",
"Solution_9": "since no one answered this, i wont hold this anymore, here is my proof( i will do half of the problem and leave the other half to you! :) ):\r\n$ p^2=\\frac{b^2}{16}\\frac{2a-b}{2a+b}$ \r\nlet $b=2c$ so:\r\n$p^2=\\frac{c^2(a-c)}{4(a+c)}\\Rightarrow 4p^2=c^2(\\frac{a+c-2c}{a+c})=c^2(1-\\frac{2c}{a+b})\\Rightarrow 4p^2=c^2-\\frac{2c^3}{a+b} \\Rightarrow c^2-4p^2=\\frac{2c^3}{a+b}\\Rightarrow a+c=\\frac{2c^3}{c^2-4p^2}\\Rightarrow c^2-4p^2|2c^3$\r\n$(c^2-4p^2)(-2c)+(2c^3)=8p^2c-2c^3+2c^3=8p^2c \\Rightarrow c^2-4p^2=(c^2-4p^2, 2c^3)|8p^2c$\r\nnow there is two situations, first: $p\\not |c$ and second: $p|c$\r\nif $p\\not |c$ then $c^2-4p^2$ and $p^2$ are primitive thus:\r\n$c^2-4p^2|8p^2c \\Rightarrow c^2-8p^2|8c$\r\n$(c^2-4p^2)(-8)+(8c)(c)=-8c^2+32p^2+8c^2=32p^2 \\Rightarrow c^2-4p^2=(c^2-4p^2, 8c)|32p^2 \\Rightarrow c^2-p^2|32$ (because $c^2-4p^2$ and $p^2$ are perimitive)\r\n$c^2-4p^2=(c-2p)(c+2p)|2^5$ because $c-2p$ and $c+2p$ are the same, so both must be even, also $c-2p 0, the parabola opens upward, therefore the vertex is minimum. When a < 0, the parabola opens downward, so the vertex is maximum. I hope this helps.\r\n\r\nTo find the intercepts, if you want to find x, substitute y to 0, the reverse (of course) if otherwise.",
"Solution_2": "hERES A PROBLEM GIVEN TO US AT mATHpATH...FOR SOME REASON, ITS NO LETTIN ME WRITE IS LOWERCASE\r\n\r\nONE STORMY NIGHT, A PIRATES SHIP GETS CAUGHT IN A STORM. THE SHIP IS SHIPRECKED BAND EVERYONE DIES...EXCEPT 5. THEY SWIM TO SHORE SAFELY AND START GATHERING AS MANY COCONUITS AS THEY CAN FIND. AT THE END IOF THE DAY, THEY PILE ALL THE COCONUTS TOGETHER AND DECIDE TO SPLIT THE PILE TOMMOROW. THE FIRST SAILER DOESNT TRUST THE OTHERS SO HE TAKES ONE-FIFTH OF THE PILE AND BURYS IT WILE EVERYONE IS SLEEPING. THERE IS ONE LEFT OVER SO HE GIVES IT TO THE MONKEY. SAILER 2 DOES THE SAM...SO DOES 3,4, AND 5\r\n\r\nIN THE MORNING, THEY START TO SLIT THE PILE...NONE OF THEM SAY ANYTHING BECAUSE THEY ALL TOOK SOME..AFTER SPLITTING THE PILE, THERE IS ONE LEFT OVER SO THEY GIVE IT TO THE MONKEY.\r\n\r\nWHAT IS THE SMALLEST POSSIBLE NUMBER OF COCONUTS THEY COULD HAVE FOUND.\r\n\r\np.s. TRY TO ALSO FIND A NEGATIVE SOLUTION"
}
{
"Tag": [
"trigonometry"
],
"Problem": "tan^2(1) + tan^2(2) + tan^2(3) + ......... + tan^2(89) = ?\r\n\r\n sin^2(1) + sin^2(2) + sin^2(3) + ..... +sin^2(89) = ?\r\n\r\n sin(20)sin(40)sin(80) = ?",
"Solution_1": "For second use $ sin \\theta \\equal{} \\cos(\\frac {\\pi}{2} \\minus{} \\theta)$\r\nand $ \\sin^{2}\\theta \\plus{} \\cos^{2}\\theta \\equal{} 1$\r\n\r\nFor third use $ 4\\sin A\\sin (60 \\minus{} A)\\sin (60 \\plus{} A) \\equal{} \\sin 3A$\r\nso answer will be $ \\frac {\\sin60^\\circ}{4} \\equal{} \\frac {\\sqrt {3}}{8}$\r\n\r\nand 1^{st }is very trivial",
"Solution_2": "$ 4\\sin A\\sin (60 \\minus{} A)\\sin (60 \\plus{} A) \\equal{} \\sin 3A$ is a useful identity, but back in the day (hah, :roll: ) when I was going through high school, people didn't promote that identity much.\r\n\r\nIn general, a useful approach is to try to write the expressing using as few \"weird\" angles and as many \"common\" angles as possible, and then try to simplify the expression. Oftentimes, the \"weird\" angles will cancel out, or sometimes, such as in this case, you'll end up with \"common\" angles.\r\n\r\nLet $ S_\\theta \\equal{} \\sin \\theta$ and $ C _\\theta \\equal{} \\cos \\theta$, for shorthand.\r\n\r\nLet's see how this plays out in this case:\r\n\r\n[hide=\"Weird angle is 10\"]$ S_{20} S_{40} S_{80}$\n\n$ \\equal{} S_{(30 \\minus{} 10)} S_{(30 \\plus{} 10)} C_{10}$\n\n$ \\equal{} (S_{30} C_{10} \\minus{} C_{30} S_{10})(S_{30} C_{10} \\plus{} C_{30} S_{10} ) C_10$\n\n$ \\equal{} \\left( S^2 _{30} C^2 _{10} \\minus{} C^2 _{30} S^2 _{10} \\right) C_{10}$\n\n$ \\equal{} \\left( \\frac {1}{4} c^2 _ {10} \\minus{} \\frac {3}{4} s^2 _{10} \\right) C_{10}$\n\n$ \\equal{} \\frac {1}{4} \\left( C^3 _{10} \\minus{} 3 S^2 _{10} C_{10} \\right)$\n\n$ \\equal{} \\frac {1}{4} C_{(3 \\cdot 10)} \\equal{} \\frac {\\sqrt {3}}{8}$[/hide]"
}
{
"Tag": [
"geometry"
],
"Problem": "One side of a square is 12 + 3x centimeters and another side is 32 + x centimeters. How many centimeters is the length of the diagonal of the square? Express your answer in simplest radical form.",
"Solution_1": "[hide] We have $ 12\\plus{}3x\\equal{}32\\plus{}x \\implies x\\equal{}10$. Therefore, the side length of the square is $ 32\\plus{}10\\equal{}42$ and the diagonal is $ \\boxed{42\\sqrt{2}}$.[/hide]",
"Solution_2": "Just set the two expressions equal. 12+3x=32+x\r\n\r\n2x=20, x=10.\r\n\r\nTherefore, the length of the diagonal is $ 42\\sqrt{2}$."
}
{
"Tag": [
"function",
"inequalities",
"inequalities solved"
],
"Problem": "Show that if $x,y,z>0$ we have:\r\n\r\n$ \\frac{x}{x+2y+2z} + \\frac{y}{2x+y+2z} + \\frac{z}{2x+2y+z} \\geq \\frac{3}{5}$\r\n\r\nI have found this solution:\r\n\r\nWLOG $x+y+z=1$\r\n\r\nwe know that the function $f(x)= \\frac{1}{x}$ is convex (you can show this using derivates) and you apply Jensen's inequality with weights $x,y,z$.\r\n\r\n$ \\frac{x}{x+2y+2z} + \\frac{y}{2x+y+2z} + \\frac{z}{2x+2y+z} \\geq \\frac{1}{1+2( \\sum(xy))} \\geq \\frac{3}{5}$ because $ ( \\sum(x))^2 \\geq 3 \\sum(xy)$\r\n\r\nwhat is your solution?\r\n\r\ncheers",
"Solution_1": "If you put a=x+2y+2z,b=2x+y+2z,c=2x+2y+2 you obtain\r\n\r\nx=(2b+2c-3a)/5\r\ny=(2c+2a-3b)/5\r\nz=(2b+2c-3a)/5\r\n\r\nso that your inequality is tranformed in\r\n\r\nb/a +a/b +c/a + a/c +b/c + c/b \\geq 6 \r\n\r\nwhich is true as a/b +b/a \\geq 2 by AM-GM."
}
{
"Tag": [
"geometry",
"3D geometry",
"algebra",
"factorization",
"difference of cubes",
"special factorizations"
],
"Problem": "Simplify $ \\frac{\\sqrt[3]{2}}{1\\plus{}\\sqrt[3]{2}\\plus{}\\sqrt[3]{4}}$",
"Solution_1": "[hide]Multiplying both sides by $ \\sqrt[3]{2}\\minus{}1$ gives $ \\sqrt[3]{4}\\minus{}\\sqrt[3]{2}$[/hide]",
"Solution_2": "How do I know what to multiply to the denominator to get rid of the radicals?",
"Solution_3": "[hide=\"sub\"]We let $ \\sqrt[3]{2}\\equal{}x$. Thus we want $ \\dfrac{x}{1\\plus{}x\\plus{}x^2}$. We recognize the denominator from the difference of cubes: $ x^3\\minus{}1\\equal{}(x\\minus{}1)(x^2\\plus{}x\\plus{}1)$. Thus we can multiply the numerator and the denominator by $ \\sqrt[3]{2}\\minus{}1$ to get $ \\dfrac{\\sqrt[3]{4}\\minus{}\\sqrt[3]{2}}{2\\minus{}1}\\equal{}\\sqrt[3]{4}\\minus{}\\sqrt[3]{2}$.[/hide]",
"Solution_4": "Ok got it thanks",
"Solution_5": "Mmm okay a new one\r\n\r\nSimplify\r\n\r\n$ \\frac{1}{1\\plus{}\\sqrt{2}\\plus{}\\sqrt{3}}$",
"Solution_6": "You'll notice the trick used above no longer works. What to do?\r\n\r\n[hide=\"Idea\"] Get rid of one radical first and then the other. In other words, let's try multiplying by $ (1 \\plus{} \\sqrt{2} \\minus{} \\sqrt{3})$. This gives\n\n$ (1 \\plus{} \\sqrt{2})^2 \\minus{} 3 \\equal{} 2 \\sqrt{2}$.\n\nNow we just have to multiply by $ \\sqrt{2}$. [/hide]\n[hide=\"Generalization\"] Rationalize $ a \\plus{} b \\sqrt{c} \\plus{} d \\sqrt{e}$ by multiplying by\n\n$ (a \\plus{} b \\sqrt{c} \\minus{} d \\sqrt{e})(a \\minus{} b \\sqrt{c} \\plus{} d \\sqrt{e})(a \\minus{} b \\sqrt{c} \\minus{} d \\sqrt{e})$. \n\nThese three quantities are the (generalized) [url=http://en.wikipedia.org/wiki/Conjugate_element_%28field_theory%29]conjugates[/url] of $ a \\plus{} b \\sqrt{c} \\plus{} d \\sqrt{e}$. [/hide]"
}
{
"Tag": [
"USAMTS"
],
"Problem": "1) If you quote a definition from a book, how should you state the source? (Where would you put it, what would you put--author, title, etc.)\r\n\r\n2) If you use a CAS (computer algebra systems) like Derive, should you explain what it is? What should I write? If it was Mathematica, then you wouldn't need to, but I don't think Derive is as popular as Mathematica, etc.\r\n\r\n3) If you use a calculator (TI-83+) program, should you give a URL for where you can download it? How should you mention it?\r\n\r\nThanks! :)",
"Solution_1": "The general rule of citation, not just on the USAMTS but in math and science in general, is to give enough information so that the reader can verify your work if he/she wishes. Meaning if you use a program, you should include the source code and/or the URL where it can be found. If you use a \"well-known\" software package (and Derive is in that category as far as I'm concerned), you should state it by name. \r\n\r\nCiting reference books should use a standard citation: author, title, publisher, year, page numbers.",
"Solution_2": "Thanks. :)"
}
{
"Tag": [
"modular arithmetic",
"summer program",
"PROMYS",
"number theory",
"greatest common divisor",
"relatively prime",
"prime numbers"
],
"Problem": "hello i'm new here and don't know where this might go...but anywayshere is a good problem i just solved recently:\r\nif in a game there are only 2 possible scores $a$ and $b$, (u can choose to get either one each time) then:\r\ni)what is the largest score that can not be made by these numbers?(proof needed)\r\nii)what is the number of impossible scores?(proof needed)\r\n\r\nplease don't say for part (i), based on the Chicken McNugget Theorem it has to be $ab-a-b$--it would be unacceptable :) \r\n\r\ngood luck.",
"Solution_1": "It is important to note that $a$ and $b$ must be relatively prime. Else, their sum would have to be a multiple of some prime number, so there will always be a number not obtainable.\r\n\r\nFor part i (there was a similar proof in AoPS volume 2), write:\\[\\begin{array}[b]{ccccc}\r\n0 & a & 2a & 3a & \\ldots\\\\\r\nb & b+a & b+2a & b+3a & \\ldots\\\\\r\n2b & 2b+a & 2b+2a & 2b+3a & \\ldots\\\\\r\n\\vdots & \\vdots & \\vdots & \\vdots & \\ddots\\\\\\end{array}\\]\r\nYou can get all $n\\equiv0\\pmod{a}$ from the first row.\r\n$n\\equiv b\\pmod{a}$ from the second row for $n\\ge b$.\r\n$n\\equiv 2b\\pmod{a}$ from the third row for $n\\ge 2b$.\r\nEventually, all numbers can be obtainable in $\\bmod{a}$ after row $a$ (that is, we can get all $n\\ge(a-1)b$.\r\nSince it is the last row, that is the smallest number of that remainder in $\\bmod{a}$ that is obtainable. Since all the other remainders are already obtainable, the largest number that cannot be reached is then $ab-a-b$.\r\n\r\nNo idea yet for the second part...",
"Solution_2": "Just FYI, this is very similar to a question on the PROMYS application. Though PROMYS starts tomorrow, they like to reuse problems. Not accusing or anything, but a heads up to people around here.",
"Solution_3": "this is the coin exchange problem of Frobenius, and has been discussed before.",
"Solution_4": "Does a closed form for the second part exist?",
"Solution_5": "Yes. One can show that if two nonnegative integers $ k,\\ell$ sum to $ ab \\minus{} a \\minus{} b$, then exactly one of $ k$ or $ \\ell$ is an impossible score (and one is possible). From this, a closed form for ii) follows immediately.",
"Solution_6": "So in that case the answer is just $ \\frac{ab\\minus{}a\\minus{}b\\plus{}1}2$, right?\r\n\r\nI think I see the reasoning behind it: it cannot be two numbers that can both be expressible as a non-negative linear combo of $ a,b$ or otherwise $ ab\\minus{}a\\minus{}b$ will be able to also. So it must be at least one that is not expressible, and this number must be in the form $ N\\equal{}mb\\minus{}na>0$ for $ 00$ (considering the complete residue system mod a $ \\{0,b,2b,...(a\\minus{}1)b\\}$), hence we have $ (a\\minus{}1)b\\minus{}a\\minus{}N\\equal{}(a\\minus{}1\\minus{}m)b\\plus{}(n\\minus{}1)a$, which is a valid non-negative linear combo of $ a,b$.\r\n\r\nIs this proof good enough (of course the formulation for N follows directly from the proof of the chicken mcnugget theorem)?\r\nBTW why is it called chicken mcnugget theorem? Can't they give it a more appetizing name? :P",
"Solution_7": "People were wondering the largest number of Chicken McNuggets that cannot be attained when you order small, medium, or large orders of Chicken McNuggets, which came in orders of 6, 9, and 20. Hence the name.\r\n\r\nAlso the answer for (ii) seems right because it's odd iff a and b are not both even, and if a and b are even then the GCD is more than 1.\r\n\r\nEDIT: Wait a minute, if $ ab\\minus{}a\\minus{}b$ is even (which it is), then we have it over 2 plus it over 2 equals it. $ \\frac{ab\\minus{}a\\minus{}b}{2}$ can't be both obtainable and unobtainable at the same time...",
"Solution_8": "[quote=\"1=2\"]EDIT: Wait a minute, if $ ab \\minus{} a \\minus{} b$ is even (which it is)...[/quote]\r\nCorrection: which it never is if $ a$ and $ b$ are relatively prime.",
"Solution_9": "Just wondering, can this be generalized to more than 2 variables?",
"Solution_10": "If you look at possible scores of $ a,b,c$, the problem is pretty open. But if you look at possible scores of $ ab,bc,ca$ or $ abc,bcd,cda,dab$, interesting stuff happens.",
"Solution_11": "[hide=\"idea for three numbers\"] Ask the same question using three relatively prime numbers $ a, b$ and $ c$, where $ a < b < c$. Consider all numbers now $ \\pmod{a}$. Now, using x $ b$s and y $ c$s, where x and y may be as large as needed, construct a complete residue class $ \\pmod{a}$, using the smallest value of $ bx \\plus{} cy$ for each $ d\\pmod{a}$ (note that a immediately gives the $ 0\\pmod{a}$).Then take the largest $ bx \\plus{} cy$, subtract a from it, and use that as your answer.\n\nFor example, consider 4, 9, and 11. We have to find the smallest sum of 4s, 9s, and 11s so we end up with numbers congruent to $ 0, 1, 2, 3 \\pmod{4}$.\n\nSmallest 0 (mod 4): 4\nSmallest 1 (mod 4): 9\nSmallest 2 (mod 4):18\nSmallest 3 (mod 4): 11\n\nOur answer is 18-4=14.\n\nUnfortunately, this may take some time to calculate for larger numbers...[/hide]",
"Solution_12": "[quote=\"MellowMelon\"]But if you look at possible scores of $ ab,bc,ca$ or $ abc,bcd,cda,dab$, interesting stuff happens.[/quote] Could you please elaborate some interesting stuff about this? Thanks.",
"Solution_13": "i am very sry to express my ignorance but wat is the chicken mcnuggets theorem?",
"Solution_14": "[quote=\"The Enigma\"]i am very sry to express my ignorance but wat is the chicken mcnuggets theorem?[/quote]\r\nsome one help me..",
"Solution_15": "[quote=\"The Enigma\"][quote=\"The Enigma\"]i am very sry to express my ignorance but wat is the chicken mcnuggets theorem?[/quote]\nsome one help me..[/quote]\r\n\r\nhttp://lmgtfy.com/?q=chicken+mcnugget+theorem"
}
{
"Tag": [
"geometry",
"vector",
"analytic geometry",
"geometry unsolved"
],
"Problem": "It might be (and I know it actually is) a simple problem but my skill in geometry is not sharp. So, what I have in my program is a line and a segment given with their two points respectively. The question is, how do I find out whether they intersect without computing the intersection point itself? In other words, I need a condition of their intersection.\r\nI'd be very grateful for your help.",
"Solution_1": "The line intersects the segment iff the two end points of the segment are on opposite sides of the line. How do we know if they are on opposite sides of the line? Denote two arbitrary points on your line A, B and the end points of the segment C, D. Compute (B-A)x(C-A), that is, a vector in the direction of the line crossed with a vector from one of the end points to the line. Then compute (B-A)x(D-A), or the same thing for the other end point. Note that both of these products will have only a z coordinate. If both z coordinates are the same sign (+ or -) then the segment does not intersect the line. Otherwise, they do.\r\n\r\nBy the way, I think this topic would be better suited for the comp sci forums but ah well.",
"Solution_2": "Yes, it greatly helped, thanks a lot!"
}
{
"Tag": [
"conics",
"projective geometry",
"geometry proposed",
"geometry"
],
"Problem": "Given three circles in the plane, none of which lies inside another (nonintersecting). Prove that the main diagonals of the convex hexagon determined by three pairs of internal common tangents of the three circles are concurrent. ( where the main diagonal means the one that exactly divides the hexagon into two convex quadrilaterals.)",
"Solution_1": "My solution:\n\nDenote these three circles as $ (O_1), (O_2), (O_3) $\n\nBy Monge's theorem we get\nthe intersection of two external tangents of $ \\{(O2),(O3) \\}, \\{(O3),(O1) \\}, \\{(O1),(O2)\\} $ are collinear ,\nso by [url=http://mathworld.wolfram.com/FourConicsTheorem.html]Dual of Four Conics theorem[/url] $ \\Longrightarrow $ six internal tangents of $ (O1), (O2), (O3) $ form an envelope of a conic .\nBy Brianchon's theorem we get the main diagonals of the convex hexagon bounded by three pairs of internal common tangents of these three circles are concurrent .\n\nQ.E.D"
}
{
"Tag": [
"algebra",
"polynomial",
"advanced fields",
"advanced fields unsolved"
],
"Problem": "Hi all, \r\n\r\nI have the following problem:\r\n\r\nLet $ p \\in \\mathbb{C}[z]$ be a polynomial with complex coefficients, which has no roots in $ S^1$. Show that the number of zeros of $ p$ in the interior of the unit disc is the degree of the continuous map $ \\hat{p}: S^1 \\rightarrow S^1$ defined by $ \\hat{p}(z)\\equal{}\\frac{p(z)}{|p(z)|}$.\r\n\r\n\r\nOne may factor $ p(z)\\equal{}f(z)g(z)$ where $ f(z)$ has no roots $ z$ with $ |z| \\geq 1$ and $ g(z)$ has no roots $ z$ with $ |z| \\leq 1$. One may do this because of the Fundamental Theorem of Algebra and by reordering of the linear factors. If I could show that $ \\hat{p}$ is homotopic to $ f/|f|$, then I would be done since that would imply that $ \\deg(\\hat{p})\\equal{}\\deg(f/|f|)$, which is the number of zeros in the interior of the disc. But that is where I'm encountering trouble. How can I do this? \r\n\r\nThanks in advance.",
"Solution_1": "Your notation is unclear - what does $ S_1$ mean? I thought it meant the unit circle, but clearly that's not right.",
"Solution_2": "Here, $ S^1: \\equal{} \\{z \\in \\mathbb{C} | |z|\\equal{}1\\}$ is the boundary of the $ 2$-disc.",
"Solution_3": "Why is that clearly not correct?",
"Solution_4": "I was getting worried about what would happen to the map p-hat near zeros of p, but the fact that it is continuous immediately implies that all =discontinuities are removable and we fill them in as dictated.",
"Solution_5": "The fact that $ p$ has no roots on $ S^1$ implies that $ \\hat{p}: S^1 \\rightarrow S^1$ is well-defined. But how is it homotopic to $ f/|f|$?",
"Solution_6": "argument principle.\r\nconsider $ log(\\frac{p}{|p|})$, and see how many times will it go around the $ S$ when $ z$ run around $ S$.",
"Solution_7": "The degree of $ p$ is the integer $ d$ such that $ p_*(x) \\equal{} dx$ for every $ x \\in \\pi_1(S^1)$. \r\n\r\nYou can show that \r\n$ \\mathrm{deg}(pq) \\equal{} \\mathrm{deg}(p) \\plus{} \\mathrm{deg}(q)$: since $ S^1$ is a topological group, multiplication\r\nof loops is homotopic to concatenation, from which this result follows. (This is problem 3 in May Ch. 1; do it \r\nfirst to make things easier.)\r\n\r\nGiven this, you only have to show the result for linear polynomials $ (x \\minus{} z)$. From here this should be much\r\neasier. (There are many ways to proceed - since you are probably getting this problem from May's book,\r\nreference the above argument why $ \\pi_1(S^1) \\equal{} \\mathbb{Z}$.)"
}
{
"Tag": [
"probability",
"geometry",
"3D geometry",
"real analysis",
"real analysis unsolved"
],
"Problem": "Suppose that $a_1,...,a_k$ are distinct irrational numbers and $N$ is a positive integer. Is it true that the probability for $n$ to verify that $[na_1]+...+[na_k]$ is congruent to $j$ (mod $N$) is the same for all $j$? Does anyone have any idea about a proof?",
"Solution_1": "I don't think it's true. Set $a_1 = \\sqrt{2}$, $a_2 = \\sqrt{2} + 2$, and $N = 2$.",
"Solution_2": "Sorry, I was idiot, I should have realized that. But what if $a_1,...,a_k$ are linearly independent over $Q$?",
"Solution_3": "Then the answer is \"Yes\". All you need to do is to use the fact that the points $na_1,\\dots, na_k$ taken modulo $N$ are uniformly distributed in the cube $(0,N)^k$."
}
{
"Tag": [
"function",
"integration",
"limit",
"calculus",
"derivative",
"real analysis",
"geometry"
],
"Problem": "Can anyone help prove Riemann Theorem?\r\nThe definition is as following:\r\nAssume the symbols using here are well-defined.\r\nAnd $G(x)$ is a cyclic function with a cycle of $T$.\r\nProve:\r\n$\\displaystyle \\int\\limits_{a}^{b}F(x)G(px)dx=\\frac{1}{T}\\int\\limits_{a}^{b}F(x)dx\\int\\limits_{0}^{T}G(x)dx$",
"Solution_1": "I forget something,the left hand side should be the limit when $p$ tends to $+\\infty$.\r\nHowever,I don't know how to type that symbol in LaTeX. :blush:",
"Solution_2": "Is this what you mean?:\r\n\r\n$\\lim_{p\\rightarrow\\infty}\\int_{a}^{b}F\\left(x\\right)G\\left(px\\right)\\,dx=\\frac{1}{T}\\int_{a}^{b}F\\left(x\\right)\\,dx\\int_{0}^{T}G\\left(x\\right)\\,dx$",
"Solution_3": "Yes,but I didn't know how to type in that limit when I was posting that.",
"Solution_4": "[Note on language: what you called \"cyclic\" is usually called \"periodic\" in English]\r\n\r\nYou might as well start by subtracting off the average value- write $G(x)=h(x)+c$, where $h$ has integral zero over one period. $\\int_a^b F(x)\\cdot c\\,dx=\\frac1T\\int_a^b F(x)\\,dx\\int_0^T c\\,dx$, and we are left with the taxk of proving that $\\int_a^b F(x)h(px)\\,dx\\to 0$ as $p\\to\\infty$ when $h$ has average value zero.\r\n\r\nIf $F$ is continuously differentiable, we can use integration by parts. Let $H$ be an antiderivative of $h$; since $\\int_0^T h(x)\\,dx=0$, $H(a+T)=H(a)$ and $H$ is bounded. Let $F'=f$.\r\nWe have $\\int_a^b F(x)h(px)\\,dx=\\frac{F(b)H(pb)}{p}-\\frac{F(a)H(pa)}{p}-\\frac1p\\int_a^b f(x)H(x)\\,dx$. Each term is bounded by some constant multiple of $\\frac1p$ by boundedness of $M$, and everything goes to zero.\r\n\r\nFor uglier functions $F$, we use a density result; the \"nice\" functions ($C^1$ in this case) are dense in the space we're working in (such as $L^1$ functions).",
"Solution_5": "Firstly,thanks for the advice on language.\r\n[quote=\"jmerry\"]\nWe have $\\int_a^b F(x)h(px)\\,dx=\\frac{F(b)H(pb)}{p}-\\frac{F(a)H(pa)}{p}-\\frac1p\\int_a^b f(x)H(x)\\,dx$. Each term is bounded by some constant multiple of $\\frac1p$ by boundedness of $M$, and everything goes to zero.\n[/quote]\nI don't really understand this,can you give a more detailed explanation?\nI can prove the cases that $F(x)$ is a continuous function.But the problem is,$F(x)$ can be a lot uglier.\nThe original expression is that $F(x)$ and $G(x)$ are Riemann integrable...\n[quote=\"jmerry\"]\nFor uglier functions $F$, we use a density result; the \"nice\" functions ($C^1$ in this case) are dense in the space we're working in (such as $L^1$ functions).[/quote]\r\nWhat's the meaning of $C^1$ and $L^1$ :?:\r\nAnd what is the meaning of this density result?",
"Solution_6": "Firstly,thanks for the advice on language.\r\n[quote=\"jmerry\"]\nWe have $\\int_a^b F(x)h(px)\\,dx=\\frac{F(b)H(pb)}{p}-\\frac{F(a)H(pa)}{p}-\\frac1p\\int_a^b f(x)H(x)\\,dx$. Each term is bounded by some constant multiple of $\\frac1p$ by boundedness of $M$, and everything goes to zero.\n[/quote]\nI don't really understand this,can you give a more detailed explanation?\nI can prove the cases in which $F(x)$ is a continuous function.But the problem is,$F(x)$ can be a lot uglier.\nThe original expression is that $F(x)$ and $G(x)$ are Riemann integrable...\n[quote=\"jmerry\"]\nFor uglier functions $F$, we use a density result; the \"nice\" functions ($C^1$ in this case) are dense in the space we're working in (such as $L^1$ functions).[/quote]\r\nWhat's the meaning of $C^1$ and $L^1$ :?:\r\nAnd what is the meaning of this density result?Can you give an explanation?Thank you.\r\nSorry that I've posted the same thing twice,that's a mistake.",
"Solution_7": "$C^{1}$ just means that the function's derivative is continuous. I don't know what $L^{1}$ means...",
"Solution_8": "$f \\in L^1(\\Omega) \\Rightarrow \\int_{\\Omega} |f(x)|dx <\\infty$\r\n$f$ is any function.\r\n$\\Omega$ is some space (I am used to subspaces of $R^n$ but you can do other spaces if you have a measure).",
"Solution_9": "And if you want $L^1$ to be a metric space, then its members are not functions but equivalence classes of functions under the relation that two functions are equivalent if they are equal almost everywhere. (Example: the function which is 1 on the rationals and 0 on the irrationals belongs to the equivalence class of the the zero function.)\r\n\r\nFinding a nice dense subset would be saying that for any $\\epsilon>0,$ we can find a \"nice\" function $g$ such that $\\int|f-g|<\\epsilon.$ If you take for granted that you can do that, it shouldn't be that hard to see how to extend jmerry's argument.\r\n\r\nActually proving the density would be part of the Lebesgue theory of integration.",
"Solution_10": "[url=http://www.artofproblemsolving.com/Forum/viewtopic.php?&t=26568]This[/url] is the best explanation I can find. Look up the Riemann-Lebesgue lemma for more information.",
"Solution_11": "Thanks a lot.It seems liyi's solution,that classical one, is exactly what I'm looking for.\r\nBy the way,can any of you help give a proof to that dense result?",
"Solution_12": "You prove the density step by step.\r\n\r\n1. given a positive measurable function, one can construct a sequence of measurable simple functions the increases to the given function. (A simple function is a function with a finite range. It's a linear combination of indicator functions of sets. At this point in the construction, we have no control over the shape of the sets.)\r\n\r\n2. Prove that if this original positive function is integrable ($L^1$), then the sequence of simple functions constructed in #1 converges to the original function not just pointwise but also in the $L^1$ norm. (This proof would use either Fatou's Lemma or the Dominated Convergence Theorem.)\r\n\r\n3. Given a complex-valued integrable function, write it as the linear combination of four functions (positive part of real part, negative part of real part, etc.) Approximate each as in #2. We now have that simple functions are dense in $L^1.$\r\n\r\n4. Now we need this result from measure theory: Given a measurable set $E$ of finite measure and $\\epsilon>0,$ there exists a set $B$ which is a finite union of bounded intervals such that $|E\\Delta B|<\\epsilon.$ Here I'm using $|\\cdot|$ to mean measure and $E\\Delta B=(E\\setminus B)\\cup(B\\setminus E)$ is the symmetric difference of the sets.\r\n\r\n5. Given #4, we can approximate a characteristic function of a set arbitrarily well in $L^1$ by a step function. Now it's just an exercise in epsilon-gymnastics to say that we can approximate any simple function arbitrarily well in $L^1$ by a step function.\r\n\r\n6. Now that we have the step functions dense in $L^1,$ we can proceed along any of several paths. One thought is to \"bevel the edges\" of the step functions to produce continuous functions or even $C^1$ functions. Another is this:\r\n\r\n6'. Suppose we're working on all of $\\mathbb{R}.$ (Cutting down to some interval isn't too hard.) Then translation of functions is continuous in $L^1.$ What I mean by that: Suppose $f\\in L^1(\\mathbb{R}).$ Define $f_t(x)=f(x-t).$ Then $\\lim_{t\\to0}\\|f_t-f\\|_1=0.$ The proof uses #5, the density of step functions.\r\n\r\n7'. Suppose $\\varphi$ is a $C^{\\infty},$ compactly supportd function such that $\\int_{\\mathbb{R}}\\varphi=1.$ Define $\\varphi_{\\epsilon}(x)=\\frac1{\\epsilon}\\varphi\\left(\\frac{x}{\\epsilon}\\right).$ Then $\\varphi_{\\epsilon}*f\\in C^{\\infty}$ (that's convolution) and $\\lim_{\\epsilon\\to0^+}\\|\\varphi_{\\epsilon}*f-f\\|_1=0.$ The proof uses #6'.\r\n\r\nNow we have that $C^{\\infty}$ is dense in $L^1.$"
}
{
"Tag": [
"number theory",
"prime factorization"
],
"Problem": "How many divisors does $ 2\\cdot2\\cdot3\\cdot3\\cdot3$ have?",
"Solution_1": "Every factor of a number must share its prime factors and have no extra ones. For example, the number 18 has prime factorization $ 2 \\cdot 3^2$. A factor of it, 9, shares those prime factors with no extras: $ 2^0 \\cdot 3^2$.\r\n\r\nThis implies that for the first prime factor of the number with exponent $ n$, there are $ n\\plus{}1$ ways to \"choose\" it. In our 18 example, the 2 could have had an exponent of 0 or 1, and the 3 could have been 0, 1, or 2. Thus, the number of factors is simply the product of the exponents added to 1.\r\n\r\nIn this problem, it's already prime factorized, so we have $ (2 \\plus{} 1)(3 \\plus{} 1) \\equal{} 12$."
}
{
"Tag": [
"MATHCOUNTS",
"AMC",
"AIME",
"AMC 10"
],
"Problem": "How do you prepare for State?\r\n\r\nI use the MATHCOUNTS Drills on that one website.\r\n\r\nThe night before the competition, I eat some food with protein.\r\nI also do the past years competition (So I would do the 2005 State Competition for the 2006 State).\r\n\r\nThe morning of the competition, I eat a banana.\r\nI also look over the formula sheet I downloaded and printed from the Formula forum.\r\n\r\nWhat do you do?",
"Solution_1": "It is a good idea to get a lot of rest several nights before the competition. You should also eat a good breakfast before the competition so you do not become hungry and have plenty of energy.\r\n\r\nLooking over a formula sheet the morning of the competition won't do you much good. You should find how the formulas are derived and make sure you understand that, because then you are not very likely to forget the formula, but even if you do, you could derive it.\r\n\r\nIt is a good idea to practice multiple state rounds before the competition, so you have more practice. I do every sprint round from about 1995 to 2005 (all that I have) and do some twice to make sure I'm ready. Practicing several different rounds will give you an advantage. If you have done multiple rounds, you will be likely to be able to pace yourself well so you can finish the test and get the most possible correct. If you have done only one round as practice, you won't have a good enough idea of the range the of difficulty of the test, so you probably won't do as well.\r\n\r\nAnother good idea is to do all of the warm ups and work outs provided for the current year. The same people that write the warm ups and work outs write the actual competitions, so if you do them, you will have a better idea of what types of problems might be on the contest.\r\n\r\nBasically, practice will help you do the best.",
"Solution_2": "Our MATHCOUNTS coach is also our Algebra 2 teacher. 2 weeks before the competiton, we take time from math class and turn it into MATHCOUNTS practice time. We do all the chapter, state, and national competitions.\r\nOne day we'll do all the sprints of one year.\r\nAnother day we'll do all the targets of one year.\r\nWe take home another year of competition for homework.\r\nWe also practice countdown and team after school.\r\nOur coach has finally been able to project the countdown problems on a board, so we can practice it the real way.\r\n\r\nDo you think this will help us at State? (we're in Kentucky and our school has won it six years in a row)(at least I think so)",
"Solution_3": "I try to win.\r\n\r\nhere are some real tips:\r\nFirst, eat a good dinner the night before.\r\nGet 9+ hours of sleep.\r\nEat a good breakfast.\r\n\r\nTry to stay as relaxed as possible all that day, it really helps.\r\n\r\nSPRINT:\r\nBefore you get the sprint round, make numbered boxes 1-30 on a few sheets of paper.\r\nUse the boxes as scratch paper for specific problems. Circle your answer.\r\nRead each question at least twice, and make sure you get the key information. Trust me, this is important. Many a time I have given radius instead of diameter, speed instead of distance...\r\nDon't spend too long on one problem, if you think it will take more than a minute, skip it and come back later.\r\nEither have a watch or timer to keep time, and with about 5 minutes left, start checking your work. You will probably get more right this way than spending the 5 minutes on the two hardest problems.\r\nMake sure the circled answer that you have on scratch paper is what you have for an answer.\r\nMake sure your answers are in the right form/units.\r\nThen go back and do the problems you skipped.\r\n\r\nTARGET:\r\nYou don't need the numbered paper thing here; you have enough scratch paper on the sheet itself.\r\nDo whatever problem seems easier first.\r\nThen do the harder one.\r\nGo back to the first problem and try it a different way. Do that for the other problem.\r\nKeep confirming your answer in different ways until time is called.\r\n\r\nTEAM:\r\nMost of this is based on what your team's strategy is and how strong you are in comparison of he rest of the group.\r\nHowever, this is something that I think really helps.\r\nMake boxes on paper like in the sprint round, except one sheet of paper per problem.\r\nWrite a decent solution so the other person can follow your steps.\r\nWhen someone or a group is done, give to another person/group to check it over.\r\nWhen there are 2 minutes left, make sure that the captain has all of the answers written on the sheet. Make sure its the captain's sheet though. At nats last year, we almost got 0 on the team round because the captain wrote on the wrong sheet (stupid girl).\r\n\r\nCOUNTDOWN:\r\nIf you make it this far congrats!\r\nBefore you go up, observe who your opponents may be.\r\nNote their speed and whether they are prone to giving a wrong answer too quickly.\r\nWhen you are up there, don't get nervous.\r\nRead the question on the projector (or whatever else) quickly, noting the key words/numbers. If you think you can do it in your head, do so. If you need paper, use it.\r\nMake sure your answer is in the right form, then buzz in. If it isn't, convert it and buzz in immediately before you have the right answer.\r\nIf the other guy buzzes in and gets it wrong, then do the question on paper, and take all of the time you need. Check over it several ways, and buzz in a few seconds before time is called (but don't cut it too close :) )",
"Solution_4": "Just wondering, what AoPS chapters would be useful for preparing for states?",
"Solution_5": "I think mainly\r\n$1$ if you forget how to simplify radicals.\r\n$3$\r\n$4$\r\n$\\boxed{5}$ \r\n$7$ but its really easy\r\n$9-18$\r\n$25$\r\n$26$\r\nThats all i think but the rest is useful a bit",
"Solution_6": "When you start the sprint round, should you scan the first page and look for the easiest question (the one with the least words)?\r\n\r\nI did this at chapter and I got 8th overall.",
"Solution_7": "[quote=\"Aznwithabrain\"]When you start the sprint round, should you scan the first page and look for the easiest question (the one with the least words)?\n\nI did this at chapter and I got 8th overall.[/quote]\r\n\r\nWell, the shortest problems aren't always the easiest. I would say as a general rule, go through every question in order, and if you need to spend more than 10 seconds visualizing what to do, skip it and come back. What I did at chapter was work on the problems backward (I posted this on a thread somewhere, I just don't remember which one) and I got second on written.",
"Solution_8": "I wouldn't do that. If you can't answer all of the low numbered problems, then :oops:",
"Solution_9": "Well how bout at State?",
"Solution_10": "at any level.\r\nNot to be mean, but if you can't answer the easy questions, then you aren't gonna win.\r\nFor the last problems, I do that a lot, especially if one looks easy",
"Solution_11": "Another question\r\n\r\nOther than past competitions, 2006 warm-ups and workouts and etc., and the MATHCOUNTS drill page, and the AoPS books.\r\n\r\nWhat do you use for practice?",
"Solution_12": "other warmups and workouts",
"Solution_13": "Is that it?",
"Solution_14": "mathematical circles a little. also a few amc-10 tests, but the format is so different that >.<\r\nI don't know what else there is :?:",
"Solution_15": "how often do you meet per week with your team? and what do you do during team practices? does anyone have suggestions for team round?\r\n\r\nand looking at these schedules, when do you do homework or extracurricular activities?",
"Solution_16": "state in illinois was in a holiday inn. free jolly ranchers on all the tables. we played poker and used ours as chips. they gave us a boxed lunch, it was okay I guess. Afterwards we had an ice cream social and everyone was coming by congratulating me on the nationals thing...i guess everyone's really nice in illinois!\r\n\r\nWe practiced every monday wednesday friday morning from 7:00 to 7:50. mostly other competitions that our coach has picked up from places. around AMC 10 time she made sure we each understood all the problems, it was really random. sometimes we'd take a session and do countdown (most fun)\r\nfor team round we always set up a group answer sheet the same way. I'm not sure how widespread this is but we take a sheet of paper and make a chart with our initials at the top and rows for 1-10. when we get the answer we put it in the right box. when we get 2 or three answers the same in a row we put it down, when we get 4 different answers we make sure we're covered on all the other ones and work on that...every team round is different (duh) so that's kind of inefficient on some of them. also two people work 10-1 and two work 1-10. That way we at least get to every problem...supposedly.",
"Solution_17": "We have practice wednesday mornings from 7:15 to 8:00, but it's canceled every other week, and we don't really do anything...",
"Solution_18": "My team has practice Monday and Friday afternoons and Wednesday mornings from 7:00 to 7:45",
"Solution_19": "we had 5 practices (literally) before states.\r\n\r\nThey're 90 minutes, but be usually get in only 2 team rounds or countdown,target,team.\r\n\r\nSo we did 5 team rounds total and 3 countdowns, 3 targets (ALL SCHOOL ROUND)",
"Solution_20": "To prepare for state, I go over everything I've learned the night and day of State",
"Solution_21": "what place did you get at chapter?",
"Solution_22": "[quote=\"camathkid\"]To prepare for state, I go over everything I've learned the night and day of State[/quote]\r\n\r\nActually, that's not necessarily a good idea, having to look over all that tires out your brain, and also shows you haven't prepared enough :(.",
"Solution_23": "How do you prepare?",
"Solution_24": "i win :) :D",
"Solution_25": "very funny",
"Solution_26": "Yeah, considering that he'll get pwnd at nats.",
"Solution_27": "Does the handbook with warm-ups and work-outs really help?",
"Solution_28": "yeah. There are 300 or so problems per year in those, as compared to 144 in chapter, state, national. (sprint target team)",
"Solution_29": "Warm-ups and Workouts are the building blocks for being good at MATHCOUNTS."
}
{
"Tag": [
"LaTeX",
"\\/closed"
],
"Problem": "The way I save it is click Print when on the transcript, then hit command-s (control-s for windows) and click save as Webpage, complete.\r\n\r\nThe only problem with this is the latex doesnt show up. It downloads as a folder with all the latex pictures and a html file with the transcript, and i'm wondering if there's an easier way.",
"Solution_1": "The way I did it was copy-paste onto Word and save as PDF. OR you can use something like CutePDF and print it to that.",
"Solution_2": "Use opera, then save it as an MHTML file, it will retain all the images like the original. This should work in IE as well but Opera Pwns IE :P",
"Solution_3": "Just copy it onto a Word document...what's so hard about doing that?",
"Solution_4": "[quote=\"mathemagician1729\"]Use opera, then save it as an MHTML file, it will retain all the images like the original. This should work in IE as well but Opera Pwns IE :P[/quote]\r\n\r\nI believe AIME15 has Safari.\r\n\r\n@AIME15: Those virtual printers that print into PDF, etc. are probably the best choice. With me, they retained formatting. I'm not sure that CutePDF is compatible with Mac, but you can always Google and see."
}
{
"Tag": [
"geometry",
"3D geometry",
"function"
],
"Problem": "in real numbers\r\n$x_{1}+x_{2}+....+x_{1997}=1997$\r\n$x_{1}^{4}+x_{2}^{4}+...+x_{1997}^{4}=x_{1}^{3}+x_{2}^{3}+...+x_{1997}^{3}$",
"Solution_1": "This seems to easy, but if $x_{1},x_{2},x_{3},..x_{1997}=1$ it works",
"Solution_2": "But there is more than one solution, right?",
"Solution_3": "I believe it's 1, and -1.",
"Solution_4": "$-1$ does not work with the either of the statements.",
"Solution_5": "[quote=\"AstroPhys\"]$-1$ does not work with the either of the statements.[/quote]\r\nHow so?\r\n\r\nI just tried it and it worked..",
"Solution_6": "[quote=\"ashrafmod\"]in real numbers\n$x_{1}+x_{2}+....+x_{1997}=1997$\n$x_{1}^{4}+x_{2}^{4}+...+x_{1997}^{4}=x_{1}^{3}+x_{2}^{3}+...+x_{1997}^{3}$[/quote]\r\n\r\n[hide=\"An idea?\"]I think from the second equation, $\\sum_{n=1}^{1997}x_{n}^{2}(x_{n}^{2}-x_{n})=0$...so...maybe either $x_{n}=0$...or $x_{n}^{2}-x_{n}=0$...or $x_{n}=1$...? I dunno, that's what I'm thinking...[/hide]",
"Solution_7": "Wait, asto, I did it wrong.\r\n\r\nNevermind.",
"Solution_8": "lol its 1. the cube vs. squares thing.",
"Solution_9": "A less-than-clever way to do it is to use Lagrange multipliers on the function $f = x_{1}+x_{2}+\\cdots+x_{1997}$ subject to the constraint $(x_{1}^{4}-x_{1}^{3})+(x_{2}^{4}-x_{2}^{3})+\\cdots+(x_{1997}^{4}-x_{1997}^{3}) = 0$ which tells us that the extrema must occur when\r\n\r\n$1 = 4x_{i}^{3}-3x_{i}^{2}\\Rightarrow (x_{i}-1)(4x_{i}^{2}+x_{i}+1) = 0$\r\n\r\nfor all $i$, i.e. $x_{i}= 1$ is the only real extremum (we can verify that this is a maximum). Thus we have $f \\le 1997$ with equality iff $x_{i}= 1$ for all $i$.",
"Solution_10": "If all $x_{i}\\in \\mathbb{R}^{+}$:\r\n\r\n$S_{1}= \\sum_{i=1}^{1997}x_{i}$\r\n$S_{2}= \\sum_{i=1}^{1997}x_{i}^{2}$\r\nsame for $S_{3},S_{4}$\r\n\r\nBy Cauchy-Schwarz we have $S_{1}S_{3}\\ge S_{2}^{2}$. By AM-GM we have $S_{2}^{2}\\ge 1997S_{4}$\r\n\r\nHence $S_{1}S_{3}= S_{1}S_{4}\\ge 1997S_{4}\\Leftrightarrow S_{1}\\ge 1997$ with equality iff all $x_{i}$ are equal (obviously to 1)\r\n\r\nif one or more are negative then I dunno...maybe try splitting into negative and positive subsets...w/e."
}
{
"Tag": [],
"Problem": "Shubrashis Nyogi aka tittu has won the gold medal at Int Bio olympiad!!!!!! :) :D",
"Solution_1": "well done tittu! :clap: :omighty:",
"Solution_2": "Congratulations Tittu :clap2: :first: :10:",
"Solution_3": "congratulations :thumbup: :clap: :first:",
"Solution_4": "and deserves muccchhhhhh mooooooreeeee\r\n\r\n:1:",
"Solution_5": "and you are also going to make us do the same(congratulating) next year :P",
"Solution_6": "correct skand \r\nbut tittu \r\nvery very very very well done ................................................................"
}
{
"Tag": [],
"Problem": "A quarter weighs the same as two pennies. The quarters in one pound of quarters have a total value of $ \\$25$. How many dollars would a pound of pennies be worth?",
"Solution_1": "the stack of quarters is worth $ \\$25$, which means that there are $ 25 \\times 4$ quarters, or $ 100$ quarters\r\n\r\nsince a quarter weighs $ 2$ pennies, that means there are $ 100 \\times 2$ pennies in that stack, or $ 200$ pennies\r\n\r\n$ 200$ pennies is $ \\$2$, so our answer is $ \\boxed {2}$",
"Solution_2": "i got this right so i'm gonna explain:the stack is $25 and 2 pennies weigh the same as 1 quarter,2500/25=100,and 100 *2=200,200/100=2 so the answer would be 2"
}
{
"Tag": [
"algebra",
"polynomial",
"function",
"complex analysis",
"complex analysis unsolved"
],
"Problem": "Let\r\n\\[ D \\equal{} \\{z\\parallel{}z| \\le 1\\},\\]\r\n\r\n\\[ A_0 \\equal{} \\{f: D \\to \\mathbb{C}| f\\text{ is analytic in the interior of } D,f\\text{ is continous on D}\\}\\]\r\nDoes the polynomials dense in $ A_0$ with the maximum norm?",
"Solution_1": "The Cesaro means of the Fourier series of a continuous function converge uniformly. Given $ f(z)\\equal{}\\sum a_nz^n$ (on the open disk), the polynomials $ f_n(z)\\equal{}\\sum_{k\\equal{}0}^{n\\minus{}1} \\frac{n\\minus{}k}{n}a_kx^k$ converge uniformly to $ f$ on the circle, and thus also on the whole disk.",
"Solution_2": "Another solution is to note that $ f(rz)$ is close to $ f$ if $ r<1$ is close to 1, by uniform continuity. Since $ f(rz)$ is analytic for $ {|z| < 1/r}$, the power series of $ f(rz)$ at 0 converges uniformly to $ f$ on the closed unit disc. This leads to the desired polynomial."
}
{
"Tag": [
"trigonometry"
],
"Problem": "$\\frac{\\sin 10^\\circ + \\sin 20^\\circ}{\\cos 10^\\circ + \\cos 20^\\circ}$ equals\r\n\r\nA. $\\tan 10^\\circ + \\tan 20^\\circ$\r\nB. $\\tan 30^\\circ$\r\nC. $\\frac{1}{2} (\\tan 10^\\circ + \\tan 20^\\circ)$\r\nD. $\\tan 15^\\circ$\r\nE. $\\frac{1}{4} \\tan 60^\\circ$",
"Solution_1": "[hide]we know $\\sinh +\\ sin b =2sin(a+b)/2*cos(a-b)/2$\nand \n$\\cosh + \\ cosb = 2cos(a+b)/2*cos(a-b)/2$\nthen the ansewr is D. $\\tan 15^\\circ$[/hide]",
"Solution_2": "[hide=\"Answer\"]$\\sin 10=\\sin (15-5)=\\sin 15\\cos 5 -\\sin 5\\cos 15$, $\\sin 20=\\sin (15+5)=\\sin 15\\cos 5+\\sin 5\\cos 15$\n$cos 10=\\cos (15-5)=\\cos 15\\cos 5+\\sin 15\\sin 5$, $\\cos 20=\\cos (15+5)=\\cos 15\\cos 5-\\sin 15\\sin 5$\n$\\frac{\\sin 10+\\sin 20}{\\cos 10+\\cos 20}=\\frac{2\\sin 15\\cos 5}{2\\cos 15\\cos 5}=\\tan 15\\Rightarrow \\boxed{D}$[/hide]",
"Solution_3": "[hide]$\\frac{\\sin 10^\\circ + \\sin 20^\\circ}{\\cos 10^\\circ + \\cos 20^\\circ}$ \n\n$=\\frac{2sin15^\\circ * cos(-5)^\\circ}{2cos15^\\circ * cos(-5)^\\circ}$\n\n$=\\frac{sin15^\\circ}{cos15^\\circ}$\n\n$=tan15^\\circ$\n\n$\\boxed{D}$[/hide]",
"Solution_4": "wow, we have a calculator for the amc, it takes no thinking to put those into the calculator, and would be faster (maybe)\r\n\r\nbut yes, the other ways are better",
"Solution_5": "I dont think you could use calculators in 80s AMCs"
}
{
"Tag": [
"function",
"real analysis",
"real analysis unsolved"
],
"Problem": "Is every real derivable and convex function on an open convex subset A of R\u207f , C\u00b9 ?",
"Solution_1": "ok no one likes this kind of question : is ..? :oops: \r\njust show that instead in R :\r\n\r\nlet f : R -> R derivable and convex . show that f' is continuous . :roll:",
"Solution_2": "no one ? :maybe: it might be classical one .."
}
{
"Tag": [
"function",
"algebra",
"domain",
"inequalities",
"calculus",
"derivative",
"functional equation"
],
"Problem": "How do you figure out if a function one-to-one algebraically instead of graphically (using the horizontal line test).\r\n\r\nThanks!",
"Solution_1": "Prove the implication \"$ f(a) \\equal{} f(a')\\Rightarrow a \\equal{} a'$\", or its contrapositive \"$ a\\ne a'\\Rightarrow f(a)\\ne f(a')$\"",
"Solution_2": "[quote=\"10000th User\"]Prove the implication \"$ f(a) \\equal{} f(a')\\Rightarrow a \\equal{} a'$\", or its contrapositive \"$ a\\ne a'\\Rightarrow f(a)\\ne f(a')$\"[/quote]\r\n\r\n\r\ncan u explain it in a better way please",
"Solution_3": "[quote=\"b555\"][quote=\"10000th User\"]Prove the implication \"$ f(a) \\equal{} f(a')\\Rightarrow a \\equal{} a'$\", or its contrapositive \"$ a\\ne a'\\Rightarrow f(a)\\ne f(a')$\"[/quote]\n\n\ncan u explain it in a better way please[/quote]Ex: Show that $ f(x)\\equal{}x^5$ is 1-1.\r\n\r\nIf you graph it, it looks like a snake, a curve similar to $ f(x)\\equal{}x^3$. For any $ ao for all x then it is stritcly increasing.\r\n2.if f'(x)o for all x then it is stritcly increasing.\n2.if f'(x)_128S^2",
"Solution_1": "you can prove that:\r\n$ (2a^2\\plus{}2b^2\\plus{}2c^2)^2 \\geq 198S^2$",
"Solution_2": "$ 4(a^2 \\plus{} b^2 \\plus{} c^2)^2\\ge 12abc(a \\plus{} b \\plus{} c)\\ge 192S^2$",
"Solution_3": "Make it stronger.\r\n$ 16S^2\\equal{}(a\\plus{}b\\plus{}c)(a\\plus{}b\\minus{}c)(a\\minus{}b\\plus{}c)(\\minus{}a\\plus{}b\\plus{}c)\\leq \\frac{(a\\plus{}b\\plus{}c)^4}{27}$ :lol:",
"Solution_4": "We have:\r\n2a^2 + 2c^2 + b^2 \u2265 [(a+c)^2 \u2013 b^2 ] + 2b^2\r\n\u22652\u221a[2(a+c-b)(a+b+c)b^2]\r\n\u22652\u221a[2(a+b+c)(b^2-(c-a)^2)(a+c-b)]=8S\u221a2"
}
{
"Tag": [
"geometry"
],
"Problem": "Here is a proof of this theorem [url=http://www.its.caltech.edu/~ari/paper-folding.html]Proof[/url].\r\n\r\nIt have an approach of the final length of the paper folded is equal to the initial length.\r\n\r\nI tried to prove this in another way. The area of the paper folded is equal to the area of the initial paper not folded. The area here is a transverse one.\r\n\r\nSo, using the same assumption that the bend in the paper is a semi circle we need to sum all those semi-circles.\r\n\r\nThe radius of each semi circle is $ 2^{i \\minus{} 1}$ where $ i$ is the number of folds. [url=http://www.pomonahistorical.org/12times.htm](See the image of this site if you have doubts about this)[/url]\r\n\r\nSo, by this conservation of area.\r\n\r\n$ \\sum _{i \\equal{} 1}^n \\frac {1}{2} \\pi \\left(2^{i \\minus{} 1} h\\right)^2 \\equal{} L h$\r\n\r\nSolving for $ L$ in the last equation we have:\r\n\r\n$ L \\equal{} \\frac {h \\pi }{6} \\left(2^n \\minus{} 1\\right) \\left(2^n \\plus{} 1\\right)$\r\n\r\nBut the equation founded by Britney Gallivan is:\r\n\r\n$ L \\equal{} \\frac {h \\pi }{6} \\left(2^n \\minus{} 1\\right) \\left(2^n \\plus{} 4\\right)$\r\n\r\nSo, where this difference from 1 to 4 came from?\r\nThe formula above is true, one the length is conserved but the other the area is conserved, in theory the 2 need to be equal and give the same result.\r\n\r\nThe only thing I can think that make this difference is that the length of the part bended is not actually $ j \\pi h$, but less than this or the area is not $ \\frac {1}{2} \\pi \\left(2^{i \\minus{} 1} h\\right)^2$ but a little bit greater (don't know how).\r\n\r\nI would be glad if someone help me to find my mistake and correct the formula to give the correct result.\r\n\r\nNote: If we think about volumes the result would be the same, because in this case the volume is Area times $ w$, and the factor $ w$ would cancell out",
"Solution_1": "The difference between your and Britney's formula is the difference between the assumption that the area is preserved and the assumption that the length of the outer edge is preserved when making a fold. I do not know which of these two assumptions is more realistic for paper. You both have no mistakes in mathematics but your physical models of the fold are different.",
"Solution_2": "hm...\r\n\r\nNow I know the difference between. The result needed to be equal, because the paper is conserved itself.\r\n\r\nThe thing is, Britney used the radius of the paper $ j$. So it is the external part of the paper. Physically it would be more realistic to use the middle of the paper insted of the external part, (the mean of the external and internal), so the radius would be $ j \\minus{} \\frac{1}{2}$. After that I found this:\r\n\r\n$ \\sum _{i\\equal{}1}^n \\sum _{j\\equal{}1}^{2^{i\\minus{}1}} \\left(j\\minus{}\\frac{1}{2}\\right) \\pi h\\equal{}\\sum _{i\\equal{}1}^n \\frac{\\pi \\left(h 2^{i\\minus{}1}\\right)^2}{2 h}$\r\n\r\nIt give the equality if we change this. The thing confusing is where you mesure the length matter, but the area not.\r\n\r\nThe 2 conservative laws are rigth and actually the same law.\r\n\r\nDo you agree fedja?",
"Solution_3": "I certainly agree that when you bend metal or plastic, your law makes more sense, so, probably, you are right about paper too. :) I also wonder if the thickness of the paper really changes a bit when folding (at least, I see no reason why it shouldn't happen :?, pehaps I should refresh my very rusty knowledge of \"solid body physics\" or to leave this discussion to someone who knows it better :P).",
"Solution_4": "Hm... I was also thinking about this.\r\n\r\nIf we fold the paper and unfold it, the thickness in the fold will be less than in others parts of paper. I think this is just sensible for one fold, If you continuosly fold and fold, it will not make a big differente, and if the thickness of the paper is too great this differente in thickness will also be small.\r\n\r\nSo one of the consequences of folding and unfolding a paper is that the length of the paper increass (just a little)\r\n\r\nI just made a very very approximation of the real case (don't know if this hold)\r\n\r\nYou have a paper of length $ L_0$ and thickness $ h$. After you fold it you have basically a retacgle of thickness $ 2 h$ and length $ L_1$ and a semi-circle of radius $ h$. If we use the law of conservation of area (mass) we have.\r\n\r\n$ L_0 h\\equal{} L_1 (2h) \\plus{} \\frac{\\pi }{2} h^2$\r\n\r\nSo, If we unfold the length of the paper would be $ 2 L_1 \\plus{} \\delta L$\r\n\r\nA good approximation of this $ \\delta L$ would be \"Cut the paper when you bend it and the put it thogeher\". So we would joing 2 quartes of circle side by side.\r\n\r\nThe final lenth would be so: $ 2 L_1 \\plus{} 2 h \\equal{} L_0\\plus{}\\frac{1}{2} h (4\\minus{}\\pi )$\r\n\r\nAnd this approximation would've be only valid if $ h$ is small.\r\n\r\nIf the paper has thickness of $ 1mm$ the difference of length would've be only $ 0.43 mm$\r\n\r\nI don't think this is a precise result, but we could've play with it. Find the average thickness of the fold, how may chemical bonds were broken when you fold (this explain why the paper is too sensive in the fold, chemical bonds are broken when you fold a paper)\r\n\r\nThis is not a study in paper, just some interesting models to work out.\r\n\r\nI don't know too much about physics of bending materials, so I can't say that those approximations are good or not."
}
{
"Tag": [
"calculus",
"integration",
"calculus computations"
],
"Problem": "What is the sum from 1 to infinity of $ \\frac{1}{n \\sqrt{n^2\\plus{}2}}$? Please prove it.",
"Solution_1": "Numerically, it's about $ 1.1588006.$ Is there some reason why it should be a more recognizable number than that?",
"Solution_2": "It was on my friend's homework.",
"Solution_3": "1. For what class?\r\n\r\n2. Are you sure it wasn't an integral?\r\n\r\n3. Are you sure it wasn't an estimate?",
"Solution_4": "Well, that's what she told me 4 calc bc, no, it's definitely a sum, but the problem was probably just to see whether it converges or not which is trivial, although she asked me 4 the exact value.",
"Solution_5": "At that level, you're definitely not asking for the exact value on something like this. Most of the time, that exact value can't be written in closed form."
}
{
"Tag": [
"induction",
"number theory proposed",
"number theory"
],
"Problem": "Let $ a_1\\equal{}a_2\\equal{}1$ and \\[ a_{n\\plus{}2}\\equal{}\\frac{n(n\\plus{}1)a_{n\\plus{}1}\\plus{}n^2a_n\\plus{}5}{n\\plus{}2}\\minus{}2\\]for each $ n\\in\\mathbb N$. Find all $ n$ such that $ a_n\\in\\mathbb N$.",
"Solution_1": "First I get $ na_n\\minus{}(n\\minus{}1)^2a_{n\\minus{}1}\\equal{}\\left\\{\r\n\\begin{array}{ll}\r\n\\minus{}n\\plus{}2\\ (n: odd) &\\quad \\\\\r\n\\minus{}n\\plus{}3\\ (n: even) &\\quad\r\n\\end{array}\r\n\\right.$",
"Solution_2": "Prove by induction that \\[ a_n\\equal{}\\frac{(n\\minus{}1)!\\plus{}1}n\\]\r\nThe rest is obvious.",
"Solution_3": "$ a_{3}=\\frac{2}{3} $. your formula does not work for $ n=1,3 $. your formula works for the sequence $ a_{1}=2,a_{2}=1 $...so it was a typo in the initial problem?",
"Solution_4": "[quote=\"anonymouslonely\"]$ a_{3}=\\frac{2}{3} $. your formula does not work for $ n=1,3 $. your formula works for the sequence $ a_{1}=2,a_{2}=1 $...so it was a typo in the initial problem?[/quote]\nNo in the initial problem $ a_{1} $ is 2 and $ a_{2} $ is 1.Right!"
}
{
"Tag": [
"inequalities",
"calculus",
"trigonometry",
"logarithms",
"calculus computations"
],
"Problem": "Prove that :$ (sin x)^{cos x}<(cos x)^{sinx}\\ for \\ 0(sin(x))^{cos(x)}$\n[/hide]\r\nthank u",
"Solution_2": "[quote=\"pankajsinha\"]Prove that :$ (sin x)^{cos x} < (cos x)^{sinx}\\ for \\ 0 < x < \\frac {\\pi}{4}$[/quote]\r\nYou could do it taking the logarithm.\r\nLet $ f(x) \\equal{} (\\sin x)( \\log \\cos x)$ and $ g(x) \\equal{} (\\cos x)( \\log \\sin x)$\r\nNow divide both by $ \\log \\cos x$.\r\nThat way, you have logarithms with a base and you can compare the values accordingly for the interval $ \\left[0, \\frac {\\pi}{4}\\right]$.",
"Solution_3": "For $ 0ab-a-b and x,y>0",
"Solution_1": "For the first: when I remember correctly: yes (possibly I forgot some other condition that we need)\r\n\r\nFor the second (but please post only one problem per thread in future):\r\nSome people call it (I don't know why) the Chicken McNugget theorem, so searching for it will help.",
"Solution_2": "I remember posting a proof for the affirmative answer to your first question some time ago, but it's not worth the effort of looking for it :).\r\n\r\nSuppose $x$ is not nilpotent. Our aim is to show that there is a prime ideal which does not contain $x$. Let $\\mathcal I$ be the collection of ideals which do not contain any of the elements $x,x^2,\\ldots$. Since $x$ is not niplotent, the zero ideal is an element of $\\mathcal I$, so, in particular, $\\mathcal I$ is non-empty. It's easy to see that this collection satisfies the hypothesis necessary for applying Zorn's Lemma: every chain (with respect to inclusion) in $\\mathcal I$ has an upper bound, namely the union of all the ideals in the chain. By Zorn's Lemma, $\\mathcal I$ has a maximal element $P$. We're done if we can show that $P$ is prime.\r\n\r\nTake $a,b\\not\\in P$. By the maximality of $P$, we have $x^m\\in P+(a),\\ x^n\\in P+(b)$ for some positive integers $a,b$. We then clearly have $x^{m+n}\\in P+(ab)\\Rightarrow ab\\not\\in P$, so whenever $a,b\\not\\in P$, we get $ab\\not\\in P$, which means precisely that $P$ is prime."
}
{
"Tag": [
"inequalities",
"calculus",
"derivative",
"algebra",
"function",
"domain"
],
"Problem": "show that for any x natural>=1 we have:\r\n\r\nx=[1/a^2]; where a=arctan(1/sqrt(x)).\r\n\r\nand [t]=t-{x}.",
"Solution_1": "spoiler:\n\n[hide]\n\nconsider the right triangle below with side lengths 1, :sqrt: x, and :sqrt: x+1. It is clear that arctan(1/ :sqrt: x)= the angle D. So what we want to prove is that x = [1/D^2] or equivalently x<=1/D^21/ :sqrt: x+1 or D>sinD, so we need to prove the equivalent identity, for all positive angles D, 0sinD for all D>0. for the rhs, consider tanD-D with derivative sec^2(D)-1, now this derivative is positve for 0 D for 00$, and any $n\\in\\mathbb N$, we can find $t_n>n$ such that $|x'(t_n)|<\\varepsilon$: simply take $n1$, with $|x(s)-L|<\\frac\\varepsilon 2,\\ \\forall s\\ge t$, and choose $t_n\\in(t,t')$ such that $(t'-t)x'(t_n)=x(t')-x(t)$. We have $|u'(x(t_n))|=|x'(t_n)|<\\varepsilon$, and the conclusion that $u'(L)=0$ follows.",
"Solution_2": "nice :) \r\nand you're right, it's a bit more difficult (i think) for the case $d>1$. But it's still true."
}
{
"Tag": [
"AMC",
"AIME",
"geometry",
"circumcircle",
"3D geometry",
"algebra",
"polynomial"
],
"Problem": "[For reference: the contest page: [url]http://www.artofproblemsolving.com/Forum/viewtopic.php?t=126603[/url]]\r\n\r\n[size=167]Mock AIME 5 for 2007[/size]\r\n\r\nWritten by Alex Anderson\r\nwith contributions from Jacob Steinhardt\r\n\r\n[hide=\"Answers:\"]1.\t299\n2.\t322\n3.\t972\n4.\t997\n5.\t181\n6.\t204\n7.\t670\n8.\t601\n9.\t275\n10.\t502\n11.\t936\n12.\t003\n13.\t252\n14.\t647\n15.\t219[/hide]\r\n\r\nResults:\r\n\r\n1st place: :winner_first: \r\nMysticTerminator, 14/15\r\n2,3,4,5,6,7,8,9,10,11,12,13,14,15\r\n\r\n2nd place: :winner_second: \r\npaladin8, 12/15\r\n1,2,3,4,5,6,7,8,9,10,12,14\r\n\r\n3rd place [tie]: :winner_third: \r\nbeta 10/15\r\n2,3,4,5,6,7,8,11,12,13\r\nnat mc 10/15\r\n1,2,3,6,7,9,10,11,12,13\r\n\r\nComments:\r\n\r\nThanks a lot for jsteinhardt for helping me check the test. Also he submitted 4th and 14th problems and thought of the idea for problem 5. \r\n\r\nProblem 12 had a small error...I decided to let the test go on, rather than trying to change anything. It was pretty clear to find the largest lower bound. Regardless, if you even looked at that problem, you should have just asked with a private message; I was quite clear about this.\r\n\r\nI suspect that some people used a calculator for problem 5. I probably should have said that there were 7 prime factors, find the second largest one. \r\n\r\nMany people were confused about problem #3. A geometric sequence of integers with a finite number of terms does not have to have an integer ratio. \r\n\r\nProblem # and percentage of people who answered it correctly:\r\n1.\t75\r\n2.\t87.5\r\n3.\t71.875\r\n4.\t56.25\r\n5.\t50\r\n6.\t68.75\r\n7.\t62.5\r\n8.\t18.75\r\n9.\t46.875\r\n10.\t21.875\r\n11.\t21.875\r\n12.\t12.5\r\n13.\t12.5\r\n14.\t6.25\r\n15.\t3.125\r\n\r\noverall average:\t6.15625 of 15 correct\r\n\r\nDifficulty:\r\n\r\nThe first two problems were significantly easier than the actual AIME. I would say that the next 7 problems [3-9] problems or so were on par with an official AIME. #10 was particularly difficult to find the actual solution. Many people found the pattern and solved that way. Perhaps I could have put this problem later. I feel that many people saw this problem, got discouraged, and stopped working on problems. I feel that more people could have solved #12. Anyway, the later geometry was pretty difficult. Also #14 was tough and interesting problem; one of my favorites on the test.\r\n\r\nI thought that the general content was similar. I did not have any \"annoying casework problems\" that are prevalent on the AIME contests. I also felt that I could have included more counting problems. #6 was pretty interesting, but #8 was a standard application of recursion for expected value.\r\n\r\nSolutions:\r\n\r\nI don't plan to type out official solutions. Every problem has been answered by somebody. Furthermore, I have sent people solutions to some of the harder problems, like #10,14,15. Perhaps I could get some help writing out solutions...I can compile them into a one thread...maybe wiki entries too...\r\n\r\nFinally,\r\nI appreciate the positive feedback. I worked hard on this contest, and I think it turned out well. \r\n\r\n--Alex Anderson",
"Solution_1": "Cool. I really liked #14 as well :).\r\n\r\nBTW, nice job to Arnav for getting them all except #1 :P.",
"Solution_2": "Yes, that is rather ironic, isn't it? Problem 1 [b]was[/b] quite an easy problem.\r\nI did 1, 2, 3, and then got too lazy and didn't finish it. I did get all three of them right, though. :lol:",
"Solution_3": "I'm willing to bet he submitted 42 or the like for #1.",
"Solution_4": "Since some people were wondering...\r\n\r\n[hide=\"15\"]\nDenote $DE=x$. Consider just $P$, $A$, $B$, $D$, $E$, and $C$. You can either use similar triangles, or some tricky polar stuff [somebody will have to fill in the pole/polar stuff] to show that $P$, $D$, $E$, and $C$ are in a harmonic progression. In particular this means [without directed lengths] that\n\n\\[\\frac{PD}{DE}*\\frac{EC}{CP}=1\\]\n\nHence $CE=2x$, and using power of a point on $E$, \n\n\\[(x)(2x)=12\\implies x=\\sqrt{6}\\]\n\nIt follows directly that $PD=3\\sqrt{6}$, $PA^{2}=(3\\sqrt{6})(6\\sqrt{6})\\implies PA=PB=6\\sqrt{3}$.\n\nNow we have to deal with $F$. Let $I$ be an inversion centered at $P$, with the circle passing through $A$ and $B$. This maps $w_{1}$ to itself [since the circles are orthogonal, or equivalently using power of a point]. Hence, $I(A)=A$ and $I(D)=C$. Furthermore, we know that the circumcircle of $I(ADFP)$ is a line because any circle through the center of inversion becomes a line, this line being $AC$. We need to find $I(F)$. We know that this must lie on $PF$ and $AC$. Hence, $I(F)=G$. Then by definition of inversion, \n\n\\[PA^{2}=(PF)(PG)\\implies 108=PF*(6\\sqrt{3}+10)\\implies PF=81\\sqrt{3}-135\\]\n\nIt follows that the answer is $81+3+135=\\boxed{219}$.\n\nComment: Obviously this problem was quite difficult. I wanted to go all out, so to speak, on the geometry at the end.[/hide]",
"Solution_5": "Woah totally forgot to even look at the questions.\r\n\r\n5 looks way too easy. I don't want to work it out, but can't we be all \"WOAH DIFFERENCE OF SQUARES!!!111lololol\"?",
"Solution_6": "[quote=\"PenguinIntegral\"]Woah totally forgot to even look at the answers.\n\n5 looks way too easy. I don't want to work it out, but can't we be all \"WOAH DIFFERENCE OF SQUARES!!!111lololol\"?[/quote]\r\n\r\nUnless you can factor 40001 magically, you need some other techniques...I think that these problems are reasonable, and could be on an aime, but since there is nobody preventing the use of a calculator during a mock test, I don't think I will use this type of problem again.",
"Solution_7": "You can do #15 using only basic angle-chasing/power-of-a-point, but it's still fairly involved.",
"Solution_8": "[quote=\"probability1.01\"]You can do #15 using only basic angle-chasing/power-of-a-point, but it's still fairly involved.[/quote]\r\n\r\nyea...i realize...but i prefer the concise solution...the harmonic progression directly follows from triangle similarity...the inversion thing is equivalent to showing that $\\triangle APF\\sim\\triangle GPA$...",
"Solution_9": "One way to factor $40001$ is to write it as $40401-400$.",
"Solution_10": "[quote=\"Ravi B\"]One way to factor $40001$ is to write it as $40401-400$.[/quote]\r\n\r\n[hide]in general, that would be 4a^4+1=(2a^2-2a+1)(2a^2+2a+1)\n\nthis is the sophie-germain identity[/hide]",
"Solution_11": "[quote=\"Altheman\"][quote=\"PenguinIntegral\"]Woah totally forgot to even look at the answers.\n\n5 looks way too easy. I don't want to work it out, but can't we be all \"WOAH DIFFERENCE OF SQUARES!!!111lololol\"?[/quote]\n\nUnless you can factor 40001 magically, you need some other techniques...I think that these problems are reasonable, and could be on an aime, but since there is nobody preventing the use of a calculator during a mock test, I don't think I will use this type of problem again.[/quote]\r\nWhy would I need magic? It only took me a few minutes to find 13, 17, and checking everything under the square root of 181 finds that it is prime. What was your intended solution, and could you give a hint for 12?",
"Solution_12": "[hide=\"hint for 12\"] $x^{3}-x(y^{2}+z^{2}) = y^{3}-y(z^{2}+x^{2})$ tells you a lot more than it looks like it tells you. [/hide]",
"Solution_13": "[quote=\"paladin8\"][hide=\"hint for 12\"] $x^{3}-x(y^{2}+z^{2}) = y^{3}-y(z^{2}+x^{2})$ tells you a lot more than it looks like it tells you. [/hide][/quote]\r\nSet the difference of cubes equal to the other terms?",
"Solution_14": "[quote=\"PenguinIntegral\"][quote=\"paladin8\"][hide=\"hint for 12\"] $x^{3}-x(y^{2}+z^{2}) = y^{3}-y(z^{2}+x^{2})$ tells you a lot more than it looks like it tells you. [/hide][/quote]\nSet the difference of cubes equal to the other terms?[/quote]\r\n\r\nYes... and what do you get?",
"Solution_15": "[quote=\"paladin8\"][quote=\"PenguinIntegral\"][quote=\"paladin8\"][hide=\"hint for 12\"] $x^{3}-x(y^{2}+z^{2}) = y^{3}-y(z^{2}+x^{2})$ tells you a lot more than it looks like it tells you. [/hide][/quote]\nSet the difference of cubes equal to the other terms?[/quote]\n\nYes... and what do you get?[/quote]\r\nSomething like $(x-y)(x^{2}+xy+y^{2})=x(y^{2}+z^{2}-y(z^{2}+x^{2})$\r\n\r\nIf we expand that, $(x-y)(x^{2}+xy+y^{2})=x(y^{2}+z^{2}-y(z^{2}+x^{2})=xy^{2}+xz^{2}-yz^{2}-yx^{2}=xz^{2}-yz^{2}=z^{2}(x-y)$\r\n$(x^{2}+xy+y^{2})=z^{2}$?",
"Solution_16": "[hide=\"5.\"]1,599,999,999 can be written as $16*a^{8}-1$ if we let $a=10$, then factoring: \n\n\\[16*a^{8}-1=(4a^{4}+1)(4a^{4}-1) =(2a^{2}-2a+1)(2a^{2}+2a+1)(2a^{2}+1)(2a^{2}-1)\\]\n\nHence, we get the factorization $(181)(221)(201)(199)$. It is easy to check these factors for divisibility by $2,3,5,7,11,13$ [we need not go higher because $15^{2}>181,221,201,199$]. We obtain the prime factorization as $3*13*17*67*181*199$. Hence the answer is $\\boxed{181}$[/hide]\n\n[hide=\"12.\"]\n\nSubtracting equations gives:\n\n\\[(x^{3}-y^{3})-z^{2}(x-y)-(xy)(x-y)=(x-y)(x^{2}-2xy+y^{2}-z^{2})\\]\n\nThe second condition gives us that $x\\ne y$, so we have that $(x+y)^{2}=z^{2}$, likewise, we have $(y+z)^{2}=x^{2}$, and $(z+x)^{2}=y^{2}$. Adding these and dividing by 2 gives: \n\n\\[x^{2}+y^{2}+z^{2}+2(xy+yz+zx)=0\\]\n\nSubtracting $(x+y)^{2}=z^{2}$ gives:\n\n\\[2yz+2zx+2z^{2}=0\\implies (z)(x+y+z)=0\\]\n\nWe throw out $z=0$, and have $x+y+z=0$. Plugging this into the last statement gives $x=1$. Then if $y=t$, then $z=-1-t$. We can finish by completing the square:\n\n\\[(1)^{2}+(t)^{2}+(-1-t)^{2}=2t^{2}+2t+2=2\\left(t+\\frac{1}{2}\\right)^{2}+3\\]\n\nThe minimum does not occur...but it can be approached...this was my mistake on the test. Anyway, the answer is $\\boxed{3}$.\n[/hide]",
"Solution_17": "Is 8 just a recursive mess?",
"Solution_18": "Oh hah I forgot to submit this...Well lucky for an AIME...no careless mistakes, i got 15!\r\n\r\nI liked it: pretty much all clean problems, no major computation. But one question I had: I got that there's only onep ossible real part for k in #10...",
"Solution_19": "[quote=\"K81o7\"]Oh hah I forgot to submit this...Well lucky for an AIME...no careless mistakes, i got 15!\n\nI liked it: pretty much all clean problems, no major computation. But one question I had: I got that there's only onep ossible real part for k in #10...[/quote]\r\n\r\no rly?\r\n\r\nin retrospect, #10 was not so good for an aime...the point was to prove that the sum was invariant...i could not think of how to phrase the problem so that you would need to find the invariant...\r\n\r\n#8 is pretty simple recursion, if you do it right, it becomes 1.1^n...",
"Solution_20": "[quote=\"Altheman\"]\n#8 is pretty simple recursion1.1^n...[/quote]\r\nO rly?",
"Solution_21": "ya rly",
"Solution_22": "Can someone show me how to do this one: \r\n\r\n\r\n9. If $p(x)=x^{3}-5x^{2}+4x-1$ has roots $a$, $b$, and $c$, then find $\\left(a^{2}-\\frac{1}{a^{2}}\\right)\\left(b^{2}-\\frac{1}{b^{2}}\\right)\\left(c^{2}-\\frac{1}{c^{2}}\\right)$.",
"Solution_23": "[quote=\"pkothari13\"]Can someone show me how to do this one: \n\n\n9. If $p(x)=x^{3}-5x^{2}+4x-1$ has roots $a$, $b$, and $c$, then find $\\left(a^{2}-\\frac{1}{a^{2}}\\right)\\left(b^{2}-\\frac{1}{b^{2}}\\right)\\left(c^{2}-\\frac{1}{c^{2}}\\right)$.[/quote]\r\n\r\n\r\n[hide=\"9\"]Note that $p(x)=(x-a)(x-b)(x-c)$ by the factor theorem. Then we factor to employ that fact. [The product is taken over a,b, and c].\n\n\\begin{eqnarray*}\\left(a^{2}-\\frac{1}{a^{2}}\\right)\\left(b^{2}-\\frac{1}{b^{2}}\\right)\\left(c^{2}-\\frac{1}{c^{2}}\\right)&=&\\frac{1}{(abc)^{2}}(a^{4}-1)(b^{4}-1)(c^{4}-1)\\\\ &=&\\frac{1}{(abc)^{2}}\\prod (1-a)(-1-a)(i-a)(-i-a)\\\\ &=&\\frac{1}{(abc)}*p(1)p(-1)p(i)p(-i)\\\\ &=&\\frac{1}{(-1)^{2}}*(-1)(-11)(4+3i)(4-3i)\\\\ &=&275\\\\ \\end{eqnarray*}[/hide]\r\n\r\nEDIT: typo",
"Solution_24": "[quote=\"Altheman\"][hide=\"9\"]Note that $p(x)=(x-a)(x-b)(x-c)$ by the factor theorem. Then we factor to employ that fact. [The product is taken over a,b, and c].\n\n\\begin{eqnarray*}\\left(a^{2}-\\frac{1}{a^{2}}\\right)\\left(b^{2}-\\frac{1}{b^{2}}\\right)\\left(c^{2}-\\frac{1}{c^{2}}\\right)&=&\\frac{1}{(abc)^{2}}(a^{4}-1)(b^{4}-1)(c^{4}-1)\\\\ &=&\\frac{1}{(abc)^{2}}\\prod (1-a)(-1-a)(i-a)(-i-a)\\\\ &=&\\frac{1}{(abc)}*p(1)p(-1)p(i)p(-1)\\\\ &=&\\frac{1}{(-1)^{2}}*(-1)(-11)(4+3i)(4-3i)\\\\ &=&275\\\\ \\end{eqnarray*}[/hide][/quote]\r\n\r\nAh! That's a great solution! Thanks man :)",
"Solution_25": "[quote=\"pkothari13\"][quote=\"Altheman\"][hide=\"9\"]Note that $p(x)=(x-a)(x-b)(x-c)$ by the factor theorem. Then we factor to employ that fact. [The product is taken over a,b, and c].\n\n\\begin{eqnarray*}\\left(a^{2}-\\frac{1}{a^{2}}\\right)\\left(b^{2}-\\frac{1}{b^{2}}\\right)\\left(c^{2}-\\frac{1}{c^{2}}\\right)&=&\\frac{1}{(abc)^{2}}(a^{4}-1)(b^{4}-1)(c^{4}-1)\\\\ &=&\\frac{1}{(abc)^{2}}\\prod (1-a)(-1-a)(i-a)(-i-a)\\\\ &=&\\frac{1}{(abc)}*p(1)p(-1)p(i)p(-1)\\\\ &=&\\frac{1}{(-1)^{2}}*(-1)(-11)(4+3i)(4-3i)\\\\ &=&275\\\\ \\end{eqnarray*}[/hide][/quote]\n\nAh! That's a great solution! Thanks man :)[/quote]\r\nCan't we just use standard AoPS2 methods to find the polynomial with those roots then find the constant term?",
"Solution_26": "abc is 1(from vietas), not -1, but it doesnt make a difference when you are squaring it..\r\n\r\nI just kept on playing around with a+b+c, ab+ac+bc, and abc..until you get the answer..",
"Solution_27": "[quote=\"PenguinIntegral\"]Can't we just use standard AoPS2 methods to find the polynomial with those roots then find the constant term?[/quote]\r\n\r\nbut that is ugly...and this technique is useful in other contexts...like this year's hmmt algebra #10, for example",
"Solution_28": "Wow I would have had 11, but I missed #5 because i din't see that $3|201$. I feel dumb, but not as dumb as arnav missing number one lol.",
"Solution_29": "[quote=\"Altheman\"][quote=\"PenguinIntegral\"]Can't we just use standard AoPS2 methods to find the polynomial with those roots then find the constant term?[/quote]\n\nbut that is ugly...and this technique is useful in other contexts...like this year's hmmt algebra #10, for example[/quote]\r\nBut the elegant method takes thought, and it's a nine. I really don't see the AMC putting that kind of textbook question that late in the test.\r\n\r\n@nat mac\r\n\r\nArnav does not \"miss\" math problems. He just typed in the wrong answer.",
"Solution_30": "Sorry, PenguinIntegral, could you explain your (the AoPS 2) method more fully?\r\n\r\nI solved the problem the Altheman way.",
"Solution_31": "[quote=\"Ravi B\"]Sorry, PenguinIntegral, could you explain your (the AoPS 2) method more fully?\n\nI solved the problem the Altheman way.[/quote]\r\nI think he means newton's sums",
"Solution_32": "Heh, I had a pretty ugly solution to 9\r\n\r\n[hide]\nMatching coefficients with the roots, we have\n\\begin{eqnarray*}a+b+c &=& 5\\\\ ab+bc+ac &=&4\\\\ abc &=& 1 \\end{eqnarray*}\n\nExpand out the product to eventually get\n\\[(abc)^{2}-\\left(\\frac{1}{abc}\\right)^{2}+\\frac{a^{4}+b^{4}+c^{4}}{(abc)^{2}}-\\frac{(ab)^{4}+(bc)^{4}+(ac)^{4}}{(abc)^{2}}. \\]\n\nTo find $a^{4}+b^{4}+c^{4}$, apply Newton's sums on the original polynomial. For the last term, the polynomial with roots $ab, bc, ca$ is\n\\begin{eqnarray*}(x-ab)(x-bc)(x-ac) &=& x^{3}-(ab+bc+ac)x^{2}+(a^{2}bc+ab^{2}c+abc^{2})x-(abc)^{2}\\\\ &=& x^{3}-4x^{2}+5x-1 \\end{eqnarray*}\nSo to find $(ab)^{4}+(bc)^{4}+(ac)^{4}$, apply Newton's sum on the above polynomial.\n[/hide]\r\n\r\nOh, and I thought question 1 was kinda hard, relative to its position in the test (I got it wrong, :blush: )",
"Solution_33": "I tried Newton's Sums but they don't like me :( . (I had to look them up in AoPS 2 and I still got it wrong :( )",
"Solution_34": "[quote=\"Ravi B\"]Sorry, PenguinIntegral, could you explain your (the AoPS 2) method more fully?\n\nI solved the problem the Altheman way.[/quote]\r\nLook somewhere in the polynomial section where it shows how to construct polynomials that have, for example, roots that are the reciprocals of the original polynomial.",
"Solution_35": "Thanks. I looked at AoPS 2, and see how to construct a polynomial whose roots are reciprocals of the originals, or a multiple of the originals. But constructing a polynomial with roots $a^{2}-\\frac{1}{a^{2}}$ (and same for $b$ and $c$) seems harder. At least I don't see a nice method. Do you have one?",
"Solution_36": "Hm, I remember having a nice way of doing this, but I don't remember what it was, and I threw away my scratch work... :wallbash:",
"Solution_37": "[quote=\"Ravi B\"]Thanks. I looked at AoPS 2, and see how to construct a polynomial whose roots are reciprocals of the originals, or a multiple of the originals. But constructing a polynomial with roots $a^{2}-\\frac{1}{a^{2}}$ (and same for $b$ and $c$) seems harder. At least I don't see a nice method. Do you have one?[/quote]\r\nI'm sure there is a way to do it. I haven't really thought about it much...",
"Solution_38": "[quote=\"PenguinIntegral\"][quote=\"Altheman\"][quote=\"PenguinIntegral\"]Can't we just use standard AoPS2 methods to find the polynomial with those roots then find the constant term?[/quote]\n\nbut that is ugly...and this technique is useful in other contexts...like this year's hmmt algebra #10, for example[/quote]\nBut the elegant method takes thought, and it's a nine. I really don't see the AMC putting that kind of textbook question that late in the test.[/quote]\r\n\r\nConstructing a polynomial with such roots is complicated, cumbersome, and prone to error. Even for simple problems, it's usually easier to use an alternate approach; a problem with roots as relatively complicated as it was in Altheman's AIME is even more so unwieldy to solve with construction methods. \r\n\r\nOn the other hand, the elegant solution is quick, extremely nice, and FAR more applicable; even simple problems are slaughtered by the factor theorem approach.",
"Solution_39": "what are the mock AIME for?",
"Solution_40": "[quote=\"yhh3002\"]what are the mock AIME for?[/quote]\r\npractice and fun"
}
{
"Tag": [
"calculus",
"integration",
"trigonometry",
"real analysis",
"real analysis solved"
],
"Problem": "I think that the integral $\\displaystyle \\int \\sqrt{ \\tan x} dx$ was discussed last week here. So I was thinking about computing:\r\n\r\n$\\displaystyle \\int \\sqrt[3]{ \\tan x} dx$ and $\\displaystyle \\int \\sqrt[3]{ \r\nctan x} dx$ for $\\displaystyle x \\in \\left( 0, \\frac{ \\pi}{2} \\right).$\r\n\r\nThen try:\r\n\r\n$\\displaystyle \\int \\sqrt[n]{ \\tan x} dx$ and $\\displaystyle \\int \\sqrt[n]{ ctan x} dx$ for $\\displaystyle x \\in \\left( 0, \\frac{ \\pi}{2} \\right),$ $n \\geq 2,$ $n \\in N.$\r\n\r\ncheers! :D :D",
"Solution_1": "Let $\\sqrt[3]{\\tan x} = u$, $x = \\arctan u^3$, thus\r\n$dx = \\frac{3 u^2}{1 + u^6}$\r\n\r\n$\\int \\sqrt[3]{\\tan x} dx = \\int \\frac{3 u^3}{1+u^6} du$\r\n\r\nWe have reduced it to a rational form.\r\n\r\n$\\frac{u^3}{1+u^6} = \\frac{A}{u^2+1} + \\frac{B}{u^2-\\sqrt3 u+1} + \\frac{C}{u^2+\\sqrt3 u+1}$ \r\n\r\nFind $A, B, C$ in this way... and we get the primitive.",
"Solution_2": "For $A=\\int \\sqrt[n]{\\tan x} dx$, $x=\\arctan u^n$ \r\n$A=n\\int\\frac{u^n}{1+u^{2n}}du$\r\n\r\nPartial fraction in $\\mathbb C[X]$ and coming back on $\\mathbb R[X]$\r\n\r\ngives\r\n\r\nif $m,n\\geq 2$ are positive integer with $m<2n$\r\n\r\n$\\int \\frac{u^{m-1}}{1+u^{2n}}du = \\frac{-1}{2n}\\sum_{k=1}^{n}\r\n\\cos(a_k)\\ln(1-2x\\cos(b_k)+x^2)+\\frac{1}{n}\\sum_{k=1}^{n}\\sin(a_k)\\arctan(\\frac{x-\\cos(b_k)}{sin(b_k)})$\r\n\r\nWith $a_k = \\frac{m\\pi(2k-1)}{2n}$, $b_k=\\frac{(2k-1)\\pi}{2n}$"
}
{
"Tag": [
"number theory unsolved",
"number theory"
],
"Problem": "$ a$, $ b$ are positive non-integers numbers such a+b is an integer:\r\na)is it possible to a^20 + b^20 be an integer?\r\nb)and a^21 + b^21 ?",
"Solution_1": "If $ a\\plus{}b$ and $ ab$ integers, then $ a^n\\plus{}b^n$ integer too for any natural n. For example $ a\\equal{}3\\minus{}3\\sqrt 2, b\\equal{}3\\plus{}2\\sqrt 2$."
}
{
"Tag": [
"number theory unsolved",
"number theory"
],
"Problem": "Find all integers n such that\r\n2^(n-1)/n!",
"Solution_1": "$V_{2}(n!)=\\sum_{k=1}^{+\\infty}[n/2^{k}]$\r\n\r\n$2^{n-1}|n!$ if $V_{2}(n!)\\geq n-1$\r\n\r\n$V_{2}(n!) < n\\sum_{k=1}^{+\\infty}1/2^{k}=n$\r\n$V_{2}\\ge n-1$ => $V_{2}(n!)=n-1$\r\nwe mut have for any k, $E(n/2^{k})=n/2^{k}$ or $E(n/2^{k})=0$\r\n\r\nn must be a power of 2\r\n\r\nS={$2^{m}$ / m de IN}"
}
{
"Tag": [
"function",
"analytic geometry"
],
"Problem": "I'm looking over functions and graphs. can anyone explain to me why you move the graph $y=x^{2}$, two to the right to graph this:$f(x)=(x-2)^{2}$why is it that you move right when its$x-\\textbf{3}$? and why does the graph steches horizontally by a factor of 2 when the graph is $f(x)=(2x)^{2}$ ? seems like everything for x is backwards...\r\n\r\nThank you :P\r\n\r\n[color=red][size=59]Preview your posts before posting :wink: \nb-flat[/color][/size]",
"Solution_1": "Don't EVER think graph. Think COORDINATE SYSTEM.\r\n\r\nIf you put $x-2$ instead of $x$, you effectively move the x-axis for $-2$, that is for $2$ to the left, which appears as the graph moved to the right. Similarly, if you put $x+3$, you move x-axis for $+3$, that is $3$ to the right, which apparently moves the graph $3$ to the left.\r\n\r\nIf you put $2x$ instead of $x$, you multiply every number on x-axis by $2$, which shrinks it twice, and that looks like the graph has been stretched twice.\r\n\r\nAnd so on.\r\n\r\nRemember: Whatever happens to the coordinates, it's applicable to the COORDINATES (not the graph), therefore from the graph point of view, exactly the \"opposite\" happens.",
"Solution_2": "but that doesnt happen for the y values. $y=2(x)^{2}$ The numbers on the y axis aren't multiplied by 2. and why are the numbers on the x axis doubled for $y=(2x)^{2}$ ? \r\n\r\nthanks for your help",
"Solution_3": "It DOES happen: Just as you put $2x$ instead of $x$, you put $2y$ instead of $y$, hence the y-axis is shrank twice, meaning the graph is stretched twice along the y-axis.\r\n\r\nTherefore, if you have the graph of $y=f(x)$, then the graph of $y=Cf(ax+b)+D$ is obtained by writing $ax+b$ instead of every $x$ on x-axis and writing $Cy+D$ instead of every $y$ on y-axis. That results in the following: (1) x-axis is shrank $a$ times (\"shrinking\" is to be understood relatively, i.e. it IS shrinking if $a>1$ and becomes stretching if $a<1$) and moved right by $b$ (again relatively: \"right\" by a negative number is \"left\"), hence the graph is stretched $a$ times horizontally and moved left by $b$; (2) y-axis is shrank $C$ times (relatively) and moved up by $D$ (where \"up\" by a negative number is \"down\"), hence the graph is stretched $C$ times vertically and moved down by $D$"
}
{
"Tag": [
"algebra",
"polynomial",
"calculus",
"derivative",
"geometry",
"3D geometry",
"calculus computations"
],
"Problem": "Okay, so I had to estimate the sq. root of 7.9 with a Taylor polynomial of degree three.\r\n\r\nI based the polynomial at 8.\r\n\r\nFirst I calculated the 1st-3rd derivatives of x^(1/2):\r\ng'(x) = 1/2(x)^(-1/2)\r\ng''(x) = -1/4(x)^(-3/2)\r\ng'''(x) = 3/8(x)^(-5/2)\r\nThen there's g(x), which is just the sq. root of 8.\r\n\r\nWhen plugging in 8 for x, you get for the Taylor polynomial of degree three:\r\n\r\nT3(x) = sqrt[8] + 1/2(8)^-1/2 (x-8) + 1/2 * -1/4(8)^(-3/2) (x-8)^2 + 1/6 * 3/8(8)^(-5/2) (x-8)^3\r\n\r\nPlugging in 7.9 for x in the polynomial above, you get 2.810693867, which is not far off from what you get when you enter the square root of 7.9 into a calculcator. When you enter it into the calculator, you get 2.810693865.\r\n\r\nAnd now for the question that is confusing me:\r\nApply Taylor's theorem to find an upper bound on the error in this approximation.\r\n.\r\nIf someone could explain how this is done, I would REALLY appreciate it. Thanks in advance!\r\n\r\n(And one more thing: I think I might have been wrong in using 8 for approximating the square root of 7.9 since 8 isn't a perfect square. Would it be more appropriate to use 9 instead?)",
"Solution_1": "[quote=\"SharkBites77\"]I think I might have been wrong in using 8 for approximating the square root of 7.9 since 8 isn't a perfect square. Would it be more appropriate to use 9 instead?)[/quote]\r\n\r\nYes. Notice that in your polynomial you had to evaluate the square root of $8$ implicitly several times, which is not an integer. \r\n\r\nThe choice of $7.9$ is unusual for a square root, though. Are you sure the problem didn't say [i]cube[/i] root?",
"Solution_2": "Yeah, it did say sq. root of 7.9.",
"Solution_3": "Hmm.\r\n\r\nAnyway, once you've found the correct taylor series centered around $9$, consult\r\n\r\nhttp://en.wikipedia.org/wiki/Taylor's_theorem\r\n\r\nTaylor's theorem essentially says that an upper bound for the error when you take the series to $n$ terms is given by the $(n+1)^{th}$ term, except that instead of\r\n\r\n$\\frac{ f^{(n+1)}(a)}{(n+1)!}(x-a)^{n+1}$\r\n\r\nThe input for $f^{(n+1)}$ is some $\\zeta$ between $a, x$ that maximizes $f^{(n+1)}$.",
"Solution_4": "Thanks! I'm a bit confused to how I would find this maximum value, though."
}
{
"Tag": [
"inequalities",
"inequalities unsolved"
],
"Problem": "[color=blue][size=150]a, b and c belongs to the interval [1/3,1].\nprouve that a/(a+b)+b/(b+c)+c/(c+a)>=7/5 [/size][/color]",
"Solution_1": "See here http://www.mathlinks.ro/Forum/viewtopic.php?t=134826",
"Solution_2": "also you can uyse it http://www.mathlinks.ro/Forum/viewtopic.php?t=135954"
}
{
"Tag": [
"search"
],
"Problem": "If anybody has links to sites which have practice SATs, it would greatly be appreciated, thanx :)",
"Solution_1": "Why not buy a book of the \"10 Real SATs\" sort? They're cheap at $20 and contain a lot of tests.",
"Solution_2": "Kaplan also has good practice SAT's.",
"Solution_3": "Yes, but I've bought three: Kaplan's and Baron's, and Master the SAT, and finished all of them.",
"Solution_4": "Ok, [url=http://www.educationplanner.com/education_planner/preparing_article.asp?sponsor=2859&articleName=SAT_Practice_Tests]try this one[/url] :) It has two practice tests (in pdf files). Does it help?\r\n\r\nYou can go to google and search \"SAT practice tests\" etc. ;)",
"Solution_5": "[quote=\"frt\"]Ok, [url=http://www.educationplanner.com/education_planner/preparing_article.asp?sponsor=2859&articleName=SAT_Practice_Tests]try this one[/url] :) It has two practice tests (in pdf files). Does it help?\n\nYou can go to google and search \"SAT practice tests\" etc. ;)[/quote]\r\n\r\nYup, good link frt :D",
"Solution_6": "Really the one and only SAT prep book you need is [url=http://www.amazon.com/exec/obidos/tg/detail/-/0874477182/learninfreed]The Official SAT Study Guide: For the New SAT[/url] by the College Board. It is very easy to waste a lot of time on test prep until test prep becomes your only EC--which makes your college application look kind of lame. Just use the REAL materials from the College Board for test prep, and use the rest of your time for worthwhile activities like reading books you enjoy on subjects you are interested in, participating in your favorite extracurricular activities, keeping up your school grades, and sports. It is DEFINITELY possible to study too much for the SAT. \r\n\r\nGood luck on your tests, and on your college applications.",
"Solution_7": "Yeah, after a certain amount of work, practicing for the (P)SAT becomes a waste."
}
{
"Tag": [
"inequalities",
"geometry",
"circumcircle",
"geometry proposed"
],
"Problem": "Given are triangle $ABC$ and $I$ inside it. Called $R_{a},R_{b},R_{c}$ be circumradius of triangles $BIC,CIA,AIB$ and $R$ is circumradius of $ABC$ prove that\r\n$R\\ge \\min\\{R_{a},R_{b},R_{c}\\}$ :)",
"Solution_1": "Let the points $O_{A},O_{B},O_{C}$ and $O$ be the circumcentres of the triangles $BIC,AIC,AIB$ and $ABC$.Then what means $R\\angle{BO_{A}C}$!\r\n=>$2\\angle{A}>360^{0}-2\\angle{BIC}$\r\nSo,if we would have that $R3*180^{0}-(\\angle{BIC}+\\angle{AIC}+\\angle{AIB})=180^{0}$ :P :lol: \r\n\r\nIn the solution above I supossed that all the angles $AIC,BIC,AIB$ are $>90^{0}$.I think the other case is easier.",
"Solution_2": "Ok that's right, thank Tiks for good solution! :blush:"
}
{
"Tag": [
"calculus",
"integration",
"function",
"limit"
],
"Problem": "I am not sure whether I should post this in the Calculus forum or not but I feel that maybe the solution won't require calculus at all.\r\n\r\nI proved the following results.\r\n\r\n1. $\\int^{ 1}_{0}x^{\\alpha}e^{x}. dx < \\frac{3}{\\alpha+1}$ for $\\alpha > 0$\r\n\r\n2. There exist integers $a_{n}$, $b_{n}$ such that $\\int^{1}_{0}x^{n}e^{x}. dx = a_{n}+b_{n}e$ where $n$ is a non-negative integer.\r\n\r\n3. If $r$ is a positive rational number so that $r= \\frac{p}{q}$, $(p,q)=1$ where $p,q$ are positive integers then for all integers $a,b$:\r\n\r\n$|a+br| = 0$ or $|a+br| \\geq \\frac{1}{q}$.\r\n\r\nHence, prove $e$ is irrational. \r\n\r\nI have a feeling that I am missing something completely trivial because it is so late.",
"Solution_1": "If anyone wants to solve this who doesn't know calculus, here are the relevant facts:\r\n\r\nThere is a function $f$ with the following properties:\r\n\r\n1) $f(\\alpha) < \\frac{3}{\\alpha+1}$ for $\\alpha > 0$\r\n2) For $n \\in \\bf N$, $0 < f(n) = a_{n}+b_{n}e$ for some integers $a_{n}, b_{n}$ dependent on $n$. (The $0 < f(n)$ was missing in the original question, but it's important. Maybe this is where the trouble came from. We also could have put it with given-the-first.)\r\n3) For any rational number $r = \\frac{p}{q}$ and any integers $a, b$, either $|a+br| = 0$ or $|a+br| \\geq \\frac{1}{q}$.\r\n\r\nGiven this knowledge, prove that $e$ is irrational.",
"Solution_2": "See http://en.wikipedia.org/wiki/Euler's_continued_fraction_formula#The_exponential_function for another interesting approach.",
"Solution_3": "[hide=\"hint\"]\nUse the well known series for $e$:\n\\[S_{n}=\\sum_{k=0}^{n}\\frac{1}{k!}\\Rightarrow e=\\lim_{n\\rightarrow \\infty}S_{n}\\]\nNow come up with bounds to show $n!(e-S_{n})$ is never an integer for each positive integer $n$, and deduce that $n!\\cdot e$ is never an integer.\n[/hide]"
}
{
"Tag": [
"combinatorics unsolved",
"combinatorics"
],
"Problem": "Let $ n$, $ k$, $ p$ be positive integers with $ 2 \\le k \\le \\frac {n}{p \\plus{} 1}$. Let $ n$ distinct points on a circle be given. These points are colored blue and red so that exactly $ k$ points are blue and, on each arc determined by two consecutive blue points in clockwise direction, there are at least $ p$ red points. How many such colorings are there?",
"Solution_1": "This is rather easy, no? :maybe: \r\n\r\nWell, if not then my solution is probably wrong.\r\n\r\n[hide]Place, the blue points anywhere. Now, place in between every $ k$ consecutive pairs of blue points the neccessary $ p$ red points. Now, we have $ n\\minus{}k\\minus{}kp$ points left, all of which must be colored red. The distinctness of the coloring will depend on how many red points are between each pair of consecutive blue points. So, label the blue points around the circle $ b_1,...,b_k$. Then, let $ p_i$ be the number of red points between $ b_i$ and $ b_{i\\plus{}1}$ excluding the neccessary $ p$. Thus, we have \n\n$ \\sum_{i\\equal{}1}^k p_i\\equal{}n\\minus{}k\\minus{}kp$\n\nThe number of nonnegative integer solutions for $ \\{p_i\\}$ is $ \\binom{n\\minus{}k\\minus{}kp\\plus{}k\\minus{}1}{k\\minus{}1}\\equal{}\\binom{n\\minus{}kp\\minus{}1}{k\\minus{}1}$. So, now since we overcounted because of the symmetries of the circle, we retract that by dividing by $ k$. Thus, there are a total of\n\n$ \\frac{1}{k}\\binom{n\\minus{}kp\\minus{}1}{k\\minus{}1}$\n\ncolorings.[/hide]",
"Solution_2": "[quote=\"cosinator\"]\n\n[hide]Now, we have $ n \\minus{} k \\minus{} kp$ points left[/hide][/quote]\r\n\r\nisn't it $ n \\minus{} k \\minus{} (k \\minus{} 1)p$ points left?",
"Solution_3": "[quote=\"cosinator\"]This is rather easy, no? :maybe: \n[/quote]\nI don't think so,though it's not hard.But I still haven't found a proof :blush: .\n[quote=\"cosinator\"]\nWell, if not then my solution is probably wrong.\n[/quote]\r\nIt's definitely incorrect,even the answer is incorrect.Just check some particular cases. :wink:",
"Solution_4": "woops :blush:",
"Solution_5": "Bump ......"
}
{
"Tag": [],
"Problem": "Supposed A, B, and C are three numbers for which 1001C-2002A=4004 and 1001B+3003A=5005. The average of the numbers A, B, and C is\r\n(A) 1\r\n(B) 3\r\n(C) 6\r\n(D) 9\r\n(E) not uniquely determined",
"Solution_1": "[quote=\"Totally Zealous\"]Supposed A, B, and C are three numbers for which 1001C-2002A=4004 and 1001B+3003A=5005. The average of the numbers A, B, and C is\n(A) 1\n(B) 3\n(C) 6\n(D) 9\n(E) not uniquely determined[/quote]\r\n\r\n[hide]Add the two equations and divide by 3003 to get B[/hide]",
"Solution_2": "Add the two equations to get:\r\n1001A + 1001B + 1001C = 9009\r\nDivide by 3003:\r\n(A+B+C)/3=3\r\nTHE ANSWER IS B.",
"Solution_3": "[quote=\"vishalarul\"]Add the two equations to get:\n1001A + 1001B + 1001C = 9009\nDivide by 3003:\n(A+B+C)/3=3\nTHE ANSWER IS B.[/quote]\r\n\r\nCant you just divide by 1001 in the first place.",
"Solution_4": "Yeah, I guess you are right. :|",
"Solution_5": "Supposed A, B, and C are three numbers for which 1001C-2002A=4004 and 1001B+3003A=5005. The average of the numbers A, B, and C is \r\n(A) 1 \r\n(B) 3 \r\n(C) 6 \r\n(D) 9 \r\n(E) not uniquely determined\r\n\r\n1001C-2002A=4004\r\n= 1001C-4004= 2002A\r\n\r\nAnd \r\n\r\n1001B+3003A-2002A= 5005- (1001C-4004)\r\n\r\n1001B+1001A+1001C= 5005 + 4004\r\n\r\n1001(B+A+C) = 9009\r\nB+A+C=9\r\n9/3= 3\r\nThe answer is B",
"Solution_6": "[hide]Adding the two equations, dividing by 3003 to get 3.\n\n$3$[/hide]"
}
{
"Tag": [
"calculus",
"integration",
"limit",
"calculus computations"
],
"Problem": "Find\r\n$ \\lim_{a \\to 0} \\int_{ \\minus{} a}^{a} \\frac {1}{\\sqrt {a^{2} \\plus{} 2ba^{n} \\minus{} x^{2} \\minus{} 2bx^{n}}}dx$,$ n$ is an integer",
"Solution_1": "case 1: n=1\r\n$ I_a\\equal{}\\int_{ \\minus{} a}^{a} \\frac {1}{\\sqrt {(a\\plus{}b)^2\\minus{}(x\\plus{}b)^2}}dx$\r\n\r\n$ \\equal{}\\frac{1}{a\\plus{}b}\\int_{ \\minus{}a}^{a} \\frac {1}{\\sqrt {1\\minus{}\\left(\\frac{x\\plus{}b}{a\\plus{}b}\\right)^2}}dx$\r\n\r\n$ \\equal{}\\int_{\\frac{b\\minus{} a}{b\\plus{}a}}^{1} \\frac {1}{\\sqrt {1\\minus{}u^2}}dx\\equal{}\\frac{\\pi}{2}\\minus{}\\arcsin \\left( \\frac{b\\minus{} a}{b\\plus{}a}\\right)$\r\n\r\nso we get \r\n\r\n$ \\lim_{a\\to 0} I_a \\equal{}\\frac{\\pi}{2}\\minus{}\\arcsin (1) \\equal{}0$\r\n\r\ncase 2: $ n \\ge 2$\r\n\r\n$ I_a\\equal{}\\int_{ \\minus{} a}^{a} \\frac {1}{\\sqrt {(a^2\\minus{}x^2) \\plus{} 2b(a^n \\minus{} x^n)}}dx$\r\n\r\n$ \\equal{}\\int_{ \\minus{} 1}^{1} \\frac {1}{\\sqrt {(1\\minus{}u^2) \\plus{} 2ba^{n\\minus{}2}(1 \\minus{} u^n)}}du$\r\n\r\nso i would GUESS\r\n\r\n$ \\lim_{a\\to 0} I_a \\equal{} \\int_{ \\minus{} 1}^{1} \\frac {1}{\\sqrt {1\\minus{}u^2}}du \\equal{} \\pi$"
}
{
"Tag": [
"LaTeX",
"geometry proposed",
"geometry"
],
"Problem": "All these posts have fancy formal notation. My question is two-fold, where is this formal documentation written up? In other words, where can I learn it other than deciphering it from the posts and 2), most importantly, how do you type 0 and a+b+c=3\r\nprove that:\r\n (a+b^2)(b+c^2)(c+a^2) =<13\r\nhelp me!",
"Solution_1": "I can prove this if $ a^2\\plus{}b^2\\plus{}c^2 \\le 4$, can anyone prove it given $ a^2\\plus{}b^2\\plus{}c^2 \\ge 4$ ?",
"Solution_2": "The critical points lie within $ a^2 \\plus{} b^2 \\plus{} c^2 > 4$, so that should prove to be the more difficult part. One proof strategy might be this: Prove first that the maximum is achieved when one of the values is $ 0$, then the problem reduces to proving\r\n\\[ a(3\\minus{}a)^2(3\\minus{}a\\plus{}a^2)\\leq 13\\]\r\nHow you would go about accomplishing either of these without calculus, however, is beyond me. Using calculus, you can find the inequality is a bit tighter actually:\r\n\\[ (a \\plus{} b^2)(b \\plus{} c^2)(c \\plus{} a^2)\\leq 12 \\plus{} \\frac {9308 \\plus{} 8309\\cdot 2^{\\frac {2}{3}} 7^{\\frac {1}{3}} \\minus{} 5936\\cdot 2^{\\frac {1}{3}}7^{\\frac {2}{3}}}{9375}\\approx 12.76496\r\n\\]\r\nthe inequality being strict equality for exactly 3 points.",
"Solution_3": "Using calculus, how do you prove the max is when one of them is 0?",
"Solution_4": "yes,why one of them is 0?",
"Solution_5": "See also here:\r\nhttp://www.artofproblemsolving.com/Forum/viewtopic.php?t=58379",
"Solution_6": "Well if we've given up on a non-calculus solution and taken the gloves off, then the proof sketch is as follows. Let $ f(a,b) \\equal{} (a \\plus{} b^2)(b \\plus{} (3 \\minus{} a \\minus{} b)^2)(3 \\minus{} a \\minus{} b \\plus{} a^2)$. $ f$ is of fifth-degree in both $ a$ and $ b$, so $ \\frac {\\partial f}{\\partial a}$ and $ \\frac {\\partial f}{\\partial b}$ are both quartic. Thus, we can find using any algorithm for quartics exact algebraic expressions for the solutions of\r\n\\[ \\frac {\\partial f}{\\partial a} \\equal{} \\frac {\\partial f}{\\partial b} \\equal{} 0\r\n\\]\r\nAfter some ridiculously tedious algebra, you find the critical points, evaluate $ f$ at those points and find that $ f(a^*,b^*) < 9$ for each of them. Comparing this to, say, $ f(1,0) \\equal{} 12$, you see that the maximum cannot be at a critical point, and therefore the maximum must be on the boundary of the region $ 0\\leq a \\plus{} b \\leq 3$. This boundary is composed of three line segments, one of which is $ \\lbrace (a,b): b \\equal{} 0,0\\leq a \\leq 3 \\rbrace$. You then argue by symmetry that $ f$ takes on the same range of values on each line segment (i.e. interchanging any of a,b,c doesn't change the problem), so the maximum must lie on the line $ \\lbrace (a,b): b \\equal{} 0,0\\leq a \\leq 3 \\rbrace$. This is simply the one dimensional problem of finding the maximum of\r\n\\[ f(a,0) \\equal{} a(3 \\minus{} a)^2(3 \\minus{} a \\plus{} a^2)\r\n\\]\r\non $ 0\\leq a \\leq 3$. Again, this is a fifth degree polynomial, so its derivative with respect to $ a$ is quartic, and we can obtain exact solutions. It has two real solutions, one is $ a \\equal{} 3$, which corresponds to a minimum, the other is\r\n\\[ a \\equal{} \\frac {1}{15}\\left(13 \\plus{} 4\\cdot 2^{\\frac {2}{3}} 7^{\\frac {1}{3}} \\minus{} 2^{\\frac {1}{3}} 7^{\\frac {2}{3}}\\right)\r\n\\]\r\nwhich is where $ f$ obtains its maximum. Substituting this back into $ f$ yields the upper bound I stated in a previous post.",
"Solution_7": "[quote=\"Math pro\"]Let $ x,y,z$ are nonnegative real numbers such that $ x \\plus{} y \\plus{} z \\equal{} 3,$ prove that\n\n$ \\left(x \\plus{} y^2\\right)\\left(y \\plus{} z^2\\right)\\left(z \\plus{} x^2\\right) < 13.$[/quote]Using the arithmetic-mean \u2014 geometric-mean inequality,\r\n\r\n$ \\frac {1}{3}\\sum_{cyc}{(3x \\plus{} 2y \\plus{} 4z)^3\\left(x \\plus{} y^2\\right)}\\geq\\prod_{cyc}{(3x \\plus{} 2y \\plus{} 4z)}\\sqrt [3]{\\prod_{cyc}{\\left(x \\plus{} y^2\\right)}}.$\r\n\r\nThis comes down to prove that \r\n\r\n$ 3\\sqrt [3]{13}\\prod_{cyc}{(3x \\plus{} 2y \\plus{} 4z)}>\\sum_{cyc}{(3x \\plus{} 2y \\plus{} 4z)^3\\left(x \\plus{} y^2\\right)}.$\r\n\r\nId est, it remains to the following [color=darkred]quintic[/color] homogeneous inequality:\r\n\r\n$ 47(x \\plus{} y \\plus{} z)^2\\prod_{cyc}{(3x \\plus{} 2y \\plus{} 4z)}>20\\sum_{cyc}{(3x \\plus{} 2y \\plus{} 4z)^3\\left[x(x \\plus{} y \\plus{} z) \\plus{} 3y^2\\right]},$\r\n\r\nsince $ x \\plus{} y \\plus{} z \\equal{} 3$ and $ \\sqrt [3]{13} \\equal{} 2.3513\\cdots > 2.35 \\equal{} \\frac {47}{20}.$ But\r\n\r\n$ 47(x \\plus{} y \\plus{} z)^2\\prod_{cyc}{(3x \\plus{} 2y \\plus{} 4z)} \\minus{} 20\\sum_{cyc}{(3x \\plus{} 2y \\plus{} 4z)^3\\left[x(x \\plus{} y \\plus{} z) \\plus{} 3y^2\\right]}$\r\n\r\n$ \\equal{} \\frac {83}{8}\\sum_{cyc}{x(3x \\minus{} 4y)^2\\left(x^2 \\plus{} 6xy \\plus{} 6y^2\\right)} \\plus{} \\frac {117}{8}\\sum_{sym}{x^5} \\plus{} \\frac {75}{4}\\sum_{cyc}{x^4y} \\plus{} \\frac {95}{4}\\sum_{cyc}{x^3y^2}$\r\n\r\n$ \\plus{} 6441\\sum_{sym}{x^3yz} \\plus{} 8676\\sum_{sum}{xy^2z^2} > 0.$\r\n\r\nsee also here : http://www.mathlinks.ro/viewtopic.php?t=146734"
}
{
"Tag": [
"function"
],
"Problem": "Prove that $ \\pi$ is irrational!",
"Solution_1": "http://en.wikipedia.org/wiki/Proof_that_%CF%80_is_irrational\r\n\r\nThere are more proofs out there, and few can be found searching here on ML (and please try searching before you post these quite well known questions - this one, and the one with Zeta function in College Playground).\r\n\r\nhttp://www.mathlinks.ro/viewtopic.php?t=162010\r\nhttp://www.mathlinks.ro/portal.php?t=162051\r\n\r\n\r\n[hide=\"Something to laugh about\"] \n\nhttp://in.answers.yahoo.com/question/index?qid=20090430032537AA7F1RP\n\n :rotfl: [/hide]"
}
{
"Tag": [
"limit",
"trigonometry",
"function",
"calculus",
"calculus computations"
],
"Problem": "Compute the ff. limits:\r\n\r\n$ 1.) \\lim_{x\\rightarrow\\infty} \\cos (\\pi \\sqrt{4x^4\\plus{}3x^2\\plus{}1}\\minus{}2\\pi x^2)$\r\n\r\n$ 2.) \\lim_{x\\rightarrow 0} \\frac{| {\\cos(\\sin(3x)) } | \\minus{}1}{x^2}$",
"Solution_1": "1. $ L \\equal{} \\lim_{x \\to \\plus{} \\infty} \\cos [\\pi (\\sqrt {4x^{4} \\plus{} 3x^{2} \\plus{} 1} \\minus{} 2 x^{2})]$\r\n\r\n$ \\equal{} \\lim_{x \\to \\plus{} \\infty} \\cos \\left(\\pi \\frac {4x^4 \\plus{} 3x^2 \\plus{} 1 \\minus{} 4x^4}{\\sqrt {4x^{4} \\plus{} 3x^{2} \\plus{} 1} \\plus{} 2 x^{2}}\\right)$\r\n\r\n$ \\equal{} \\lim_{x \\to \\plus{} \\infty} \\cos \\left(\\pi \\frac {3 \\plus{} \\frac {1}{x^2}}{\\sqrt {4 \\plus{} \\frac {3}{x^2} \\plus{} \\frac {1}{x^4}} \\plus{} 2}\\right)$\r\n\r\n$ \\equal{} \\cos \\left(\\frac {3 \\pi}{4}\\right)$\r\n\r\n$ \\equal{} \\minus{} \\frac {1}{\\sqrt {2}}$.\r\n\r\n\r\n2. For x close to zero, the function cos is positive, and the function $ \\frac{\\cos(\\sin(3x)) \\minus{} 1}{x^2}$ is even. Hence,\r\n\r\n$ L \\equal{} \\lim_{x \\to 0} \\frac{\\cos(\\sin(3x)) \\minus{} 1}{x^2} \\equal{} \\minus{} \\lim_{x \\to 0} \\frac{1 \\minus{} \\cos^2(\\sin(3x))}{x^2(1 \\plus{} \\cos(\\sin(3x)))}$\r\n\r\n$ \\equal{} \\minus{} \\frac12 \\lim_{x \\to 0} \\frac{\\sin^2(\\sin(3x))}{x^2} \\equal{} \\minus{} \\frac92 \\lim_{x \\to 0} \\left[\\frac{\\sin(\\sin(3x))}{\\sin(3x)}\\right]^2 \\cdot \\left[\\frac{\\sin(3x)}{3x}\\right]^2 \\equal{} \\minus{} \\frac92$",
"Solution_2": "yes, that was easy :)"
}
{
"Tag": [
"limit",
"number theory",
"prime factorization",
"number theory unsolved"
],
"Problem": "[i]Is there any elementary proofs to prove that : $ \\lim_{ n \\to \\infty } \\frac{ \\pi(n)}{n} \\equal{} 0$ :maybe: [/i]",
"Solution_1": "$ \\displaystyle S: =\\lim_{n \\to +\\infty}{\\frac{\\pi(n)}{n}= \\lim_{k \\to +\\infty}{\\prod_{i=1}^k{\\left(1-\\frac{1}{p_i}\\right)} \\ge 0}}$ where $ p_i$ is the $ i$-th prime (it is true by inclusion-esclusion principle)\r\nNow if $ y \\in \\mathbb{R}^+$ then $ 1-y \\le e^{-y}$, with equality iff y=0. It follows that $ S \\le e^{\\displaystyle -\\lim_{k \\to +\\infty}{\\sum_{i=1}^k{\\left(\\frac{1}{p_i}\\right)}}}$.\r\nNow assume that $ \\sum_{i=1}^{\\infty}{\\left(\\frac{1}{p_i}\\right)}$ converges, then there exists $ v \\in \\mathbb{N}$ such that $ \\sum_{i=v+1}^{+\\infty}{\\left(\\frac{1}{p_i}\\right)}<\\frac{1}{2}$. Define $ t: =\\prod_{i=1}^v{p_i}$; because of $ p_j \\nmid nt+1$ forall $ 1 \\le j \\le v$ we must have $ \\displaystyle \\sum_{n=1}^{+\\infty}{\\frac{1}{nt+1}} \\le \\prod_{i=v+1}^{\\infty}{ \\sum_{k=0}^{\\infty}{\\frac{1}{p_i^k}}}$(*), but the last rhs is $ \\le \\prod_{i=v+1}^{\\infty}{\\left(1+\\frac{2}{p_i}\\right)}$ that converges for assumption, contracting that lhs of (*) diverges. So such $ v \\in \\mathbb{N}$ does not exist. It implies that $ \\sum_{i=1}^{\\infty}{\\left(\\frac{1}{p_i}\\right)}$ diverges, and so $ \\displaystyle S: =\\lim_{n \\to +\\infty}{\\frac{\\pi(n)}{n}}$ is exactly 0.",
"Solution_2": "Your first statement isn't quite rigorous since we don't know that the limit exists; what you're computing is an upper bound on the $ \\limsup$. \r\n\r\nThe second half of the argument can be made more elementary. Equivalently, it suffices to show that\r\n\r\n$ \\lim_{k \\to \\infty} \\prod_{i \\equal{} 1}^{k} \\left( \\frac {1}{1 \\minus{} \\frac {1}{p_i} } \\right) \\equal{} \\infty$.\r\n\r\nBut this is classic: by unique prime factorization we have the bound\r\n\r\n$ \\prod_{i \\equal{} 1}^{k} \\left( \\frac {1}{1 \\minus{} \\frac {1}{p_i} } \\right) \\ge \\sum_{i \\equal{} 1}^{p_k} \\frac {1}{i}$\r\n\r\nand it is elementary to show that the harmonic series diverges, i.e. by Cauchy condensation. The conclusion follows."
}
{
"Tag": [
"function",
"limit",
"induction",
"real analysis",
"real analysis solved"
],
"Problem": "Let the functions f_n:[e,oo)-->R, defined recurently like so f_0(x)=ln(x),\r\nf_(n+1) (x)=ln(x*f_n(x)), n is natural. Then we have:\r\n\r\n \\lim (x-->e) (f_n(x))^(1/(x-e)=e^(n+1)/e, n is natural.\r\n\r\ncheers!!",
"Solution_1": "I used L'Hospital for this. We are trying to prove that ln(f_(n+1)(x))/x-e tends to (n+2)/e as x->e. It's obvious by induction that f_n(x)->1 as x->e so we have an indetermination so we can apply L'Hospital. \r\nLet's assume that we have proved P_n which states that (g_n)'(x)->(n+1)/e as x->e, where g_n(x)= ln(f_n(x)). Then, by L'Hospital we also have g_n(x)/x-e -> (n+1)/e as x->e. Now let's prove that (g_(n+1))'(x)->(n+2)/e. (g_(n+1))'(x)=(1/ f_(n+1)(x))* f_(n+1)'(x)= (1/ f_(n+1)(x))* (1/x+f_n'(x)/f_n(x)). But we can observe that f_n'(x)/f_n(x) = (g_n)'(x)->(n+1)/e as x->e, and 1/x->1/e as x->e and 1/ f_(n+1)(x)->1 as x->e, so by combining these we get that (g_(n+1))'(x) -> (n+2)/e. So by induction (g_n)'(x)->(n+1)/e as x->e for any n. Now by applying L'Hospital to these we get g_n(x)/(x-e) -> (n+1)/e as x->e. If we put e^(this limit we just got) we obtain the limit we wanted.\r\n\r\nSee ya!\r\n\r\n :D"
}
{
"Tag": [
"geometry",
"power of a point",
"radical axis",
"geometry proposed"
],
"Problem": "Let $w=C(O,r)$ be a fixed circle and let $A$ be a fixed point such that $OA>r$.\r\n\r\nFor a variable circle $x,\\ A\\in x,\\ x\\cap w\\ne \\emptyset$, I note the tangent $t$ in the point $A$ to the circle $x$\r\n\r\nand the common chord $c$ of the circles $w,x$. Ascertain the geometrical locus of the point $L\\in t\\cap c$.\r\n\r\nSee http://www.mathlinks.ro/Forum/viewtopic.php?t=54479",
"Solution_1": "$A$ is a circle of zero radius. The tangent $t$ is the radical axis of the circles $(A, x)$, the chord $c$ is the radical axis of the circles $(w, x)$. $L \\equiv t \\cap c$ is the radical center of the circles $(A, w, x)$ which lies on the radical axis of the circles $(A, w)$ perpendicular to their center line AO. Let the last radical axis intersect the center line AO at a point R.\r\n \r\n$AR^2 = (RO + r)(RO - r) = RO^2 - r^2 = (AO - AR)^2 - r^2$\r\n\r\n$0 = AO^2 - 2AO \\cdot AR - r^2,\\ \\ \\ AR = \\frac{AO^2 - r^2}{2 AO}$"
}
{
"Tag": [
"MATHCOUNTS",
"geometry",
"3D geometry",
"prism"
],
"Problem": "Hey who do you think is going to pwn this year?\r\nI practiced and did alot but I think Mark Sellke, Yushi and David are going to pwn. I don't think Vivian Gu does MC this year.\r\n\r\n\r\nBlah got 35th-60th last year, lots of people with 25-28",
"Solution_1": "duh they're gonna pwn. especially selke, as he should make masters round in '10. sorry for the slight invasion of your forum, guys.",
"Solution_2": "That's fine as long as we're not not talking about say pythag.\r\n\r\nBy the way is masters top 4 cd or written?",
"Solution_3": "[quote=\"AwesomeToad\"]That's fine as long as we're not not talking about say pythag.\n\nBy the way is masters top 4 cd or written?[/quote]\r\n\r\nwritten\r\n\r\ncuz I made it...",
"Solution_4": "dnkwin did u really make the nats?\r\nwhat is ur real name?",
"Solution_5": "yes he did and he did very well\r\njust pm him instead of posting here\r\nno names to usernames mentioned on the forum\r\n\r\n=)",
"Solution_6": "bleh. In 6th grade I got 63rd(23/46. Blah. Careless mistakes. :( )\r\nNow I'm in 7th grade, trying again......I'm probably gonna get like 25th this time at best.\r\n\r\nI would really love to make nats and that would be so awesome, but I live in Indiana. CURSE THAT! IF only I lived in say Oklahoma or North Dakota.......then I'd actually have a chance. bleh. :(\r\n\r\nAlso, if any of you see Doom Monkey, he's in my school.\r\n\r\nWhat do you think the difficulty of the test will be? Easyish or hardish?",
"Solution_7": "[quote=\"AwesomeToad\"]Hey who do you think is going to pwn this year?\nI practiced and did alot but I think Mark Sellke, Yushi and David are going to pwn. I don't think Vivian Gu does MC this year.\n\n\nBlah got 35th-60th last year, lots of people with 25-28[/quote]\\\r\n\r\nI think Vivian Gu is still doing Mathcounts this year. In 2009, she was a 7th grader. :wink:",
"Solution_8": "[quote=\"maybach\"]Easyish or hardish?[/quote]\r\n\r\nI think it's gonna be middle-ish.",
"Solution_9": "I think I'm going to win",
"Solution_10": "[quote=\"ernie\"][quote=\"maybach\"]Easyish or hardish?[/quote]\n\nI think it's gonna be middle-ish.[/quote]\r\n\r\nUm.........similar to, harder than, or easier than the 2008 State?(I hope the latter)",
"Solution_11": "[quote=\"gauss1181\"][quote=\"AwesomeToad\"]Hey who do you think is going to pwn this year?\nI practiced and did alot but I think Mark Sellke, Yushi and David are going to pwn. I don't think Vivian Gu does MC this year.\n\n\nBlah got 35th-60th last year, lots of people with 25-28[/quote]\\\n\nI think Vivian Gu is still doing Mathcounts this year. In 2009, she was a 7th grader. :wink:[/quote]\r\n\r\nVivian is NOT doing MC this year. I go to her school and she told me. She said something about GMO.",
"Solution_12": "[quote=\"maybach\"][quote=\"ernie\"][quote=\"maybach\"]Easyish or hardish?[/quote]\n\nI think it's gonna be middle-ish.[/quote]\n\nUm.........similar to, harder than, or easier than the 2008 State?(I hope the latter)[/quote]\r\n\r\nMaybe a bit harder?",
"Solution_13": "[quote=\"ernie\"][/quote][quote=\"maybach\"][quote=\"ernie\"][quote=\"maybach\"]Easyish or hardish?[/quote]\n\nI think it's gonna be middle-ish.[/quote]\n\nUm.........similar to, harder than, or easier than the 2008 State?(I hope the latter)[/quote][quote=\"ernie\"]\n\nMaybe a bit harder?[/quote]\r\n2008 State was kinda hard... I only got a 35 on it (compared to 46 this year)... so at the rate mathcounts problems are going I would say a LOT easier...",
"Solution_14": "It's an unwritten rule that they can only repeat every problem five times so they're running out of ideas.\r\n\r\n[quote=\"AwesomeToad\"]Vivian is NOT doing MC this year. I go to her school and she told me. She said something about GMO.[/quote]\r\n\r\n...Do you mean CGMO? I can't think of anything else...\r\n\r\nAlso, that would be a pretty silly reason (bleh maybe not) not to do MC?",
"Solution_15": "[quote=\"math154\"]It's an unwritten rule that they can only repeat every problem five times so they're running out of ideas.\n\n[quote=\"AwesomeToad\"]Vivian is NOT doing MC this year. I go to her school and she told me. She said something about GMO.[/quote]\n\n...Do you mean CGMO? I can't think of anything else...\n\nAlso, that would be a pretty silly reason (bleh maybe not) not to do MC?[/quote]\r\n\r\nIt is pretty silly... Also wait what unwritten rule? I thought it was two, not five? (exact wording, of course.)",
"Solution_16": "[quote=\"AwesomeToad\"]\n\n\nBlah got 35th-60th last year, lots of people with 25-28[/quote]\r\n\r\nArgh. I got 23/46 in state. But I took 2009 again and got 28/46. Still total fail though.............",
"Solution_17": "I think Vivian meant CGMO. There was something else.\r\n\r\nSomeone at my school said the stupidest thing ever:\r\n[quote]Whats the point of doing MAthcounts again if you already got a trip to Orlando?[/quote]",
"Solution_18": "[quote=\"AwesomeToad\"]I think Vivian meant CGMO. There was something else.\n\nSomeone at my school said the stupidest thing ever:\n[quote]Whats the point of doing MAthcounts again if you already got a trip to Orlando?[/quote][/quote]\r\n\r\ngetting another trip to nationals..............",
"Solution_19": "Besides, most people at my school don't do math at all (mostly jocks)\r\nI'd like to raise my national ranking if I got another trip",
"Solution_20": "heh yeah. we all experience these ppl on a daily basis. \r\n\r\nand, other than the whole nats thing, mc is fun! yay",
"Solution_21": "[quote=\"pinkmuskrat\"]heh yeah. we all experience these ppl on a daily basis. \n\nand, other than the whole nats thing, mc is fun! yay[/quote]\r\n\r\nWait so the nationals is at Disney again this year?\r\n\r\nThe guy who came 3rd at AAPotter (Purdue) chapter goes to my school, came 7th state\r\n\r\nDoes some really weird stuff at my school.\r\n\r\nNot want to mention his name though...",
"Solution_22": "AwesomeToad, is that the kid who won state countdown?\r\n\r\nI was there.",
"Solution_23": "[quote=\"maybach\"]AwesomeToad, is that the kid who won state countdown?\n\nI was there.[/quote]\n\nYes.\nExamples: [quote]Oh I hit you. I get 5 points[/quote]\n???\nor\n[quote]I was up all night playing starcraft blah pwns a korean oh i'm braindead go to bed at 5: 60[/quote]",
"Solution_24": "Yeah. I was there feeling really sad watching cause' I really wanted to be up there and I actually thought I did a good job(Man I need to practice on sprint, but I've imrpoved my average score 14 points since). I saw that dude. He looked like he was praying and he looked really sad when they read \"7th place\".....feel really sorry for him......poor guy :(\r\n\r\nAlso, could I make cd(not nats) for states with 35/46? Because that's about what I can get on a good day assuming the test was made recently(2006-).",
"Solution_25": "[quote=\"AwesomeToad\"]Besides, most people at my school don't do math at all (mostly jocks)\nI'd like to raise my national ranking if I got another trip[/quote]\r\n\r\nif you are supposedly in vivian's school, then you are in my school too. i have no clue who you are though.",
"Solution_26": "People in 8th grade now.\r\nBut the guy who came 7th was sure he got at least a 27 on sprint and perfect target.\r\n\r\nBut he got 24/14 so yeah",
"Solution_27": "I totally agree. But I'll win.",
"Solution_28": "[quote=\"maybach\"]Yeah. I was there feeling really sad watching cause' I really wanted to be up there and I actually thought I did a good job(Man I need to practice on sprint, but I've imrpoved my average score 14 points since). I saw that dude. He looked like he was praying and he looked really sad when they read \"7th place\".....feel really sorry for him......poor guy :(\n\nAlso, could I make cd(not nats) for states with 35/46? Because that's about what I can get on a good day assuming the test was made recently(2006-).[/quote]\r\n\r\nI hate to say this but don't feel to sorry for him.\r\n\r\nAlso the 35/46 would probably just barely make state cd better not count on it.\r\nOf course, the level of the competition and test also matter",
"Solution_29": "[quote=\"AwesomeToad\"][quote=\"maybach\"]Yeah. I was there feeling really sad watching cause' I really wanted to be up there and I actually thought I did a good job(Man I need to practice on sprint, but I've imrpoved my average score 14 points since). I saw that dude. He looked like he was praying and he looked really sad when they read \"7th place\".....feel really sorry for him......poor guy :(\n\nAlso, could I make cd(not nats) for states with 35/46? Because that's about what I can get on a good day assuming the test was made recently(2006-).[/quote]\n\nI hate to say this but don't feel to sorry for him.\n\nAlso the 35/46 would probably just barely make state cd better not count on it.\nOf course, the level of the competition and test also matter[/quote]\r\n\r\nI agree. Only something with a 50/55 would be enough to win state mc.",
"Solution_30": "Generally on an average test you would need 43/46\r\n\r\nBut this year Sellke got 45 and year before best was 41.\r\nAlso, what do you mean by 50/55?",
"Solution_31": "Maybe he was referring to the team score? But team round doesn't matter in state. Also, it's too bad they cut the scholarship money. I think with some practice I could get 20th place, but that wouldn't be good enough for a scholarship anymore :(\r\n\r\nAlso, my sister went to MC State in 8th grade at Rose Hulman and got 163rd place(I got 63rd :rofl:). I thought the Rose Hulman auditorium seemed nicer. But only 2 kids get scholarships though..........",
"Solution_32": "Team Round determines the state coach wow my school did better than last year.\r\n\r\n2 people in CD!!/Top 10",
"Solution_33": "[quote=\"AwesomeToad\"]Team Round determines the state coach wow my school did better than last year.\n\n2 people in CD!!/Top 10[/quote]\r\n\r\nState Coach doesn't really matter. \r\n\r\nAwesome Toad, you're from West Laffayette, right?\r\n\r\nAnyways the state coach will always be Bob Fischer no matter what. \r\n\r\nAlso, I actually got a 4 on target, but I got a 5 because they misread my answer to the rectangular prism question. :)\r\n\r\nStill did horribly though.........",
"Solution_34": "Are all of you guys 8th graders and below?",
"Solution_35": "Yeah. I'm currently in 7th grade!(thank god, with selkie, I know I have know chance this year)",
"Solution_36": "Sellke, \r\nLiang, and homma are gonna pwn this year.\r\n\r\nVivians not doing mc, so everyone wants the 4th spot",
"Solution_37": "[quote=\"AwesomeToad\"][quote=\"maybach\"]AwesomeToad, is that the kid who won state countdown?\n\nI was there.[/quote]\n\nYes.\nExamples: [quote]Oh I hit you. I get 5 points[/quote]\n???\nor\n[quote]I was up all night playing starcraft blah pwns a korean oh i'm braindead go to bed at 5: 60[/quote][/quote]\n\nCorrections:\n\nOh I hit you. I get 2 points. (Currently Retired)\n\nI was up 1/3rd of the night playing starcraft, I pwn koreans, I'm braindead go to bed at 1:00. (Retired as of Friday)\n\nAnyways, meh I'm doing more programming than math right now, but I'll still try for 4th (although there are some really good 8th graders ready to pwn me) :P. Does anyone know what grade 5th/6th place are in?\n\n[quote=\"Maybach\"]Yeah. I was there feeling really sad watching cause' I really wanted to be up there and I actually thought I did a good job(Man I need to practice on sprint, but I've imrpoved my average score 14 points since). I saw that dude. He looked like he was praying and he looked really sad when they read \"7th place\".....feel really sorry for him......poor guy :(\n\nAlso, could I make cd(not nats) for states with 35/46? Because that's about what I can get on a good day assuming the test was made recently(2006-).[/quote]\r\n\r\nNah, I didn't really expect to make nats (although it would be awesome if it happened). My goal was to pwn people in CD, and not fail too bad in written.",
"Solution_38": "Alex Barksdale came in 5th if I remember right\r\nCorrect me if I'm wrong...",
"Solution_39": "Does anyone have a list of the top 16? 2009 State MC?"
}
{
"Tag": [
"probability",
"geometry"
],
"Problem": "A circle of radius $ r$ is concentric with and outside a regular hexagon of side length $ 2$. The probability that $ 3$ entire sides of the hexagon are visible from a random point on the cirlce is $ 1/2$. What is $ r$?",
"Solution_1": "just to be sure...\r\n\r\nThe area of visibiliy is enclosed by the tangent to the circle at that point and the closest part of the hexagon right??\r\n\r\n\r\n\r\n :)",
"Solution_2": "Yes thats correct."
}
{
"Tag": [
"trigonometry",
"algebra proposed",
"algebra"
],
"Problem": "Given $ (x_n)$ and $ (y_n)$ defined as\r\n\\[ x_1 \\equal{} y_1 \\equal{} \\sqrt {3}\\ ; \\ x_{n \\plus{} 1} \\equal{} x_n \\plus{} \\sqrt {1 \\plus{} \\left ({x}_n \\right)^2}\\ ; \\ y_{n \\plus{} 1} \\equal{} \\frac {y_n}{1 \\plus{} \\sqrt {1 \\plus{} \\left (y_n \\right)^2}} \\ \\forall n\\ge 2\\]\r\nProve that $ x_n.y_n$ $ \\in$ $ (2,3)$",
"Solution_1": "[quote=\"mathVNpro\"]Given $ (x_n)$ and $ (y_n)$ defined as\n\\[ x_1 \\equal{} y_1 \\equal{} \\sqrt {3}\\ ; \\ x_{n \\plus{} 1} \\equal{} x_n \\plus{} \\sqrt {1 \\plus{} \\left ({x}_n \\right)^2}\\ ; \\ y_{n \\plus{} 1} \\equal{} \\frac {y_n}{1 \\plus{} \\sqrt {1 \\plus{} \\left (y_n \\right)^2}} \\ \\forall n\\ge 2\\]\nProve that $ x_n.y_n$ $ \\in$ $ (2,3)$[/quote]\r\n\r\nhint :\r\n\r\nfor $ x_n$ : $ \\cot \\frac{x}{2}\\equal{}\\cot x\\plus{} \\sqrt{1\\plus{}\\cot^2 x}$\r\n\r\nfor $ y_n$ : $ \\tan \\frac{x}{2}\\equal{}\\frac{\\tan x}{1\\plus{}\\sqrt{1\\plus{}\\tan^2x}}$",
"Solution_2": "[quote=\"hophinhan\"][quote=\"mathVNpro\"]Given $ (x_n)$ and $ (y_n)$ defined as\n\\[ x_1 = y_1 = \\sqrt {3}\\ ; \\ x_{n + 1} = x_n + \\sqrt {1 + ({x}_n )^2}\\ ; \\ y_{n + 1} = \\frac {y_n}{1 + \\sqrt {1 + (y_n )^2}} \\ \\forall n\\ge 2\\]\nProve that $ x_n.y_n$ $ \\in$ $ (2,3)$[/quote]\n\nhint :\n\nfor $ x_n$ : $ \\cot \\frac {x}{2} = \\cot x + \\sqrt {1 + \\cot^2 x}$\n\nfor $ y_n$ : $ \\tan \\frac {x}{2} = \\frac {\\tan x}{1 + \\sqrt {1 + \\tan^2x}}$[/quote]\r\n\r\nWe have\r\n\r\n$ \\displaystyle 2 < x_{n}\\cdot y_{n} = {\\frac {2 \\cos^2 (\\frac{\\alpha}{2^n})}{2 \\cos^2 (\\frac{\\alpha}{2^n})-1}} < 3 \\qquad n\\ge2, \\alpha=\\frac{\\pi}3$"
}
{
"Tag": [
"induction",
"number theory unsolved",
"number theory"
],
"Problem": "prove that \r\n \r\n 0 < sum g(k)/k from 1 to n - 2n/3 < 2/3\r\n\r\n where g(k) is the greatest odd divisor of k",
"Solution_1": "[quote=\"marko avila\"]prove that \n \n 0 < sum g(k)/k from 1 to n - 2n/3 < 2/3\n\n where g(k) is the greatest odd divisor of k[/quote]\r\n\r\nUse induction on n.",
"Solution_2": "Maybe this is related:\r\nhttp://www.mathlinks.ro/Forum/viewtopic.php?t=73591"
}
{
"Tag": [],
"Problem": "Prove that if x is an integer, than x^2+6x+5/x+5 is also an integer.",
"Solution_1": "note that $\\frac{ x^2+6x+5}{x+5}=x+1 \\in Z$ if $x \\in Z$\r\n\r\nThat's it",
"Solution_2": "that's correct. nice job!",
"Solution_3": "what does $\\in Z$ mean?",
"Solution_4": "It means the number is rational :)\r\nN -> natural\r\nZ -> rational\r\nD -> decimal\r\nQ -> irrational\r\nR -> real\r\n(C -> complex)"
}
{
"Tag": [
"calculus",
"trigonometry",
"function",
"search"
],
"Problem": "Okay, so I'm thinking of doing a Family-Feud type of thing. Math related, of course. But I need to do some surveys.\r\n\r\n1. Name a word that you will often find in a problem that tells you that it will most likely require calculus to solve.\r\n\r\n2. Besides \u201cQ.E.D.\u201d, name a word or phrase that is often found in proofs.\r\n\r\n3. Name the branch of mathematics that\u2019s the hardest to understand.\r\n\r\n4. Name the most widely known theorem.\r\n\r\n5. Name something that you can do with most graphing calculators, but not with scientific calculators.\r\n\r\n6. Name a value for $x$ between 0 and 90 exclusive that you have $\\sin x^{\\circ}$ memorized.\r\n\r\n7. Name a function that, when graphed, could be identified right away.\r\n\r\n8. Name a mathematical feat that Chuck Norris is capable of.\r\n\r\n9. Name a widely known mathematics competition.\r\n\r\n10. Name a formula that any math student should have memorized.\r\n\r\n11. Name a branch of mathematics that many people hate.\r\n\r\n12. Name a famous mathematician.\r\n\r\n13. Rational or irrational, name one of the most important real numbers.\r\n\r\nPM me your answers, and once we get enough votes, we can play!\r\n\r\nYou may submit multiple answers.\r\nHowever, these answers will be weighted so that everyone who votes gets an equal say.\r\nSo let's say one member votes for answer A, and another nominates answer B and answer C. Answer A will count for twice as much points as B or C.",
"Solution_1": "Who is Chuck Norris?",
"Solution_2": "These should explain it:\r\nhttp://en.wikipedia.org/wiki/Chuck_Norris\r\nhttp://en.wikipedia.org/wiki/Chuck_Norris_Facts\r\n\r\nSo far, 5 people, including me, voted. I probably will stop once I get a decent size of people, maybe 20 or 25 or something, and then just play around with the numbers.",
"Solution_3": "[quote=\"bubka\"]Who is Chuck Norris?[/quote]\r\nread my signature. :D search for chuck norris random fact generator on google.",
"Solution_4": "[quote=\"fireemblem13\"][quote=\"bubka\"]Who is Chuck Norris?[/quote]\nread my signature. :D search for chuck norris random fact generator on google.[/quote]\r\n\r\nOk so your signature has plenty of mathematical feats Chuck Norris is capable of. But that does not help me as regards his identity. I am too lazy for a google search now, so somebody clarify please.",
"Solution_5": "as far as i know, he's a made-up person, but im not the best person to ask. my friend told me a chuck norris joke(non-mathematical) one day and it was funny... so yea...",
"Solution_6": "There is a real Chuck Norris and I have seen him on tv ;) I have signed up, too.",
"Solution_7": "oh. what did he do?\r\ni sign up too",
"Solution_8": "He played Walker in \"Walker, Texas Ranger\"",
"Solution_9": "So far, 11 people voted. I'm looking for more, still.",
"Solution_10": "so, once there are a lot of ppl who have done this, what do we do?",
"Solution_11": "And who is Chuch norris, fireemblem?",
"Solution_12": "let's see.... the wikipedea link above is pretty useful. basically, he's someone you associate impossible and crazy talsks with. for example, some random facts\r\n1. Chuck Norris doesn't brush his teeth. He points his fist at his mouth and the plaque jumps out. :rotfl: :rotfl: \r\n2. If you are eating Alphabits and the letters somehow spell Chuck Norris, DO NOT attempt to take another bite if you value your life.\r\n3. Contrary to popular belief, a meteor did not end the dinosaurs' existance. It was Chuck Norris' roundhouse kick; it was so powerful it left a massive crater in the earth. :rotfl:",
"Solution_13": "Once we get a bunch of votes, we'll start the game. Most likely in another topic, though.",
"Solution_14": "How many players are there so far?",
"Solution_15": "So far, 12 votes. If you're okay with this, we could get something going.",
"Solution_16": "Sure. What topic are you going to make?"
}
{
"Tag": [
"trigonometry",
"number theory"
],
"Problem": "resolve this equation \r\n$sin^3$x(1+cotangx)+$cos^3$x(1+tanx) = cos2x",
"Solution_1": "$\\sin^3x(1+\\cot x)+\\cos^3x(1+\\tan x)=\\cos 2x$\r\n$\\sin^2 x(\\sin x+\\cos x)+\\cos^2 x(\\cos x+\\sin x)=\\cos 2x$\r\n$\\sin x+\\cos x=\\cos 2x$\r\n$\\sin x\\pm\\sqrt{1-\\sin^2 x}=1-2\\sin^2x$\r\n$\\pm\\sqrt{1-\\sin^2 x}=1-\\sin x-2\\sin^2 x$\r\n$1-\\sin^2 x=(1-\\sin x-2\\sin^2 x)^2$\r\nAfter expansion and factorization, we have\r\n$\\sin x(\\sin x+1)(2\\sin^2 x-1)=0$\r\n$\\sin x=0$ or $\\sin x=-1$ or $\\sin x=\\pm\\frac{\\sqrt{2}}{2}$\r\nHowever, from the original equation, we know that $\\sin x$ and $\\cos x$ must be both non-zero or otherwise $\\tan x$ and $\\cot x$ will be undefined. So the case $\\sin x=0$ and $\\sin x=-1$ (which leads to $\\cos x=0$) should be rejected. So the case left is $\\sin x=\\pm\\frac{\\sqrt{2}}{2}$.\r\nWe now get $x=\\frac{\\pi}{4}+\\frac{k\\pi}{2}$ where $k\\in\\mathbb{Z}$\r\nBut when we put x into $\\sin x+\\cos x=\\cos 2x$ adn check, we find that only when $x=\\frac{3}{4}\\pi,\\frac{7}{4}\\pi,etc$ the equation is satified.\r\nTherefore, $x=\\frac{3}{4}\\pi+k\\pi$ where $k\\in\\mathbb{Z}$\r\nPS1 This is my 100th post! :D \r\nPS2 Is this a number theory problem?",
"Solution_2": "first of all $x\\neq \\frac{k\\pi}{2},k\\in Z$ ..... [b](*)[/b]\n\nequation is equivalent with:\n\n$\\sin x+\\cos x=\\cos 2x$ \n\n$\\sin x+\\cos x=-(\\sin x+\\cos x)(\\sin x-\\cos x)$ \n\n$(\\sin x+\\cos x)(\\sin x-\\cos x+1)=0$\n\n$(\\sin x+\\cos x)\\left(\\cos\\left(x+\\frac{\\pi}{4}\\right)-\\frac{\\sqrt{2}}{2}\\right)=0$\n\nsecond factor of LHS is $\\neq 0$ (condition [b](*)[/b]), so solution is \n\n$\\boxed{x=m\\pi-\\frac{\\pi}{4}\\ ,\\ m\\in Z}$"
}
{
"Tag": [
"algebra",
"polynomial",
"number theory",
"relatively prime",
"AMC"
],
"Problem": "The polynomial \r\n $ P(x) \\equal{} (1\\plus{}x \\plus{} x^2 \\plus{} . . . \\plus{}x^{17})^2\\minus{}x^{17}$\r\nhas $ 34$ complex zeros of the form $ z_k\\equal{}r_k[cos(2\\pi\\alpha_k) \\plus{}isin(2\\pi\\alpha_k)]$\r\n$ k\\equal{}1,2,3,...,34$ with $ 00$. given that \r\n$ a_1\\plus{}a_2\\plus{}a_3\\plus{}a_4\\plus{}a_5\\equal{}m/n$, where $ m$ and $ n$ are relatively prime positive integers, find $ m\\plus{}n$.",
"Solution_1": "[url]http://www.artofproblemsolving.com/Forum/viewtopic.php?p=378129#378129[/url]"
}
{
"Tag": [
"search",
"number theory unsolved",
"number theory"
],
"Problem": "Prove that there are infinitely many positive integers $ n$ such that $ n(n\\plus{}1)$ can be represented as a sum of two positive squares in at least two different ways. (Here $ a^{2}\\plus{}b^{2}$ and $ b^{2}\\plus{}a^{2}$ are considered as the same representation.)",
"Solution_1": "http://www.mathlinks.ro/viewtopic.php?search_id=820928092&t=122895",
"Solution_2": "http://www.mathlinks.ro/viewtopic.php?p=695192#695192",
"Solution_3": "one sol is $n=\\left(\\frac{(2+\\sqrt{5})^{2k}(1+\\sqrt{5})-(2-\\sqrt{5})^{2k}(1-\\sqrt{5})}{2\\sqrt{5}}\\right)^2$ for all $k\\in\\mathbb{N}$.Pretty bad looking,huh? :D\nIn this case,$n(n+1)=p^4+p^2$ and $(p^2-2)^2+(5p^2-4)$.This $5p^2-4$ is a square.",
"Solution_4": "[quote=sankha012]one sol is $n=\\left(\\frac{(2+\\sqrt{5})^{2k}(1+\\sqrt{5})-(2-\\sqrt{5})^{2k}(1-\\sqrt{5})}{2\\sqrt{5}}\\right)^2$ for all $k\\in\\mathbb{N}$.Pretty bad looking,huh? :D\nIn this case,$n(n+1)=p^4+p^2$ and $(p^2-2)^2+(5p^2-4)$.This $5p^2-4$ is a square.[/quote]\nHey (sorry to revive this thread).. But I seriously wanna ask How the hell did you come up with that? Plz. give the motivation behind it...\n Since there is no reachable solution to this prob on this thread till now here's my idea:-\n[hide]Set $n=m^2$ as then $n(n+1)$ is already a sum of two squares.\nNow if we get $m^2=p^2+q^2,$ then notice that it becomes $(pm+q)^2 - (p-qm)^2$..\nNow it's trivial as it's well know that there are infinite pythagorean triplets...So there are infinite $(m,n,p)$ where m,n,p are integers..\nQED[/hide]\n\n",
"Solution_5": "This is my solution",
"Solution_6": "What about this?\n\n$n=4m^4$\n$n^2+n=(4m^4)^2+(2m^2)^2$\n$=(4m^4-2m^2)^2+(4m^3)^2$",
"Solution_7": "[quote=makar]Prove that there are infinitely many positive integers $ n$ such that $ n(n\\plus{}1)$ can be represented as a sum of two positive squares in at least two different ways. (Here $ a^{2}\\plus{}b^{2}$ and $ b^{2}\\plus{}a^{2}$ are considered as the same representation.)[/quote]\n\nIf n is such a number, then so is 4n(n+1).\n\n"
}
{
"Tag": [
"function",
"geometry"
],
"Problem": "An open box with a square base is required to have a volume of 10 cubic feet.\r\n\r\nNOTE: What exactly is a cubic foot?\r\n\r\n(a) Express the amount A of material used to make such a box as a function of the length x of a side of the square base.\r\n\r\n(b) How much material is required for a base 1 foot by 1 foot?\r\n\r\nNOTE: Is one foot by foot the same as 144 inches?",
"Solution_1": "The box have a square base.\r\nThe volume of the box is the area of the base times the heigth.\r\nOr\r\n\r\n${x^{2}h = 10}$\r\n\r\nIt is equal 10, because the volume was given.\r\n\r\nThe amout of material is proportional to the sufarce area. so\r\n\r\n${S=x^{2}+4 h x}$\r\n${h=\\frac{10}{x^{2}}}$\r\n${S=\\frac{x^{3}+40}{x}}$\r\n\r\n\r\n${A=\\alpha S}$\r\n\r\nI'm not english. so i dont know how much is one feet, and one inches. but one foot by one foot could be 144 square inches",
"Solution_2": "dude, omfg. let me introduce you to my bestest cyber friends, yahoo and his sidekick google. you could meet them at yahoo.com and google.com. they would love to meet you because thats how they make their money and im sure they would be extreely helpful to you. \r\n\r\nyou would be saving us a buch of time if you used them\r\n\r\none foot is 12 inches. that means 1 square foot is 144 square inces and 1 cubic foot is 12^3 cibic inches."
}
{
"Tag": [
"AMC",
"AIME",
"\\/closed"
],
"Problem": "Hello!\r\n\r\nI have two questions. \r\n\r\n\r\n1. I really don't get the difference between AIME A and AIME B Problem Series, diverting from Aime A is not in the summer. \r\n\r\n2. Which one should I take AMC 12 Problem Series or AIME B? \r\n\r\nI am going to finish Volume 1 and 2 this summer. And I really want to do well on the AIME next year. I also might take WOOT, but I am not positively sure.\r\n\r\nThanks for your help! :lol: :)",
"Solution_1": "[quote]Description\nOur two AIME Problem Series classes prepare students for the AIME, which is the second in the series of tests used to determine the United States team at the International Math Olympiad. Many top colleges also request AIME scores as part of the college application process. Each day consists of a discussion of specific problems from past AIME competitions, or from other contests of a similar difficulty level. Important general problem solving strategies and specific math facts are taught through solving these problems.\n\nPlease note, we offer two versions of this class, AIME Problem Series A and AIME Problem Series B. These two courses are roughly the same difficulty, but the problems covered in the two courses are completely different. Many students opt to take both classes. We typically offer one in the summer and one in the fall/winter.[/quote]\r\n\r\nthat should answer question number 1.",
"Solution_2": "if anyone could clear this up for me at least \r\n\r\n#2 \r\n\r\nthanks! :)",
"Solution_3": "To answer #2, we'll probably need some more background info on you.",
"Solution_4": "what would you mean by some background information",
"Solution_5": "What are your goals? Have you qualified for AIME before? Do you want a higher AIME score or higher AMC score? What grade are you in? etc.\r\n\r\nAlso, I would suggest holding off on WOOT until you are positive that you will qualify for USAMO.",
"Solution_6": "[quote=\"crazypianist1116\"]What are your goals? Have you qualified for AIME before? Do you want a higher AIME score or higher AMC score? What grade are you in? etc.\n\nAlso, I would suggest holding off on WOOT until you are positive that you will qualify for USAMO.[/quote]\r\n\r\nI agree on more background info needed, but I don't think should avoid WOOT until you are certain you're ready for the USAMO. We've had plenty of people take WOOT fruitfully before being sure they could pass the AIME. Before taking on WOOT, you should be sure that you are dedicated enough to put in the time to get the most out of the course. Mathematically, it would be best if you are confident you can get at least 5 AIME questions. So, you should have a fighting chance on the AIME before starting WOOT, but you needn't wait until you are sure you can pass the AIME to join."
}
{
"Tag": [
"search",
"probability",
"expected value"
],
"Problem": "An experiment has n possible outcomes, all equally likely. Each trial is independent. We repeat until we have two trials with the same outcome. For example, we would stop after getting 1, 4, 3, 4. What is the expected number of trials f(n)? Find an asymptotic expression for f(n).",
"Solution_1": "My apologies for the incorrect link. Here is the link but a similiar (though different) problem posted eariler.\r\n\r\nhttp://www.artofproblemsolving.com/Forum/viewtopic.php?search_id=1695488873&t=158138",
"Solution_2": "This is a different (although similar) problem.\r\n\r\n[hide=\"Solution\"]The probability we finish in exactly $ m+1$ trials, $ 0 \\leq m \\leq n$ is $ \\frac{m}{n}\\cdot \\prod_{i = 0}^{m-1}\\frac{n-i}{n}= \\frac{m \\cdot n!}{n^{m+1}(n-m)!}$. Thus, the answer is $ \\sum_{m = 0}^{n}\\frac{m(m+1) \\cdot n!}{n^{m+1}(n-m)!}$. I don't, off the top of my head, see a nice form for this expression (although that shouldn't stop others from looking for one).[/hide]"
}
{
"Tag": [
"AMC",
"AIME",
"ARML",
"USA(J)MO",
"USAMO",
"algebra",
"functional equation"
],
"Problem": "So I've been working through AoPS vol 1&2 over the summer and have been doing random aime problems (usually around 5~6 tough ones a day). I don't know but I think I'm starting to lose my interest in math since I haven't really been doing much else. I am scared that I will burnout and abandon math altogether. Can someone please tell me how much math I should be doing. Right now I do about 7-8 hours of math per day but most of it is done in the early mornings or at night.\r\n\r\n\r\nThanks in Advance",
"Solution_1": "There's no amount you \"should\" be doing. Do as much as interests you. I can say that 7-8 hours would be too much for me, at least.",
"Solution_2": "I would suggest taking a break from math a little bit and start doing something else. I was burned out just after this year's AIME and didn't feel like doing math anymore. Usually, when you take breaks, you start to remember what you enjoyed about math and then you'll do math again. You just need some variety in your life once in a while so it wouldn't hurt to take an entire day or two off of math. And I also do 7-8 hrs per day (during winter, summer, and spring break) so I know how you feel. The chances that you will [b]completely[/b] lose your interest is very low if you really do have a true passion for math.",
"Solution_3": "You can always diversify and study something like science one day, then math for the next few days. Take breaks from math, but you don't necessarily have to take breaks from studying.",
"Solution_4": "Personally, working through a whole problem book is too intimidating for me. I usually do problems from various sources to reduce fatigue (e.g. aops forums).",
"Solution_5": "the problem is that you're doing 7-8 hours of math a day. you should barely even be awake for that long. add in meals, school, practicing instruments, sports practices, and studying/homework for other subjects and you're up until midnight\r\nyou should probably get some rest",
"Solution_6": "Yeah, 7-8 is kind of overkill. \r\n\r\nWhen I get bored, I do things I shouldn't be doing (cough group theory/axiomatic set theory/etc...), or I just do olympiad problems instead, which are very different from AIME problems. There's always something out there.",
"Solution_7": "How quickly you burn out I think depends more on how much you force yourself to do, as opposed to how much you're naturally inclined to do. As an example of the latter, when doing a stupid assignment like writing a paper my idea of a break is finding a functional equation on these forums and busting my brain on it for a bit. So that 7-8 hours is going to read differently for different people. But if you wouldn't normally do something like 4-5 hours per day anyway, unforced, you might want to cut back.\r\n\r\n[OFFTOPIC]\r\n[quote=\"Zhero\"]I do things I shouldn't be doing (cough group theory/axiomatic set theory/etc...)[/quote]\r\n\"Shouldn't be doing\"? That is perfectly legitimate math, maybe more so than contest math, and for the olympiad problems motivated by advanced concepts* you may be well prepared.\r\n\r\n* such as [url=http://www.artofproblemsolving.com/Forum/viewtopic.php?t=159989]this IMO problem[/url] whose solution is elementary only in its presentation, though such a problem [i]should[/i] never appear on an olympiad.\r\n[/OFFTOPIC]\r\n\r\nEDIT to below: Of course. I suppose I implicitly meant \"on average\" several times in this post.",
"Solution_8": "Well the amount you would do \"unforced\" can also vary. For example I might feel inclined to do a lot of math one day and then much less the next.\r\n\r\n@Palmer: Is this not the case for you?",
"Solution_9": "You should no when you feel tired. After you feel tired you will never remember the concepts you just studied. So if you do not feel tired doing 7-8 hours of math then I guess I should be fine.\r\n\r\n :surf: This is my 50th post(no celebration).",
"Solution_10": "[quote]\"Shouldn't be doing\"? That is perfectly legitimate math, maybe more so than contest math, and for the olympiad problems motivated by advanced concepts* you may be well prepared. [/quote]\r\nHmm yeah, now that you mention it, I think I'll have to retract my statement. Though I didn't think much of it before, I did benefit from doing axiomatic set theory. For one, it helped me to understand mathematical rigour more (e.g., what axiomatic systems are and how to write formal proofs), and I learned much about functions. \r\n\r\nAnd it certainly provides for a fine break away from the stuff like that the OP's been doing. And it does teach invaluable techniques. \r\n\r\nI suppose this notion arose from my putting competition math above all the other math... in the past few months, in my eyes, all the other math is just a distraction from competition math. \r\n\r\nBut my eyes have opened now. It's summer now, no competitions to worry about. Time to indulge...",
"Solution_11": "A great way to memorize new formulas/concepts is to do them right before you sleep. Believe it or not, your brain thinks about what you just did while you're asleep, and when you wake up again, what you just learned will stay with you for a long time.",
"Solution_12": "If I do math right before I sleep, I can't sleep.\r\n\r\nI recommend 1-3 hours of math on weekdays (depending on how much homework you have) and 4-6 hours on weekends. (each day)\r\n\r\nThat's like all I have time for. But the week after ARML (and the month before it) I barely did any math at all, because preparing for USAMO basically made me sick of it. (temporarily, of course)",
"Solution_13": "Wow, do it like, few hours. 1, 1.5, 2.Something like that. I'd die if I did math stuff 7-8 hours a day. It s like, a part time job for you :D\r\nI'd say forget about maths for like a few weeks, go do some sports, hang out with your buddies more. The reason why you lose interest is because you dedicate too much time to it, you feel a routine in it, it becomes boring. I know what I'm talking about :roll: \r\n\r\nFor me, I deal with maths in the evening. Open up the unsolved problems section and work my magic. But I can't do it for 7 hours a day :(\r\nYou know if you do maths so long, your head is like, exhausted, usually when I feel that way I go outside jogging or riding a bike or something and when I think about the math problem I had, the answer seems to somewhat more easily reveal itself :P\r\n\r\nSo all in all, you just have to take up something else if you don't wanna lose interest. \r\n\r\nHave fun."
}
{
"Tag": [
"linear algebra",
"matrix"
],
"Problem": "Determine the number of possible values for the determinant of $ A$, given that $ A$ is a $ n*n$ matrix with real entries such that $ A^3\\minus{}A^2\\minus{}3A\\plus{}2I\\equal{}0$, where $ I$ is the identity and $ 0$ is the all-zero matrix. [/b][/hide][/code]",
"Solution_1": "All eigenvalues of this matrix are roots of $ x^3 \\minus{} x^2 \\minus{} 3x \\plus{} 2 \\equal{} 0.$ That equation has three real roots, $ 2$ and $ \\frac { \\minus{} 1\\pm\\sqrt {5}}2.$ Let's just call these numbers $ r_1,r_2,$ and $ r_3.$\r\n\r\nThe possible values of $ \\det(A)$ are $ r_1^ir_2^jr_3^k,$ where $ i,j,$ and $ k$ are nonnegative integers such that $ i \\plus{} j \\plus{} k \\equal{} n.$\r\n\r\nWe can form such a product in $ \\binom{n \\plus{} 2}{2}$ ways. The only remaining question is whether different values of $ (i,j,k)$ could result in the same determinant - I don't think so, but I haven't proven that yet.",
"Solution_2": "Hello,\r\n\r\nI am sure that there is an easier way to finish the last part, but this is one way:\r\n\r\nLet $ i_1\\plus{}j_1\\plus{}k_1\\equal{}n$ and $ i_2\\plus{}j_2\\plus{}k_2\\equal{}n$. Then if we have two values of the determinant that are equal, we get\r\n\r\n$ \\begin{array}{lrl}\r\n &2^{i_{1}}\\left(\\frac{\\minus{}1\\plus{}\\sqrt{5}}{2}\\right)^{j_{1}}\\left(\\frac{\\minus{}1\\minus{}\\sqrt{5}}{2}\\right)^{k_{1}}&\\equal{}2^{i_{2}}\\left(\\frac{\\minus{}1\\plus{}\\sqrt{5}}{2}\\right)^{j_{2}}\\left(\\frac{\\minus{}1\\minus{}\\sqrt{5}}{2}\\right)^{k_{2}}\\\\\r\n\\Leftrightarrow & 2^{i_{i}\\minus{}j_{1}\\minus{}k_{1}\\minus{}(i_{2}\\minus{}j_{2}\\minus{}k_{2})}&\\equal{}\\left(\\minus{}1\\plus{}\\sqrt{5}\\right)^{j_{2}\\minus{}j_{1}}\\left(\\minus{}1\\minus{}\\sqrt{5}\\right)^{k_{2}\\minus{}k_{1}}\\\\\r\n\\Leftrightarrow &4^{i_{1}\\minus{}i_{2}}&\\equal{}\\left(1\\minus{}5\\right)^{j_{2}\\minus{}j_{1}}\\left(\\minus{}1\\minus{}\\sqrt{5}\\right)^{j_{1}\\minus{}k_{1}\\minus{}(j_{2}\\minus{}k_{2})}\\\\\r\n\\Leftrightarrow &\\left(\\minus{}1\\right)^{j_{1}\\minus{}j_{2}}4^{i_{1}\\plus{}j_{1}\\minus{}(i_{2}\\plus{}j_{2})}&\\equal{}\\left(\\minus{}1\\minus{}\\sqrt{5}\\right)^{j_{1}\\minus{}k_{1}\\minus{}(j_{2}\\minus{}k_{2})}\\\\\r\n\\Leftrightarrow & \\left(\\minus{}1\\right)^{j_{1}\\minus{}j_{2}}4^{k_{2}\\minus{}k_{1}}&\\equal{}\\left(\\minus{}1\\minus{}\\sqrt{5}\\right)^{j_{1}\\minus{}k_{1}\\minus{}(j_{2}\\minus{}k_{2})}\r\n\\end{array}$\r\n\r\nNow one can argue that the left hand side of the last equation always is rational for integer exponents while the right hand side always is irrational except when the exponent is zero (I am not giving a proof to this statement though) which forces the RHS to be equal to 1. From the LHS we therefore get $ k_1\\equal{}k_2$ which, if put into the condition $ j_1\\minus{}k_1\\equal{}j_2\\minus{}k_2$, shows that $ j_1\\equal{}j_2$, but then $ i_1\\equal{}i_2$ as well and we are done. The answer is thus $ \\binom{n\\plus{}2}{2}$, the one Kent Merryfield anticipated."
}
{
"Tag": [
"number theory proposed",
"number theory"
],
"Problem": "I know this problem is posted before but as I know ,noone posted solution.\r\nProve that for all $ n\\geq 3$ and $ n\\in Z_{\\plus{}}$ we have $ S(9^{n})>9$ .where $ S(n)$ denote sum of all digits of $ n$.",
"Solution_1": "If n is odd last digit is 9, therefore (because $ 9^{n}>9$) $ S(9^{n})>9$. If n=2k, then $ S(9^{n})\\equal{}S(81^{k})$.\r\n$ 81^{k}\\equal{}1\\plus{}10*k*8\\plus{}100*\\frac{k(k\\minus{}1)}{2}64\\plus{}...$\r\nBy considering many variants of k we can prove, that $ S(81^{k})>9$ for k>1."
}
{
"Tag": [
"inequalities open",
"inequalities"
],
"Problem": "Ask for help!",
"Solution_1": "[quote=\"teanleen\"]Ask for help![/quote]\r\nSee here:\r\nhttp://www.artofproblemsolving.com/Forum/viewtopic.php?t=57683",
"Solution_2": "[quote=\"arqady\"][quote=\"teanleen\"]Ask for help![/quote]\nSee here:\nhttp://www.artofproblemsolving.com/Forum/viewtopic.php?t=57683[/quote]Thank you!\r\nBut ,I don't think it is very nice!"
}
{
"Tag": [
"limit",
"LaTeX",
"calculus",
"calculus computations"
],
"Problem": "[color=darkblue]Let $ f(n) \\equal{} (n^2 \\plus{} n \\plus{} 1)^2 \\plus{} 1$, with $ n \\in N$ and $ n\\ge 1$.\n\nLet $ x_n \\equal{} \\frac {f(1).f(3).f(5)...f(2n \\minus{} 1)}{f(2).f(4).f(6)...f(2n)}$, with $ n \\in N$ and $ n\\ge 1$.\n\nLet $ u_n \\equal{} n^2.x_n$\n\nFind $ \\lim_{n \\to \\plus{} \\infty}(u_n)$[/color]",
"Solution_1": "Hint: $ \\frac{f(2n\\minus{}1)}{f(2n)} \\equal{} \\frac{2n(n\\minus{}1) \\plus{} 1}{2(n\\plus{}1)n \\plus{} 1}$",
"Solution_2": "$ \\text{\\LaTeX}$ note: use \\cdot for the multiplication dot. $ f(1)\\cdot f(2)$ looks much better than $ f(1).f(2)$. Also, \\dots is a better spaced ellipsis, and \\cdots is a vertically centered one."
}
{
"Tag": [
"geometry",
"area of a triangle",
"Heron\\u0027s formula"
],
"Problem": "In triangle $ABC$, find $\\frac{\\cot{\\frac{A}{2}}}{s-a}$ where $s$ is the semi-perimeter. (you can answer in side lengths, etc.)",
"Solution_1": "We have \r\n\r\n\\[\\cot \\frac{A}{2}+\\cot \\frac{B}{2}=\\frac{c}{r}\\]\r\n\r\n\\[\\cot \\frac{B}{2}+\\cot \\frac{C}{2}=\\frac{a}{r}\\]\r\n\r\n\\[\\cot \\frac{C}{2}+\\cot \\frac{A}{2}=\\frac{b}{r}\\]\r\n\r\nso, we get $\\cot \\frac{A}{2}=\\frac{b+c-a}{2r}$,therefore using Heron's formula\r\n\r\n$\\frac{\\cot \\frac{A}{2}}{s-a}=\\frac{1}{r}=\\frac{1}{2}\\sqrt{\\frac{(a+b-c)(b+c-a)(c+a-b)}{a+b+c}}$",
"Solution_2": "how did you get these realtions??? :(",
"Solution_3": "I believe you can get it from drawing a triangle, look at $tan {\\frac{A}{2}}$ and then you'll get the relation.",
"Solution_4": "See the attachment."
}
{
"Tag": [
"algebra",
"polynomial",
"algebra unsolved"
],
"Problem": "Find all $P(x)\\in{R[x]}$ such that \r\n$P(x^{2}+1)=P^{2}(x)-1$",
"Solution_1": "(*) $P(x^{2}+1)=P(x)^{2}-1$\r\nDeriving (*) we get \r\n(**) $xP'(x^{2}+1)=P(x)P'(x)$.\r\n\r\nTake $g(x)=x^{2}+1$ and for all $n\\geq 0$\r\n$g_{n}(x)=\\underbrace{g(g(\\cdots g(x)))}_{n-{\\text{times}}}$.\r\n\r\nThen $00$ for all $\\lambda\\in\\mathbb{R}$,\r\nand for all $\\lambda\\in\\mathbb{C}$ we have \r\n$P(\\lambda)=0\\ \\ \\Rightarrow P(g(\\lambda))=P(\\lambda)^{2}-1=-1$ and\r\n$P(\\lambda)=-1\\Rightarrow P(g(\\lambda))=P(\\lambda)^{2}-1=0$.\r\n\r\n(1) $\\gcd (P(x),g_{n}(x))=1$ for all $n\\geq 0$:\r\nOtherwise $P(x)$ and $g_{n}(x)$ have a common root $\\lambda\\in\\mathbb{C}$ for some $n\\geq 0$.\r\nBut then $P(\\lambda)=0$ and $P(0)=P(g_{n}(\\lambda))\\in\\{0,-1\\}$.\r\nSo $P(0)=P(g_{2}(0))=P(g_{4}(0))=\\cdots$ and the polynomial $P(x)-P(0)$ has an infinite number of roots.\r\nThis forces $P\\equiv P(0)\\in\\{0,-1\\}$, which contradicts (*).\r\n\r\n(2) $g_{n}(x)\\ |\\ P'(x)$ for all $n\\geq 0$:\r\nWe have $g_{0}(x)=x\\ |\\ xP'(g(x))=P(x)P'(x)$ and so $g_{0}(x)\\ |\\ P'(x)$ by (1)\r\nand from $g_{n}(x)\\ |\\ P'(x)$ follows $g_{n+1}(x)=g_{n}(g(x))\\ |\\ xP'(g(x))=P(x)P'(x)$ \r\nand $g_{n+1}(x)\\ |\\ P'(x)$ for all $n\\geq 0$ again using (1).\r\n\r\nNow $\\deg g_{n}=2^{n}\\mathop{\\longrightarrow}_{n\\rightarrow\\infty}\\infty$, so $P'\\equiv 0$ and $P\\equiv c$ constant. Then (*) gives $c=c^{2}-1$.\r\nSo (*) has $2$ solutions $P\\equiv{1+\\sqrt{5}\\over 2}$ and $P\\equiv{1-\\sqrt{5}\\over 2}$."
}
{
"Tag": [],
"Problem": "If $ x$ is positive and $ x^2 \\equal{} 729$, what is the value of $ x$?",
"Solution_1": "The equation $ x^2\\equal{}729$ has solutions $ x\\equal{}\\pm 27$. They tell you $ x$ is positive, so $ x\\equal{}\\boxed{27}$",
"Solution_2": "$ 7+2+9=18=9 \\times 2 \\ \\text{i.e.} \\ 9 \\mid 729$.\r\nSo,\r\n$ \\begin{eqnarray*}729 & = & 9\\times 81 \\\\ & = & 9 \\times 9 \\times 9 \\\\ & = & 9^3 \\\\ & = & \\left(3^2\\right)^2 \\\\ & = & 3^{2 \\times 3} \\\\ & = & 3^{3 \\times 2} \\\\ & = & \\left(3^3\\right)^2 \\\\ & = & 27^2.$",
"Solution_3": "[quote=\"Kouichi Nakagawa\"]$ 7+2+9=18=9 \\times 2 \\ \\text{i.e.} \\ 9 \\mid 729$.\nSo,\n$ \\begin{eqnarray*}729 & = & 9\\times 81 \\\\ & = & 9 \\times 9 \\times 9 \\\\ & = & 9^3 \\\\ & = & \\left(3^2\\right)^2 \\\\ & = & 3^{2 \\times 3} \\\\ & = & 3^{3 \\times 2} \\\\ & = & \\left(3^3\\right)^2 \\\\ & = & 27^2.$[/quote]\n@Kouichi Nakagawa, for the fourth line in the equation, it should be $=\\left(3^2\\right)^3$, not $=\\left(3^2\\right)^2$.",
"Solution_4": "[quote=\"JSGandora\"][quote=\"Kouichi Nakagawa\"]$ 7+2+9=18=9 \\times 2 \\ \\text{i.e.} \\ 9 \\mid 729$.\nSo,\n$ \\begin{eqnarray*}729 & = & 9\\times 81 \\\\ & = & 9 \\times 9 \\times 9 \\\\ & = & 9^3 \\\\ & = & \\left(3^2\\right)^2 \\\\ & = & 3^{2 \\times 3} \\\\ & = & 3^{3 \\times 2} \\\\ & = & \\left(3^3\\right)^2 \\\\ & = & 27^2.$[/quote]\n@Kouichi Nakagawa, for the fourth line in the equation, it should be $=\\left(3^2\\right)^3$, not $=\\left(3^2\\right)^2$.[/quote]\n\nYes that's right.\nI've made a typo."
}
{
"Tag": [
"calculus",
"calculus computations"
],
"Problem": "Solve the following, given that y(1) = 1\r\n\r\nx(-5y^2 + 9x^2)y' = 2(y^2)(3x-5y)",
"Solution_1": "Hello there. This is a first order homogeneous ODE that can be solved using the substitution, y = vx."
}
{
"Tag": [
"geometry",
"circumcircle",
"ratio",
"geometry unsolved"
],
"Problem": "please post solutions to----\r\n\r\nhttp://www.mathlinks.ro/viewtopic.php?t=238434",
"Solution_1": "Mmm, It 's easy. You can figure it out for your case. In case, you need the solution. Click on the hidden message.\r\n\r\n[hide=\"Solution\"]\n$ ND$ is the diameter of the circumcircle of triangle $ BFD$ and $ \\triangle up BFD$ is the image of triangle $ ABC$ under the scale of half. (or $ BFD$ is similar to $ ABC$ with ratio $ \\frac{1}{2}$).\n[/hide]"
}
{
"Tag": [
"number theory",
"least common multiple",
"prime factorization"
],
"Problem": "This should set an example; if you want harder problems look somewhere else.\r\n\r\n1. Two friends are playing tag. The first runs at 20 feet per minute and the other at 25 feet per minute. If the first friend has a minute head start how long will it take the second friend to catch up?\r\n\r\n2. There are two brothers. Right now one is three times as old as the other. In three years he will only be twice as old. What are their ages now?\r\n\r\nI hope these problems show an appropriate level of difficulty for Getting Started problems (they're easy). ;) \r\n\r\nEdited to change speed (but it doesn't change the answer to the first problem) and to correct spelling.",
"Solution_1": "These problems look good to me. Although 20 feet per hour is kind of slow :D .",
"Solution_2": "Random question but are American children taught feet and inches in school?!",
"Solution_3": "Yeah, America mostly uses feet, inches and the like. I personally believe that we should switch to the metric system(and most other ppl who know science will agree). We have to spend 1-2 weeks in science class just to learn how to convert from the English system to the metric...all you need to know for the metric system are the prefixes! We Americans are too stubborn.",
"Solution_4": "1.\n\n[hide]First guy has a 20 ft head start, since he has a minute's head start. After one minute, the first guy will have traveled 40 ft, and the 2nd guy will have traveled 25 ft. So if we keep on going and listing numbers, we find that once 4 minutes are over, they will both have traveled 100 ft.\n\n\n\nEdited, I found a nicer way using the least common multiple of 20 and 25. I think there's a way to do this problem using the distance formula, but oh well. \n\n20's prime factorization is 5*2 2 , and 25's is 5 2 . So the LCM of these two numbers needs to have a 5 2 and a 2 2 . Multiplying 25 and 4 out we get 100, our answer from before. Now, we just need to figure out how long it'll take. We can use the distance formula I think. Distance=100, so 100=25x, x=4. If you wanted to do it for the first guy, his distance is 80 feet, so 80=20x, and both ways we get 4.[hide][/hide]\n\n\n\n2. \n\n[hide] Let a be the older brother and b be the younger brother. Thus, a = 3b. In 3 years a will be twice as old as b. So a + 3 = 3 (b + 3), which gives a + 3 = 3b + 6. Using substitution, 3b + 3 = 3b + 6, so b = 3, and a = 9.[/hide][/hide]",
"Solution_5": "Let's try something. If at time x the faster brother catches up to the slower brother, what's the distance that the faster brother has covered? What's the distance that the slower brother has covered? Make this into an equation and solve for x.",
"Solution_6": "[quote=\"mikewd\"]Random question but are American children taught feet and inches in school?![/quote]\r\n\r\nYes, as my fellow moderator explained, United States schools still mostly teach English customary units. Interestingly, the metric system is the only system of measurement ever OFFICIALLY acknowledged by Congress (back in the 1860s), but the force of custom is such that most Americans are still much more familiar with inches than with centimeters, with miles than kilometers, and so on.",
"Solution_7": "[quote=\"joml88\"]Yeah, America mostly uses feet, inches and the like. I personally believe that we should switch to the metric system(and most other ppl who know science will agree). We have to spend 1-2 weeks in science class just to learn how to convert from the English system to the metric...all you need to know for the metric system are the prefixes! We Americans are too stubborn.[/quote]\r\n\r\nEnglish system.. how ironic, we use metric in the UK (well in school at least, my parents don't)",
"Solution_8": "I tried what you suggested, Osiris, and this is what I got:\n\n[hide]x is the time it takes for 2nd guy to catch up with 1st guy, so 25x = 20 + 20x, since the 1st guy had a 20 ft head start. Solving for x gives x = 4. Wow that was a lot easier.[/hide]",
"Solution_9": "Just cause I'm bored... #1 is sort of ambiguous (is that the right word for this?) because they could make turns, etc..\r\n\r\n#2: Care to clarify?\r\n[quote]In three years he will only be twice as old.[/quote]\r\nTwice as old as he was, or the older brother was, or the older brother will be?",
"Solution_10": "[quote=\"Duck-Billed Platypus\"]Just cause I'm bored... #1 is sort of ambiguous (is that the right word for this?) because they could make turns, etc..\n\n#2: Care to clarify?\n[quote]In three years he will only be twice as old.[/quote]\nTwice as old as he was, or the older brother was, or the older brother will be?[/quote]\r\n\r\nIn three years the older brother will be twice as old as the younger brother.",
"Solution_11": "#2: [hide]Let first brother's age be A, second brother's age be B.\n\nOne is 3 times as old as the other, so A=3B.\n\nIn 3 years he will be twice as old as the other, so A+3=2(B+3).\n\nSubstitute.. 3B+3=2B+6\n\nB=3\n\nTheir ages now are 9 and 3.[/hide]",
"Solution_12": "1.[hide]\n\nThe second friend catches up to the first friend 5 feet / minute, since 25-20 is 5. Since the first friend has a minute, or 20 feet head start it will take 20/5, or 4 minutes for the second friend to catch up.[/hide]\n\n\n\n2.[hide]\n\na will represent the first brother's age right now, and b will represent the second brother's age right now.\n\n\n\n3a = b\n\n\n\n2(a+3) = b+3\n\n2a+3 = b\n\n\n\n6a+9 = 3b\n\n6a+0= 2b\n\n-------------\n\n0a+9=b\n\n\n\nSo the older brother is 9 years old. We can substitue that into 3a = b.\n\n\n\n3a = 9\n\na = 3\n\n\n\nSo the younger brother is 3 years old.\n\n[/hide]",
"Solution_13": "1.[hide]20x + 20 = 25x\n\n20 = 5x\n\n4=x\n\n\n\nthe + 20 is the fact that 20 ft/min* 1 min = 20 ft, thus the y-interecept at 0 min mark of when the faster of the two starts to run.[/hide]\n\n\n\n2.[hide]2 brothers x and \n\n\n\nx = 3y\n\n\n\nx + 3 = 2(3 + y)\n\nx + 3 = 6 + 2y\n\n3y + 3= 6 + 2y\n\ny = 3\n\n\n\nx = 3 (3) = 9\n\n\n\nThe older brother (x) is 9 years of age, while the younger brother (y) is 3 years of age.\n\n[/hide]",
"Solution_14": "1. [hide] let x = minutes they have been running for. Since the first boy (20ft per minute) has a minute head start, he is 20 feet ahead. The first boy's distance is represented by 20x, and the second boy's distance is represented by 25x. So I set up an equation: 20 + 20x = 25x. After doing some algebra, x comes out to be 4, so therefore the answer is 4 minutes.[/hide]\n\n\n\n2. [hide] I made the younger one's age x, and the older one's 3x (seeing that he is 3 times the age of the younger one. Three years later, the older one's age will be 3x+3. The problem says at this time, the older one will only be twice the age of the younger one. I set up this equation: 3x+3 = 2(3+x) , or 3x+3=(6+2x). x comes out to be 3, so if you plug that in for the two original ages, you'll get 3 and 9.[/hide]\n\n\n\nThanks for these problems. The other ones in this forum are too hard for me at this point, but these are easy (maybe even a bit too easy)."
}
{
"Tag": [
"geometry",
"parallelogram",
"geometry unsolved"
],
"Problem": "Could someone give a solid/rigorous proof for the following:\r\n\r\nABCD is a convex quadrilateral. The midpoints of AB, BC, CD, DA are E, F, G, H. Segments AF, BG, CH, DE are drawn. The points at which these segments intersect are W, X, Y, Z. Then the midpoints of WX, XY, YZ, ZW form a parallelogram.\r\n\r\n\r\n\r\n[geogebra]c2f4825bba5facac77e0148a3d467d5d29c0a8f7[/geogebra]",
"Solution_1": "[quote=\"bbgun34\"]Could someone give a solid/rigorous proof for the following:\n\nABCD is a convex quadrilateral. The midpoints of AB, BC, CD, DA are E, F, G, H. Segments AF, BG, CH, DE are drawn. The points at which these segments intersect are W, X, Y, Z. Then the midpoints of WX, XY, YZ, ZW form a parallelogram.\n\n\n\n[geogebra]c2f4825bba5facac77e0148a3d467d5d29c0a8f7[/geogebra][/quote]\r\nIf $ \\square ABCD$ is a convex quadrialteral then the midpoints of $ AB,BC,CD,DA$ form a parallelogram, so there is probably something wrong in the statement as one is given much more information than needed."
}
{
"Tag": [
"inequalities",
"inequalities proposed"
],
"Problem": "Let $a_k\\geq0$ for all $k=1,2,...,n$.Prove the following inequality:\r\n\r\n $ e^(a_1+a_2+...+a_n-n)\\geq \\frac^n {a_1+a_2+...+a_n}{n}$ ;)",
"Solution_1": "Please, learn Latex. I learned quickly,although I know the first notions from you, for which I thank you !",
"Solution_2": "Oh,levi I'm so lazy.But in the end I must learn it.Anyway,thanks for the advice. ;)"
}
{
"Tag": [
"MATHCOUNTS",
"Pythagorean Theorem",
"geometry",
"exterior angle"
],
"Problem": "anyone who has intersting geometric proofs, could ou please post them here?\r\nfor example, the pythagorean theorm.\r\n\r\nIf possible, please post a problem using that proof in your post. only if possible. Thanks.",
"Solution_1": "For proofs of the Pythagorean Theorem, check [url=http://www.artofproblemsolving.com/Forum/viewtopic.php?t=275754]this[/url] thread.",
"Solution_2": "IDK how intense you are with geometric proofs. Heres a really really basic one:\r\n\r\n\"Show that he shortest distance from a line AB to a point C not on AB is the segment CD, where line CD is perpendicular to line AB and point D lies on line AB.\"\r\n\r\nThats not very interesting, but I came up with it (yeah, obviously someone noticed it waaaaayyyyy before me and its sorta obvious, but I hadnt heard of any theorem like that before, even though im sure there is one out there)",
"Solution_3": "are you talking about like, MC proofs, if higher move it there mods.",
"Solution_4": "mathcounts level proofs i mean",
"Solution_5": "One such example:-\r\n\r\nProve that the sum of the 3 exterior angles of a triangle is equal to 360 degrees.",
"Solution_6": "To prove arbalas exterior angle prove \r\n\r\nlet say we have a triangle with angle measures a b and c.\r\n\r\nWe know that the exterior angle is equal two the sum of the two interior angles so\r\n\r\nexterior a=b+c\r\nexterior b=a+c\r\nexterior c=a+b \r\n\r\nThe sum is 2a+2b+2c and since a+b+c=180 the sum of the exterior angles ina triangle is 360 degrees",
"Solution_7": "random proof that if two lines have consecutive interior angles(the ones created by the transversal) that add up to 180, then the lines are parallel: Contradiction- Draw the lines such that they intersect. This creates a triangle with a zero degree angle, which is impossible so we are done.",
"Solution_8": "you guys can also post formulas also and theorms.",
"Solution_9": "here's one of the most useful formulas I know:\r\n\r\nThe Pythagorean Theorem states that in a right triangle, where a and b are its legs and c is its hypotenuse, a^2+b^2=c^2\r\n\r\nlol. So basic yet so helpful. :)"
}
{
"Tag": [
"geometry unsolved",
"geometry"
],
"Problem": "Given a circle $ k$ and a point $ P$ outside it, an arbitrary line $ s$ passing through $ P$ intersects $ k$ at the points $ A,B$. Let $ M,N$ be the midpoints of the arcs determined by the points $ A,B$. Let $ C$ be the point on $ AB$ such that $ PC^2\\equal{}PA\\cdot PB$. Prove that $ \\angle MCN$ doesn't depend on the choice of $ s$.",
"Solution_1": "[img]http://i006.radikal.ru/0804/04/4c243cc021f5.jpg[/img]\r\nLet $ K,L \\in k$, s.t. $ PK$ and $ PL$ are tangent to $ k$. Then we have $ PK^2 \\equal{} PL^2 \\equal{} PA\\cdot PB \\equal{} PC^2$, so $ PK \\equal{} PC \\equal{} PL$, so $ C$ always lie on the circle $ t$ with radius $ PK$. Let $ \\angle{CKL} \\equal{} \\alpha$, then $ \\angle{BPL} \\equal{} 2\\alpha$. But $ 2\\alpha \\equal{} \\angle{BPL} \\equal{} \\dfrac{\\breve{BL} \\minus{} \\breve{AL}}{2} \\equal{} \\dfrac{\\breve{BM} \\plus{} \\breve{ML} \\minus{} \\breve{AL}}{2} \\equal{} \\dfrac{\\breve{LA} \\plus{} 2\\breve{ML} \\minus{} \\breve{AL}}{2} \\equal{} \\breve{ML}$\r\nso $ \\angle{MKL} \\equal{} \\dfrac{\\breve{ML}}{2} \\equal{} \\alpha$ and $ \\angle{CKM} \\equal{} \\alpha \\minus{} \\alpha \\equal{} 0$, so $ K,C,M$ are collinear.\r\n Let $ \\angle{CLK} \\equal{} \\beta$, then $ \\angle{BPK} \\equal{} 2\\beta$. But $ 2\\beta \\equal{} \\angle{BPK} \\equal{} \\dfrac{\\breve{BK} \\minus{} \\breve{AK}}{2} \\equal{} \\dfrac{\\breve{BN} \\plus{} \\breve{KN} \\minus{} \\breve{AK}}{2} \\equal{} \\dfrac{\\breve{AK} \\plus{} 2\\breve{NK} \\minus{} \\breve{AK}}{2} \\equal{} \\breve{NK}$\r\nso $ \\angle{NLK} \\equal{} \\dfrac{\\breve{NK}}{2} \\equal{} \\beta$ and $ \\angle{CLN} \\equal{} \\beta \\minus{} \\beta \\equal{} 0$, so $ N,C,L$ are collinear.\r\n\r\nBecause $ P$ and $ k$ are constant, $ \\angle{KPL} \\equal{} const$. But $ \\angle{MCN} \\equal{} \\angle{KCL} \\equal{} 180^\\circ \\minus{} \\dfrac{\\angle{KPL}}{2} \\equal{} const.$, q.e.d.",
"Solution_2": "Thank [color=darkblue][b]Ahiles[/b][/color] for nice solution."
}
{
"Tag": [
"ARML",
"algebra",
"polynomial",
"geometry",
"circumcircle",
"LaTeX",
"inequalities"
],
"Problem": "Post ARML problems/solutions to previous ARML problems. To make the difficulty more varried, I suggest we post easier problems (numbers 1-3) first, then moderate difficulty (4,5), then the harder problems (6-8). The proccess starts over every 10, so just look at the units digit to determine the difficulty of the problem.\r\n\r\n[color=blue][size=200]PROBLEM 1[/size][/color]\r\nSource: 2000 problem 1\r\n\r\nFor $1 0$, $j, k > 0$.\n\nTherefore, $c-a = j^{2}-k^{2}= (j+k)(j-k) = 25 \\cdot 1$.\n\n(Since j, k are positive, 5*5 wouldn't work since k would be 0)\n\nTherefore, $j=13$, $k=12$.\n\nSo, \\[a=169, b=13^{3}, c=144, d=1728\\] (too lazy to calculate $13^{3}$...) :lol: \n\n[/hide]\r\n\r\nAgain, I have no problems to give :( \r\n\r\nMaybe I should check my ARML book....",
"Solution_13": "I feel like such a fool\u2026 The problem above was not an ARML problem (sorry :blush: ). \r\nAnd I have never taken any national or state math tests. Therefore, I did not make it to the ARML nationals. :noo: \r\n\r\nOnce again I am sorry for my ignorance and any deceptiveness I may have portrayed on my part. I am just a teenage who likes math.",
"Solution_14": "[quote=\"Imperial Effect\"]I feel like such a fool\u2026 The problem above was not an ARML problem (sorry :blush: ). \nAnd I have never taken any national or state math tests. Therefore, I did not make it to the ARML nationals. :noo: \n\nOnce again I am sorry for my ignorance and any deceptiveness I may have portrayed on my part. I am just a teenage who likes math.[/quote]\r\n\r\ner what it is all ok this is just a forum for like hanging out and stuff",
"Solution_15": "*ahem* any idea? :oops: \r\n\r\nanyway I have an easy inequality...\r\n\r\n[b]Problem 22[/b]\r\n\r\n$\\frac{1}{x}+\\frac{1}{y}+\\frac{1}{z}=1 \\Rightarrow x^{2}+y^{2}+z^{2}\\ge 27$",
"Solution_16": "Let $a=\\frac{1}{x}, b=\\frac{1}{y},$ and $c=\\frac{1}{z}$.\r\n\r\nNow, $f(x)=\\frac{1}{x^{2}}$ is convex in (0,infinity), so\r\n\r\n$f(a)+f(b)+f(c) \\ge 3 \\cdot f(\\frac{a+b+c}{3})=3 \\cdot f(1/3)=27$",
"Solution_17": "For the people here who aren't familiar with Jensen's (or, for that matter, convexity), QM-HM is probably a more illustrative proof. Yes, I know it is a special case of Jensen's...\r\n\r\n[hide=\":weightlift:\"]$\\sqrt{\\frac{x^{2}+y^{2}+z^{2}}3}\\ge \\frac 3{\\frac{1}{x}+\\frac{1}{y}+\\frac{1}{z}}=3$, from which the result follows immediately.[/hide]\r\nYou got to it first, Jeremy, so I grant you the burden of posting the next problem. ARML marathon, right? :roll:",
"Solution_18": "Ok, while we are on the subject:\r\n\r\nLet $a,b,c>0$ be such that $a+b+c=1$\r\n\r\nProve that $a^{b}\\cdot b^{c}\\cdot c^{a}\\le ab+bc+ca$\r\n\r\nFor those who want a more ARML-style problem, compute the sum (from k=1 to infinity) of\r\n\r\n$\\frac{F_{k}}{2^{k}}$\r\n\r\nwhere $F_{1}=1$, $F_{2}=1$, and the $F_{k}$ are the Fibonacci numbers",
"Solution_19": "[quote=\"n^4+4\"]Ok, while we are on the subject:\n\nLet $a,b,c>0$ be such that $a+b+c=1$\n\nProve that $a^{b}\\cdot b^{c}\\cdot c^{a}\\le ab+bc+ca$[/quote]\r\n[hide=\" :rotfl: \"]It's AM-GM on $a,b,c$ with weights $b,c,a$.[/hide]\r\nI think your second problem is harder :P",
"Solution_20": "[quote=\"not_trig\"]*ahem* any idea? :oops: \n\nanyway I have an easy inequality...\n\n[b]Problem 22[/b]\n\n$\\frac{1}{x}+\\frac{1}{y}+\\frac{1}{z}=1 \\Rightarrow x^{2}+y^{2}+z^{2}\\ge 27$[/quote]\r\n\r\nwell my solution used AM-GM and GM-HM:\r\n\r\n$\\sqrt[3]{xyz}\\ge\\frac{3}{\\frac{1}{x}+\\frac{1}{y}+\\frac{1}{z}}=3$; $\\frac{1}{x}+\\frac{1}{y}+\\frac{1}{z}\\Leftrightarrow xyz=xy+yz+xz \\Rightarrow xy+yz+yz\\ge27$\r\n\r\nnow $x^{2}+y^{2}+z^{2}\\ge xy+yz+xz = 27$\r\n\r\nEDIT: BERMAN YOU FOXY PERSON YOU KEEP USING SMILEYS AS HIDE TITLES",
"Solution_21": "@boy soprano: out of curiosity, why did you (falsely) change the inequality to be strict?\r\n\r\nActually, I think the second problem is easier... I guess I'm just weird like that. Anyhow, it's pretty cool, so it deserves more than the workout smiley.\r\n\r\n[hide=\":starwars:\"]S=1/2+1/4+2/8+3/16+5/32+...\n[u]2S=1+1/2+2/4+3/8+5/16+8/32+...[/u]\nS+2S=1+2/2+3/4+5/8+8/16+13/32+...\nBut just by quadrupling the original:\n4S=3+2/2+3/4+5/8+8/16+13/32+...\nSubtracting, S=2.[/hide]",
"Solution_22": "erm i have another problem:\r\n\r\nFind $a-b$ if $a^{2}-b^{2}=45$ adn $a^{3}-b^{3}=945$, $a, b \\in \\mathbb{Z}$.\r\n\r\nSure, this was #2 on our comprehensive regional, but even Vivek skipped it...",
"Solution_23": "That problem was really easy to cheat on (i.e. just check every one of the 5 answer choices they gave us)\r\n\r\nAs it is I see nothing better than working through about 10 cases",
"Solution_24": "[quote=\"jb05\"]@boy soprano: out of curiosity, why did you (falsely) change the inequality to be strict?[/quote]\r\nWhat do you mean?\r\n\r\n$\\frac{1}{3}^{1/3}\\frac{1}{3}^{1/3}\\frac{1}{3}^{1/3}= \\frac{1}{3}\\cdot\\frac{1}{3}+\\frac{1}{3}\\cdot\\frac{1}{3}+\\frac{1}{3}\\cdot\\frac{1}{3}= \\frac{1}{3}$?",
"Solution_25": "When you quoted it, you changed <= to <.\r\n\r\nThis new problem looks disgusting. Who's up for trying all 12 cases? :wink:",
"Solution_26": "[quote=\"not_trig\"]erm i have another problem:\n\nFind $a-b$ if $a^{2}-b^{2}=45$ adn $a^{3}-b^{3}=945$, $a, b \\in \\mathbb{Z}$.\n\nSure, this was #2 on our comprehensive regional, but even Vivek skipped it...[/quote]\r\n\r\n[hide]The key part that makes it easy is the $a,b\\in\\mathbb{Z}$.\n\n$a^{2}-b^{2}=45$\n\n$(a+b)(a-b)=3^{2}\\cdot5$\n\nThere are only 3 pairs of solutions $(a^{2},b^{2})$ that solve this; $(23^{2},22^{2})$, $(9^{2},6^{2})$, and $(7^{2},2^{2})$.\n\n$(23^{2},22^{2})$ can be thrown out because $23^{3}-22^{3}$ is way greater than 945.\n\n$(9^{2},6^{2})$ produces $(\\pm9,\\pm6)$, which could only possibly be true if $9^{3}+6^{3}=945$, and it does.\n\nThus, $a-b=3$.[/hide]",
"Solution_27": "[quote=\"jb05\"]When you quoted it, you changed <= to <.\n\nThis new problem looks disgusting. Who's up for trying all 12 cases? :wink:[/quote]\nOh, no, I think it just looks like that because the bottom of the quote box makes it hard to see the equality line. Compare:\n[quote]$a^{b}\\cdot b^{c}\\cdot c^{a}\\le ab+bc+ca$[/quote]\n[quote]\n$a^{b}\\cdot b^{c}\\cdot c^{a}\\le ab+bc+ca$\n\n[/quote]\r\n\r\n(And in my previous post, I forgot what \"strict\" meant : I thought strict => sharp => equality included :roll: )",
"Solution_28": "The second problem is just plugging 1/2 into the generating function, is it not? So it would be $\\frac{1/2}{1-1/2-1/3}= 2$",
"Solution_29": "[quote=\"b-flat\"][quote=\"not_trig\"]erm i have another problem:\n\nFind $a-b$ if $a^{2}-b^{2}=45$ adn $a^{3}-b^{3}=945$, $a, b \\in \\mathbb{Z}$.\n\nSure, this was #2 on our comprehensive regional, but even Vivek skipped it...[/quote]\n\n[hide]The key part that makes it easy is the $a,b\\in\\mathbb{Z}$.\n\n$a^{2}-b^{2}=45$\n\n$(a+b)(a-b)=3^{2}\\cdot5$\n\nThere are only 3 pairs of solutions $(a^{2},b^{2})$ that solve this; $(23^{2},22^{2})$, $(9^{2},6^{2})$, and $(7^{2},2^{2})$.\n\n$(23^{2},22^{2})$ can be thrown out because $23^{3}-22^{3}$ is way greater than 945.\n\n$(9^{2},6^{2})$ produces $(\\pm9,\\pm6)$, which could only possibly be true if $9^{3}+6^{3}=945$, and it does.\n\nThus, $a-b=3$.[/hide][/quote]\r\n\r\nlol i think \"use calculator\" would be a slightly easier solution, but this is good too (kinda lucky that $9^{3}+6^{3}=945$)"
}
{
"Tag": [
"inequalities",
"rearrangement inequality"
],
"Problem": "ARGH, I have spent ages trying to prove this.\r\n\r\n\r\nab + bc + ca \u2264 $a^2 + b ^2 + c^2$\r\n\r\nfor all real non-zero numbers a,b,c\r\n\r\nI just cant do it, im missing something simple.\r\n\r\nYour help is appreciated.\r\n\r\nThanks",
"Solution_1": "Have you heard of the rearrangement inequality?\r\n\r\nWell first assume WLOG that $a \\ge b \\ge c$. Then by rearrangement, this is trivial.",
"Solution_2": "Actually, an easier proof for you would be to multiply 2 on both sides, and then subract the $2ab+2bc+2ac$ terms over. Factor into squares, and then you're done.",
"Solution_3": "$(a-b)^2 + (b-c)^2 + (c-a)^2 \\geq 0$\r\nBut you should learn \"rearrangement inequality\" too. It's usefull in BW :)",
"Solution_4": "WLOG, assume $a \\geq b \\geq c$, then: $a \\cdot a + b \\cdot b + c \\cdot c \\geq a\\cdot b + b \\cdot c + c \\cdot a$.\r\n\r\nBTW, what is BW?",
"Solution_5": "Probably the Baltic Way competition? Am I right?",
"Solution_6": "Another solution:$a^2+b^2+c^2-ab-bc-ca=\\left(a-\\frac{b+c}{2}\\right)^2+\\frac{3}{4}(b-c)^2\\geq 0$",
"Solution_7": "Joining in flogging ...\r\nif (x,y,z) are absolute values of (a,b,c), and thus non-negative, by AM-GM\r\nwe get $x^3+y^3+z^3 - 3 xyz \\ge 0$ \r\nor $(x+y+z)(x^2+y^2+z^2 -xy -yz-zx) \\ge 0$\r\nsince $x+y+z \\ge 0$ we have $a^2+b^2+c^2 \\ge xy+yz+zx$ but $xy \\ge ab $ etc.. so you get what you want..."
}
{
"Tag": [
"geometry",
"inequalities",
"AMC",
"AIME",
"blogs",
"USA(J)MO",
"USAMO",
"\\/closed"
],
"Problem": "I heard that the Olympiad classes (not WOOT, of course):\r\n\r\n- Olympiad Problem Solving\r\n- Olympiad Geometry\r\n- Olympiad Inequalities\r\n\r\nare going to be cancelled. Is this true? :noo: :(\r\n\r\nOf course, I may be wrong, and this might just be a whole rumour that's spreading around. If so, I completely apologize and someone can just delete this post. But please say whether the Olympiad classes are going to be continued or not, because I can't afford to pay for WOOT.",
"Solution_1": "According to the classes information under \"Online School,\" Olympiad Problem Solving will occur during Fall 2006 and Olympiad Geometry and Inequalities will occur during Winter 2006/7.",
"Solution_2": "We have decided to discontinue the Olympiad classes in order to be able to spend more time on other projects. We strongly encourage students interested in the Olympiad classes to participate in WOOT. In order to facilitate this, we have added any early-bird discount on the WOOT price. Students who sign up by July 15 will receive a 100 dollar discount, as now reflected on the [url=http://www.artofproblemsolving.com/Classes/AoPS_C_Enroll.php]Enrollment page[/url].",
"Solution_3": "WOOT! I think I will probably sign up for WOOT now! I just have to convince my parents, lol :)",
"Solution_4": "So are the Olympiad classes the only ones being discontinued (of course along with the independent study)? Will there be any options besides WOOT for those who were planning to take an olympiad class? I guess those new books are coming along then.",
"Solution_5": "Does this mean that the classes already planned for this year will be cancelled? (EDIT: Nvm, I see that they've been taken off the classes page.)\r\n\r\nAlso, I won't have the time or money next school year to do WOOT, but I would have enough for olympiad classes, does anyone have any recommendations as to what I should do? I'm considering taking another AIME problem series, doing an intermediate class, though for some of those classes there would be a lot of review (especially seeing as I plan to go through AoPS 2), possibly trying WOOT anyway, or even not taking an AoPS classes.",
"Solution_6": "While I understand the concerns of those of you who would like to take the old Olympiad classes, I ask that you understand our position as well. We've been heavily subsidizing the school for a very long time (and will still be subsidizing WOOT). In order to continue to be able to produce any resources at all, we have to focus more of our time on properly priced resources, instead of spending 50-60% of our time on resources we give away either for free or for a very, very low price. I've posted some of my thoughts about this in my blog, [url=http://www.artofproblemsolving.com/Forum/weblog_entry.php?e=6098]here[/url].",
"Solution_7": "Thankyou for your response on pricing.\r\n\r\nHowever, assuming that I can afford it, I'm also worried about time, seeing as I'll have a particularly busy sophmore (next) year (I have been planning to do WOOT in junior year). How much time does one have to spend on WOOT? Could I still get as much from WOOT if I didn't spend as much time as I should on it? Or would it be recommended that I do something lighter next year(thinking of AIME Problem Series B as well as A, which I'm doing over the summer)?",
"Solution_8": "[quote=\"calc rulz\"]Thankyou for your response on pricing.\n\nHowever, assuming that I can afford it, I'm also worried about time, seeing as I'll have a particularly busy sophmore year. How much time does one have to spend on WOOT? Could I still get as much from WOOT if I didn't spend as much time as I should on it? Or would it be recommended that I do something lighter next year(thinking of AIME Problem Series B as well as A, which I'm doing over the summer)?[/quote]\r\n\r\nI think the answer to this depends on how well you can budget your time. WOOT is somewhat more spread out than our other classes. As I remember from your earlier postings in the AMC area, you are already pretty proficient with the AIME. Due to the USAMO expansion, you'll also have some more breathing room. I'd suggest you start spending more of your time with the more challenging material. In other words, I'd recommend *not* taking the AIME PS B and focusing more of your efforts on Olympiad material. If you dive into AIME PS A over the summer, and spend some time with books like Zeitz and such, you should put yourself in a good position with respect to the AIME in 2007. (I'm assuming you're on your school's math team and will have plenty of opportunity to stay sharp for the AMC.)\r\n\r\nOne of our goals in expanding the USAMO was to get more students to spend some time working on olympiad-style problems. Far too many would spend far too much time cramming away for the AIME. I would guess that at least half the students who barely qualified for the AIME (if not more) had never tried an olympiad problem before qualifying for the USAMO. We'd like to change that, and encourage students to start developing problem solving stamina, writing skills, and the courage to attack problems that appear impossible.",
"Solution_9": "Very understandable. Okay. Well, one final question that's been bugging me though: Why was inversion taught in the olympiad geometry class? To my understanding [url=http://olympiads.win.tue.nl/imo/]inversion isn't tested at the IMO[/url] but [url=http://en.wikipedia.org/wiki/Mathematical_Olympiad_Program]Wikipedia says[/url] MOP poses questions about it. Is it just for mathematical enrichment, or can it be useful in some problems although it's not required? Thanks.\r\n\r\nP.S. The \"Naming controvesy\" article on wikipedia is quite humorous",
"Solution_10": "Inversion was taught for one of the reasons you surmise - it can be extremely useful, but it's not ever required. (In fact, this is true of a lot of the advanced techniques in all areas of Olympiad problem solving.)\r\n\r\nI also taught it because it's way cool stuff.",
"Solution_11": "Mr. Rusczyk,\r\n\r\nAlthough I understand the cost issue in cancelling the Olympiad classes, it is a loss nevertheless for students. I have no doubt WOOT is a great course, but it will not go as deep as the Olympiad subject classes therefore it is not really able to replace the Olympiad classes. But again I understand you have no choice. \r\n\r\nBut I have an idea that may benefit both students and AoPS. Are you willing or able to sell transcritpts of past Olympiad classes (have you nicely archived those materials?)? Students purchasing those transcripts can study on their own pace. It may not benefit as much as actually participating in the classes, but it should be next to the best. I have no idea how many students will be willing to buy, but I will be willing to pay for it. \r\n\r\nThanks,\r\n\r\nKevin Yuan (I'm using my son Allen's account to post this)",
"Solution_12": "We will be writing Olympiad books in the coming years, and there will be beginning Olympiad material in the Intermediate texts, the first two of which will be out within a year. Putting the transcripts in a format (and configuring the ordering system, etc) that would allow selling those is unfortunately too time-consuming to merit what they would produce. We will be drawing on the highlights of these for WOOT, and I expect the entirety of a few years' WOOT participation will be much broader and deeper than the Olympiad classes were.",
"Solution_13": "Is there a chance that they will come out in early 2007 or late 2006? :D",
"Solution_14": "I am very sorry that this is a little bit off topic and if this has been announced before, although what is the date when the Intro to Number Theory book is going to be released? :?",
"Solution_15": "[quote=\"Knuth's_check\"]I am very sorry that this is a little bit off topic and if this has been announced before, although what is the date when the Intro to Number Theory book is going to be released? :?[/quote]\r\n\r\nAround the 27th of this month.",
"Solution_16": "[quote=\"rrusczyk\"]We will be writing Olympiad books in the coming years, and there will be beginning Olympiad material in the Intermediate texts, the first two of which will be out within a year. Putting the transcripts in a format (and configuring the ordering system, etc) that would allow selling those is unfortunately too time-consuming to merit what they would produce. We will be drawing on the highlights of these for WOOT, and I expect the entirety of a few years' WOOT participation will be much broader and deeper than the Olympiad classes were.[/quote]\r\n\r\nThen would AoPS mind if we tried to get past transcripts/problems from people who took the courses when they were still offered?",
"Solution_17": "No, we can't encourage that. We may bring the olympiad classes back some day, and we'll be drawing on some of that material for WOOT. Moreover, we cannot encourage any sort of file sharing of our online school material, as it jeopardizes the future of the online school (and has been a contributor to our discontinuing the olympiad classes for now).",
"Solution_18": "That's perfectly reasonable, of course. Looks like this generation of problem solvers will have to make do without them :(",
"Solution_19": "Speaking of books and classes and the like, what are the next books that are going to come out?",
"Solution_20": "[quote=\"horselvr101\"]Speaking of books and classes and the like, what are the next books that are going to come out?[/quote]Introduction to Algebra and Intermediate Counting & Probability should come out in early 2007. Intermediate Number Theory should come out in Spring 2007.",
"Solution_21": "[quote=\"DPatrick\"][quote=\"horselvr101\"]Speaking of books and classes and the like, what are the next books that are going to come out?[/quote]Introduction to Algebra and Intermediate Counting & Probability should come out in early 2007. Intermediate Number Theory should come out in Spring 2007.[/quote]\r\nWhat exactly will the intro to algebra include?",
"Solution_22": "Basically all of algebra I, much of the algebraic portion of pre-algebra, and a healthy chunk of algebra II. \r\n\r\nLinear equations, quadratics, functions, word problems, proportions, special factorizations, arithmetic and geometric sequences and series, radicals, exponentials, plenty of basic analytic geometry.",
"Solution_23": "What about Intermediate Number Theory? I just took the Intro to Number Theory course and I was wondering where I should go next to continue studying that area...",
"Solution_24": "[quote=\"horselvr101\"]What about Intermediate Number Theory? I just took the Intro to Number Theory course and I was wondering where I should go next to continue studying that area...[/quote]\r\n\r\nIt'll be a while before we have an Interm. Number Theory book. The class will probably be offered again next summer.",
"Solution_25": "Will Interm Algebra come out before or after the class starts?",
"Solution_26": "Are the olympiad classes ever going to come back or are they gone for good?\r\nWhat is the tentative schedule for books to come out this year?\r\n\r\n\r\n-bpms",
"Solution_27": "[quote=\"bpms\"]Are the olympiad classes ever going to come back or are they gone for good?\nWhat is the tentative schedule for books to come out this year?\n[/quote]\r\n\r\nThe olympiad classes are closed for the foreseeable future. Some of the material will be folded into WOOT.\r\n\r\nTentative books schedule: Intro Algebra March, Interm Counting May, Interm Algebra June."
}
{
"Tag": [
"geometry",
"3D geometry"
],
"Problem": "I've come up with the following conjecture but I'm not sure how to prove it. Take a pre-solved Rubik's cube (normal 3x3x3) and perform any number of moves. Prove that for some number n, you can iterate that sequence n times such that the cube will end in a solved position. I've tried this for several sequences and so far I haven't found one that doesn't solve in less than 200 (note the sequences I've been doing are at most 4-5 moves and not very complicated). If anyone has a program simulating a cube I'd appreciate the source. I don't know too much about orbitals so if they're used to approach this problem, please tell me.",
"Solution_1": "one i know off top of head; R U' repeated 64 times; 128 moves.",
"Solution_2": "Let a move be any given sequence of what you normally call a move.\r\n\r\nThe order of the rubik-group is finite, so you will ever reach the solved position (=neutral element). You can even show (by Lagranges theorem) that the number of moves you did is a divisor of the total number of different positions possible.\r\n\r\nTo show it more elementary: you can't get a new constellation everytime after doing the move, since that would mean that the cube has $\\infty$ many positions, which is clearly not to case. So after some time you get a position you already had before, but what is the difference between that position and a full solved cube after some recoloring\u00bf Nothing! So you will ever reach the position you started with (solved one) again.",
"Solution_3": "Its not too hard to find a maximal period for your re-iterations. The something about it in the strategy section"
}
{
"Tag": [
"algebra",
"polynomial"
],
"Problem": "find square root of:-\r\n\r\n$11+2\\sqrt{7}+\\sqrt{12}+\\sqrt{84}$",
"Solution_1": "[hide=\"Solution\"]\\begin{eqnarray*}\\sqrt{11+2\\sqrt{7}+\\sqrt{12}+\\sqrt{84}} &=& \\sqrt{11+2\\sqrt{7}+2\\sqrt{3}+2\\sqrt{21}}\\\\ &=& \\sqrt{1^2+\\sqrt{3}^2+\\sqrt{7}^2+2\\cdot 1\\cdot\\sqrt{7}+2\\cdot 1\\cdot\\sqrt{3}+2\\cdot\\sqrt{3}\\cdot\\sqrt{7}}\\\\ &=& \\sqrt{(1+\\sqrt{3}+\\sqrt{7})^2}\\\\ &=& 1+\\sqrt{3}+\\sqrt{7}\\end{eqnarray*}[/hide]",
"Solution_2": "farenhajt , in that, how did you get to know that it would be consisiting of 3 terms , 1, sqrt3 and sqrt7 ? i mean if it was a much more complex expression then wouldnt it be hard to get to know the terms or to split them like you did?I mean isnt their some sure-shot way like using variables that you can get the answer?",
"Solution_3": "Well, I think there's no general formula, since the expression must have special properties. In this example it's really obvious that we have $2\\sqrt{3}, 2\\sqrt{7}$ and $2\\sqrt{3}\\cdot\\sqrt{7}$, and that points to a square-of-a-trinomial formula. Once you know where to go, the rest is easy. And of course, you must have working knowledge of polynomial factorization, in order to recognize the patterns."
}
{
"Tag": [
"probability"
],
"Problem": "not sure where to post this people told me this was one of the good books on glambling and stuff. If anybody has it le me know what you think of it.",
"Solution_1": "gambling???????",
"Solution_2": "[quote=\"dreambold22\"]gambling???????[/quote]\r\n\r\nwhy not",
"Solution_3": "i'm not interested in this topic. good bye.",
"Solution_4": "[quote=\"Nick31\"][quote=\"dreambold22\"]gambling???????[/quote]\n\nwhy not[/quote]\r\n\r\nErm..... :what?: :what?: :what?:",
"Solution_5": "Is it a probability book? :|",
"Solution_6": "gambling??????? [color=white][size=0]lol copy/paste[/size][/color]",
"Solution_7": "[quote=\"leoxnlin\"]Is it a probability book? :|[/quote]\r\n\r\nprobabilities applied to various card games, but I don't have the book so I can't tell exactly"
}
{
"Tag": [
"trigonometry",
"geometry proposed",
"geometry"
],
"Problem": "Given quadrilateral $ ABCD$, $ AC\\cap BD=O$. Prove that $ ABCD$ is cyclic iff $ OA\\sin A+OC\\sin C=OB\\sin B+OD\\sin D$.",
"Solution_1": "http://www.mathlinks.ro/Forum/viewtopic.php?t=26474\r\nSee, for the solution,\r\nhttp://www.mathlinks.ro/Forum/viewtopic.php?t=15584 post #2 file IMO_1997_shortlist.pdf or\r\nhttp://www.kalva.demon.co.uk/short/soln/sh9723.html\r\n\r\n darij"
}
{
"Tag": [
"inequalities proposed",
"inequalities"
],
"Problem": "Let $ x_1,x_2,...,x_n$ be real numbers such that $ x_i > 1$ for $ i \\equal{} 1,2,...,n$. Prove that\r\n\\[ \\frac {x_1x_2}{x_3 \\minus{} 1} \\plus{} \\frac {x_2x_3}{x_4 \\minus{} 1} \\plus{} ... \\plus{} \\frac {x_nx_1}{x_2 \\minus{} 1} \\ge 4n.\r\n\\] Determine when the equality occurs.",
"Solution_1": "I am so bored today!\r\nNow, we have\r\n$ \\sum_{i \\equal{} 1}^n \\frac {x_ix_{i \\plus{} 1}}{x_{i \\plus{} 2} \\minus{} 1} \\geq 4\\frac {x_ix_{i \\plus{} 1}}{x_{i \\plus{} 2}^2}$\r\nwith the notations are $ n \\plus{} 1 \\equal{} 1$ and $ n \\plus{} 2 \\equal{} 2$\r\nIt is mere to prove that\r\n$ \\sum_{i \\equal{} 1}^n \\frac {x_ix_{i \\plus{} 1}}{x_{i \\plus{} 2}^2} \\geq n$ which is true by the general form of Cauchy in my signature!! :D"
}
{
"Tag": [
"inequalities",
"ratio",
"geometry",
"algebra",
"polynomial",
"circumcircle",
"function"
],
"Problem": "Last week, the Nordic Mathematical Contest was held. Here is the problem-set.\r\n\r\n[url=http://www.mathlinks.ro/Forum/viewtopic.php?p=206646#p206646]Problem 1 - link[/url]\r\nFind all positive integers $k$ such that the product of the digits of $k$, in decimal notation, equals \\[\\frac{25}{8}k-211\\]\r\n\r\n[url=http://www.mathlinks.ro/Forum/viewtopic.php?p=206634#p206634]Problem 2 - link[/url]\r\nLet $a,b,c$ be positive real numbers. Prove that \\[\\frac{2a^2}{b+c} + \\frac{2b^2}{c+a} + \\frac{2c^2}{a+b} \\geq a+b+c\\]\r\n\r\n[url=http://www.mathlinks.ro/Forum/viewtopic.php?p=206644#p206644]Problem 3 - link[/url]\r\nThere are $2005$ young people sitting around a large circular table. Of these, at most $668$ are boys. We say that a girl $G$ has a strong position, if, counting from $G$ in either direction, the number of girls is always strictly larger than the number of boys ($G$ is herself included in the count). Prove that there is always a girl in a strong position. \r\n\r\n[url=http://www.mathlinks.ro/Forum/viewtopic.php?p=206638#p206638]Problem 4 - link[/url]\r\nThe circle $\\zeta_{1}$ is inside the circle $\\zeta_{2}$, and the circles touch each other at $A$. A line through $A$ intersects $\\zeta_{1}$ also at $B$, and $\\zeta_{2}$ also at $C$. The tangent to $\\zeta_{1}$ at $B$ intersects $\\zeta_{2}$ at $D$ and $E$. The tangents of $\\zeta_{1}$ passing thorugh $C$ touch $\\zeta_{2}$ at $F$ and $G$. Prove that $D$, $E$, $F$ and $G$ are concyclic.",
"Solution_1": "Can you say please how much time is allowed ?\r\nP.S. the inequality seems to be very easy :lol:",
"Solution_2": "4 hours.\r\n\r\nYes, the inequality is very easy!",
"Solution_3": "Does anyone has recent years' NMO problems?",
"Solution_4": "Arccosinus, were you at Baltic Way 2004? I might know you from there ;)",
"Solution_5": "No portoseb. I wanted to go, but I gave my place in the team to an alternate, because I had to go to a wedding in the U.S.",
"Solution_6": "I saw the results today, at the Finnish IMO selection in which I take part. I was the first with 18 points, so nobody got full points. As soon as the problems will be put on the internet I will let you know.\r\n\r\nEDIT: here they are:\r\n\r\nhttp://www.math.helsinki.fi/~smy/olympia/PM/NMC19.htm",
"Solution_7": "19th Nordic Mathematical Contest \t\t\t\t\t\r\nResults\t\t\t\t\t\t\r\n\t\t\t\t\t\t\t\r\n\t\t\tProblem\tSum\r\n\t\t\t1\t2\t3\t4\t\u3000\r\n1.\tDumitrescu, Sebastian\tFIN\t5\t4\t5\t4\t18\r\n2.\tAlanko, Samu\tFIN\t5\t5\t5\t0\t15\r\n\u3000\tPettersson, Ville\tFIN\t5\t5\t5\t0\t15\r\n\u3000\tRennemo, J\u00f8rgen Vold\tNOR\t5\t5\t5\t0\t15\r\n\u3000\tStef\u00e1nsson, \u00d6rn\tICE\t5\t5\t5\t0\t15\r\n6.\tXing, Chen\tSWE\t4\t5\t5\t0\t14\r\n7.\tHalld\u00f3rsson, H\u00f6skuldur P\u00e9tur\tICE\t3\t5\t3\t2\t13\r\n\u3000\tTroeng, Olof\tSWE\t5\t3\t5\t0\t13\r\n9.\tJacobsen, Martin Wedel\tDEN\t5\t5\t0\t2\t12\r\n\u3000\tLiu, Zihan\tSWE\t5\t0\t2\t5\t12\r\n\u3000\tReeh, Sune Nikolaj Precht\tDEN\t5\t5\t0\t2\t12\r\n\u3000\tVass, Marton\tFIN\t5\t0\t5\t2\t12\r\n13.\tEdlund, Erik\tSWE\t5\t5\t1\t0\t11\r\n14.\tKolu, Iiris\tFIN\t5\t4\t1\t0\t10\r\n\u3000\tRudberg, Tore\tSWE\t5\t5\t0\t0\t10\r\n\u3000\tSigstad, Henrik\tNOR\t5\t4\t1\t0\t10\r\n\u3000\tSireni, Jukka\tFIN\t5\t5\t0\t0\t10\r\n\u3000\tTerelius, H\u00e5kan\tSWE\t5\t0\t5\t0\t10\r\n\u3000\tThelin, Sam\tSWE\t3\t5\t2\t0\t10\r\n20.\tHolden, Nina\tNOR\t4\t5\t0\t0\t9\r\n\u3000\tJohnsen, Hilde Galleberg\tNOR\t3\t5\t0\t1\t9\r\n\u3000\tJohnson, Kerstin\tSWE\t4\t5\t0\t0\t9\r\n\u3000\tNors Andersen, Drees\tDEN\t3\t4\t0\t2\t9\r\n24.\tEdlund, Bj\u00f6rn\tSWE\t3\t0\t5\t0\t8\r\n\u3000\tHelgad\u00f3ttir, Inga Steinunn\tICE\t3\t3\t0\t2\t8\r\n\u3000\tMeuller, Simon\tSWE\t0\t3\t5\t0\t8\r\n\u3000\tVesalainen, Esa\tFIN\t5\t3\t0\t0\t8\r\n\u3000\t\u00c5rdal, Atle Rygg\tNOR\t3\t5\t0\t0\t8\r\n29.\tAbrahamsen, Mikkel\tDEN\t4\t0\t2\t1\t7\r\n\u3000\tGrythe, Thomas Berge\tNOR\t2\t5\t0\t0\t7\r\n\u3000\tM\u00fcller, Jeppe Krarup\tDEN\t5\t2\t0\t0\t7\r\n32.\tBl\u00e5sten, Eemeli\tFIN\t0\t3\t3\t0\t6\r\n\u3000\tJacobsen, K\u00e5re\tDEN\t1\t5\t0\t0\t6\r\n\u3000\tKokkala, Janne\tFIN\t1\t5\t0\t0\t6\r\n\u3000\tPeng, Gunnar\tSWE\t1\t4\t0\t1\t6\r\n\u3000\tSiljander, Juhana\tFIN\t5\t0\t1\t0\t6\r\n\u3000\tWilson, Danny\tDEN\t5\t0\t0\t1\t6\r\n38.\tBrandt, Asbj\u00f8rn\tDEN\t5\t0\t0\t0\t5\r\n\u3000\tJensen, Morten Harding\tDEN\t5\t0\t0\t0\t5\r\n\u3000\tL\u00e4hteenm\u00e4ki, Meri\tFIN\t3\t0\t0\t2\t5\r\n\u3000\tM\u00e4ki, Sami\tFIN\t3\t0\t2\t0\t5\r\n\u3000\tPoulsen, Morten Kj\u00e6r\tDEN\t5\t0\t0\t0\t5\r\n\u3000\tSelin, M\u00e5rten\tSWE\t1\t4\t0\t0\t5\r\n\u3000\tSjamsutdin, Aslanbek\tNOR\t3\t0\t1\t1\t5\r\n\u3000\tSpars\u00f8, Eske\tDEN\t5\t0\t0\t0\t5\r\n46.\tEladhari, Matti\tSWE\t4\t0\t0\t0\t4\r\n\u3000\tGu\u00f0mundsson, J\u00f3n Emil\tICE\t3\t0\t0\t1\t4\r\n\u3000\tJohannesen, David\tDEN\t3\t0\t0\t1\t4\r\n\u3000\tLaaksonen, Antti\tFIN\t2\t0\t0\t2\t4\r\n\u3000\tMar\u00edusd\u00f3ttir, \u00deorey Mar\u00eda\tICE\t3\t0\t0\t1\t4\r\n\u3000\tT\u00f8rholm, Sebastian Paaske\tDEN\t3\t0\t0\t1\t4\r\n\u3000\tVester, Steen\tDEN\t4\t0\t0\t0\t4\r\n53.\tAlexanderson, Per\tSWE\t2\t0\t0\t1\t3\r\n\u3000\tEgilsd\u00f3ttir, Salv\u00f6r\tICE\t3\t0\t0\t0\t3\r\n\u3000\tErnvall, Toni\tFIN\t3\t0\t0\t0\t3\r\n\u3000\tGu\u00f0mundsd\u00f3ttir, Mar\u00eda Helga\tICE\t3\t0\t0\t0\t3\r\n\u3000\tGunnarsson, Einar Bjarki\tICE\t2\t0\t0\t1\t3\r\n\u3000\tGyring, Peter\tNOR\t3\t0\t0\t0\t3\r\n\u3000\tHelgason, Einar Axel\tICE\t2\t0\t0\t1\t3\r\n\u3000\tJensen, Ole S.\tDEN\t3\t0\t0\t0\t3\r\n\u3000\tNielsen, Emil Bundgaard\tDEN\t3\t0\t0\t0\t3\r\n\u3000\t\u00d3skarsd\u00f3ttir, Mar\u00eda\tICE\t3\t0\t0\t0\t3\r\n\u3000\tSigur\u00f0sson, Baldur\tICE\t2\t0\t1\t0\t3\r\n\u3000\tSigur\u00f0sson, Dan\u00edel \u00der\u00f6stur\tICE\t3\t0\t0\t0\t3\r\n\u3000\tSun, Lei\tSWE\t3\t0\t0\t0\t3\r\n\u3000\tYilong, Li\tFIN\t0\t3\t0\t0\t3\r\n67.\tJokinen, Kosti\tFIN\t0\t0\t0\t2\t2\r\n\u3000\tKlungre, Vidar\tNOR\t1\t0\t1\t0\t2\r\n\u3000\tLi, Kang\tDEN\t0\t0\t0\t2\t2\r\n\u3000\tMertaniemi, Henrikki\tFIN\t2\t0\t0\t0\t2\r\n\u3000\tNielsen, Kr\u00e6n Vodder\tDEN\t0\t0\t0\t2\t2\r\n\u3000\tNu\u00f1ez, Walter\tSWE\t1\t0\t1\t0\t2\r\n\u3000\tParnfelt, Johan\tSWE\t1\t0\t0\t1\t2\r\n\u3000\tRydberg, Elin\tSWE\t2\t0\t0\t0\t2\r\n\u3000\tStr\u00e5hlman, Christian\tSWE\t2\t0\t0\t0\t2\r\n\u3000\tSveinbj\u00f6rnsson, Benjamin Ragnar\tICE\t2\t0\t0\t0\t2\r\n\u3000\tSvensson, Mats\tSWE\t2\t0\t0\t0\t2\r\n\u3000\t\u00c5strand, Matti\tFIN\t0\t0\t0\t2\t2\r\n79.\tLarsson. Joel\tSWE\t1\t0\t0\t0\t1\r\n\u3000\tLode, P\u00e5l S\u00e6vik\tNOR\t0\t0\t1\t0\t1\r\n\u3000\tMarcher, Jette Louise\tDEN\t0\t0\t0\t1\t1\r\n\u3000\tRantanen, Mikko\tFIN\t1\t0\t0\t0\t1\r\n83.\tBerg, Justus\tSWE\t0\t0\t0\t0\t0\r\n\u3000\tKettunen, Markus\tFIN\t0\t0\t0\t0\t0\r\n\u3000\tLauridsen, Nis Brix\tDEN\t0\t0\t0\t0\t0",
"Solution_8": "[url=http://www.artofproblemsolving.com/Forum/resources.php?c=1&cid=253&year=2005]Added.[/url]",
"Solution_9": "[b]Nordic Mathematical Contest 2009[/b]\nApril 2, 2009\n\n[url=http://www.artofproblemsolving.com/Forum/viewtopic.php?p=3028413#p3028413]Problem 1[/url]\nA point $P$ is chosen in an arbitrary triangle. Three lines are drawn through $P$ which are parallel to the sides of the triangle. The lines divide the triangle into three smaller triangles and three parallelograms. Let $f$ be the ratio between the total area of the three smaller triangles and the area of the given triangle. Prove that $f\\ge\\frac{1}{3}$ and determine those points $P$ for which $f =\\frac{1}{3}$ .\n[url=http://www.artofproblemsolving.com/Forum/viewtopic.php?p=3028409#p3028409]Problem 2[/url]\nOn a faded piece of paper it is possible to read the following:\n\\[(x^2 + x + a)(x^{15}- \\cdots ) = x^{17} + x^{13} + x^5 - 90x^4 + x - 90.\\]\nSome parts have got lost, partly the constant term of the first factor of the left side, partly the majority of the summands of the second factor. It would be possible to restore the polynomial forming the other factor, but we restrict ourselves to asking the following question: What is the value of the constant term $a$? We assume that all polynomials in the statement have only integer coefficients.\n[url=http://www.artofproblemsolving.com/Forum/viewtopic.php?p=3028392#p3028392]Problem 3[/url]\nThe integers $1$, $2$, $3$, $4$, and $5$ are written on a blackboard. It is allowed to wipe out two integers $a$ and $b$ and replace them with $a + b$ and $ab$. Is it possible, by repeating this procedure, to reach a situation where three of the five integers on the blackboard are $2009$?\n[url=http://www.artofproblemsolving.com/Forum/viewtopic.php?p=3028405#p3028405]Problem 4[/url]\n$32$ competitors participate in a tournament. No two of them are equal and in a one against one mach the better always wins. Show that the gold, silver, and bronze medal winners can be found in $39$ matches.",
"Solution_10": "[b]Nordic Mathematical Contest 2004[/b]\nApril 1, 2004\n\n[url=http://www.artofproblemsolving.com/Forum/viewtopic.php?p=3028451#p3028451]Problem 1[/url]\nTwenty-seven balls labelled from $1$ to $27$ are distributed in three bowls: red, blue, and yellow. What are the possible values of the number of balls in the red bowl if the average labels in the red, blue and yellow bowl are $15$, $3$, and $18$, respectively?\n[url=http://www.artofproblemsolving.com/Forum/viewtopic.php?p=3028439#p3028439]Problem 2[/url]\nThe Fibonacci sequence is defined by $f_1 = 0, f_2 = 1$, and $f_{n+2} = f_{n+1}+f_n$ for $n\\ge 1$. Prove that there is a strictly increasing arithmetic progression whose no term is in the Fibonacci sequence.\n[url=http://www.artofproblemsolving.com/Forum/viewtopic.php?p=3028457#p3028457]Problem 3[/url]\nGiven a finite sequence $x_{1,1}, x_{2,1}, \\dots , x_{n,1}$ of integers $(n\\ge 2)$, not all equal, define the sequences $x_{1,k}, \\dots , x_{n,k}$ by\n\\[ x_{i,k+1}=\\frac{1}{2}(x_{i,k}+x_{i+1,k})\\quad\\text{where }x_{n+1,k}=x_{1,k}.\\]\nShow that if $n$ is odd, then not all $x_{j,k}$ are integers. Is this also true for even $n$?\n[url=http://www.artofproblemsolving.com/Forum/viewtopic.php?p=3028454#p3028454]Problem 4[/url]\nLet $a, b, c$ be the sides and $R$ be the circumradius of a triangle. Prove that\n\\[\\frac{1}{ab}+\\frac{1}{bc}+\\frac{1}{ca}\\ge\\frac{1}{R^2}.\\]",
"Solution_11": "[b]Nordic Mathematical Contest 2010[/b]\nApril 13, 2010\n\n[url=http://www.artofproblemsolving.com/Forum/viewtopic.php?p=3029723#p3029723]Problem 1[/url]\nA function $f : \\mathbb{Z}_+ \\to \\mathbb{Z}_+$, where $\\mathbb{Z}_+$ is the set of positive integers, is non-decreasing and satisfies $f(mn) = f(m)f(n)$ for all relatively prime positive integers $m$ and $n$. Prove that $f(8)f(13) \\ge (f(10))^2$.\n[url=http://www.artofproblemsolving.com/Forum/viewtopic.php?p=3029733#p3029733]Problem 2[/url]\nThree circles $\\Gamma_A$, $\\Gamma_B$ and $\\Gamma_C$ share a common point of intersection $O$. The other common point of $\\Gamma_A$ and $\\Gamma_B$ is $C$, that of $\\Gamma_A$ and $\\Gamma_C$ is $B$, and that of $\\Gamma_C$ and $\\Gamma_B$ is $A$. The line $AO$ intersects the circle $\\Gamma_A$ in the point $X \\ne O$. Similarly, the line $BO$ intersects the circle $\\Gamma_B$ in the point $Y \\ne O$, and the line $CO$ intersects the circle $\\Gamma_C$ in the point $Z \\ne O$. Show that \n\\[\\frac{|AY |\\cdot|BZ|\\cdot|CX|}{|AZ|\\cdot|BX|\\cdot|CY |}= 1.\\]\n[url=http://www.artofproblemsolving.com/Forum/viewtopic.php?p=3029748#p3029748]Problem 3[/url]\nLaura has $2010$ lamps connected with $2010$ buttons in front of her. For each button, she wants to know the corresponding lamp. In order to do this, she observes which lamps are lit when Richard presses a selection of buttons. (Not pressing anything is also a possible selection.) Richard always presses the buttons simultaneously, so the lamps are lit simultaneously, too.\na) If Richard chooses the buttons to be pressed, what is the maximum number of different combinations of buttons he can press until Laura can assign the buttons to the lamps correctly?\nb) Supposing that Laura will choose the combinations of buttons to be pressed, what is the minimum number of attempts she has to do until she is able to associate the buttons with the lamps in a correct way?\n[url=http://www.artofproblemsolving.com/Forum/viewtopic.php?p=3029757#p3029757]Problem 4[/url]\nA positive integer is called simple if its ordinary decimal representation consists entirely of zeroes and ones. Find the least positive integer $k$ such that each positive integer $n$ can be written as $n = a_1 \\pm a_2 \\pm a_3 \\pm \\cdots \\pm a_k$ where $a_1, \\dots , a_k$ are simple.",
"Solution_12": "2004, 2009, 2010 added; thanks for posting! :)",
"Solution_13": "[b]Nordic Mathematical Contest 2011[/b]\nApril 4, 2011\n\n[url=http://www.artofproblemsolving.com/Forum/viewtopic.php?p=3029937#p3029937]Problem 1[/url]\nWhen $a_0, a_1, \\dots , a_{1000}$ denote digits, can the sum of the $1001$-digit numbers $a_0a_1\\cdots a_{1000}$ and $a_{1000}a_{999}\\cdots a_0$ have odd digits only?\n[url=http://www.artofproblemsolving.com/Forum/viewtopic.php?p=3029944#p3029944]Problem 2[/url]\nIn a triangle $ABC$ assume $AB = AC$, and let $D$ and $E$ be points on the extension of segment $BA$ beyond $A$ and on the segment $BC$, respectively, such that the lines $CD$ and $AE$ are parallel. Prove $CD \u0015\\ge \\frac{4h}{BC}CE$, where $h$ is the height from $A$ in triangle $ABC$. When does equality hold?\n[url=http://www.artofproblemsolving.com/Forum/viewtopic.php?p=3029951#p3029951]Problem 3[/url]\nFind all functions $f$ such that\n\\[f(f(x) + y) = f(x^2-y) + 4yf(x)\\]\nfor all real numbers $x$ and $y$.\n[url=http://www.artofproblemsolving.com/Forum/viewtopic.php?p=3029970#p3029970]Problem 4[/url]\nShow that for any integer $n \\ge 2$ the sum of the fractions $\\frac{1}{ab}$, where $a$ and $b$ are relatively prime positive integers such that $a < b \\le n$ and $a+b > n$, equals $\\frac{1}{2}$.\n(Integers $a$ and $b$ are called relatively prime if the greatest common divisor of $a$ and $b$ is $1$.)",
"Solution_14": "[b]Nordic Mathematical Contest 2012[/b]\n27 March, 2012\n\n[url=http://www.artofproblemsolving.com/Forum/viewtopic.php?p=3029993#p3029993]Problem 1[/url]\nThe real numbers $a, b, c$ are such that $a^2 + b^2 = 2c^2$, and also such that $a \\ne b, c \\ne -a, c \\ne -b$. Show that\n\\[\\frac{(a+b+2c)(2a^2-b^2-c^2)}{(a-b)(a+c)(b+c)}\\]\nis an integer.\n[url=http://www.artofproblemsolving.com/Forum/viewtopic.php?p=3030023#p3030023]Problem 2[/url]\nGiven a triangle $ABC$, let $P$ lie on the circumcircle of the triangle and be the midpoint of the arc $BC$ which does not contain $A$. Draw a straight line $l$ through $P$ so that $l$ is parallel to $AB$. Denote by $k$ the circle which passes through $B$, and is tangent to $l$ at the point $P$. Let $Q$ be the second point of intersection of $k$ and the line $AB$ (if there is no second point of intersection, choose $Q = B$). Prove that $AQ = AC$.\n[url=http://www.artofproblemsolving.com/Forum/viewtopic.php?p=3030035#p3030035]Problem 3[/url]\nFind the smallest positive integer $n$, such that there exist $n$ integers $x_1, x_2, \\dots , x_n$ (not necessarily different), with $1\\le x_k\\le n$, $1\\le k\\le n$, and such that\n\\[x_1 + x_2 + \\cdots + x_n =\\frac{n(n + 1)}{2}\\quad\\text{, and }x_1x_2 \\cdots x_n = n!,\\]\nbut $\\{x_1, x_2, \\dots , x_n\\} \\ne \\{1, 2, \\dots , n\\}$.\n[url=http://www.artofproblemsolving.com/Forum/viewtopic.php?p=3030045#p3030045]Problem 4[/url]\nThe number $1$ is written on the blackboard. After that a sequence of numbers is created as follows: at each step each number $a$ on the blackboard is replaced by the numbers $a - 1$ and $a + 1$; if the number $0$ occurs, it is erased immediately; if a number occurs more than once, all its occurrences are left on the blackboard. Thus the blackboard will show $1$ after $0$ steps; $2$ after $1$ step; $1, 3$ after $2$ steps; $2, 2, 4$ after $3$ steps, and so on. How many numbers will there be on the blackboard after $n$ steps?"
}
{
"Tag": [
"calculus",
"integration",
"algebra",
"function",
"domain",
"geometry",
"rectangle"
],
"Problem": "I'm reading about complex analysis and I have some problems translating $\\epsilon$s and $\\delta$s into a geometric interpretation. I have a clear geometric view on Lebesgue/Riemann-Stieltjes integration of real valued functions. I want to accomplish that with complex integration too. I'll state the theorem\r\n\r\n[b]Theorem:[/b] Let $\\gamma: [a,b] \\to \\mathbb{C}$ be of bounded variation and suppose that $f: [a,b] \\to \\mathbb{C}$ is continuous. Then there is a complex number $I$ such that for every $\\epsilon > 0$ there is a $\\delta > 0$ such that when $P=\\{t_{0}< t_{1}< ... < t_{m}\\}$ is a partition of $[a,b]$ with $\\parallel P\\parallel =max\\{(t_{k}-t_{k-1}): 1 \\leq k \\leq m \\}< \\delta$ then\r\n\\[| I-\\sum_{k=1}^{m}f(\\tau_{k})[ \\gamma (t_{k-1})] | < \\epsilon \\]\r\nFirst of all, I hardly believe the author made an awfull typo and he meant to say:\r\n\\[{| I-\\sum_{k=1}^{m}f(\\tau_{k})[ \\gamma (t_{k})-\\gamma(t_{k-1})}] | < \\epsilon \\]\r\nHe calls $I$ the integral of $f$ with respect to $\\gamma$ over $[a,b]$. I don't see the geometric meaning. First of all we are not \"integrating\" over the domain of $f$, which is kinda weird... \r\n\r\nLater on, the author defines the line integral by considering the function $f \\circ \\gamma$. This makes already some more sense to me. But I still don't feel comfortable. If someone can tell me how to make a drawing to clear things up, that would be most helpful :)",
"Solution_1": "[quote=\"Jacobson\"]\nFirst of all, I believe the author made an awfull typo and he meant to say:\n\\[{| I-\\sum_{k=1}^{m}f(\\tau_{k})[ \\gamma (t_{k})-\\gamma(t_{k-1})}] | < \\epsilon \\]\n[/quote]\nYou are correct :).\n[quote]\nHe calls $I$ [b]the integral of $f$ with respect to $\\gamma$ over $[a,b]$ [/b].\n[/quote]\nThis is the first time I see such a combination of words. Of course, formally it makes perfect sense and allows one to reduce the integration over rectifiable paths to the Stiltjes integration very easily: you just consider your curve $\\gamma$ in the same way as you would consider $g$ in $\\int_{a}^{b}f\\,dg$: you can talk either about Riemann sums or about the (complex-valued) measure generated by $\\gamma$, which is sort of convenient, but the object that arises is of hardly any use outside the particular problem of defining the integral of a function $F: \\mathbb C\\to\\mathbb C$ over $\\gamma$ (for which you just need to apply this construction to $f=F\\circ\\gamma$).\n[quote]\nI don't see the geometric meaning. \n[/quote]\nDo you see the geometric meaning of the Stiltjes integral?\n[quote]\nFirst of all we are not \"integrating\" over the domain of $f$, which is kinda weird... \n[/quote]\nWell, this integral should be written $\\int_{a}^{b}f\\,d\\gamma$, so the integration actually is over the domain of $f$ (which is the interval $[a,b]$). Just don't try to think of it as of integral over $\\gamma$; however strange the definition may seem, it is put together correctly: it is, indeed, \"integral of $f$ over $[a,b]$ with respect to $\\gamma$\" in the same way as the Stiltjes integral $\\int_{a}^{b}f\\,dg$ is \"integral of $f$ over $[a,b]$ with respect to $g$\".\n[quote]\nLater on, the author defines the line integral by considering the function $f \\circ \\gamma$. This makes already some more sense to me. But I still don't feel comfortable. If someone can tell me how to make a drawing to clear things up, that would be most helpful :)[/quote]\r\nNote that when you take $f=F\\circ\\gamma$, you can rephrase the definition of the Riemann sum in the following way. Take the curve $\\Gamma\\subset \\mathbb C$ (the one parameterized by the mapping $\\gamma: [a,b]\\to\\mathbb C$), split it into small pieces by points $z_{0},z_{1},\\dots,z_{n}\\in \\Gamma$ ($z_{k}$ is the same as $\\gamma(t_{k})$); choose a point $\\zeta_{k}$ ($=\\gamma(\\tau_{k})$) on each piece of $\\Gamma$ between $z_{k-1}$ and $z_{k}$ and consider $\\sum_{k}F(\\zeta_{k})(z_{k}-z_{k-1})$. In the case, when $\\Gamma$ is a line segment, you'll get the usual Riemann sum. The problem with the drawing is that the product of two complex numbers doesn't have a clear \"geometric meaning\" unlike the product of two real numbers, which can be represented as a (signed) area of a rectangle, but, I hope, you see that the formula for the Riemann sums didn't change compared to the one for the real-valued functions defined on an interval.\r\n\r\nHope, this helps a bit :).",
"Solution_2": "thanks a lot :) it helped"
}
{
"Tag": [
"function",
"absolute value",
"calculus",
"calculus computations"
],
"Problem": "1. using the definition of cauchy, how do I show that every cauchy sequence of real numbers is a bounded sequence?\r\n2. if these are true prove them otherwise provide a counterexample:\r\na. every unbounded sequence of positive real numbers diverges to [size=150]\u221e[/size]\r\nb. every unbounded sequence of positive real numbers has a subsequence that diverges to \u221e",
"Solution_1": "By the Cauchy property, a Cauchy sequence is eventually bounded; there exists $N$ such that $|x_N-x_n|<1$ for $n>N$. (That specialized the property in two ways). Now, the first $N-1$ terms can't make the sequence unbounded; the first $N-1$ terms have a maximum $M$ and a minimum $C$. We have $\\min(x_N-1,C)0$, pick $N$ large enough that $|a_n|<\\frac{\\epsilon}{M}$ for $n>N$. Then $|a_nb_n|\\leq \\epsilon$ for $n>N$.\r\n\r\n$2:$ I hope you know how to construct a sequence with a single limit point ( $(n^{-1})_n$ ). Now mix this with the sequences $(n^{-1}+1)_n$ and $(n^{-1}+2)_n$. You are going to have to fiddle around with the indices to make it work, but this isn't hard.",
"Solution_4": "It looks like mikejones left out a few words in #3. I think we can figure out what he meant to say:\r\n\r\nSuppose $(a_n)$ is a Cauchy sequence and that $b_n=3a_n+7.$ Prove that $(b_n)$ is a Cauchy sequence.\r\n\r\nHere's a generalization: suppose $f: \\mathbb{R}\\mapsto\\mathbb{R}$ satisfies a Lipschitz condition. That is, suppose there is a constant $K$ such that for all $x$ and $y,\\,|f(x)-f(y)|\\le K|x-y|.$ Suppose $(a_n)$ is a Cauchy sequence and that $b_n=f(a_n).$ Prove that $(b_n)$ is a Cauchy sequence.\r\n\r\n(The connection between the specific case and the generalization: the function $f(x)=3x+7$ satisfies a Lipschitz condition with constant $K=3.)$\r\n\r\nOr take the generalization one step further out: suppose $f$ is uniformly continuous. Suppose $(a_n)$ is a Cauchy sequence and that $b_n=f(a_n).$ Prove that $(b_n)$ is a Cauchy sequence.",
"Solution_5": "[quote=\"jmerry\"]By the Cauchy property, a Cauchy sequence is eventually bounded; there exists $N$ such that $|x_N-x_n|<1$ for $n>N$. (That specialized the property in two ways). Now, the first $N-1$ terms can't make the sequence unbounded;...\n[/quote]\nI don't quite see why you can claim the first N-1 can't be unbounded? Can you explain?\n\n[quote=\"jmerry\"]2a: The answer is no. Try interlacing an unbounded sequence and a bounded sequence.\n[/quote]\r\nI have my own question for this one. Do we distiguish between [b]divergent to $\\infty$[/b] and [b]divergent[/b]?",
"Solution_6": "[quote=\"Hazel\"]I don't quite see why you can claim the first N-1 can't be unbounded? Can you explain?[/quote]\r\n\r\nthey are between $\\min_{i \\in \\overline{1,N-1}} x_i$ and $\\max_{i \\in \\overline{1,N-1}} x_i$.\r\n\r\n\r\nFor the other question, yes, divergent to $\\infty$ means that the sequence has limit $\\infty$, while divergent means that the sequence doesn't have finite limit."
}
{
"Tag": [
"function",
"integration",
"calculus",
"derivative",
"geometry",
"rectangle",
"real analysis"
],
"Problem": "Suppose \r\n$u(x,t)$ is continous functions \r\n$\\frac{\\partial u}{\\partial t}$ is continous\r\n$u(x+1,t)=u(x,t), \\; \\frac{\\partial u(x+1,t)}{\\partial x}=\\frac{\\partial u(x,t)}{\\partial x}$ \r\n$\\frac{\\partial^2 u}{\\partial x^2}=\\frac{\\partial^2 u}{\\partial t^2}$\r\n\r\nProve that $E(t)=\\frac{1}{2}\\int_{0}^{1}(\\frac{\\partial u(x,t)}{\\partial t})^2+(\\frac{\\partial u(x,t)}{\\partial x})^2)dx$ is constant independent of $t$",
"Solution_1": "Here's a formal calculation, assuming we have as many derivatives as we need:\r\n\r\n$\\frac{dE}{dt}=\\int_0^1\\frac{\\partial u}{\\partial t}\\cdot \\frac{\\partial^2u}{\\partial t^2} + \\frac{\\partial u}{\\partial x}\\cdot\\frac{\\partial^2u}{\\partial t\\partial x}\\,dx.$\r\n\r\nNow integrate the second term by parts:\r\n\r\n$\\int_0^1\\frac{\\partial u}{\\partial x}\\cdot\\frac{\\partial^2u}{\\partial t\\partial x}\\,dx = \\frac{\\partial u}{\\partial x}\\cdot\\frac{\\partial u}{\\partial t}|_0^1 - \\int_0^1\\frac{\\partial^2u}{\\partial x^2}\\cdot\\frac{\\partial u}{\\partial t}\\,dx.$\r\n\r\nNow note that the boundary terms in this integration by parts are zero by the periodicity of $u$ and use the fact that $u$ satisfies the wave equation to replace the second derivative with respect to $x$ with the second derivative with respect to $t.$\r\n\r\nHence, $\\frac{dE}{dt}=\\int_0^1\\frac{\\partial u}{\\partial t}\\cdot \\frac{\\partial^2u}{\\partial t^2}-\\frac{\\partial u}{\\partial t}\\cdot \\frac{\\partial^2u}{\\partial t^2}\\,dx = 0.$\r\n\r\nGiven the density of $C^{\\infty}$ functions, this identity would now hold in the appropriate Sobolev space, with the derivatives interpreted as weak derivatives.",
"Solution_2": "A second calculation, using a similar level of formality. We use the technique of separation of variables to find the general solution of the PDE.\r\n\r\n$u(x,t)=\\sum_{n=-\\infty}^{\\infty} e^{2\\pi inx}\\left(c_ne^{2\\pi i nt}+d_ne^{-2\\pi int}\\right).$\r\n\r\nTake the appropriate partial derivatives:\r\n\r\n$\\frac{\\partial u}{\\partial x}=\\sum_{n=-\\infty}^{\\infty} 2\\pi i ne^{2\\pi inx}\\left(c_ne^{2\\pi i nt}+d_ne^{-2\\pi int}\\right).$\r\n\r\n$\\frac{\\partial u}{\\partial t}=\\sum_{n=-\\infty}^{\\infty} 2\\pi i ne^{2\\pi inx}\\left(c_ne^{2\\pi i nt}-d_ne^{-2\\pi int}\\right).$\r\n\r\nBy Parseval's identity, if $g(x)=\\sum_{n=-\\infty}^{\\infty}a_ne^{2\\pi inx}$ then\r\n\r\n$\\int_0^1|g(x)|^2\\,dx=\\sum_{n=-\\infty}^{\\infty}|a_n|^2.$\r\n\r\nThus, \r\n\r\n$\\int_0^1\\left|\\frac{\\partial u}{\\partial x}\\right|^2\\,dx = \\sum_{n=-\\infty}^{\\infty} 4\\pi^2n^2\\left|c_ne^{2\\pi i nt}+d_ne^{-2\\pi int}\\right|^2$ and\r\n\r\n$\\int_0^1\\left|\\frac{\\partial u}{\\partial t}\\right|^2\\,dx = \\sum_{n=-\\infty}^{\\infty} 4\\pi^2n^2\\left|c_ne^{2\\pi i nt}-d_ne^{-2\\pi int}\\right|^2$\r\n\r\nBy direct calculation, \r\n\r\n$\\left|c_ne^{2\\pi i nt}+d_ne^{-2\\pi int}\\right|^2=|c_n|^2 + c_n\\overline{d_n} e^{4\\pi int}+\\overline{c_n}d_ne^{-4\\pi int}+|d_n|^2$\r\n\r\n$\\left|c_ne^{2\\pi i nt}-d_ne^{-2\\pi int}\\right|^2=|c_n|^2 - c_n\\overline{d_n} e^{4\\pi int}-\\overline{c_n}d_ne^{-4\\pi int}+|d_n|^2$\r\n\r\nAdd these together, and the result is independent of $t$. In the end,\r\n\r\n$E(t) = \\sum_{n=-\\infty}^{\\infty}4\\pi^2n^2\\left(|c_n|^2+|d_n|^2\\right).$",
"Solution_3": "Now that I look at it, this one has a Green's theorem solution too:\r\nWrite $E(0)-E(T)=\\int_{dR}\\frac12\\left(\\left(\\frac{\\partial u(x,t)}{\\partial t}\\right)^2+\\left(\\frac{\\partial u(x,t)}{\\partial x}\\right)^2\\right)dx+0\\, dt$, where $R$ is the rectangle with vertices $(0,0),(1,0),(1,T),(0,T)$. By Green's theorem, this becomes\r\n$E(0)-E(T)=\\int_R\\frac{\\partial u(x,t)}{\\partial t}\\cdot\\frac{\\partial u(x,t)}{\\partial t^2}+\\frac{\\partial u(x,t)}{\\partial x}\\cdot\\frac{\\partial u(x,t)}{\\partial x\\partial t}\\, dx\\, dt$\r\n$=\\int_0^T\\int_0^1\\frac{\\partial u(x,t)}{\\partial t}\\cdot\\frac{\\partial u(x,t)}{\\partial x^2}+\\frac{\\partial u(x,t)}{\\partial x}\\cdot\\frac{\\partial u(x,t)}{\\partial x\\partial t}\\, dx\\, dt$\r\n$=\\int_0^T\\int_0^1\\frac{\\partial}{\\partial x}\\left(\\frac{\\partial u(x,t)}{\\partial x}\\cdot\\frac{\\partial u(x,t)}{\\partial t}\\right)\\, dx\\, dt=0$\r\nby periodicity of $\\frac{\\partial u(x,t)}{\\partial x}\\cdot\\frac{\\partial u(x,t)}{\\partial t}$ in $x$.\r\n\r\nThis assumes that the mixed partial derivatives are continuous, so we can set them equal to each other."
}
{
"Tag": [
"Gauss"
],
"Problem": "Consider the sequence below where each third number is missing.\r\n\r\n$1, 2, 4, 5, 7, 8, 10, 11 ...$\r\n\r\nWhat is the the sum of the first one hundered terms? Show as many solutions as possible.",
"Solution_1": "[quote=\"guile\"]Consider the sequence below where each third number is missing.\n\n$1, 2, 4, 5, 7, 8, 10, 11 ...$\n\nWhat is the the sum of the first one hundered terms? Show as many solutions as possible.[/quote]\r\n[hide=\"hint\"]\nFirst find the sum when each third number is not missing. And then subtract $3+6+9+...$.\n[/hide]",
"Solution_2": "[hide=\"Solution\"] I believe it ends in 149 and there are 49 numbers missing, so $\\frac{149(150)}{2}-3\\frac{49(50)}{2}=149\\times75-49\\times25=\\boxed{7500}$. :D I think. [/hide]",
"Solution_3": "[hide]Well I think that we all know to add the first and last terms together and multiply that by half the number of terms to get 5050 for the sum of 1-100. We can use the same concept for this. \n\n$\\displaystyle{(3+99)\\cdot\\116}$ But because there are an odd number of terms we add the middle term, the 17th.\n$\\displaystyle{3\\cdot\\117=41}$ Add these two together to find out much to subtract from 5050. So we do $\\displaystyle{5050-1673=\\boxed{3377}}$ :D [/hide]",
"Solution_4": "oops... I answered the sum of 1-100 not including the multiples of 3... sorry :blush:",
"Solution_5": "You can make this the sum of the first 50 terms of this sequence--3,9,15..., which ends in $3+(49*6)=297$. Now we find the sum of such terms, $\\frac{(3+297)(50)}{2}=\\fbox{7500}$",
"Solution_6": "[quote=\"jhollenbeck\"]You can make this the sum of the first 50 terms of this sequence--3,9,15..., which ends in $3+(49*6)=297$. Now we find the sum of such terms, $\\frac{(3+297)(50)}{2}=\\fbox{7500}$[/quote]\r\n\r\nDarn it, you stole my method!",
"Solution_7": "[quote=\"DanK\"][quote=\"jhollenbeck\"]You can make this the sum of the first 50 terms of this sequence--3,9,15..., which ends in $3+(49*6)=297$. Now we find the sum of such terms, $\\frac{(3+297)(50)}{2}=\\fbox{7500}$[/quote]\n\nDarn it, you stole my method![/quote]\r\n\r\nJajajaa!",
"Solution_8": "well.....\r\n[hide]you add the first and second terms, you get 3. Third and fourth, 9. Fifth and sixth, 15.\nThere are 50 of those pairs.\nYou have $((2*(50*51)/2)-50)*3=50*50*3=[7500]$[/hide]\r\n\r\nsorry, I didn't post all of the solution, because my hands are cold. Oh. I didn't steal anyone's method.",
"Solution_9": "[quote=\"mbabbitt2003\"]well.....\n[hide]you add the first and second terms, you get 3. Third and fourth, 9. Fifth and sixth, 15.\nThere are 50 of those pairs.\nYou have $((2*(50*51)/2)-50)*3=50*50*3=[7500]$[/hide]\n\nsorry, I didn't post all of the solution, because my hands are cold. Oh. I didn't steal anyone's method.[/quote]\r\n\r\nNice method!!!!",
"Solution_10": "[hide=\"solution using gauss formula\"]\nyou can make this into another sequence\n$1+2+3+4+5+...$\nby adding $3+6+9...$ to the original sequence\nsince we are adding the first 100 terms of the original sequence\nwe can add all the numbers from 1-150\nand subtract the first 50 multiples of 3\nthe sum of 1-150 is\n$\\frac{(150)(150+1)}{2} = 11325$\nthe 3+6+9 sequence can be expressed as\n$\\frac{3\\times(50)(50+1)}{2} = 3825$\nour answer is then $11325-3825=\\fbox{7500}$[/hide]"
}
{
"Tag": [
"calculus",
"inequalities",
"calculus computations"
],
"Problem": "We have sets $A$ and $B$ of real numbers.\r\nThe elements of the set $A+B$ are given as $a+b$ where $a\\in{A}$ and $b\\in{B}$.\r\nProve that $supA+supB=sup(A+B)$ where supA=supremum of the set A.",
"Solution_1": "This is a standard result proved in most good calculus textbooks, e.g. Apostol.",
"Solution_2": "[quote=\"spider_boy\"]We have sets $A$ and $B$ of real numbers.\nThe elements of the set $A+B$ are given as $a+b$ where $a\\in{A}$ and $b\\in{B}$.\nProve that $supA+supB=sup(A+B)$ where supA=supremum of the set A.[/quote]\r\n\r\nI did not say that it is so hard.I am interested in people's solutions. :)",
"Solution_3": "[quote=\"spider_boy\"][quote=\"spider_boy\"]We have sets $A$ and $B$ of real numbers.\nThe elements of the set $A+B$ are given as $a+b$ where $a\\in{A}$ and $b\\in{B}$.\nProve that $supA+supB=sup(A+B)$ where supA=supremum of the set A.[/quote]\n\nI did not say that it is so hard.I am interested in people's solutions. :)[/quote]\r\nLet $\\sup A = x$ and $\\sup B = y.$\r\n\r\nWe have $a \\le x \\ \\forall a \\in A$ and $b \\le y\\ \\forall b \\in A,$ which implies $a+b \\le x+y.$ Hence, an upperbound for $A+B$ exists and $x+y$ is one of the upperbounds. \r\n\r\nClaim: $\\sup (A+B) = x+y.$ Suppose, for the sake of contradiction, $\\sup (A+B) = x+y-\\epsilon$ for some $\\epsilon > 0 .$ \r\n\r\nSince $\\sup A = x, \\ \\exists a' \\in A$ such that $a' > x-\\frac{\\epsilon}{2}.$ Again, since $\\sup B = y, \\ \\exists b' \\in B$ such that $b' > y-\\frac{\\epsilon}{2}.$ Thus, we have $a'+b' > x+y-\\epsilon,$ which contradicts the assumption that $x+y-\\epsilon = \\sup (A+B).$ And, we are done.\r\n\r\n[I can't think of a simpler proof. May be there is one.]",
"Solution_4": "mine was completely this one. :)",
"Solution_5": "1) max(A)$16 \\geq (a^{2}+b^{2}+c^{2}+d^{2})^{2}$\r\n<=>$(a+b+c+d)^{2}\\geq (a^{2}+b^{2}+c^{2}+d^{2})^{2}$\r\n<=>$a+b+c+d \\geq a^{2}+b^{2}+c^{2}+d^{2}$\r\n<=>$a(1-a)+b(1-b)+c(1-c)+d(1-d) \\geq0$\r\nSuppose that:$a \\geq b \\geq c \\geq d$,then we have:\r\n$1-a \\leq 1-b \\leq 1-c \\leq 1-d$\r\nThen by Chebyshev's inequality we have:\r\n$a(1-a)+b(1-b)+c(1-c)+d(1-d) \\geq0$\r\n(note$a+b+c+d=4$)",
"Solution_2": "You use Chebyshev inequality in a wrong way. If $a+b+c+d=4$ then certainly\r\n\\[a^{2}+b^{2}+c^{2}+d^{2}\\ge 4.\\]",
"Solution_3": "Sorry!\r\nI have other solution:\r\nWe have:\r\n$\\frac{1}{a^{2}}+\\frac{1}{b^{2}}+\\frac{1}{c^{2}}+\\frac{1}{d^{2}}-a^{2}-b^{2}-c^{2}-d^{2}=(\\frac{1}{a}+a)(\\frac{1}{a}-a)+(\\frac{1}{b}+b)(\\frac{1}{b}-b)+(\\frac{1}{c}+c)(\\frac{1}{c}-c)+(\\frac{1}{d}+d)(\\frac{1}{d}-d) \\geq 2(\\frac{1}{a}+\\frac{1}{b}+\\frac{1}{c}+\\frac{1}{d}-a-b-c-d) \\geq 2(\\frac{16}{a+b+c+d}-a-b-c-d)=0$\r\n(note that $x+\\frac{1}{x}\\geq 2$)\r\nI belive this solution will be true!",
"Solution_4": "But $(\\frac{1}{a}-a)$ can be $<0$ !!! :wink:",
"Solution_5": "[quote=\"manlio\"]But $(\\frac{1}{a}-a)$ can be $<0$ !!! :wink:[/quote]\r\nIf $(\\frac{1}{a}-a) <0$,then?",
"Solution_6": "I believe that this way is hardly true. You should try another one, chienthan.",
"Solution_7": "For example if $(\\frac{1}{a}-a)<0$ you cannot write $(\\frac{1}{a}+a)(\\frac{1}{a}-a) \\geq 2(\\frac{1}{a}-a)$",
"Solution_8": "if numbers are in $(0,3)$ i can prove it by convexiti of the function .but i will find an other solution.\r\nhi [b]hungkhtn[/b] can u post ur theorem ( i mean smvt and E?) because i do not now anything about them $thinks$. :D",
"Solution_9": "About SMV, you can see\r\n\r\nhttp://www.mathlinks.ro/Forum/viewtopic.php?t=113248",
"Solution_10": "[quote=\"hungkhtn\"]Let $a,b,c,d$ be positive real numbers such that $a+b+c+d=4$. Prove that\n\\[\\frac{1}{a^{2}}+\\frac{1}{b^{2}}+\\frac{1}{c^{2}}+\\frac{1}{d^{2}}\\ge a^{2}+b^{2}+c^{2}+d^{2}. \\]\n[/quote]\r\n\r\nI think the following stronger inequality holds for $a+b+c+d=4$:\r\n\r\n${\\frac{1}{ab}+\\frac{1}{bc}+\\frac{1}{cd}+\\frac{1}{da}}\\ge a^{2}+b^{2}+c^{2}+d^{2}$.",
"Solution_11": "Yes, Vasc, it was also on my note few minutes ago. The solution is based on an idea of Vardach.",
"Solution_12": "I would like to see your solution, Thuan and VASC, if it is not mixing variable.",
"Solution_13": "i remember mr. Schwarz told us that Vasc's inequality holds even for $5$ variables and for the degree $3,4$ also.",
"Solution_14": "I proved already that the inequality\r\n\\[\\frac{1}{a_{1}^{2}}+\\frac{1}{a_{2}^{2}}+...+\\frac{1}{a_{n}^{2}}\\ge a_{1}^{2}+a_{2}^{2}+...+a_{n}^{2}\\]\r\nholds for all $n\\in\\{1,2,...,9\\}$.\r\n\r\nHowever, I would like to see a simple solution for $n=4$, that the reason why I post this problem. I am also very curious of how Cauchy-Schwarz can be applied to prove this case $n=4$ and even $n=5$ as Pohoatza's post. Could you show it? Pohoatza?\r\n\r\nI also wait a solution of VASC and Thuan about their improvement. Hope that I have not to wait long then.\r\n\r\nThe following inequality is true and stronger\r\n\r\n\\[\\frac{1}{a^{2}}+\\frac{1}{b^{2}}+\\frac{1}{c^{2}}+\\frac{1}{d^{2}}-4 \\ge \\sqrt{3}(a^{2}+b^{2}+c^{2}+d^{2}-4).\\]",
"Solution_15": "[quote=\"hungkhtn\"] The following inequality is true and stronger\n\\[\\frac{1}{a^{2}}+\\frac{1}{b^{2}}+\\frac{1}{c^{2}}+\\frac{1}{d^{2}}-4 \\ge \\sqrt{3}(a^{2}+b^{2}+c^{2}+d^{2}-4). \\]\n[/quote]\r\nI think the inequality below is true and stronger\r\n\\[\\frac{1}{a^{2}}+\\frac{1}{b^{2}}+\\frac{1}{c^{2}}+\\frac{1}{d^{2}}+4 \\ge 2(a^{2}+b^{2}+c^{2}+d^{2}). \\]",
"Solution_16": "[quote=\"Vasc\"][quote=\"hungkhtn\"] The following inequality is true and stronger\n\\[\\frac{1}{a^{2}}+\\frac{1}{b^{2}}+\\frac{1}{c^{2}}+\\frac{1}{d^{2}}-4 \\ge \\sqrt{3}(a^{2}+b^{2}+c^{2}+d^{2}-4). \\]\n[/quote]\nI think the inequality below is true and stronger\n\\[\\frac{1}{a^{2}}+\\frac{1}{b^{2}}+\\frac{1}{c^{2}}+\\frac{1}{d^{2}}+4 \\ge 2(a^{2}+b^{2}+c^{2}+d^{2}). \\]\n[/quote]\r\n\r\nNice one, VASC,\r\n\r\nI build this inequality from the general case of $n$ so I don't check the case $n=4$ much. I just use the general result to replace this particular case. For all, we only need to check the inequality when $n-1$ numbers equal, so your problem can be done similarly as mine, VASC, just a bit different in the last step. I prove it by elementary method, not mixing variable, EV or SMV, too.",
"Solution_17": "Dear VASC and others,\r\n\r\nDo you think that it is time now to show a solution to the initial problem? We did discuss a lot, with strong problem, but a nice solution to the first is still hidden,\r\n\r\n[quote=\"hungkhtn\"]Let $a,b,c,d$ be positive real numbers such that $a+b+c+d=4$. Prove that\n\\[\\frac{1}{a^{2}}+\\frac{1}{b^{2}}+\\frac{1}{c^{2}}+\\frac{1}{d^{2}}\\ge a^{2}+b^{2}+c^{2}+d^{2}. \\]\n[/quote]",
"Solution_18": "The stronger\r\n\\[\\frac{1}{a\\sqrt{a}}+\\frac{1}{b\\sqrt{b}}+\\frac{1}{c\\sqrt{c}}+\\frac{1}{d\\sqrt{d}}\\ge a^{2}+b^{2}+c^{2}+d^{2}\\]\r\n:)",
"Solution_19": "[quote=\"Vasc\"][quote=\"hungkhtn\"]Let $a,b,c,d$ be positive real numbers such that $a+b+c+d=4$. Prove that\n\\[\\frac{1}{a^{2}}+\\frac{1}{b^{2}}+\\frac{1}{c^{2}}+\\frac{1}{d^{2}}\\ge a^{2}+b^{2}+c^{2}+d^{2}. \\]\n[/quote]\n\nI think the following stronger inequality holds for $a+b+c+d=4$:\n\n${\\frac{1}{ab}+\\frac{1}{bc}+\\frac{1}{cd}+\\frac{1}{da}}\\ge a^{2}+b^{2}+c^{2}+d^{2}$.[/quote]\r\nUsing the known $x^{2}+y^{2}\\le \\frac{(x+y)^{4}}{8xy}\\quad \\forall x,y \\ge 0$ $(*)$, we have\r\n\\[a^{2}+c^{2}\\le \\frac{(a+c)^{4}}{8ac}=\\frac{(a+c)^{4}bd}{8abcd}\\le \\frac{(a+c)^{4}(b+d)^{2}}{32abcd}, \\qquad b^{2}+d^{2}\\le \\frac{(b+d)^{4}(a+c)^{2}}{32abcd}\\]\r\nWe need to prove\r\n\\[{\\frac{1}{ab}+\\frac{1}{bc}+\\frac{1}{cd}+\\frac{1}{da}}\\ge \\frac{(a+c)^{4}(b+d)^{2}}{32abcd}+\\frac{(b+d)^{4}(a+c)^{2}}{32abcd}\\]\r\nOr\r\n\\[32 \\ge (a+c)(b+d)((a+c)^{2}+(b+d)^{2}) \\]\r\nIt is true by $(*)$. :) Equality holds if and only if $a=b=c=d=1$. :)",
"Solution_20": "Yes, toanhocmuonmau. :lol: It is also my solution.",
"Solution_21": "A general problem:\r\n Let $a_{1},a_{2},...,a_{n}\\geq 0 such that: \\sum a_{i}=n(n\\geq 4)$.\r\nProve that:\r\n $\\sum\\frac{1}{a_{i}^{n-2}}\\geq \\sum a_{i}^{2}$",
"Solution_22": "[quote=\"hungkhtn\"][quote=\"Vasc\"][quote=\"hungkhtn\"] The following inequality is true and stronger\n\\[\\frac{1}{a^{2}}+\\frac{1}{b^{2}}+\\frac{1}{c^{2}}+\\frac{1}{d^{2}}-4 \\ge \\sqrt{3}(a^{2}+b^{2}+c^{2}+d^{2}-4). \\]\n[/quote]\nI think the inequality below is true and stronger\n\\[\\frac{1}{a^{2}}+\\frac{1}{b^{2}}+\\frac{1}{c^{2}}+\\frac{1}{d^{2}}+4 \\ge 2(a^{2}+b^{2}+c^{2}+d^{2}). \\]\n[/quote]\n\nNice one, VASC,\n\nI build this inequality from the general case of $n$ so I don't check the case $n=4$ much. I just use the general result to replace this particular case. [/quote]\r\n\r\nMy general case of $n$ is the following.\r\n\r\n[i]If $a_{1},a_{2},...,a_{n}$ ($n\\ge 4$) are positive numbers such that $a_{1}+a_{2}+...+a_{n}=n$, then\n\\[\\frac{1}{a_{1}^{2}}+\\frac{1}{a_{2}^{2}}+...+\\frac{1}{a_{n}^{2}}-n \\ge \\frac 8{n}(a_{1}^{2}+a_{2}^{2}+...+a_{n}^{2}-n). \\]\n[/i]\r\nThe cases $n=4$ and $n=8$ are very interesting.",
"Solution_23": "In the topic about how to construct an inequality here\r\n\r\nhttp://www.mathlinks.ro/Forum/viewtopic.php?t=143631\r\n\r\nI did post the following stronger inequality\r\n\r\n$\\bigstar$ (Hungkhtn's, SI Inequalities A story of Maths, problem 5.3.4) \r\nLet $a_{1},a_{2},...,a_{n}$ be non-negative real numbers such that $a_{1}+a_{2}+...+a_{n}=n$. Prove that\r\n\\[n^{2}\\left(\\frac{1}{a_{1}^{2}}+\\frac{1}{a_{2}^{2}}+...+\\frac{1}{a_{n}^{2}}-n\\right) \\ge 10(n-1)\\left(a_{1}^{2}+a_{2}^{2}+...+a_{n}^{2}-n\\right).\\]\r\n\r\nMy solution is very elementary.",
"Solution_24": "The following is more sharper than the first\r\n\\[\\sum\\frac{1}{a}\\ge 3+\\frac{3}{4}\\sum a^{2}\\]\r\nand it has 2 equalities :)",
"Solution_25": "Maybe\r\n\r\n$\\sum \\frac 1{a}\\ge 1+\\frac 3{4}\\sum{a^{2}}$ for $\\sum a=4$.\r\n\r\nThis was posted in 2004 on this forum for $n$ numbers.",
"Solution_26": "Yes, it is an old inequality\r\n\r\n$\\bigstar$ [5.2.1, SII]\r\nLet $a_{1},a_{2},...,a_{n}\\ (n\\ge 3)$ be non-negative real numbers with sum $n$. Prove that\r\n\\[\\frac{1}{a_{1}}+\\frac{1}{a_{2}}+...+\\frac{1}{a_{n}}-n \\ge \\frac{4(n-1)}{n^{2}}\\left t(a_{1}^{2}+a_{2}^{2}+...+a_{n}^{2}-n).\\]",
"Solution_27": "[quote=\"toanhocmuonmau\"][quote=\"Vasc\"][quote=\"hungkhtn\"]Let $a,b,c,d$ be positive real numbers such that $a+b+c+d=4$. Prove that\n\\[\\frac{1}{a^{2}}+\\frac{1}{b^{2}}+\\frac{1}{c^{2}}+\\frac{1}{d^{2}}\\ge a^{2}+b^{2}+c^{2}+d^{2}. \\]\n[/quote]\n\nI think the following stronger inequality holds for $a+b+c+d=4$:\n\n${\\frac{1}{ab}+\\frac{1}{bc}+\\frac{1}{cd}+\\frac{1}{da}}\\ge a^{2}+b^{2}+c^{2}+d^{2}$.[/quote]\nUsing the known $x^{2}+y^{2}\\le \\frac{(x+y)^{4}}{8xy}\\quad \\forall x,y \\ge 0$ $(*)$, we have\n\\[a^{2}+c^{2}\\le \\frac{(a+c)^{4}}{8ac}=\\frac{(a+c)^{4}bd}{8abcd}\\le \\frac{(a+c)^{4}(b+d)^{2}}{32abcd}, \\qquad b^{2}+d^{2}\\le \\frac{(b+d)^{4}(a+c)^{2}}{32abcd}\\]\nWe need to prove\n\\[{\\frac{1}{ab}+\\frac{1}{bc}+\\frac{1}{cd}+\\frac{1}{da}}\\ge \\frac{(a+c)^{4}(b+d)^{2}}{32abcd}+\\frac{(b+d)^{4}(a+c)^{2}}{32abcd}\\]\nOr\n\\[32 \\ge (a+c)(b+d)((a+c)^{2}+(b+d)^{2}) \\]\nIt is true by $(*)$. :) Equality holds if and only if $a=b=c=d=1$. :)[/quote]\r\n\r\nIt is just now I can read your proof. Very nice one, toanhocmuonmau. Congras!\r\n\r\nDo you find a nice proof for $n=5$?",
"Solution_28": "[quote=\"hungkhtn\"]I proved already that the inequality\n\\[\\frac{1}{a_{1}^{2}}+\\frac{1}{a_{2}^{2}}+...+\\frac{1}{a_{n}^{2}}\\ge a_{1}^{2}+a_{2}^{2}+...+a_{n}^{2}\\]\nholds for all $n\\in\\{1,2,...,9\\}$.\n[/quote]\nIt's true for $n=10$ and it's wrong for all $n\\geq11$. :wink:",
"Solution_29": "[b]Ji Chen:[/b]\nLet $a,b,c,d$ be positive real numbers such that $a+b+c+d=4$. Prove that$$ a^{2}+b^{2}+c^{2}+d^{2}\\leq 2\\left(\\frac{1}{ac}+\\frac{1}{bd}\\right).$$",
"Solution_30": "[quote=hungkhtn]Let $a,b,c,d$ be positive real numbers such that $a+b+c+d=4$. Prove that\n\\[\\frac{1}{a^{2}}+\\frac{1}{b^{2}}+\\frac{1}{c^{2}}+\\frac{1}{d^{2}}\\ge a^{2}+b^{2}+c^{2}+d^{2}.\\][/quote]\nLet $a,b,c,d$ be real numbers such that $abcd=1$. Prove that$$ \\frac{1}{a^2}+\\frac{1}{b^2}+\\frac{1}{c^2}+\\frac{1}{d^2}\\geq ab+bc+cd+da$$\n(Proposed by Mircea Becheanu, University of Bucharest, Rom\u00e2nia)\n\nLet $a,b,c$ be real numbers such that $abc=1$. Prove that:$$a^2+b^2+c^2 \\geq a+b+c .$$",
"Solution_31": "[quote=hungkhtn]Let $a,b,c,d$ be positive real numbers such that $a+b+c+d=4$. Prove that\n\\[\\frac{1}{a^{2}}+\\frac{1}{b^{2}}+\\frac{1}{c^{2}}+\\frac{1}{d^{2}}\\ge a^{2}+b^{2}+c^{2}+d^{2}.\\][/quote]\n[url=https://artofproblemsolving.com/community/c6h181229p996138]the stronger one[/url]\n[url=https://artofproblemsolving.com/community/c6h1370494p10534020]here\n[/url]",
"Solution_32": "Let $a,b,c,d$ be positive real numbers such that $a 0$ then we have:\r\n\r\n$ \\frac{3}{2} ( \\sum \\frac{x^{2}}{y^{2}} + 1) \\geq \\frac{x}{y} +2 \\sqrt{ \\frac{y}{z} }+3 \\sqrt[3]{ \\frac{z}{x} } \\geq 6$.\r\n\r\nmade by Ioan Dancila.\r\n\r\ncheers! :D :D",
"Solution_1": "3+3sum(x^2/y^2) = sum(x^2/y^2) + 3 + 2sum(x^2/y^2)\r\n>= sum(x^2/y^2) + 3 + 2*3 (xyz/xyz)^2 by AM-GM\r\n= (x^2/y^2 + 1) + (y^2/z^2 + 3) + (z^2/x^2 + 5)\r\n>=2 x/y + 4 (y/z)^1/2 + 6 (z/x)^1/3 by AM-GM on each bracket, proving the left inequality (obviously, equality occurs iff x = y = z = 1).\r\n\r\nTo prove the right inequality, note that \r\n\r\n1/6 x/y + 1/3 (y/z)^1/2 + 1/2 (z/x)^1/3\r\n<=(xyz/xyz)^1/6 = 1, by the weighted AM-GM inequality. Again, equality occurs iff x = y = z = 1."
}
{
"Tag": [
"inequalities unsolved",
"inequalities"
],
"Problem": "$a+b+c = 1 , a,b,c \\geq 0$\r\n\r\nShow that \r\n\r\n$2(\\frac{b}{a}+\\frac{a}{c}+\\frac{c}{b}) \\geq \\frac{1+a}{1-a}+\\frac{1+b}{1-b}+\\frac{1+c}{1-c}$",
"Solution_1": "It's \\frac, not frac.\r\nSee http://www.mathlinks.ro/Forum/viewtopic.php?t=25780 .\r\n\r\n darij"
}
{
"Tag": [
"algebra unsolved",
"algebra"
],
"Problem": "the following problem maybe a Olympiad one:\r\n\r\nhttp://www.mathlinks.ro/viewtopic.php?p=1685878#1685878",
"Solution_1": "Do not double post."
}
{
"Tag": [],
"Problem": "Balanced chemical equation:\r\nC3H5(NO3)3(s) \u2192 3CO2(g) + 2.5H2O(g) + 1.5N2(g) + 0.25O2(g)\r\n\r\nI've been thinking for a while, and can't quite answer why it's so explosive. Any help?",
"Solution_1": "Hrmm.. your question sparked my interest.. taken from wikipedia:\r\n\r\n\"Nitroglycerin and any or all of the diluents mentioned above can certainly deflagrate, or burn. However, the explosive power of nitroglycerin is derived from detonation: a shock propagates through the fuel-rich medium at a supersonic speed. In other words, the initial decomposition sets up a pressure gradient that induces decomposition in contiguous material, creating a fast-moving transition zone, which (due to the nature of the material) can detonate any unstable or explosive material it encounters. This generates a self-sustained cascade of near instantaneous pressure-induced decomposition into gas of the explosive material which grows upon itself exponentially. This is quite unlike deflagration, which depends solely upon available fuel, regardless of pressure or shock.\"",
"Solution_2": "A couple of reasons...\r\n\r\nFirst, obviously a very negative $\\Delta H$.\r\n\r\nSecond, a very positive $\\Delta S$\r\n\r\nThird, (very likely) a very low-energy transition state, in turn leading to a high reaction rate.\r\n\r\nFourth, (likely) a low specific heat (Kopp's rule gives $265\\ \\text{J deg}^{-1}\\text{ mol}^{-1}$; the actual value is unknown to me).",
"Solution_3": "One must be very specific in one's questions if one is to obtain a precise answer. Unfortunately, in most cases, one must be more familiar to the answer than one is to know enough to ask a percise answer.\r\n\r\nThere are two dimensions to the question about nitroglycerin (the reactive chemical in dynamite that made blasting much safer and fueled the Nobel prizes, and the start of the Nobel chemical company). The first dimension is the amount of energy released, which is driven by the high bond strength of the formed nitrogen-oxygen bonds formed in the reaction of nitroglycerin with oxygen in burning or exploding. (This has been addressed in the other responses to your question.) \r\n\r\nThe second dimension is the speed of the reaction in other words how fast the energy is released. This is an independent physical property. One traditional way to measure this is to determine how fast a flame front will progress in an open trench. This is why nitroglycerin is not used as a fuel for internal combustion engines -- it simply delivers the energy too fast. It is also not used as a gunpower substitute for the same reason -- it delivers the energy too fast. The mechanics of converting the released chemical energy is insufficient for mechanical or projectile use. If one were to put kerosene in a open trench and lit one end of the vapor, one could walk faster than the flame front progressed. This is a relatively slow release of energy, useful in jet engines and the like. One would have to run to keep up with an methyl ether fire (a member of my lab did so and got his beaker into the hood before the entire beaker and the flame front coincided, avoiding a more memorable incident.) If one were to do this with nitroglycerin, the flame front would progress at a speed greater than mach 1- instantaneous for most real world considerations. In this speed of progression lies the explosive force (and great danger) of nitroglycerin. The explosion is due to the generation of great quantities of gas that puts great pressure on whatever is about to be impacted, be it a beaker, hand or face.\r\n\r\nThis is too dangerous for casual experiementation. If one is not working with folks who have experience with these chemicals, one may well end up earning a Darwin Award, perhaps even demonstrating the theory that led to the naming of this award.\r\n\r\nExplosives should not be experiemented with casually.\r\n\r\nExplosives should not be experiemented with casually.\r\n\r\nExplosives should not be experiemented with casually.\r\n\r\nI hope I have addressed your question in a way that will lead to you and the readers of this to not conduct experiements that you and all who love you will regret."
}
{
"Tag": [
"geometry",
"percent",
"probability",
"puzzles"
],
"Problem": "A larger square table top divided into 100 squares (on a 10 square by 10 square set-up) of 12 cm. on a side of each smaller square is marked out on the larger sheet of the table top.\r\n\r\nIn order to win a prize, a player has to roll one coin down a chute so that the coin rolls onto the large sheet and lands completely within a smaller square and not touching a line.\r\n\r\nIf a player rolls a 5 cm. diameter coin down the chute, are the chances less than even or greater than even, that the coin will come to rest not touching a line?",
"Solution_1": "Assuming that the coin stays center on the table:\r\n[hide]\nThe locus of the points where the coin's center can be winning is a lot of 7 by 7 squares in the middle of the grid squares. Area: 49. So, the odds are about $ \\frac{49}{144} \\approx 34.0 percent < \\frac{1}{2}$ for one square. Since this stays the same for all of the interior of the board, and is only slightly different at the edges (that is, if the coin stays completely on the table), my answer would be less than even.\n[/hide]\r\nIs that right? Sorry, can't figure out how to put the percent symbol.",
"Solution_2": "[quote=\"math_explorer\"]Sorry, can't figure out how to put the percent symbol.[/quote]\r\n\r\nThe code for $ \\%$ is \\%. :)",
"Solution_3": "Wait, math_explorer,\r\n\r\nassuming the coin stays on the table,\r\n\r\njust condense this problem down to one 5 cm. diameter coin\r\n\r\nbeing dropped onto one 12 cm. by 12 cm. square. It must\r\n\r\nstay inside this one square and not touch any of the four\r\n\r\nlines of the square. Please reconsider with this clarification/hint.",
"Solution_4": "I thought that was what I did.",
"Solution_5": "Your post looked right, except you would want \"probability\" instead of \"odds.\"\r\n\r\nGood work.",
"Solution_6": "Oops... I think my subconscious did that. :blush:"
}
{
"Tag": [
"trigonometry"
],
"Problem": "Prove that\r\n\r\n$ i \\tan(3\\pi/11) \\equal{} (x^3\\minus{}1)/(x^3\\plus{}1) \\equal{} x^{10}\\plus{}x^9\\minus{}x^8\\minus{}x^7\\minus{}x^6\\plus{}x^5\\plus{}x^4\\plus{}x^3\\minus{}x^2\\minus{}x$ \r\n\r\nwhere $ x\\equal{}e^{2\\pi i/11}$.",
"Solution_1": "Note that $ x^{11}\\equal{}1$, so $ x^{10}\\plus{}x^9\\minus{}x^8\\minus{}x^7\\minus{}x^6\\plus{}x^5\\plus{}x^4\\plus{}x^3\\minus{}x^2\\minus{}x$\r\n$ \\equal{}x^{11}\\plus{}x^{10}\\plus{}x^9\\minus{}x^8\\minus{}x^7\\minus{}x^6\\plus{}x^5\\plus{}x^4\\plus{}x^3\\minus{}x^2\\minus{}x\\minus{}1$\r\n$ \\equal{}(x^9\\minus{}x^6\\plus{}x^3\\minus{}1)(x^2\\plus{}x\\plus{}1)\\equal{}(x^3\\minus{}1)(x^2\\plus{}x\\plus{}1)(x^6\\plus{}1)$\r\n$ \\equal{}\\frac{(x^3\\minus{}1)(x^6\\minus{}1)(x^6\\plus{}1)}{(x\\minus{}1)(x^3\\plus{}1)}\\equal{}\\frac{(x^3\\minus{}1)(x^{12}\\minus{}1)}{(x\\minus{}1)(x^3\\plus{}1)}\\equal{}\\frac{(x^3\\minus{}1)(x\\minus{}1)}{(x\\minus{}1)(x^3\\plus{}1)}\\equal{}\\frac{x^3\\minus{}1}{x^3\\plus{}1}$.\r\n\r\nAlso, $ i\\tan{\\frac{3\\pi}{11}}\\equal{}\\minus{}i\\tan{\\frac{8\\pi}{11}}\\equal{}\\frac{x^7\\minus{}x^4}{x^7\\plus{}x^4}\\equal{}\\frac{x^3\\minus{}1}{x^3\\plus{}1}$, and we are done."
}
{
"Tag": [],
"Problem": "A friend of mine spoke to me about someone he knew a few years ago who could track down sports games with mathlab. For instance he could track down where shots were taken from and so on but I have no clue how to do this and neither did my friend, just wondering if any of you have experience doing this.",
"Solution_1": "anyone has any idea?",
"Solution_2": "I think I've found it\r\n\r\nhttp://www.cs.ubc.ca/nest/lci/thesis/fhli/videoclips/"
}
{
"Tag": [
"factorial",
"algebra",
"polynomial",
"probability and stats"
],
"Problem": "Say $ X$ is distributed Poisson($ \\lambda$). \r\n\r\nWhat is $ E(X^r)$, $ r$ a nonnegative integer?\r\n\r\nMy idea is to express $ E(X^r)$ in terms of $ E(X),E((X)(X \\minus{} 1)),E((X)(X \\minus{} 1)(X \\minus{} 2))$,etc, but I can't find a simple formula for it.",
"Solution_1": "you're on the right track.\r\n\r\nLet $ (x)_n$ denote the falling factorial, that is\r\n\r\n\\[ (x)_n \\triangleq x(x-1)(x-2)\\ldots(x-n+1)\r\n\\]\r\n\r\n1) it's straightforward to verify that if $ X\\sim\\text{Poisson}(\\lambda)$, then\r\n\r\n\\[ E((X)_n) = \\lambda^n\r\n\\]\r\n\r\n2) if you know some combinatorics (i actually had to look this up), then, as polynomials,\r\n\r\n\\[ t^n = \\sum^n_{k=0} S(n,k)(t)_k\r\n\\]\r\n\r\nwhere $ S(n,k)$ are the Stirling numbers of the second kind.\r\n\r\n3) it follows from linearity of expectations that\r\n\r\n\\begin{eqnarray*}\r\nE(X^n) &=& E\\left(\\sum^n_{k=0} S(n,k)(X)_k\\right) \\\\\r\n&=&\\sum^n_{k=0} S(n,k)E((X)_k) \\\\\r\n&=&\\sum^n_{k=0} S(n,k)\\lambda^k\r\n\\end{eqnarray*}\r\n\r\nI don't think you're going to get a much more satisfying answer than this."
}
{
"Tag": [
"videos"
],
"Problem": "To play this game, watch the video in the previous post and wait until the related videos comes up at the end. Or just press the menu button. Pick one of those videos and post it, preferrably [i]not[/i] the same video. :P \r\n\r\n[youtube]pcYbnxA-qDY[/youtube].",
"Solution_1": "Okay...\r\n\r\n[youtube]SXx2VVSWDMo[/youtube]",
"Solution_2": "[youtube]Ooa8nHKPZ5k[/youtube]",
"Solution_3": "[youtube]Tr1qee-bTZI[/youtube]\r\n\r\nPlease, take 15 minutes out of your day to watch this.",
"Solution_4": "[quote=\"miyomiyo\"][youtube]Tr1qee-bTZI[/youtube]\n\nPlease, take 15 minutes out of your day to watch this.[/quote]\r\n\r\nActually, for arithmetic, use MCGrawHill, and for Algebra, use AoPS text.\r\n\r\n\r\n[youtube]I-b5D5941AM[/youtube]",
"Solution_5": "[youtube]VjgidAICoQI[/youtube]\r\n\r\nWhat in the world...?",
"Solution_6": "[youtube]1bSZslEDUl0[/youtube]",
"Solution_7": "[youtube]Pfs4Rd5f_IQ[/youtube]."
}
{
"Tag": [
"inequalities",
"inequalities unsolved"
],
"Problem": "If $ a,b,c$ are positive reals such that $ abc \\equal{} a \\plus{} b \\plus{} c$ prove that\r\n\\[ (2ab \\minus{} 1)(2bc \\minus{} 1)(2ca \\minus{} 1)\\ \\geqslant\\ \\frac1{2(a \\plus{} b \\plus{} c)^2}\\]",
"Solution_1": "Is equivalent with :\r\n\r\n\\[ 2(a\\plus{}b\\plus{}c)(2a\\plus{}2b\\plus{}c)(2a\\plus{}b\\plus{}2c)(a\\plus{}2b\\plus{}2c)\\ge 1\\]",
"Solution_2": "It seems too weak. \r\n$ 2ab \\minus{} 1 \\equal{} \\frac {2abc}{c} \\minus{} 1 \\equal{} \\frac {2(a \\plus{} b \\plus{} c)}{c} \\minus{} 1 \\equal{} \\frac {2(a \\plus{} b)}{c} \\plus{} 1$\r\nSo the product on the left side must be greater then 1.\r\nSo we have to show $ 2(a \\plus{} b \\plus{} c)^2 \\ge 1$, or $ 2(a \\plus{} b \\plus{} c)^3 \\ge abc$, which is obvious.",
"Solution_3": "[quote=\"dgreenb801\"]It seems too weak. \n$ 2ab \\minus{} 1 \\equal{} \\frac {2abc}{c} \\minus{} 1 \\equal{} \\frac {2(a \\plus{} b \\plus{} c)}{c} \\minus{} 1 \\equal{} \\frac {2(a \\plus{} b)}{c} \\plus{} 1$\nSo the product on the left side must be greater then 1.\nSo we have to show $ 2(a \\plus{} b \\plus{} c)^2 \\ge 1$, or $ 2(a \\plus{} b \\plus{} c)^3 \\ge abc$, which is obvious.[/quote]\r\n\r\nIndeed , the stronger :\r\n\\[ (2ab \\minus{} 1)(2bc \\minus{} 1)(2ca \\minus{} 1)> \\frac {2916}{(a \\plus{} b \\plus{} c)^2}\\]\r\nis also true for $ a,b,c > 0$ with $ abc \\equal{} a \\plus{} b \\plus{} c$ , but very easy again :( .",
"Solution_4": "I think\r\n$ (2ab \\minus{} 1)(2bc \\minus{} 1)(2ca \\minus{} 1) \\ge \\frac {3375}{(a \\plus{} b \\plus{} c)^2}$ is true.\r\nWe have\r\n$ 2ab \\minus{} 1 \\equal{} \\frac {2abc}{c} \\minus{} 1 \\equal{} \\frac {2(a \\plus{} b \\plus{} c)}{c} \\minus{} 1 \\equal{} \\frac {2(a \\plus{} b)}{c} \\plus{} 1$\r\nSo we have to show\r\n$ (\\frac {2(a \\plus{} b)}{c} \\plus{} 1)(\\frac {2(b \\plus{} c)}{a} \\plus{} 1)(\\frac {2(c \\plus{} a)}{b} \\plus{} 1) \\ge \\frac {6750}{(a \\plus{} b \\plus{} c)^2}$\r\nBy Holder's inequality,\r\n$ (\\frac {2(a \\plus{} b)}{c} \\plus{} 1)(\\frac {2(b \\plus{} c)}{a} \\plus{} 1)(\\frac {2(c \\plus{} a)}{b} \\plus{} 1) \\ge (2 \\sqrt [3] {\\frac {(a \\plus{} b)(b \\plus{} c)(c \\plus{} a)}{abc}} \\plus{} 1)^3$\r\nBut $ (a \\plus{} b)(b \\plus{} c)(c \\plus{} a) \\ge 8 abc$, so this is $ \\ge 125$. So we have to show\r\n$ 125 \\ge \\frac {3375}{(a \\plus{} b \\plus{} c)^2}$, or $ (a \\plus{} b \\plus{} c)^3 \\ge 27abc$, which is obvious.",
"Solution_5": "[quote=\"dgreenb801\"]I think\n$ (2ab \\minus{} 1)(2bc \\minus{} 1)(2ca \\minus{} 1) \\ge \\frac {6750}{(a \\plus{} b \\plus{} c)^2}$ is true.[/quote]\r\n\r\nIt fails for $ a\\equal{}b\\equal{}c\\equal{}\\sqrt{3}$ ;)",
"Solution_6": "Thanks for all your help! :)"
}
{
"Tag": [
"geometry",
"AMC",
"USA(J)MO",
"USAMO"
],
"Problem": "Let's start with geometry.\r\n\r\n[size=150]Problem 1[/size]\r\n\r\nLet $ BB', CC'$ be altitudes of triangle $ ABC,$ and assume $ AB\\not\\equal{}AC$. Let $ M$ be the midpoint of $ BC, H$ the orthocenter of $ ABC$, and $ D$ the intersection of $ BC$ and $ B'C'$. Show that $ DH$ is perpendicular to $ AM$.\r\n\r\nPreparation for USAMO :wink:.",
"Solution_1": "OK...\r\n\r\n[hide=\"Minor Hint\"]Radical axes either all concur or are parallel.[/hide] \n[hide=\"Hint\"]\nFirst note what happens if we consider the point $ E$ on $ AM$ such that $ EH$ is perpendicular to $ AM$. And then go from there.[/hide]\n[hide=\"Bigger hint\"]Extend $ AM$ to form a parallelogram. Cyclic quads![/hide]",
"Solution_2": "sweet marathon you have here.\r\n\r\nand no i can't do it.",
"Solution_3": "I guess I need to start practicing now, since its only 3 weeks away.\r\n\r\nIll try to post as many proof problems as I can.\r\n\r\n^unimpossible, I feel for you and youkow. You two are probably more deserving than me of making it, seriously.\r\n\r\nBtw, did you guys see David Liang (7th grade) on the list? That guys my hero, even though Ive never met him."
}
{
"Tag": [
"LaTeX",
"quadratics",
"arithmetic series",
"algebra",
"quadratic formula",
"arithmetic sequence"
],
"Problem": "Lance Armstrong wants to ride to his friend's house 5000 miles away. He rides his bike 10 miles on September 1. He then rides another 20 miles on September 2. Each subsequent day, he rides 10 miles more than he rode the day before. On what day will he show up at his friend's house?",
"Solution_1": "I think you can solve this with an arithmetic sequence. I am not good at using latex so I will use regular typing. ar[hide]ith sequence formula. \nan= a1 + (n-1)d\n5000=10+10n-10\nn=50\nI hope I am correct.[/hide]",
"Solution_2": "[quote]Lance Armstrong wants to ride to his friend's house 5000 miles away.\nHe rides his bike 10 miles on September 1.\nHe then rides another 20 miles on September 2.\nEach subsequent day, he rides 10 miles more than he rode the day before.\nOn what day will he show up at his friend's house?[/quote][hide]\nWe have the [b]sum[/b] of an arithmetic series: $S_n = \\frac{n}{2}[2a + (n-d)d]$\n\nWe have: $a = 10,\\;d = 10,\\;S_n = 5000$\n\nHence: $\\frac{n}{2}[20 + 10(n-1)] = 5000\\;\\;\\;\\Rightarrow\\;\\;\\;n^2 + n - 1000 = 0$\n\nwhich has the positive root: $n \\;= \\;\\frac{-1 + \\sqrt{4001}}{2} \\;= \\;31.126...$\n\n\nTherefore, Lance arrives on the 32nd day: $\\text{October 2.}$[/hide]",
"Solution_3": "[hide]well first you can divide everything by 10, the numbers will then be a bit easier to work with so you have:\nn(n+1)/2=500\nn^2+n-1000=0\nUsing the quadratic formula, I find x is 31.126...so he will arrive after 32 days. There are 32 days in September, so he will arrive [b]October 2nd[/b][/hide]",
"Solution_4": "[color=darkred]\nBrute force =D !\n\n[hide=\"yummy solution o_o\"]\n10\n30\n60\n100\n150\n210\n280\n360\n450\n550\n660\n780\n...\ntoo... tired...\n[/hide]\n\n[hide=\"screw that and try a different method o_o\"]\nFormula for triangular numbers:\n\nn(n+1)/2\n\nYou have to find the smallest value of n where that expression > 5000/10.\n\nn(n+1)/2 > 500\nn(n+1) > 1000\nI think n = 32 (please don't eat me if I'm wrong, I don't taste good)\n\nAfter I wonder how the heIl Lance Armstrong managed to bike 310 miles in one day (he doesn't bike a full 320 miles on October 2nd), I got...\nThirty days hath April, September, June and November.\nOctober 2nd.\n[/hide]\n[/color]",
"Solution_5": "I don't understand what is wrong with my solution though.",
"Solution_6": "Hello hello! What you noticed was that the distances he travels are in arithmetic sequence, which is correct. Thus, you did arrive at the (correct) result that on the 50th day, Lance will travel 5000 miles. However, you neglected to take into account that at that time he will already have ridden for 49 days, covering a considerable distance. Thus, the question you needed to solve was not, \"What value of n makes 10n greater than or equal to 5000?\" but rather, \"Which value of n makes 10 + 20 + 30 + ... + 10n greater than or equal to 5000?\"",
"Solution_7": "[quote]Thus, you did arrive at the (correct) result that on the 50th day, Lance will travel 5000 miles.[/quote]hmm...There is a small typo... missing an extra zero between 500 and 5000?\r\nHope this helps ..",
"Solution_8": "Haha, I realized that just as I sat down to dinner. That was another mistake, which made your first seem more reasonable -- you divided 5000 by 10 and got 50.",
"Solution_9": "Thanks for the corrections. Sorry for showing my mathematical weakness.",
"Solution_10": "Hey, don't apologise.. making mistakes is the best way to learn!"
}
{
"Tag": [
"complex numbers"
],
"Problem": "I'm going to do some self-independent studying and that I'm going to use my textbooks to take notes on important thing and maybe like create a problem and solve by myself. I'm going to upload it on website so other people can see and point out if I'm doing wrong or not.\r\n\r\nI won't post on AoPS. I would post on my own website and I guess it's both good for me and others because it would be like reading notes to other people and getting ideas on in case going wrong during my study.\r\n\r\nI'll start today and if I make some progress, I'll post on here.\r\n\r\nWish me luck :)",
"Solution_1": "Good luck.\r\n\r\nI'd look at it if you put a link.",
"Solution_2": "There are two lessons (or notes :) for myself) that I made.\r\n\r\nThe link to there is:\r\n\r\nhttp://www.dhost.info/jwec05/My%20Websites/Number%20one/math%20lesson.htm\r\n\r\nThe first one is polygon and the second one is on complex number. Please read through and find if there is anything wrong.",
"Solution_3": "What's weird is that some textbooks (like mine) call b the 'imaginary part' of a+bi, and other textbooks (like whatever you're using, apparently) call bi the imaginary part. Maybe you should mention that...",
"Solution_4": "[quote=\"Silverfalcon\"]I won't post on AoPS. I would post on my own website and I guess it's both good for me and others because it would be like reading notes to other people and getting ideas on in case going wrong during my study.\n\nI'll start today and if I make some progress, I'll post on here.\n\nWish me luck :)[/quote]First of all good luck :) \r\nSecond of all I don't understand how you will not post on AoPS-MathLinks, but still \"I'll post on here\". \r\nFinally, maybe it's hard for people to tell you their opinions when the subject at hand is somewhere else (it would be easier to quote) ;)",
"Solution_5": "Um.. For the second one, Valentine Vornicu, I posted on my website nothing but in case it will get messed up or it might take a lot of space on this public forum.. You know, this note is my stuff and well, if this takes too much space on the aops forum, it would be like taking spaces off from public forum, ya know..\r\n\r\nThat's all... But thanks! :)",
"Solution_6": "[quote=\"Silverfalcon\"]Um.. For the second one, Valentine Vornicu, I posted on my website nothing but in case it will get messed up or it might take a lot of space on this public forum.. You know, this note is my stuff and well, if this takes too much space on the aops forum, it would be like taking spaces off from public forum, ya know..\n\nThat's all... But thanks! :)[/quote]I didn't understood what you said. You first said you ain't gonna write it here, then you said you will :P :) \r\n\r\nI think that if you keep everything into a single topic nobody will mind ;)",
"Solution_7": "[quote=\"LynnelleYe\"]What's weird is that some textbooks (like mine) call b the 'imaginary part' of a+bi, and other textbooks (like whatever you're using, apparently) call bi the imaginary part. Maybe you should mention that...[/quote]\r\n\r\nThe imaginary part of $a+bi$ is $b$. That is quite universal.",
"Solution_8": "I agree that the imaginary part of $a + bi$ should be $b$, which is how I've seen it in almost all books. Unfortunately, the AoPS book says that the imaginary part is $bi$. On the other hand, it says that $\\mathrm{Im}(a + bi) = b$. That distinction is confusing to me, because I pronounce \"Im\" as \"the imaginary part of\".\r\n\r\nPerhaps the authors felt that, from the point of view of English, it is weird for the imaginary part to be a real number. I agree it is weird from that point of view, but it is much more convenient mathematically.",
"Solution_9": "[quote=\"Silverfalcon\"]360 = sum of ANY exterior angles in polygon[/quote]\r\n\r\nI don't suppose it's wrong mathematically, but it sounds funny. Although, if you're using for yourself, it doesn't matter."
}
{
"Tag": [
"geometry",
"calculus"
],
"Problem": "Hi, I was wondering why there are no solid geometry problems in the IMO.",
"Solution_1": "I was looking thruogh the past years' IMO questions , and there were some really good questions on geometry .... But somehow , the weightage for geometry seems very little .... { I could get only about 1 or 2 geometry questions in one paper. }\r\n :?",
"Solution_2": "how does that answer my question?",
"Solution_3": "They chose not to include solid geometry because people didn't like them?",
"Solution_4": "[quote=\"G-UNIT\"]how does that answer my question?[/quote]\r\n\r\nI laffed.",
"Solution_5": "my guess is that some of the problems can be greatly simplified with multivariable calculus. and these olympiad stuff promote non-calculus algebra",
"Solution_6": "[quote]I was wondering why there are no solid geometry problems in the IMO.[/quote]\r\n\r\nSolid geometry is not taught in most countries. The IMO jury and problem committees are dominated by people from the majority of countries that do not teach this subject, and many of them consider such problems unfair to \"their\" students.",
"Solution_7": "It's a bit harder to illustrate and put clear proofs for solid geometry. People would run into complications while grading.",
"Solution_8": "Solid geometry not being taught in most countries, was the reason stated to me by a couple of IMO team leaders. I don't know if that is the full story, but I doubt that difficulty of writing up the solutions rigorously has much to do with it (there are several other subjects with the same difficulty, that make it onto the IMO).",
"Solution_9": "i have noticed that there is at least one solid geometry problem at early IMO.",
"Solution_10": "Solid geometry was taught in the early-IMO countries at the time of the early IMO.",
"Solution_11": "[quote=\"fleeting_guest\"]Solid geometry was taught in the early-IMO countries at the time of the early IMO.[/quote]\r\nSolid geometry is an important part in Chinese high school course. What about other countries?"
}
{
"Tag": [
"factorial"
],
"Problem": "Evaluate \\[(\\sum^{2002}_{k=1} \\frac{k.k!}{2^k}) - (\\sum^{2002}_{k=1} \\frac{k!}{2^k}) - (\\frac{2003!}{2^{2002}})\\]",
"Solution_1": "$\\frac{k\\cdot k!}{2^k}-\\frac{k!}{2^k}=\\frac{(k+1-1)k!}{2^k}-\\frac{k!}{2^k}=\\frac{(k+1)!}{2^k}-\\frac{2k!}{2^k}=\\frac{(k+1)!}{2^k}-\\frac{k!}{2^{k-1}}$, Using telescoping method we have \r\n\r\n$\\sum_{k=1}^{2002} \\frac{k\\cdot k!}{2^k}-\\sum_{k=1}^{2002} \\frac{k!}{2^k}-\\frac{2003!}{2^{2002}}$\r\n\r\n$=\\left(\\frac{2!}{2}-\\frac{1!}{2^0}\\right)+\\left(\\frac{3!}{2^2}-\\frac{2!}{2}\\right)+\\cdots+\\left(\\frac{2002!}{2^{2001}}-\\frac{2001!}{2^{2000}}\\right)+\\left(\\frac{2003!}{2^{2002}}-\\frac{2002!}{2^{2001}}\\right)-\\frac{2003!}{2^{2002}}$\r\n\r\n$=-1$",
"Solution_2": "[quote=\"kunny\"]$\\frac{k\\cdot k!}{2^k}-\\frac{k!}{2^k}=\\frac{(k+1-1)k!}{2^k}-\\frac{k!}{2^k}=\\frac{(k+1)!}{2^k}-\\frac{2k!}{2^k}=\\frac{(k+1)!}{2^k}-\\frac{k!}{2^{k-1}}$, Using telescoping method we have \n\n$\\sum_{k=1}^{2002} \\frac{k\\cdot k!}{2^k}-\\sum_{k=1}^{2002} \\frac{k!}{2^k}-\\frac{2003!}{2^{2002}}$\n\n$=\\left(\\frac{2!}{2}-\\frac{1!}{2^0}\\right)+\\left(\\frac{3!}{2^2}-\\frac{2!}{2}\\right)+\\cdots+\\left(\\frac{2002!}{2^{2001}}-\\frac{2001!}{2^{2000}}\\right)+\\left(\\frac{2003!}{2^{2002}}-\\frac{2002!}{2^{2001}}\\right)-\\frac{2003!}{2^{2002}}$\n\n$=-1-\\frac{2001!}{2^{2000}}$[/quote]\r\n\r\nThe ans. is -1..u can recheck ur calculations...",
"Solution_3": "Yes, the answer is -1 :D",
"Solution_4": "Yeh, I was mistaken.I will edit. :lol:"
}
{
"Tag": [
"geometry",
"circumcircle",
"incenter",
"angle bisector",
"geometry proposed"
],
"Problem": "Let $ ABC$ be equilateral triangle inscribed in circumcircle $ (O)$. Let $ D$ be some point on the smaller arc of $ BC$. Let $ I_1,I_2,I_3$ be the incenters of triangle $ DBC,DCA,DAB$, respectively. Prove that the line through $ A,B,C$ perpendicular to $ I_2I_3,I_3I_1,I_1I_2$ concurrent. :D",
"Solution_1": "[b]hint[/b]: Why don't you use the [b]Carnot theorem[/b] to prove it?? :D",
"Solution_2": "I shall try solving, before receiving full proof from its author...\r\nFirst, let's re-name I_1, I_2 and I_3 as 1, 2 and 3 respectively.\r\n\r\nIt's known that the perpendiculars from the vertices of a triangle onto the sides of another triangle are concurrent iff the perpendiculars from the vertices of the 2nd triangle onto the sides of the first one are concurrent (is this Carnot's, or just a consequence?).\r\n\r\nI shall show that the perpendiculars from 1 to BC, 2 to AC and 3 to AB concur at the Fermat's point of the triangle 123..\r\n\r\nI name M and N the midpoints of the small arcs BD and CD, M' the point on the circle O such that B is the midpoint of the arc MM', and N' such as C is the midpoint of the arc NN'.\r\nThe points 1 and 3 lie on the circle C(M, BM) and, of course on the angle bisectors of brian > matt > sherry > stephen > yi > zach",
"Solution_28": "Who is Matt?\r\n\r\nBy the way, Stephen would actually beat you. :P",
"Solution_29": "Perhaps Matt McCutchen...",
"Solution_30": "hehe\r\nJ definitely wins.\r\ni wonder why y and z are so popular",
"Solution_31": "Maybe it's because it's 3:45 AM and I've been writing code for the last four hours but I found this topic exceedingly amusing.",
"Solution_32": "[quote=\"Philip_Leszczynski\"]Who is Matt?\n\nBy the way, Stephen would actually beat you. :P[/quote]\r\n\r\nturns out he did :lol:\r\n\r\ngood thing I didn't put hesterberg though (could anybody beat him?)",
"Solution_33": "Sadly, yes--looking through the list, there are two Aarons, an Abraham, and possibly another Adam who would beat me. At least I'd make the team :)",
"Solution_34": "I, too, was a J who made USAMO :) .\r\n\r\nMy total is 91.... fairly high but not a 108... JC or JL, i can think of a couple (jeffrey chen)",
"Solution_35": "[quote=\"Go Beyond\"][quote=\"randomdragoon\"]\nThere seem to be a lot of Chens taking the USAMO.[/quote]\n\nNo surprise. Chen is one of the largest family names in China. Also Li, Zhang, Liu... are the largest. :D[/quote]\r\n\r\nWang is still the most common in my opinion, my mom knows at least 20 Wangs (counting each individual family member though)\r\n\r\nHa, my initials are JC.",
"Solution_36": "Well my guess J was kinda based on the fact that my name, my brother's name, my other brother's name, my mom's name, one of my uncles' names, one of my cousins' names, and a couple other family names all start with J."
}
{
"Tag": [
"modular arithmetic"
],
"Problem": "A=2^29\r\n\r\nA has nine digits\r\nWhich digit doesn't involve in A",
"Solution_1": "[hide]4[/hide]",
"Solution_2": "[quote=\"JRav\"][hide]4[/hide][/quote]\r\n\r\nHow did you solve?",
"Solution_3": "The sum of all $ 10$ digits is $ 45\\equiv 0\\pmod{9}$, and $ 2^{29} \\equal{} 2^2\\cdot(2^3)^9\\equiv 4\\cdot( \\minus{} 1)^9\\equiv 5\\pmod{9}$, hence what's lacking from the lot is $ 4$.",
"Solution_4": "The problem statement should make clearer that $ A$ has nine digits, all of which are distinct.",
"Solution_5": "If you look closer into the wording, it's clear - a digiT (not digiTS) missing when there's nine of them can only mean all of the nine digits involved are distinct (otherwise there would be digitS missing). Though I agree it could've been made more explicit.",
"Solution_6": "Similar problem.\r\n\r\nProve that $ 2^{99}$ involves at least one digit which occurs at least 4 times.\r\nThis is also true for $ 2^{98}$ and $ 2^{97}$."
}
{
"Tag": [
"logarithms",
"ratio",
"integration",
"calculus",
"probability and stats"
],
"Problem": "Let $ X_0$ have a uniform distribution on $ [0,1]$, and for $ n \\ge 1$, $ X_{n \\plus{} 1}$ has a uniform distribution on $ [0,X_n]$. \r\nFind the almost sure limit of $ \\frac {\\ln{X_n}}{n}$.",
"Solution_1": "try to apply the law of large number ..",
"Solution_2": "Doesn't the law of large numbers usually involve some kind of sum? :maybe: Can you give more hints how to solve this?",
"Solution_3": "If $ Y_n$ are iid random variables that are uniformly distributed on $ [0,1]$, then $ X_n\\equal{}\\prod_0^n Y_j$.",
"Solution_4": ":blush: I don't understand how I missed that... That's it from me today, back to learnin PDE :lol: .",
"Solution_5": "[quote=\"fedja\"]If $ Y_n$ are iid random variables that are uniformly distributed on $ [0,1]$, then $ X_n \\equal{} \\prod_0^n Y_j$.[/quote]\r\nI m not finding it quite obvious. Can anyone explain how is the above true?",
"Solution_6": "To say $ X_{n \\plus{} 1}$ is uniformly distributed on $ [0, X_n]$ is the same as saying $ X_{n \\plus{} 1} \\equal{} Y_{n \\plus{} 1} \\cdot X_n$ where $ Y_{n \\plus{} 1}$ is uniformly distributed on $ [0, 1]$ (with appropriate independence).",
"Solution_7": "JBL, can you explain how is $ X_{n\\plus{}1}\\equal{} Y_{n\\plus{}1}\\cdot X_{n}$?",
"Solution_8": "Define $ Y_n$ to be the ratio $ \\frac {X_{n \\plus{} 1}}{X_n}$.",
"Solution_9": "Is it due to the following fact?\r\n\r\n$ P\\left(\\frac{X_{n\\plus{}1}}{X_{n}}\\le x\\right)\\equal{}P\\left(X_{n\\plus{}1}\\le x.X_{n}\\right)$\r\n$ \\equal{}\\int_{0}^{1} \\int_{0}^{x_{0}}.\\, .\\, .\\, \\int_{0}^{x_{n\\minus{}1}}\\int_{0}^{x.x_{n}}1.\\frac{1}{x_{0}}.\\frac{1}{x_{1}}.\\,.\\,.\\,\\frac{1}{x_{n}}\\, dx_{0}\\, dx_{1}\\, .\\, .\\, .\\, dx_{n\\plus{}1}\\equal{}x$\r\n\r\nTherefore $ Y_{n}\\equal{} \\frac{X_{n\\plus{}1}}{X_{n}}$ is uniform$ [0,1]$",
"Solution_10": "That integral doesn't appear to converge. We are given that $ X_{n \\plus{} 1}$ is uniform in $ [0, X_n]$, which is equivalent to saying that $ \\frac {X_{n \\plus{} 1}}{X_n}$ is uniform in $ [0, 1]$.\r\n\r\n[Minor edit for grammar.]",
"Solution_11": "Ya I m convinced. Thank you JBL."
}
{
"Tag": [
"Columbia",
"calculus",
"geometry",
"AMC",
"USA(J)MO",
"USAMO",
"counting"
],
"Problem": "This is not a post about the positives and negatives of graduating early. I am fully aware of what it would take to graduate early and the millions of reasons not to do it. What I'd like to know is how greatly it would affect my chances of getting into a top school such as Columbia which is where I'd like to go if I can get in after junior year. I am currently in tenth grade, and am greatly considering the fact of graduating after eleventh. \r\n\r\nI took AP Calculus BC last year and got a score of 5 (with an AB subscore of 5). I also took AP Physics C Mechanics and got a score of 5. This year I am taking AP Physics C Electricity and Magnetism, AP World History, AP Chemistry, AP Economics Micro and Macro, and AP Java AB. Next year I will be taking AP US Government, AP Statistics, AP Biology, AP English Language, AP Latin Literature, and AP Psychology.\r\n\r\nI am taking the PSAT in October and would take the SAT I in the Spring. I am going to take the SAT II Math IIc and the SAT II Physics as well within the next few months. At the end of the year, I'd like to take the SAT II World History and SAT II Chemistry. My GPA is 97.5/100, unweighted (my school does not weight or use class rank).\r\n\r\nLast summer, I did research in geophysics at MIT. This coming summer, I plan on continuing research except in more of a theoretical physics area hopefully regarding something like Quantum Mechanics or String Theory.\r\n\r\nMy goal is to make the USAMO this year. I am assistant captain of my school's math team, captain of my school's chess team, a member of my school's mock trial club, a member of my school's Quizbowl club, and technical manager of my school's newspaper. I have some math and chess awards, and I expect to have some science research awards by the end of this year.\r\n\r\nHow realistic are my chances? Thanks.",
"Solution_1": "You're doing many of the right things, but it's really hard to evaluate chances at a school like Columbia with a low base rate of acceptance. A lot of the applicants who don't make it in are hardly distinguishable from those who do. \r\n\r\nI definitely wish you well in trying to wrap up high school early--I would gladly have done the same at your age if only I had prepared as well as you have. I can't predict the chances for acceptance at the most selective schools, but it does seem you are on the right track. \r\n\r\nDo you have a plan B in the event of plan A not working as you desire?",
"Solution_2": "I haven't read anything above, so I'm not sure if this is the slightest relevant, nontheless its quite interesting.... :lol: My local university runs an accelarated, high-school programme called \"transition\". Its a two-year programme that covers 5-years of highschool. ( In Canada 8th Grade is the beginning of high-school). Meaning transition kids end up graduating at the end of 9th grade, and then proceed onto university.\r\n\r\n Its a pretty intense programme with very little free-time, my math teacher used to teach there and now considers it child-abuse...",
"Solution_3": "my plan B would be to stick high school out my senior year and apply again to the highly selective schools where i would have even a greater chance of being accepted (since i would be of the regular age). of course there are 2nd tier schools such as NYU I could apply to as well. i'm really hoping that i can get into columbia a year early though, because my senior year in high school would be very pointless. anything i could do to improve my chances? unfortunately, i don't do sports or anything like that. i play cello but not at any great level, so i don't think that would be of much help either.",
"Solution_4": "FWIW - There are many schools (including some Ivy ) who will admit a promising student who is in Junior year. \r\n Apart from Columbia and other Ivy leagues, you may like to check out other schools, say USC (Resident Honors Program) which will admit students who are in their Junior year (and may award a full tution scholarship - if they are really impressed with you :)) and work with your high shcool so that you get the highschool diploma too (( to work-out any remaining requirtemnts etc)... I knew this was there a few years ago, so check out their site, if the program still exists.\r\n(Of course, other places may have similar programs so check them out too)\r\n\r\nAdded later: Point to note about these special programs (eg Resident honors program of USC) is that there are additional resources (eg special housing, (with less freedom than average freshman :D) , advisers etc) which are available for those who are coming in early)",
"Solution_5": "Stanford sent this to me when I emailed then concerning applying early (I am currently a junior) \r\n\r\n[b]\n\nThank you for contacting the Office of Undergraduate Admission at \nStanford \nUniversity. We have admitted younger applicants who were academically \ncompetitive in the past. We do not discriminate against a student based \non \nage, but we will consider both the academic achievement and the \nstudent's \noverall maturity level and ability to adjust to college life regardless \nof \ntheir age.\n\nHowever, we would strongly recommend taking advantage of the full four \nyears to complete high school while making the most of your time both \nacademically and personally. If you like, you might even consider \ntaking \ncourses at a local community college during your final year.\n\nAs we do not schedule appointments, you are welcomed to drop by the \noffice \nor give us a call. Please bear in mind, however, that we will not be \nable \nto review any materials you may bring to the office.\n\nIf you have any additional questions, please feel free to contact our \noffice at (650) 725-5294.\n\nSincerely,\n\nThe Office of Undergraduate Admission\nStanford University\n\n[/b]\r\n\r\nI decided to just have a really slow and easy senior year instead. [/b]",
"Solution_6": "It's a bit like the folklore college essay listing activities such as cancer research, international peace negotiations, felling charging elephants with a single shot between the eyes, and so on. \r\n\r\nThe set of AP exams above is, obviously, overkill for a school at the level of Columbia, especially in tenth grade and with 1 or 2 years left to prolong the list. Barring a criminal record, or major deficiencies in your grades or bank account, admission is more or less a given.\r\n\r\nBear in mind, though, that \"research in geophysics\" will be decoded by anyone with a clue as \"interned at a lab\", and that the chances of doing research in string theory at this level (i.e. anything beyond working menial calculations for the actually qualified) are very, very low. It is generally advisable to discuss such things in realistic, neutral, objective and factual terms rather than exaggerations or speculations about the future, and that is especially true in the context of accelerated college admission, where maturity is a specific consideration.",
"Solution_7": "You sound way more prepared than I ever was for college.\r\nI'd like to add that in the case that you do not get in after your junior year, you might still be able to take classes at Columbia during your senior year. I took Introduction to Number Theory during the fall term of my senior year at Columbia, with the help of my school's math department assistant principal. Just check with someone in your school.\r\nThe experience was worthwhile, even though I was not officially enrolled in the class and do not plan on claiming credit. You get a feel for the school, know what you might be getting into. Doing something like this can show the colleges that you have the maturity and time management necessary to succeed. In fact, with your credentials, I'd say you might be able to do it next year, and further improve your chances of getting in early."
}
{
"Tag": [
"geometry",
"geometric transformation",
"reflection"
],
"Problem": "Pretend that you are camping. While you are searching for natural berries, you see that your tent is on fire! :) \r\nYou are 200 feet from the river, you're tent is 240 feet downstream from you're current location, and the tent is 120 ft. away from the river bank. Thankfully, you have a bucket with you. You must run to the river and to your tent.\r\n\r\n1) What is the shortest distance?\r\n2) How far did you run with the pail full and pail empty?",
"Solution_1": "[hide]\n\nLet us reflect the image off the river bank. Let A be your current location, B the tent, and B' the reflection. So, we need to find AB'. We have a right triangle with legs 320 and 240 (the distance from the riverbank to A and distance from riverbank to B'; distance tent is downstream). So, the distance is $ \\sqrt{[8(40)]^2\\plus{}[6(40)]^2}\\equal{}\\sqrt{40^2(8^2\\plus{}6^2)}\\equal{}400$ \n\nLet the river be CD, where C is closer to A and D is closer to B. Also, let the intersection of the river and AB' be X. We have $ \\triangle{ACX}\\sim\\triangle{B'DX}$. So, $ \\frac{200}{120}\\equal{}\\frac{5}{3}\\equal{}\\frac{AX}{400\\minus{}AX}$. So, AX=distance without water=250. This distance with water is 400-250=150.\n\n[/hide]"
}
{
"Tag": [
"geometry",
"geometry proposed"
],
"Problem": "Point P lies on side AB of a convex quadrilateral ABCD. Let ! be the incircle of triangle\r\nCPD, and let I be its incenter. Suppose that ! is tangent to the incircles of triangles APD\r\nand BPC at points K and L, respectively. Let lines AC and BD meet at E, and let lines\r\nAK and BL meet at F. Prove that points E, I, and F are collinear.",
"Solution_1": "see here: http://www.mathlinks.ro/viewtopic.php?p=1186805#1186805"
}
{
"Tag": [
"ARML",
"AMC",
"USA(J)MO",
"USAMO",
"AIME"
],
"Problem": "So like I'm going with the grissom team and so is dynamo. hey I CAN MEET DYNAMO (and ash and evil ed i think). Cool. Hopefully I shall get more than 50% of the questions in individual right this time. hmmm I get the feeling I have a room to myself, since there are 17 people. GO TEAM GRISSOM!\r\n\r\nThis shall be fun. :) I am very excited.\r\n\r\nOh snap if I had made usamo grissom could not be in division B, or I could not go. or we would be in division A, according to the email.",
"Solution_1": "Haha. I found out I was on ARML this week. I did one practice and got a 3/8 :dry: So if this post contains ARML stupidity it's because idk anything about ARML. \r\nJust wondering, but why are you going with grissom? and do you know, since there are 17 of us, which two are not on the team? \r\nhey I CAN MEET XINKE! The all-famous Xinke... :o You won't be impressed by me, I'm not smart, and... I'm white :(\r\nyes, you will meet ash and sid. They are not like each other except for being brown, short, and geniuses so don't expect them to be the same. \r\nhaha yeah go Grissom. weird, 3 of our people dont even go to Grissom. \r\nim excited about the competiton cuz its experience, but i'll weigh us down so i'm nervous.\r\nhow close were you to usamo?",
"Solution_2": "11 points by amc 10. Horrendous amount of points by amc 12. I need to up my average practice aime score to 12 (I should stop dreaming) since like my real aime score both years have been 2 or 3 less than average practice aime scores....\r\n\r\n\r\n\r\nhmmmm so like i wanted to go to arml and I told Mr. Crawford, and he said Grissom might send a team, and he thought that maybe I could go with grissom, and it worked out well from there.\r\n\r\nHmmmm, you have practiced more for arml than I have this year..... :o \r\n\r\nhmmm you shall be less impressed by me. I like to do problems very slowly, if at all possible, to triple check everything, and the only reason i am ok at math is cuz I kind of work hard at it. I consider you and tony and botong smarter than me.\r\n\r\nby the way they are changing arml rules this year to make it 10 problems instead of 8.",
"Solution_3": "can i go too? ask your coach dynamo...",
"Solution_4": "I would have asked you if you wanted to go if the grissom teacher had said that she had more room on the team, but right now, there's already a full team and 2 alternates. Also, registration ended 2 days ago. Sorry."
}
{
"Tag": [
"algebra",
"polynomial",
"algorithm",
"real analysis",
"real analysis unsolved"
],
"Problem": "Given an even degree polynomial in $ \\mathbb{R}[x]$, is there any deterministic way of finding out if it has a real root? ( like we use the discriminant for degree 2 polynomials)",
"Solution_1": "please see\r\n\r\n[url]http://en.wikipedia.org/wiki/Root-finding_algorithm[/url]",
"Solution_2": "[url=http://en.wikipedia.org/wiki/Sturm%27s_theorem]Sturm's theorem[/url] is even more relevant here.",
"Solution_3": "Thanks.\r\n\r\nBut more than an algorithm I was looking for a condition involving the coefficients which would give a conclusion. Rather than evaluating the polynomial at some points.",
"Solution_4": "I would hazard a guess that Sturm's theorem is the best you're going to get. Be satisfied with that.\r\n\r\nWLOG, assume that the leading coefficient of your $ 2n$ degree polynomial is positive. You're asking whether $ p(x)$ is ever negative. You could phrase that as asking about the minimum value of $ p(x).$ Where is that minimum located? At one of the roots of $ p'(x),$ which is a $ 2n\\minus{}1$ degree polynomial. Can you find the roots of $ p'(x)$ by \"a condition involving the coefficients which would give a conclusion\"? Let's put it this way: you do know about the Abel-Galois impossibility results for the solvability of polynomials of degree 5 and above?\r\n\r\nNow if you want to se if you can find something specifically for the fourth degree case, go look at the Tartaglia-Cardano-Ferrari formulas and see what you can do."
}
{
"Tag": [
"search",
"LaTeX",
"trigonometry",
"Olimpiada de matematicas"
],
"Problem": "me disculpo de entrada por que pues no me he puesto a mirar como funciona y por ende no se como usar el latex.el caso es que \r\n si x pertenece al intervalo cerrado cero, pi medios donde x es un angulo dado en radianes demostrar que \r\n\r\n cos(cosx)>=sex\r\n\r\nnuevamente les pido disculpas por no usar el latex.",
"Solution_1": "[url=http://www.artofproblemsolving.com/Forum/viewtopic.php?search_id=317043477&t=279103]contestaci\u00f3n[/url]\r\n\r\nEn latex:\r\n\r\nsi $ x$ pertenece al intervalo $ [0,\\frac {\\pi}{2}]$ donde $ x$ es un angulo dado en radianes demostrar que:\r\n \r\n$ \\cos(\\cos(x)) \\geq \\sin(x)$."
}
{
"Tag": [
"AMC",
"AIME",
"AMC 8",
"AMC 10"
],
"Problem": "Was the 2003 AMC 10B just really easy? Over winter break, I have averaged about 130 on the AMC 10. This evening, on the AMC 10 test I just did, I got a 145.5, not answering one of them from misreading. Was this to be expected from the difficulty or did I suddenly get smarter?",
"Solution_1": "Try comparing the AMC 10 statistics. [url=http://www.unl.edu/amc/e-exams/e5-amc10/e5-1-10archive/2003-10a/03stats10.html]Here is the 2003 AMC 10/12 statistics.[/url] The AIME cutoff was 121, which is higher than the usual 120, so I would assume it was slightly easier.",
"Solution_2": "Same here, I just took it and got a 144, but I usually average a 130-135.",
"Solution_3": "I just took the 2003 12A and 12B and scored the same as my AMC 10 average, i just think 2003 was a lot easier than all the other years for some odd reason",
"Solution_4": "I also remember hearing that the 2003 AIME was ridiculously easy compared to other years, so maybe the MAA decided to just make everything easier that year.",
"Solution_5": "I think that year was slightly easier because my test scores were higher that year.",
"Solution_6": "2003 amc8 was average though",
"Solution_7": "Ya, i think it was easier :P\r\nCompare to other people s score",
"Solution_8": "hmmm... Try the 2004 AMC10 B... It was quite a bit tougher than the 2003 one.",
"Solution_9": "Um, now, is the AIMe cutoff 120 or higher (100 or higher for AMC 12) or is it 1% (5% for AMC 12)",
"Solution_10": "[quote=\"AwesomeToad\"]Um, now, is the AIMe cutoff 120 or higher (100 or higher for AMC 12) or is it 1% (5% for AMC 12)[/quote]\r\n\r\nIt's either one, I believe.",
"Solution_11": "[quote=\"batteredbutnotdefeated\"][quote=\"AwesomeToad\"]Um, now, is the AIMe cutoff 120 or higher (100 or higher for AMC 12) or is it 1% (5% for AMC 12)[/quote]\n\nIt's either one, I believe.[/quote]\r\n\r\nWhichever is more less.",
"Solution_12": "What do you mean \"more less\"? which one gets most people?",
"Solution_13": "[quote=\"AwesomeToad\"]What do you mean \"more less\"? which one gets most people?[/quote]\r\n\r\nWhichever has a lesser score. If the top 1% was people with scores of 140+; then 120+ would be cutoff. If top 1% was 100+, then I think 100+ would be cutoff."
}
{
"Tag": [
"algebra",
"polynomial"
],
"Problem": "I was looking for roots for this fraction to split it into as many factors as possible.\r\n\r\n$f(x)=\\frac{2x^4+x^3-5x^2+8x-3}{x^3+x^2-2x+3}$\r\n\r\nFirst of i found out that $0.5$ was a root to the nominator.\r\n\r\n$f(x)=\\frac{(x-0.5)(2x^3+2x^2-4x+6)}{x^3+x^2-2x+3}$\r\n\r\n...but from here i couldn't split it further. Maybe some of you can help me one step further.\r\n\r\nThx in advance for your help.",
"Solution_1": "[hide=\"very big hint\"]try factoring that 2 out of the second polynomial in the numerator[/hide]",
"Solution_2": "lol, this was really great. Just two seconds after i posted the thread i found the solution to my problem (well, quite simple way when you take a little more time instead of following the book's guidelines). I clicked the delete button but i couldn't deleted my own thread since it is only allowed by the moderators. Then i wrote a reply to answer the result to my own problem and found that in the same time i wrote the reply my internet connection was gone :lol: Didn't help with a restart of my comp. Here is what i wrote in the reply (which i saved):\r\n\r\n------------------------------------------------------------------------------------\r\n\r\nOK, i can't delete it, but now i think i know how to handle this. Plz tell me if this is right, thx.\r\n\r\n$f(x)=\\frac{(x-0.5)(2x^3+2x^2-4x+6)}{x^3+x^2-2x+3}$\r\n\r\n$f(x)=\\frac{2(x-0.5)(2x^3+2x^2-4x+6)}{2(x^3+x^2-2x+3)}$\r\n\r\n$f(x)=2(x-0.5)$\r\n\r\n$f(x)=2x-1$\r\n\r\n------------------------------------------------------------------------------------\r\n\r\nWith the exception that i first timed it up by 2 and then removed the equal fractions instead of just dividing with 2 i seem to have done everything right. Thx for your reply though i didn't need it, but thx anyway (how would you know) ;)"
}
{
"Tag": [
"probability",
"algebra",
"function",
"domain"
],
"Problem": "for some who is interested in math, would you suggest acturial science or being an analyst or trader at wall street\r\n\r\nalso which is more high paying?",
"Solution_1": "If you want to make ungodly sums of money, become a successful investment banker with a big company. Of course, this requires an MBA.",
"Solution_2": "[quote=\"JRav\"]If you want to make ungodly sums of money, become a successful investment banker with a big company. Of course, this requires an MBA.[/quote]\r\nPretty much no Wall Street type jobs [i]require[/i] MBAs anymore. Also, the only way this strategy works is if you're willing to do that for your whole life. Those people at Merrill Lynch or whatever whose ratings of particular stocks can cause huge swings in those stocks once announced have spent decades moving up in those companies. If you want to get rich quick (not super quick but faster than at a big i-bank), work for a hedge fund. \r\n\r\nIf you like programming, being a Wall Street quant type person is probably about the best you can do and stay in the \"math-analytical\" sort of realm. Being an i-banker or a prop trader is not nearly as analytical a profession.",
"Solution_3": "Wel.....i wanna be an actuary co mathematician is that possible?",
"Solution_4": "How much do actuaries, quants, etc. get paid?",
"Solution_5": "[quote=\"123456789\"]How much do actuaries, quants, etc. get paid?[/quote]\r\nIt varies quite a bit, but generally a lot. This should not be your sole motivation for working such a job, though. (Probably, if it is your sole motivation, you won't get one of the best jobs.)",
"Solution_6": "Just my opinion- having worked for a large (top 10) P&C insurance company. It was not always the most interesting, but it had it's moments. What I will say is that the pay was good (10 years ago I started at $ \\$$44K with no exams passed) and the hours were very good - the only extra time I put in was studying for exams. When I decided not to pursue exams, which was fairly accepted in my department, it definitely became a job worth having with a young family. As for making money - they paid$ \\$$2k automatic raise for each exam passed. If you passed the exam on the first try, there was an added bonus. There were a few young gung-ho types (under 26) who passed all the exams first time and I know their salary was at least close to $ \\$$100k. I have had a few friends work as investment brokers and they said the money was insane (one claimed over$ \\$$1 mil in a year before bonuses) but the hours could be trying as well. He basically said there was only one relationship in his life. But he is in his late 30's and essentially retired.",
"Solution_7": "The previous poster's discussion of salary is totally consistent with my own personal knowledge of actuary salaries. Actuaries make a respectable amount of money, but to put it bluntly, it's a pittance compared to what people with comparable math skills are able make in other jobs such as investment banking. (Career actuaries have to be fairly smart people since they have to pass a series of grueling exams.) It's also less than what you could make in computer programming or engineering. On the other hand, actuaries don't have to work particularly hard and have good job security.",
"Solution_8": "Here is some salary data on actuaries:\r\n\r\n[url]http://www.actuaryjobs.com/salary.html[/url]",
"Solution_9": "It seems that actuaries stay within probability and statistical application. Does this field offer much in terms of expanding one's knowledge mathematically? In other words, is it more of a \"business\" career, or a mathematic career?",
"Solution_10": "[quote=\"mr.affable\"]It seems that actuaries stay within probability and statistical application. Does this field offer much in terms of expanding one's knowledge mathematically? In other words, is it more of a \"business\" career, or a mathematic career?[/quote]\r\n\r\nI think it is more of a business career. Probably you know most of the math but rather have to get used to certain models, assumptions and other domain knowledge.",
"Solution_11": "I heard actuaries have better job satisfaction? If you're an investment banker or analyst you might only get to sleep from 2am to 7am, and bankers have to work very long hours",
"Solution_12": "i interned with quants this summer. I mean they know math but not like we do.....(problem solving)",
"Solution_13": "I think that job satisfaction depends on the person.. everyone likes different things! You cannot conclude that every banker hates his job and has less job satisfaction than the actuaries ;)"
}
{
"Tag": [
"LaTeX",
"number theory unsolved",
"number theory"
],
"Problem": "Determine all integer solutions of the equation $ x^n\\plus{}y^n\\equal{}1994$,where $ n \\geq 2$",
"Solution_1": "Obviously, $ x,y$ are both odd. It's easy to conclude that $ n\\le3$ because $ 5^5 > 1994$ and $ 2.3^5 < 1994$ and the case $ n \\equal{} 4$ follows easily from Fermat's little theorem.Now we need to check 2cases, which are easy to do.\r\n[hide=\"Some hints\"](the main idea in the case $ n \\equal{} 3$ is to use modulo 7 and conlude that one of the numbers is divisible by $ 7$, say $ x$, and since $ 14^3 > 1994$, so $ x \\equal{} 7$ but then $ y^3 \\equal{} 1651$-impossible)[/hide]",
"Solution_2": "But x and y need not be positive, right?",
"Solution_3": "in that case we shall have to solve two equations\r\n$ x^n\\minus{}y^n\\equal{}1994$ and $ x^n\\plus{}y^n\\equal{}1994$\r\n\r\nthe first one is only possible if n is odd cuz else it ll lead to second..\r\nexpand it...ll post the solution as soon as poss..\r\nin a hurry..!",
"Solution_4": "I don't understand. Why n needs to be smaller than 3 or equal ? \r\n\r\nIf I write : 3^5 < 1994 and 5^5 > 1994 ?",
"Solution_5": "I missed one case because I thought it's obvious. It's true because we know that x,y are both odd and since $ 5^5>1994$ then they should be 1 or 3. Now, I forgot to write the fact that $ 3^6>1994$ which implies that there aren't any solution for $ n\\ge 5$. But, as I mentioned, the case $ n\\equal{}4$ is impossible because of the Fermat Little Theorem and so $ n\\le3$.",
"Solution_6": "[size=150]x^2=4a+1=3b+1, thus a=3k, x^2=12h, for some natural h, similarly for b, but 12 does not divide 1994, hence no solutions.....[/size]\r\n\r\nCan someone give me the link for using latex here?[/url]",
"Solution_7": "for latex, simply put \\$ sign in between text",
"Solution_8": "if $ n$ even, so simply\r\nif $ n$ odd, then we can assume $ n$ be a prime odd.\r\nSo $ x \\plus{} y|1994$ and $ x \\plus{} y$ be a positive integer even.\r\n$ \\rightarrow x \\plus{} y \\equal{} 2,1994$\r\n1)Case $ x \\plus{} y \\equal{} 1994$\r\nWe have $ x^n \\plus{} y^n \\equal{} (x \\plus{} y)(x^{n \\minus{} 1} \\plus{} x^{n \\minus{} 2}y \\plus{} . . . \\plus{} xy^{n \\minus{} 2} \\plus{} y^{n \\minus{} 1}) > x \\plus{} y \\equal{} 1994$.\r\n2)Case $ x \\plus{} y \\equal{} 2$, assume $ x > 0$.\r\n$ \\rightarrow n|1992$.\r\n$ \\rightarrow n \\equal{} 3,83$.\r\n$ x^n \\minus{} (x \\minus{} 2)^n \\equal{} 1994$.\r\nIf $ x \\minus{} 2\\leq 0$, impossible.\r\nIf $ z \\equal{} x \\minus{} 2 > 0$, we have:\r\n$ 1994 \\equal{} (z \\plus{} 2)^n \\minus{} z^n\\geq 2^n$, so $ n \\equal{} 3$ and we have equation:\r\n$ (z \\plus{} 2)^3 \\minus{} z^3 \\equal{} 1994$ has no root on $ Z$.",
"Solution_9": "[quote=\"Thjch Ph4 Trjnh\"]We have $ x^n \\plus{} y^n \\equal{} (x \\plus{} y)(x^{n \\minus{} 1} \\plus{} x^{n \\minus{} 2}y \\plus{} . . . \\plus{} xy^{n \\minus{} 2} \\plus{} y^{n \\minus{} 1}) > x \\plus{} y \\equal{} 1994$.[/quote] \r\n\r\nActually, for odd $ n$: $ x^n \\plus{} y^n \\equal{} (x \\plus{} y)(x^{n \\minus{} 1} \\minus{} x^{n \\minus{} 2}y \\plus{} . . . \\minus{} xy^{n \\minus{} 2} \\plus{} y^{n \\minus{} 1})$ (alternating signs). Although, it doesn't really make a difference.\r\n\r\nAnyway, here is a complete solution: \r\n[hide]WLOG, let $ x \\ge y$. If $ x \\equal{} y$, then $ 2x^n \\equal{} 1994 \\Rightarrow x^n \\equal{} 997$, which is impossible. So, $ x > y$. \n\nIf $ x$ and $ y$ are even, then $ 2^n | x^n \\plus{} y^n \\equal{} 1994$, which is a contradiction since $ n \\ge 2$ and $ 4 \\nmid 1994$. \n\nIf one of $ x,y$ is even, and the other is odd, then $ x^n \\plus{} y^n \\equal{} 1994$ is odd, a contradiction. \n\nThus, $ x,y$ are both odd. \n\nCase I: $ n$ is even. \nThen, $ x^n , y^n > 0$. Also, $ (x,y)$ is a solution iff $ (\\pm x, \\pm y)$ are all solutions. So WLOG, let $ x > y > 0$. \n\nIf $ n \\ge 8$, and $ x \\ge 3$, then $ x^n \\plus{} y^n \\ge 3^8 \\equal{} 6561$, a contradiction. \nThus, $ 1 \\le y < x \\le 1$, a contradiction. \nTherefore, there are no solutions for even integers $ n \\ge 8$\n\nIf $ n \\equal{} 6$ and $ x \\ge 5$, then $ x^6 \\plus{} y^6 \\ge 5^6 \\equal{} 15625$, a contradiction. \nThus, $ 1 \\le y \\le x \\le 3 \\Rightarrow x^6 \\plus{} y^6 \\le 2 \\cdot 3^6 \\equal{} 1458$, a contradiction. \nTherefore, there are no solutions for $ n \\equal{} 6$. \n\nIf $ n \\equal{} 4$ and $ x \\ge 7$, then $ x^4 \\plus{} y^4 \\ge 7^4 \\equal{} 2401$, a contradiction. \nThus, $ 1 \\le y < x \\le 5 \\Rightarrow x^4 \\plus{} y^4 \\le 2 \\cdot 5^4 \\equal{} 1250$, a contradiction. \nTherefore, there are no solutions for $ n \\equal{} 6$. \n\nIf $ n \\equal{} 2$ and $ x \\ge 45$, then $ x^2 \\plus{} y^2 \\ge 45^2 \\equal{} 2025$, a contradiction. \nThus, $ 1 \\le y < x \\le 43$.\nNote that $ 2y^2 \\le x^2 \\plus{} y^2 \\equal{} 1994 \\le 2x^2$ \nTherefore, $ 33 \\le x \\le 43$ AND $ 1 \\le y \\le 31$. \nTesting all $ 11$ possible values of $ x$ yields $ (x,y) \\equal{} (37,25)$ as the only possible solution with $ x > y > 0$. \nThus, the only solutions for $ n \\equal{} 2$ are $ (x,y) \\equal{} (\\pm 37 , \\pm 25) ; (\\pm 25 , \\pm 37)$.\n\nCase II: $ n$ is odd. \nObviously, either $ x > y > 0$ or $ x > 0 > y$, and $ |x| > |y|$. \n\nSince $ |x|,|y|$ are both odd integers, $ |x| \\ge |y| \\plus{} 2$.\nSince $ x^n \\plus{} y^n \\equal{} 1994$, we must have $ |x|^n \\minus{} |y|^n \\le 1994$. \nSo, we have $ (|y| \\plus{} 2)^n \\minus{} |y|^n \\le |x|^n \\minus{} |y|^n \\le 1994$. \n\nIf $ n \\ge 7$, then we have: $ (|y| \\plus{} 2)^n \\minus{} |y|^n \\ge 3^7 \\minus{} 1^7 \\equal{} 2186$, a contradiction. \nThus, there are no solutions for odd $ n \\ge 7$. \n\nIf $ n \\equal{} 5$, and $ |y| \\ge 3$, then we have: $ (|y| \\plus{} 2)^5 \\minus{} |y|^5 \\ge 5^5 \\minus{} 3^5 \\equal{} 2882$, a contradiction. \nTherefore, we must have $ |y| \\equal{} 1$, and thus, $ x^5 \\pm 1 \\equal{} 1994$ which has no integer solution for $ x$. \nThus, there are no solutions for $ n \\equal{} 5$. \n\nIf $ n \\equal{} 3$, and $ |y| \\ge 19$, then we have: $ (|y| \\plus{} 2)^3 \\minus{} |y|^3 \\ge 21^3 \\minus{} 19^3 \\equal{} 2402$, a contradiction. \nTherefore, we must have $ |y| \\le 17$. \nSince $ y < x$, we have $ 2y^3 < x^3 \\plus{} y^3 \\equal{} 1994 \\Rightarrow y \\le 9$. \nTherefore, $ \\minus{} 17 \\le y \\le 9$. Testing all $ 13$ possible values of $ y$ yield no integer solutions for $ x$. \nThus, there are no solutions for $ n \\equal{} 3$. \n\nTherefore, the only integer solutions to $ x^n \\plus{} y^n \\equal{} 1994$ with $ n \\ge 2$ are: \n$ (x,y,n) \\equal{} (\\pm 37 , \\pm 25 , 2); (\\pm 25 , \\pm 37 , 2)$. [/hide]",
"Solution_10": "[quote]\nWe have $ x^n \\plus{} y^n \\equal{} (x \\plus{} y)(x^{n \\minus{} 1} \\plus{} x^{n \\minus{} 2}y \\plus{} . . . \\plus{} xy^{n \\minus{} 2} \\plus{} y^{n \\minus{} 1}) > x \\plus{} y \\equal{} 1994$.\n[/quote]\r\nSorry.:D\r\n$ x^n \\plus{} y^n \\equal{} (x \\plus{} y)(x^{n \\minus{} 1} \\minus{} x^{n \\minus{} 2}y \\plus{} . . . \\minus{} xy^{n \\minus{} 2} \\plus{} y^{n \\minus{} 1})$.\r\n$ 2(x^{n \\minus{} 1} \\minus{} x^{n \\minus{} 2}y \\plus{} . . . \\minus{} xy^{n \\minus{} 2} \\plus{} y^{n \\minus{} 1}) \\equal{} x^{n \\minus{} 1} \\plus{}x^{n \\minus{} 3}(x \\minus{} y)^2 \\plus{} . . . \\plus{}y^{n \\minus{} 3}(x \\minus{} y)^2 \\plus{} y^{n \\minus{} 1}\\geq x^{n \\minus{} 1} \\plus{} y^{n \\minus{} 1} \\geq |x| \\plus{} |y|\\geq 1994$",
"Solution_11": "[quote=\"soumik\"][size=150]x^2=4a+1=3b+1, thus a=3k, x^2=12h, for some natural h, similarly for b, but 12 does not divide 1994, hence no solutions.....[/size]\n[/quote]\r\n\r\nthis is wrong soumik",
"Solution_12": ":D I got a bit carried away....."
}
{
"Tag": [],
"Problem": "\u039b\u03bf\u03b9\u03c0\u03cc\u03bd , \u03b8\u03b1 \u03c0\u03ad\u03c3\u03b5\u03c4\u03b5 \u03b1\u03c0\u03cc \u03c4\u03b1 \u03c3\u03cd\u03bd\u03bd\u03b5\u03c6\u03b1 !\r\n\r\n \u039f\u03b9 \u03c4\u03bf\u03cd\u03c1\u03ba\u03bf\u03b9 , \u03bc\u03cc\u03bb\u03b9\u03c2 \u03ad\u03bc\u03b1\u03b8\u03b1\u03bd \u03cc\u03c4\u03b9 \u03c3\u03c4\u03b7\u03bd \u0395\u03bb\u03bb\u03ac\u03b4\u03b1 \u03ba\u03c5\u03ba\u03bb\u03bf\u03c6\u03bf\u03c1\u03b5\u03af \u03bc\u03b5 25 \u03b5\u03c5\u03c1\u03ce \u03c4\u03bf IMO COMPENDIUM, \u03c0\u03b1\u03c1\u03ae\u03b3\u03b3\u03b5\u03b9\u03bb\u03b1\u03bd - \u03c0\u03cc\u03c3\u03b1 \u03bb\u03ad\u03c4\u03b5 ; - [color=red]20000 [/color]\u03b1\u03bd\u03c4\u03af\u03c4\u03c5\u03c0\u03b1 !!! \u039f\u03b9 \u0388\u03bb\u03bb\u03b7\u03bd\u03b5\u03c2 - \u03bc\u03b1\u03b8\u03b7\u03bc\u03b1\u03c4\u03b9\u03ba\u03bf\u03af \u03ba\u03b1\u03b9 \u03bc\u03b1\u03b8\u03b7\u03c4\u03ad\u03c2 - \u03b1\u03b3\u03cc\u03c1\u03b1\u03c3\u03b1\u03bd \u03bb\u03b9\u03b3\u03cc\u03c4\u03b5\u03c1\u03b1 \u03b1\u03c0\u03cc 10 \u03b1\u03bd\u03c4\u03af\u03c4\u03c5\u03c0\u03b1.\r\n \u0395\u03af\u03c0\u03b1\u03c4\u03b5 \u03c4\u03af\u03c0\u03bf\u03c4\u03b1 ;;;\r\n\r\n \u039c\u03c0\u03ac\u03bc\u03c0\u03b7\u03c2",
"Solution_1": "\u03a4\u0399 \u039d\u0391 \u03a0\u0395\u0399\u0399\u03a3 ? \u0395\u0394\u03a9 \u0394\u0395\u039d \u03a7\u03a9\u03a1\u039f\u03a5\u039d \u039b\u039f\u0393\u0399\u0391 \u03a0\u0391\u03a1\u0391 \u039c\u039f\u039d\u039f \u03a4\u039f \u039c\u03a5\u039d\u0397\u039c\u0391 \u039d\u0391 \u03a0\u03a1\u039f\u03a4\u03a1\u0395\u03a8\u039f\u03a5\u039c\u0395 \u039f\u039b\u039f\u03a5\u03a3 \u03a4\u039f\u03a5\u03a3 \u039c\u0391\u0398\u0397\u03a4\u0395\u03a3 \u039a\u0391\u0399 \u03a4\u039f\u03a5\u03a3 \u039c\u0391\u0398\u0397\u039c\u0391\u03a4\u0399\u039a\u039f\u03a5\u03a3 \u039d\u0391 \u0391\u0393\u039f\u03a1\u0391\u03a3\u039f\u03a5\u039d \u03a4\u039f \u0392\u0399\u0392\u039b\u0399\u039f \u0391\u039b\u039b\u0391 \u03a4\u0399 \u039d\u0391 \u0393\u0399\u039d\u0395\u0399 ? \u0394\u0395\u039d \u0391\u03a6\u039f\u03a1\u0391 \u0397 \u03a0\u03a1\u039f\u039f\u0394\u039f\u03a3 \u03a4\u0397\u03a3 \u0395\u039b\u039b\u0391\u0394\u0391\u03a3 \u03a3\u03a4\u0391 \u039c\u0391\u0398\u0397\u039c\u0391\u03a4\u0399\u039a\u0391 \u03a9\u03a3 \u039f\u039c\u0391\u0394\u0391 :mad: \u03a0\u03a1\u039f\u0397\u0393\u0395\u0399\u03a4\u0391\u0399 \u0392\u039b\u0395\u03a0\u03a4\u0395 \u0397 \u0393\u0399\u039f\u03a5\u03a1\u039f\u0392\u0399\u0396\u0399\u039f\u039d :mad: :mad: \r\n\u03a7\u0391\u0399\u03a1\u0395\u03a4\u0391\u0399"
}
{
"Tag": [
"trigonometry",
"inequalities"
],
"Problem": "In $ \\triangle ABC$\r\n\r\n$ AB=t^2, AC=t, BC=1,t>1$\r\n\r\n$ \\angle A=x^o,\\angle C=x^o+60^o,\n\\angle B=120^o-2x^o$\r\n\r\nProve that $ t <\\sqrt{2} .$",
"Solution_1": "[hide]Here's my approach which pops up from my head:\nApplying laws of cosines we have: AB^2=1+t^2-2t.cosACB.Therefore, we have:t^4-1-t^2+2t.cosACB=0(3)\nNow, let's calculate cosACB. We see that cos(ACB)=sin(90-ACB)=sin(30-x)(2). In addition we also have:sinx/1=sin(x+60)/t, therefore,t=sin(x+60)/sinx.=cos(30-x)/sinx(1)\nPlugging (1) and (2) into (3) and bashing with algebra, we can find the value of x. From that, we can calculate the value of t, which will be1 to get:\nsqrt(3)/2.cosx>1/2.sinx, which means cotg(x)>1/sqrt(3).Using this bound to prove that: [sqrt(2)-1/2].sinx>sqrt(3)/2.cosx, which means: sqrt(3)/(sqrt(2)-1/2) 1$,\n\nthus, by (***) $ t^2 \\equal{} u \\plus{} \\sqrt {u^2 \\minus{} u \\plus{} 1} \\equal{} u \\plus{} \\sqrt {1 \\minus{} u(1 \\minus{} u)} < 1 \\plus{} \\sqrt {1 \\minus{} 0} \\equal{} 2$ as desired.[/hide]",
"Solution_7": "In Chinese statements version:\r\n\r\nwhere $ r\\equal{}t ,$"
}
{
"Tag": [
"search",
"algebra",
"binomial theorem"
],
"Problem": "Can anyone help me how to claculate 2137 to the 753th power?",
"Solution_1": "[quote=\"Zootieroaz\"]Can anyone help me how to claculate 2137 to the 753th power?[/quote]\r\n\r\nExact value? \r\n\r\nThat will be by computer (I don't think calculator can handle that). If you mean last *** digits (*** is number, like two digits, etc), then you can use Fermat's or Binomial Theorem.",
"Solution_2": "[quote=\"Silverfalcon\"][quote=\"Zootieroaz\"]Can anyone help me how to claculate 2137 to the 753th power?[/quote]\n\nExact value? \n\nThat will be by computer (I don't think calculator can handle that). If you mean last *** digits (*** is number, like two digits, etc), then you can use Fermat's or Binomial Theorem.[/quote]\r\n\r\nhow would u use the binomial theorem to find the last..say 3 digits of that number?",
"Solution_3": "Well, it doesn't \"always\" work. If you have nice numbers, it's pretty nice.\r\n\r\nIf you want to find out the last three digits of the expansion of $9^{9999}$, we can write this as $(10-1)^{9999}$ and use binomial theorem to consider only few terms that will matter in calculation.",
"Solution_4": "hmm so can anyone help me?",
"Solution_5": "the answer is 42 :-)\r\n\r\n(think hitchhiker's guide to galaxy) \r\n\r\nit would take a super computer to calculate that, then u'd probly spend a year reading the answer, so just be satisfied that it will be really really big.",
"Solution_6": "what exacltly is this binomial theorem ? :blush:",
"Solution_7": "you could always deal with it in terms of logs to make its calculation simpler, or get a quick estimate",
"Solution_8": "[quote=\"sen\"]what exacltly is this binomial theorem ? :blush:[/quote]\r\n\r\nhttp://www.google.com/search?client=firefox-a&rls=org.mozilla%3Aen-US%3Aofficial_s&hl=en&q=binomial+theorem&btnG=Google+Search\r\n\r\nIt takes TWENTY-EIGHT letters and TWO clicks to find the answer."
}
{
"Tag": [
"quadratics",
"algebra",
"polynomial"
],
"Problem": "How many solutions does the equation $ \\sqrt{3x}\\equal{}x\\sqrt{3}$ have?",
"Solution_1": "1) x=0\r\n2) we may divide by sqrt3, so we get x=1\r\n2 solutions",
"Solution_2": "Square both sides.\r\n\r\n$ 3x \\equal{} 3x^2$\r\n\r\nQuadratics have two solutions.",
"Solution_3": "Of course, one must be careful to look for double roots.",
"Solution_4": "Yes, the fundamental theorem of algebra really states: there exists, for a degree n polynomial, at most n roots. The multiplicities of all roots must add up to n.",
"Solution_5": "For example $ x^2\\minus{}2x\\equal{}\\minus{}1$ has only a solution $ x\\equal{}1$, although it's quadratic. This happens when its discriminant is 0.",
"Solution_6": "hello, from the range of definition we get $ x\\geq 0$, squaring both sides of the\r\nequation we have $ 3x\\equal{}3x^2$ or $ 0\\equal{}x(x\\minus{}1)$, from here we get the solutions\r\n$ x_1\\equal{}0$ or $ x_2\\equal{}1$, which both fulfill our equation.\r\nSonnhard.",
"Solution_7": "It's a countdown, you know quadratics have $ 2$ solutions, so you guess $ \\boxed{2}$...",
"Solution_8": "But that's a critical mistake. Quadratics need not have 2 solutions. A better way is to eye the solutions 0 and 1, then conclude that the answer is 2.",
"Solution_9": "[quote=\"AIME15\"]It's a countdown, you know quadratics have $ 2$ solutions, so you guess $ \\boxed{2}$...[/quote]\r\n\r\nLook at post #6. You would make a mistake in this case.",
"Solution_10": "But you know that it's not going to be the square of a binomial, just by looking at it...",
"Solution_11": "Generally when they ask for how many solutions there are in a quadratic in a countdown, you'd guess $ 2$ and most of the times that'll work. Of course, if you're wrong, the opponent basically has the answer :P \r\n\r\nOr, hit the buzzer and solve it in the three seconds you have",
"Solution_12": "Also, the question would have to say \"how many distinct roots\" to be clear, since one could interpret it two ways if there were a double root. This discussion is getting pointless, though...",
"Solution_13": "Either it's 0 or 1. \nThere are 2 solutions.",
"Solution_14": " x can equal both 0 or 1.\n"
}
{
"Tag": [],
"Problem": "\u03bd\u03b4\u03bf \u03bf \u03b1\u03c1\u03b9\u03b8\u03bc\u03bf\u03c2 99899...9 \u03bc\u03b5 \u03bd+3 \u03c8\u03b7\u03c6\u03af\u03b1 \r\n[u]\u03b4\u03b5\u03bd [/u] \u03b4\u03b9\u03b1\u03b9\u03c1\u03b5\u03af\u03c4\u03b1\u03b9 \u03bc\u03b5 \u03c4\u03bf 37. \r\n\r\n\r\n\r\n :)",
"Solution_1": "\u039f \u03b1\u03c1\u03b9\u03b8\u03bc\u03cc\u03c2 \u03bc\u03b5\u03c4\u03ac \u03b1\u03c0\u03cc \u03c0\u03c1\u03ac\u03be\u03b5\u03b9\u03c2 \u03b3\u03c1\u03ac\u03c6\u03b5\u03c4\u03b1\u03b9 999 * 10^\u03bd -1. \r\n \u03cc\u03bc\u03c9\u03c2 \u03c4\u03bf 37 | 999 \u03ba\u03b1\u03b9 \u03ac\u03c1\u03b1 \u03c4\u03bf 37 | 999* 10^\u03bd \u03ba\u03b1\u03b9 \u03b4\u03b5\u03bd \u03b4\u03b9\u03b1\u03b9\u03c1\u03b5\u03af \u03c4\u03bf \r\n 999 * 10^\u03bd -1 :P"
}
{
"Tag": [],
"Problem": "[youtube]cXz6UJL5oDA[/youtube]\r\n\r\nvery inspirational,\r\njorian hoover",
"Solution_1": "wow\r\njust wow\r\ni'll definitely try to do it",
"Solution_2": "i know.\r\n\r\ni showed it to my mom today, and she's like \"this makes you feel bad about arguing with me over a restaurant\" and i felt really bad\r\n\r\nso i put it on here\r\n\r\njorian",
"Solution_3": "where do i send cards to?",
"Solution_4": "This has also been posted on IBScrewed. \r\n\r\nHere's a link to that page. The address is listed on there. \r\n\r\n[url]http://www.ibscrewed.net/forum/viewtopic.php?t=13019[/url]",
"Solution_5": "Maybe we could get lots a people from aops to do this if we put this in as many forums as we could, without crossing the line of \"spam\"...."
}
{
"Tag": [],
"Problem": "Consider a series starting with digits 1 and 2 and then arriving at the next number by multiplying the previous two digits.\r\n\r\nSo next one is 1*2 that is 2.\r\n\r\nNext is 2*2 that is 4.\r\n\r\nNext is 2*4 that is 8 so on .... \r\n \r\nthus you have\r\n \r\n1 2 2 4 8 32 24 6 4 8 24 24 32 16 8 8 8........\r\n\r\n\r\nThe question is: how many times the digit 9 will appear during the compilation of the first 200 digits??\r\n\r\n :?: :?:",
"Solution_1": "$0$\r\nAlso, I believe that your sequence is incorrect.",
"Solution_2": "[quote=\"LordoftheMorons\"]$0$\nAlso, I believe that your sequence is incorrect.[/quote]\r\n\r\nI am not sure if 0 is the right answer and here is how I have derived the series...\r\n\r\n1 2 2 4 8 32 24 6 4 8 24 24 32 16 8 8 8........\r\n\r\n$1*2 = 2$\r\n$2*2 = 4$\r\n$2*4 = 8$\r\n$4*8 = 32$\r\n\r\nTill now the series is...\r\n1 2 2 4 8 32\r\n\r\nNow multiple $8*3$ and get $24$. Thus, the series updates to...\r\n1 2 2 4 8 32 24\r\n\r\nNow multiple $3*2$ and get $6$. Thus, the series updates to...\r\n1 2 2 4 8 32 24 6\r\n\r\nNow multiple $2*2$ and get $4$. Thus, the series updates to...\r\n1 2 2 4 8 32 24 6 4\r\n\r\nNow multiple $4*2$ and get $8$. Thus, the series updates to...\r\n1 2 2 4 8 32 24 6 4 8\r\n\r\nNow multiple $6*4$ and get $24$. Thus, the series updates to...\r\n1 2 2 4 8 32 24 6 4 24\r\n\r\nand so on....",
"Solution_3": "Oh, you only multiply the first digit? I see. I still can't see how a $9$ would be introduced into the series, though.",
"Solution_4": "[quote=\"LordoftheMorons\"]Oh, you only multiply the first digit? I see. I still can't see how a $9$ would be introduced into the series, though.[/quote]\r\n\r\nIts not that I only multiple the first digits... The multiplecation is digit by digit and go on increasing the series for 200 multiplications.\r\n\r\nI also doubt, but still cannot point to it why 9 will not be there?",
"Solution_5": "[hide=\"solution\"] $0$\nThe largest number you can get is $9*9=81$ So the only 9's attainable are 9 and 49, which come from $3*3$ or $7*7$. 3 is prime so there must be a number ending in 3 and a number beginning in 3 next to each other. There are no numbers ending in 3 that are attainable. Going to $7*7$, since 7 is prime, there must be a number ending in 7 and a number beginning in 7 next to each other. Again, no numbers ending in 7 are attainable. *Note: I am trying to find the first 9, so 27 is not attainable because it requires $9*3$. :D [/hide]",
"Solution_6": "[quote=\"lotrgreengrapes7926\"][hide=\"solution\"] $0$\nThe largest number you can get is $9*9=81$ So the only 9's attainable are 9 and 49, which come from $3*3$ or $7*7$. 3 is prime so there must be a number ending in 3 and a number beginning in 3 next to each other. There are no numbers ending in 3 that are attainable. Going to $7*7$, since 7 is prime, there must be a number ending in 7 and a number beginning in 7 next to each other. Again, no numbers ending in 7 are attainable. *Note: I am trying to find the first 9, so 27 is not attainable because it requires $9*3$. :D [/hide][/quote]\r\n\r\nThanks !"
}
{
"Tag": [
"function",
"geometric sequence",
"algebra unsolved",
"algebra"
],
"Problem": "find all function $f(x): N \\to N$satisfy :\r\n$f(m-n)f(m+n)=f(m^2) \\forall m>n$",
"Solution_1": "This proof works if $n$ can be equal $0$....\r\nBy putting $n=0$ we obtain $f(m)^2=f(m^2)$, and by putting $n=1$ we get $f(m-1)f(m+1)=f(m^2)=f(m)^2$ so \\[ \\frac{f(m+1)}{f(m)}=\\frac{f(m)}{f(m-1} \\] which means that $f(1), f(2)\\cdots$ form a geometric sequence!!! Then, function $f$ is defined as followes: \\[ f(n)=ab^{n-1} \\] where $a$ is natural (cos' $f(1)$ is natural) and $b$ is natural, as for each $n$ term $f(n)$ is natural...and as $f(n)^2=f(n^2)$, $a=1$...\r\nOf course if $0$ isn't natural... problem gets complicated ;) then by putting $n=m-2$ and $n=m-1$ we obtain \\[ \\frac{f(2)}{f(1)}=\\frac{f(2m-1)}{f(2m-2)} \\] and \\[ \\frac{f(3)}{f(2)}=\\frac{f(2m-2)}{f(2m-3)} \\] so if we define $f(1)=a$, $f(2)=f(1)q=aq$ and $f(3)=f(2)w=aqw$ then $f(4)=f(2^2)=f(1)f(3)=a^2qw$ and so on, we can construct next terms step by step... we can easily prove, that function defined like this has a form( for$n\\geq4$) \\[ f(n)=a^2q^{[\\frac{n-1}{2}]}w^{[\\frac{n-2}{2}]} \\] now all wehave to do is to find out which functions of this form are \"good\"... I'll do it ...maybe tomorrow ;)"
}
{
"Tag": [
"trigonometry",
"AMC",
"USA(J)MO",
"USAMO",
"inequalities",
"calculus",
"vector"
],
"Problem": "A man with height [b]h[/b] fires a shell with a gun with initial speed [b]v[/b] at an angle [b]A[/b] above the horizontal . The shell hits a target which is a distance of [b]s[/b] from the man . If there is only one possible value for [b]A[/b] for which this occurs , show that \r\n\r\n$v^2 = g (-h +\\sqrt{h^2 +s^2}) $",
"Solution_1": "Let $T$ the time required for hitting target by the shell, then we have $s=v(\\cos A)T\\Longleftrightarrow T=\\frac{s}{v\\cos A}$.\r\n\r\nThe hight of the shell at the time $t=T$ can be obtained by $y=h+v(\\sin A)T-\\frac{1}{2}gT^2=h+s(\\tan A)-\\frac{1}{2}g\\frac{s^2}{v^2\\cos ^ 2 A}$. Since $y\\geq 0$, we get\r\n\r\n$v^2\\geq \\frac{1}{2}gs^2\\cdot \\frac{1+\\tan ^ 2 A}{h+s\\tan A}=\\frac{1}{2}g\\cdot \\frac{s^2+(s\\tan A)^2}{h+s\\tan A}$\r\n\r\n$=\\frac{1}{2}g\\left(s\\tan A+h+\\frac{h^2+s^2}{s\\tan A+h}-2h\\right)$\r\n\r\n$\\geq \\frac{1}{2}g\\left(2\\sqrt{(s\\tan A+h)\\cdot \\frac{h^2+s^2}{s\\tan A+h}}-2h\\right)=g(\\sqrt{h^2+s^2}-h)$, we are done.",
"Solution_2": "wow, that is some nice algebraic insight... and a very nice question!\r\n\r\nThis is the type of question where those USAMO (and up) skills pay off, of which I don't have many.\r\n\r\nNice solution Kunny.",
"Solution_3": "Thanks, Spoon. :) \r\n\r\nkunny",
"Solution_4": "[quote=\"kunny\"]Let $T$ the time required for hitting target by the shell, then we have $s=v(\\cos A)T\\Longleftrightarrow T=\\frac{s}{v\\cos A}$.\n\nThe hight of the shell at the time $t=T$ can be obtained by $y=h+v(\\sin A)T-\\frac{1}{2}gT^2=h+s(\\tan A)-\\frac{1}{2}g\\frac{s^2}{v^2\\cos ^ 2 A}$. Since $y\\geq 0$, we get\n\n$v^2\\geq \\frac{1}{2}gs^2\\cdot \\frac{1+\\tan ^ 2 A}{h+s\\tan A}=\\frac{1}{2}g\\cdot \\frac{s^2+(s\\tan A)^2}{h+s\\tan A}$\n\n$=\\frac{1}{2}g\\left(s\\tan A+h+\\frac{h^2+s^2}{s\\tan A+h}-2h\\right)$\n\n$\\geq \\frac{1}{2}g\\left(2\\sqrt{(s\\tan A+h)\\cdot \\frac{h^2+s^2}{s\\tan A+h}}-2h\\right)=g(\\sqrt{h^2+s^2}-h)$, we are done.[/quote]\r\n\r\nWhat happened to the inequality, kunny? I followed your solution but you showed $v^2 \\geq g(\\sqrt{h^2+s^2}-h)$; this isn't precisely what was asked for.",
"Solution_5": "[quote=\"shyong\"] If there is only one possible value for [b]A[/b] for which this occurs[/quote]\r\nIf only 1 possible value of [b]A[/b] is allowed, then this must be the A such that the maximum horizontal distance is achieved. Maybe this has something to do with it :\\",
"Solution_6": "To Dr.No:my solution is exactly same as that of Spoon's.\r\n\r\nkunny",
"Solution_7": "[quote=\"kunny\"]Let $T$ the time required for hitting target by the shell, then we have $s=v(\\cos A)T\\Longleftrightarrow T=\\frac{s}{v\\cos A}$.\n\nThe hight of the shell at the time $t=T$ can be obtained by $y=h+v(\\sin A)T-\\frac{1}{2}gT^2=h+s(\\tan A)-\\frac{1}{2}g\\frac{s^2}{v^2\\cos ^ 2 A}$. Since $y\\geq 0$, we get\n\n$v^2\\geq \\frac{1}{2}gs^2\\cdot \\frac{1+\\tan ^ 2 A}{h+s\\tan A}=\\frac{1}{2}g\\cdot \\frac{s^2+(s\\tan A)^2}{h+s\\tan A}$\n\n$=\\frac{1}{2}g\\left(s\\tan A+h+\\frac{h^2+s^2}{s\\tan A+h}-2h\\right)$\n\n$\\geq \\frac{1}{2}g\\left(2\\sqrt{(s\\tan A+h)\\cdot \\frac{h^2+s^2}{s\\tan A+h}}-2h\\right)=g(\\sqrt{h^2+s^2}-h)$, we are done.[/quote]\r\n\r\nThats really nice kunny ! :D :D :D I face some problems when trying to group [b]v[/b] out since I use calculus to find the max of [b]A[/b] and then plugging that [b]A[/b]\r\ninside really makes it so ugly .... Your method of solving have solve my problem ! Thanks :D",
"Solution_8": "hmm... I don't believe you can solve for this A analytically, someone correct me if I'm wrong!\r\n\r\nAlso, kunny I didn't post a solution :blush:. Were you refering to my previous post about A being the ange yielding maximum horizontal distance?",
"Solution_9": "[quote=\"Spoon\"]hmm... I don't believe you can solve for this A analytically, someone correct me if I'm wrong!\n\nAlso, kunny I didn't post a solution :blush:. Were you refering to my previous post about A being the ange yielding maximum horizontal distance?[/quote]\r\n\r\nActually it can be found that the distance is maximize when \r\n$tanA = \\frac {v}{\\sqrt{v^2 + 2gh}}$",
"Solution_10": "I'd very much like to see it, if you could please?",
"Solution_11": "from law of conservation enery , \r\n\r\n1/2 *m*V^2 = 1/2*mv^2 + mgh \r\n\r\nV^2 = v^2 + 2gh ------(1) ( here , V is the velocity just before the shell strikes the target or ground )\r\n\r\nthen suppose the time taken is [b]t[/b] then \r\n\r\nV=gt + v and d=vcos[b]A[/b]t----(2)\r\n\r\ndrawing it in vector , and you will have a triangle PQR with QP = v , PR = gt , QR = V .\r\nDrop a perpendicular from Q to PR and let the intersecting point be M . Then angle PQM = [b]A[/b] . Let the area of the triangle PQR be S then \r\n\r\nS = 1/2 * vsin(90-A)gt = 1/2 * vgtcosA=1/2 *dg (from (2) )\r\n\r\nto maximize [b]d[/b] , you need to maximise S which tell us that triangle PQR must be right angle at Q . Hence \r\n\r\ntan A = PQ/QR = v / V = v / {sqrt (v^2 + 2gh )} ( from (1) ) ...Q.E.D",
"Solution_12": "Very simple,shyong.as if Feynman's explanation. :) \r\n\r\nkunny"
}
{
"Tag": [
"percent"
],
"Problem": "Express $ 42\\frac{6}{7}\\%$ as a common fraction.",
"Solution_1": "This can be re-written as $ \\frac{300}{7}$ Since this is a percent, put it over 100, giving you $ \\frac{300}{700}\\equal{}\\frac{3}{7}$"
}
{
"Tag": [
"probability",
"trigonometry",
"arithmetic sequence",
"number theory"
],
"Problem": "Well, I took another shot at the 12A recently to see how much I've improved (I became very interested in all types of problem solving shortly after I took the test) and found I am now much better. All of which is thanks to AoPS and every member on these boards ([i]YES[/i], even sharkman). Thanks guys.\r\n\r\n[hide=\"Problem #7\"]Let [i]a[/i], [i]b[/i], [i]c[/i], [i]d[/i], and [i]e[/i] be five consecutive terms in an arithmetic sequence, and suppose that $ a+b+c+d+e=30$. Which of the following can be found?\n(A): $ a$\n(B): $ b$\n(C): $ c$\n(D): $ d$\n(E): $ e$\n[hide=\"Correct answer:\"][i]C[/i]\n\nOw, I thought it was [i]A[/i]. I was under the apparent misconception that $ a+(a+n)+(a+2n)+(a+3n)+(a+4n)=30$. Or is the answer that $ c$ is the mean, and $ c=\\frac{30}{5}$?[/hide][/hide]\n[hide=\"Problem #12\"]Integers [i]a[/i], [i]b[/i], [i]c[/i], and [i]d[/i], not necessarily distinct, are chosen independently and at random from $ 0$ to $ 2007$, inclusive. What is the probability that $ ad-bc$ is even?\n(A) : $ \\frac{3}{7}$\n(B) : $ \\frac{7}{16}$\n(C) : $ \\frac{1}{2}$\n(D) : $ \\frac{9}{6}$\n(E) :$ \\frac{5}{8}$\n[hide=\"Correct answer:\"][i]E[/i]\n\nI got this. HOWEVER, I brute-forced this answer using a 13 or 14 line program that randomed [i]a[/i], [i]b[/i], [i]c[/i], and [i]d[/i]. It did the calculation then stored to a constantly updating variable for even or odd respectively 300 times. It got a probability of .634 or something which was really close to [i]E[/i]. Can someone show how to not have to BS this problem? Thanks in advance.[/hide][/hide]\n[hide=\"Problem #16\"]How many three-digit numbers are composed of three distinct digits such that one digit is the sum of the other two?\n\n(A) : $ 96$\n(B) : $ 104$\n(C) : $ 112$\n(D) : $ 120$\n(E) : $ 256$\n[hide=\"Correct answer:\"][i]C[/i]\n\nMy work: .... :read: $ \\Rightarrow$ :stretcher: . The only number I could pit out was $ 96$, and I was pretty sure that was wrong. What am I missing?[/hide][/hide]\n[hide=\"Thoughts:\"]\nOverall, of the 21 problems I completed, 3 were wrong, but I have already taken this test. Most of the problems I seem to be having trouble with include counting, and complex permutations/probability. I am partly finished with [i]Introduction to Number Theory[/i] (AoPS). Will [i]Introduction to Counting and Probability[/i] be highly beneficial?[/hide]\r\nThat's all for now, I still want to try to bust the trigonometry problem ([i]#17[/i]) on my own.",
"Solution_1": "[quote=\"igiul\"]\nOw, I thought it was [i]A[/i]. I was under the apparent misconception that $ a+(a+n)+(a+2n)+(a+3n)+(a+4n)=30$. .[/quote]\r\nThat's not a misconception. But, [hide]there you have 2 variables in one equation. So we can't pinpoint $ a$ or $ n$.\nHowver, simplify that and you get $ 5a+10n=30$ Thus, $ a+2n=6$. Since $ a+2n=C$, we have solved for C. \n\nYou can also intuitively solve this based on the fact that the sequence has to be \"balanced\" around 30/2 (EDIT: what!? I meant balanced around the middle).[/hide]",
"Solution_2": "Oh wow, so I [i]was[/i] right...almost. Thank ya sir.",
"Solution_3": "[quote=\"igiul\"][hide=\"Problem #12\"]Integers [i]a[/i], [i]b[/i], [i]c[/i], and [i]d[/i], not necessarily distinct, are chosen independently and at random from $ 0$ to $ 2007$, inclusive. What is the probability that $ ad-bc$ is even?\n(A) : $ \\frac{3}{7}$\n(B) : $ \\frac{7}{16}$\n(C) : $ \\frac{1}{2}$\n(D) : $ \\frac{9}{6}$\n(E) :$ \\frac{5}{8}$\n[hide=\"Correct answer:\"][i]E[/i]\n\nI got this. HOWEVER, I brute-forced this answer using a 13 or 14 line program that randomed [i]a[/i], [i]b[/i], [i]c[/i], and [i]d[/i]. It did the calculation then stored to a constantly updating variable for even or odd respectively 300 times. It got a probability of .634 or something which was really close to [i]E[/i]. Can someone show how to not have to BS this problem? Thanks in advance.[/hide][/hide][/quote]\n\n[hide=\"Step 1\"] In order for $ ad$ to be odd, both $ a, d$ must be odd, so the probability of $ ad$ being odd is $ \\frac{1}{4}$. Same with $ bc$. [/hide]\n[hide=\"Step 2\"] In order for $ ad-bc$ to be even, either both $ ad, bc$ are even or both are odd. [/hide]\n[hide=\"Step 3\"] The probability that $ ad, bc$ are both even is $ \\frac{3}{4}\\cdot \\frac{3}{4}$, and the probability that $ ad, bc$ are both odd is $ \\frac{1}{4}\\cdot \\frac{1}{4}$. [/hide]\n\n[quote=\"igiul\"]\n[hide=\"Problem #16\"]How many three-digit numbers are composed of three distinct digits such that one digit is the sum of the other two?\n\n(A) : $ 96$\n(B) : $ 104$\n(C) : $ 112$\n(D) : $ 120$\n(E) : $ 256$\n[hide=\"Correct answer:\"][i]C[/i]\n\nMy work: .... :read: $ \\Rightarrow$ :stretcher: . The only number I could pit out was $ 96$, and I was pretty sure that was wrong. What am I missing?[/hide][/hide][/quote]\r\n\r\nWell, your problem statement is not correct. One digit is supposed to be the [b]average[/b] of the other two. $ A$ is the correct answer for the sum problem."
}
{
"Tag": [
"puzzles"
],
"Problem": "When can you add two to eleven and get one as the correct \r\nanswer?",
"Solution_1": "[hide]on a clock[/hide]",
"Solution_2": "[hide]On a clock, 2 plus eleven is 1.[/hide]",
"Solution_3": "[quote=\"Cartman\"]When can you add two to eleven and get one as the correct \nanswer?[/quote]\r\n[hide]Try mod 1, mod 2, mod 3, mod 4, mod 6, and mod 12.\nOr, here's a proof that \n$2+11=1$\n$a=b$\n$a^{2}=b^{2}$\n$a^{2}-b^{2}=0$\n$a+b=0$\n$a=-b$\n$1=-1$\n$2=0$\n$1=0$\n$12=0$\n$11=-1$\n$2=2$\n$2+11=1$[/hide]",
"Solution_4": "lol @ proof.\r\nBut yeah, either on a clock, or in other bases.",
"Solution_5": "[quote=\"davidyko\"]lol @ proof.\nBut yeah, either on a clock, or in other bases.[/quote]\r\n\r\nNot other bases, just other moduluses. (Moduli?)"
}
{
"Tag": [
"MATHCOUNTS",
"AMC 10"
],
"Problem": "I think our chapter competition is on the 3rd. How should I prepare? I have limited time (other important things, such as homework and stuff), and I have absolutely no idea as to how to prepare.",
"Solution_1": "Get good sleep the night before, and practice until saturday, starting now - doesn't matter if it's online here, with a teacher, a friend, or by yourself with a book.",
"Solution_2": "[quote=\"Zenox\"]Get good sleep the night before, and practice until saturday, starting now - doesn't matter if it's online here, with a teacher, a friend, or by yourself with a book.[/quote]\r\n\r\nGet a good sleep the night before the competition is really important, and do not panic before the competition! You still have several days to get ready. Good luck for the competition! :P",
"Solution_3": "What you two said is important, but I think he wants to know how to practice for the competition. I would suggest doing a lot of previous chapters and states (found at [url]http://www.mathcounts.org[/url]), and regularly going here and doing Mathcounts level problems.",
"Solution_4": "Maybe what I'm doing -- go to the AMC 10 website and download those worksheets covering types of problems and doing them without the choices and brushing up on stuff you're not good at.",
"Solution_5": "Seeing as it's in 5 days, you shouldn't be preparing anymore -- resting would be the best way to prepare.\r\n\r\nBut seriously, I doubt you need to worry about making States or doing well unless you fall into a coma on Friday or something...",
"Solution_6": "whatever you do, just don't cram the night before.\r\ni have personal experince, and Ubemaya knows exactly what i am talking about.................",
"Solution_7": "[quote=\"Ubemaya\"]Seeing as it's in 5 days, you shouldn't be preparing anymore -- resting would be the best way to prepare.\n\nBut seriously, I doubt you need to worry about making States or doing well unless you fall into a coma on Friday or something...[/quote]\r\n\r\nYeah I agree, if you want to prepare don't start 5 days before...........",
"Solution_8": "@vishalarul: you have done mathcounts before, havent you? don't you go to miller? if you do, then you've done more than enough preparation.",
"Solution_9": "No, he and I go to CMS.",
"Solution_10": "[quote=\"i_like_pie\"]No, he and I go to CMS.[/quote]\r\n\r\n\r\nLooks like you guys have a strong team.....",
"Solution_11": "uhh ohh\r\nthat's at least x-1(where x>10 and is an integer)teams better than ours.........we're going to lose\r\n1. Miller\r\n2. Harker\r\n3. Hyde\r\n4. Cupertino\r\n5. Kennedy\r\n6. Challenger Berryesa\r\n7. William Hopkins\r\n8. (reserved)\r\n9. (reserved)\r\n10.(reserved)\r\n......\r\nx. my team",
"Solution_12": "[quote=\"anirudh\"]Maybe what I'm doing -- go to the AMC 10 website and download those worksheets covering types of problems and doing them without the choices and brushing up on stuff you're not good at.[/quote]\r\nwhere? i couldn't see them anywhere.",
"Solution_13": "[quote=\"i_like_pie\"]No, he and I go to CMS.[/quote]\r\nthats as clear as me saying i go to a school in central new jersey.\r\nu no, im gonna count how many schools in this country have CMS as its abreviation. I know it stands for Cupertino MS in ilikepie's case, but there are many more. like Community MS in my district, etc\r\n\r\nedit- on the first 10 results from googling CMS middle school got 9 middle schools' websites\r\ni estimate there will be over 30 Middle schools called CMS\r\nthere are actually 2 middle schools with the string \"Community Middle School\" in it.\r\n\r\nif any1's wonderin, i go to a school with abreviation GMS, which is actually short for TGMS",
"Solution_14": "[quote=\"ProtestanT\"][quote=\"i_like_pie\"]No, he and I go to CMS.[/quote]\nthats as clear as me saying i go to a school in central new jersey.\nu no, im gonna count how many schools in this country have CMS as its abreviation. I know it stands for Cupertino MS in ilikepie's case, but there are many more. like Community MS in my district, etc\n\nedit- on the first 10 results from googling CMS middle school got 9 middle schools' websites\ni estimate there will be over 30 Middle schools called CMS\nthere are actually 2 middle schools with the string \"Community Middle School\" in it.\n\n\nif any1's wonderin, i go to a school with abreviation GMS, which is actually short for TGMS[/quote]yep, i agree. all three middle schools in Carmel are abbreviated with CMS.\r\n\r\nClay MS, Creekside MS, Carmel MS. :P",
"Solution_15": "There's a ton of WMSs in my area.",
"Solution_16": "wow...too much santaclara chapter competition...they should send a lot more teams, considering how big and compeitive our chapter is",
"Solution_17": "Our chapter competition is extremely competitive!\r\n\r\nThat's why I need to prepare quickly.\r\n\r\nAMC 12's are the way for me.",
"Solution_18": "[quote=\"anirudh\"]Our chapter competition is extremely competitive!\n\nThat's why I need to prepare quickly.\n\nAMC 12's are the way for me.[/quote]\r\n\r\nWhere is your chapter???",
"Solution_19": "The way to prepare:\r\n\r\nDo a ton of mathcounts stuff:\r\n\r\n\r\nLike problem of the day, go figure challenge, team round with your team, past sprints and targets, stuff like that...\r\n\r\n\r\nP.S. Squeeze in an AMC 10. But if you get perfects on AMC 10, try doing National sprints and targets.",
"Solution_20": "No. AMC 10's last 5 are waay harder than MC national sprints/targets. About 10-15 are targets and 1-7 are sprints\r\n\r\nTry amc 12's.",
"Solution_21": "[quote=\"anirudh\"]No. AMC 10's last 5 are waay harder than MC national sprints/targets. About 10-15 are targets and 1-7 are sprints\n\nTry amc 12's.[/quote]\r\n1. AMC 10s last five are on par with national targets.\r\n2. If you did believe that, why would you be recommending amc 12s?\r\n\r\n[size=75][color=darkred]Edited by nebula42.[/color][/size]",
"Solution_22": "don't cram too much on the last few days, else you go brain dead................\r\nthree food tips:\r\n1. chew gum. Gum has been known to increase the attemtion span and concentration up to 50%.\r\n2. eat bananas. I have no idea why, but eating bananas on average increases test score. It might be the potassium.\r\n3. no fatty foods. Starting the day before, DO NOT eat fatty foods like pizza, chips, etc. Fat clogs arteries that decrease the amount of blood flow to the brain.\r\none exercise tip:\r\n1. do a workout before the test. Only a small workout is needed, such as Ubemaya's Workout (one pushup and one situp) just to warm up the body.\r\n\r\nI know these tips may sound crazy, but they really do help and have been proven statistically. \r\n\r\nPersonal experience: at Miller Math Marathon, i followed all four steps and ended up x-th place. Judging by the fact that i expected to bomb the test and by how dumb i am, i think they really worked.",
"Solution_23": "[quote=\"davidlizeng\"]don't cram too much on the last few days, else you go brain dead................\nthree food tips:\n1. chew gum. Gum has been known to increase the attemtion span and concentration up to 50%.\n2. eat bananas. I have no idea why, but eating bananas on average increases test score. It might be the potassium.\n3. no fatty foods. Starting the day before, DO NOT eat fatty foods like pizza, chips, etc. Fat clogs arteries that decrease the amount of blood flow to the brain.\none exercise tip:\n1. do a workout before the test. Only a small workout is needed, such as Ubemaya's Workout (one pushup and one situp) just to warm up the body.\n\nI know these tips may sound crazy, but they really do help and have been proven statistically. \n\nPersonal experience: at Miller Math Marathon, i followed all four steps and ended up x-th place. Judging by the fact that i expected to bomb the test and by how dumb i am, i think they really worked.[/quote]\r\nYAY!!! easy workouts\r\n\r\nwhat do you do if you aren't allowed gum?\r\n\r\ni've heard bannanas and chocolate are good but whatever",
"Solution_24": "@bpms: I meant AMC12's. Sorry. I accidentally typed 10. :blush:",
"Solution_25": "[quote=\"#H34N1\"]Maybe what I'm doing -- go to the AMC 10 website and download those worksheets covering types of problems and doing them without the choices and brushing up on stuff you're not good at.[/quote]\r\nwhat did you mean by this.....?",
"Solution_26": "There's a webpage with a list of worksheets that are downloadable on the amc website.",
"Solution_27": "[quote=\"funcia\"][quote=\"davidlizeng\"]don't cram too much on the last few days, else you go brain dead................\nthree food tips:\n1. chew gum. Gum has been known to increase the attemtion span and concentration up to 50%.\n2. eat bananas. I have no idea why, but eating bananas on average increases test score. It might be the potassium.\n3. no fatty foods. Starting the day before, DO NOT eat fatty foods like pizza, chips, etc. Fat clogs arteries that decrease the amount of blood flow to the brain.\none exercise tip:\n1. do a workout before the test. Only a small workout is needed, such as Ubemaya's Workout (one pushup and one situp) just to warm up the body.\n\nI know these tips may sound crazy, but they really do help and have been proven statistically. \n\nPersonal experience: at Miller Math Marathon, i followed all four steps and ended up x-th place. Judging by the fact that i expected to bomb the test and by how dumb i am, i think they really worked.[/quote]\nYAY!!! easy workouts\n\nwhat do you do if you aren't allowed gum?\n\ni've heard bannanas and chocolate are good but whatever[/quote]\r\n\r\nchew it anyway...\r\nonce the test starts, there's noway you'll be forced to stop just to spit out gum. personally, i don't care if i get in trouble for chewing gum.........."
}
{
"Tag": [
"modular arithmetic",
"number theory proposed",
"number theory"
],
"Problem": "Find all pairs of possive integers $ (x;y)$ such that $ y^x\\minus{}1\\equal{}(y\\minus{}1)!$",
"Solution_1": "A less general problem was posted before:\r\nhttp://www.mathlinks.ro/viewtopic.php?t=187830\r\nbut onfortunately no proof was produced. Could you please let us know what is the source of the problem?",
"Solution_2": "See http://www.mathlinks.ro/viewtopic.php?t=150674 (containing a small but easy to fix error).",
"Solution_3": "a similar solution to what ZetaX snet the link is:\r\n\r\nlet $ d\\mid y$ and $ d < y$ then from the equation we have:\r\n\r\n$ d\\mid (y \\minus{} 1)! \\minus{} 1$\r\n\r\nbut on the other hand:\r\n\r\n$ d < y\\Rightarrow d\\leq y \\minus{} 1\\Rightarrow d\\mid (y \\minus{} 1)!$\r\n\r\nthus:\r\n\r\n$ d \\equal{} 1$\r\n\r\nso $ y$ must be a prime number...\r\n\r\nnow according to WILSON theorem we have:\r\n\r\n$ (y \\minus{} 1)!\\equiv \\minus{} 1\\pmod y$\r\n\r\nso we have:\r\n\r\n$ (y \\minus{} 1)! \\minus{} 1 \\equal{} y^x\\Rightarrow \\minus{} 1 \\minus{} 1\\equiv 0\\pmod y\\Rightarrow y\\mid 2\\Rightarrow y \\equal{} 2$\r\n------------------------------\r\nalso another solution which belongs to my friend (ML id:only math) is the following one:\r\n\r\n$ (y \\minus{} 1)! \\minus{} 1 \\equal{} y^x$\r\n\r\n$ \\Rightarrow (y \\minus{} 1)! \\minus{} 2 \\equal{} y^x \\minus{} 1$\r\n\r\n$ \\Rightarrow (y \\minus{} 1)! \\minus{} \\left(y^x \\minus{} 1\\right) \\equal{} 2$\r\n\r\n$ \\Rightarrow (y \\minus{} 1)! \\minus{} (y \\minus{} 1)\\left(y^{x \\minus{} 1} \\plus{} y^{x \\minus{} 2} \\plus{} \\ldots \\plus{} 1\\right) \\equal{} 2$\r\n\r\nbut note that $ y \\minus{} 1\\mid LHS$ so we must have $ y \\minus{} 1\\mid RHS\\Rightarrow y \\minus{} 1\\mid 2$\r\n--------------------------------\r\n\r\nthe problem was also posted here:\r\n[url]http://www.mathlinks.ro/Forum/viewtopic.php?t=183998[/url]",
"Solution_4": "BaBaK Ghalebi i do not really understand your solution. you wrote that by wilson-theorem we have $ (y\\minus{}1)! \\equiv \\minus{}1 \\bmod{y}$, where $ y \\in \\mathbb{P}$ and using this in the equation we get $ (y\\minus{}1)! \\minus{} 1 \\equiv \\minus{}2 \\not\\equiv 0 \\bmod{y}$. But the equation is $ (y\\minus{}1)! \\plus{} 1 \\equal{} y^x$ and by wlson we get $ (y\\minus{}1)! \\plus{} 1 \\equiv 0 \\bmod{y}$.",
"Solution_5": "oh,what a stupid mistake,both of my solutions are wrong,those solutions are for the following equation:\r\n\r\n$ (y\\minus{}1)!\\minus{}1\\equal{}y^x$\r\n\r\nsorry :oops:",
"Solution_6": "I'm sure the following solution is valid:\r\nas I said in my previous post,according to wilson theorem we get that $ y$ must be prime,now let $ y > 5$ and $ x\\not = 0$,note that:\r\n\r\n$ (y - 1)! = (y - 1)\\times (y - 2)\\times\\ldots\\times (\\frac {y - 1}2)\\times\\ldots\\times 2\\times 1$\r\n\r\n$ \\Rightarrow y + 1\\mid (y - 1)!$\r\n\r\nnow according to our equation we have:\r\n\r\n$ 1\\equiv ( - 1)^x\\pmod{y + 1}$\r\n\r\nso $ x$ must be even,so let $ x = 2k$ where $ k\\in\\mathbb{Z}$ now we have:\r\n\r\n$ (y - 1)! = (y^k - 1)(y^k + 1)$\r\n\r\n$ \\Rightarrow (y - 2)! = (y^{k - 1} + y^{k - 2} + \\ldots + y + 1)(y^k + 1)$ (*)\r\n\r\nbut note that $ y - 1$ is a composite number so $ y - 1\\mid (y - 2)!$ thus according to (*) we get that:\r\n\r\n${ 0\\equiv (\\underbrace{1 + 1 + \\ldots + 1}_{k\\textrm{ times }})(1 + 1)}\\pmod {y - 1}$\r\n\r\n$ \\Rightarrow 0\\equiv 2k\\pmod {y - 1}$\r\n\r\n$ \\Rightarrow 0\\equiv x\\pmod {y - 1}$\r\n\r\n$ \\Rightarrow x\\geq y - 1$\r\n\r\nbut it's easy to see that:\r\n\r\n$ y^{x}\\geq y^{y - 1} > (y - 1)! + 1$\r\n\r\nwhich is a contradiction,so we must check the cases $ y = 2,3,5$ and also the case $ x = 0$ which gives us the solution:\r\n\r\n$ (y,x) = (2,1),(3,1),(5,2)$"
}
{
"Tag": [
"LaTeX",
"inequalities unsolved",
"inequalities"
],
"Problem": "Show that if x,y,z>0 and x+y+z=1 then \r\n64<=(1+1/x )(1+1/y )(1+1/z )\r\n\r\nAlso, can anyone help me with the coding of mathematical symbols? Is there a place for a tutorial on this page somewhere? Thanks!",
"Solution_1": "The first solution that comes to mind is using Holder and obtaining that $ RHS^{1/3}>\\equal{}1\\plus{}1/ \\sqrt[3]{xyz}$ so it suffices to prove that $ 1\\plus{}1/ \\sqrt[3]{xyz}>\\equal{}4\\equal{}>1>\\equal{}3\\sqrt[3]{xyz}\\equal{}>x\\plus{}y\\plus{}z>\\equal{}3\\sqrt[3]{xyz}$ which is true by AM GM inequality.\r\n\r\nAs far as latex is concerned (although I do't know it very well myself) you can start here: http://www.artofproblemsolving.com/LaTeX/AoPS_L_About.php\r\n\r\nAnd where I used $ \\equal{}>$ i meant it in both directions, but I don't know how to type it!",
"Solution_2": "Also possible solution to this simple question is:\r\n\r\n$ \\prod (1 \\plus{} \\frac {1}{x}) \\equal{} \\prod (1 \\plus{} \\frac {x \\plus{} y \\plus{} z}{x}) \\equal{} \\prod (1 \\plus{} 1 \\plus{} \\frac {y}{x} \\plus{} \\frac {z}{x})\\geq \\prod 4 \\sqrt [4]{\\frac {yz}{x^{2}}} \\equal{} 4^{3} \\equal{} 64$",
"Solution_3": "[quote=\"salonikios\"]\n\n\nAnd where I used $ \\equal{} >$ i meant it in both directions, but I don't know how to type it![/quote]\r\n\r\nGo \\iff to get $ \\iff$."
}
{
"Tag": [
"geometry",
"rectangle",
"combinatorics proposed",
"combinatorics"
],
"Problem": "Here is a cute one :\r\n\r\nIs it possible to colour all grid points in the plane white and red so that no rectangle with vertices on grid points of one colour and sides parallel to the grid lines has area from the set $\\{1,2,4,8, \\cdots , 2^n,\\cdots, \\}$?\r\n\r\nPierre.",
"Solution_1": "[hide=Solution]\nYes. Using Cartesian coordinates, color the point $(x,y)$ white iff $x+y\\equiv 0\\pmod{3}$, otherwise color it red.\n\nSuppose to the contrary that such rectangle exists.\nLet $(p,q),(r,q),(p,s),(r,s)$ be the four vertices of the rectangle. We have $|(p-r)(q-s)|\\in \\{ 1,2,4,\\cdots \\}$.\nSince $3\\nmid p-r$, the points $(p,q),(r,q)$ can't be simultaneously white. So the four vertices must be red.\nAlso, we have $p+q\\not\\equiv r+q\\pmod{3}$. This means $(p+q)+(r+q)\\equiv 1+2\\equiv 0 \\pmod{3}$.\nSimilarly, $(p+s)+(r+s)\\equiv 0\\pmod{3}$. But this leads to $q\\equiv s\\pmod{3}\\implies 3\\mid q-s$ which is impossible.\n[/hide]"
}
{
"Tag": [],
"Problem": "Eto, nakon IMO-a dolazi i ne\u0161to sitno za nas (ve\u0107im dijelom) umirovljene natjecatelje - IMC se ove godine odr\u017eava u Budimpe\u0161ti od 25. do 30.7.\r\n\r\nNas iz Zagreba dolazi 10, kao \u0161to sam napomenuo ve\u0107 u jednom topicu, a u jednom drugom sam nam spomenuo i imena:\r\n\r\nNikola Ad\u017eaga, druga godina\r\nGoran Dra\u017ei\u0107, tre\u0107a godina\r\nNikola Grubi\u0161i\u0107, \u010detvrta godina\r\n\u017deljko Kereta, tre\u0107a godina \r\nRudi Mrazovi\u0107, \u010detvrta godina\r\nMarin Mi\u0161ur, druga godina\r\nMelkior Ornik, prva godina\r\nPetar Sirkovi\u0107, druga godina \r\nSa\u0161a Stanko, druga godina\r\nLuka \u017duni\u0107, druga godina\r\n\r\n(leader nam je Matija Ba\u0161i\u0107)\r\n\r\nKao da sam negdje pro\u010ditao da dolazi i \u010detvero ljudi iz Ljubljane, a hana je ne\u0161to spominjala da ide i njih \u010detvero.\r\n\r\nEto, toliko. Vi\u0161e nemam \u0161to pametno da ka\u017eem, pa se idem povu\u0107i da gledam Seinfeld.",
"Solution_1": "Jedan odu\u0161evljeni \u010ditatelj vam \u017eeli sre\u0107u na takmi\u010denju i sretan put :wink:",
"Solution_2": "I ovaj \u010ditalac se odu\u0161evljeno pridru\u017euje svim lijepim \u017eeljama",
"Solution_3": "Cestitke svim osvajacima medalja (i onima koji su to zamalo postali)",
"Solution_4": "\u010cestitke svima: \u010dini mi se da je svako ne\u0161to donio, od Frst Prajza do Onorabl Men\u0161na, tako da ste opravdali na\u0161a o\u010dekivanja :lol:",
"Solution_5": "Hvala na \u010destitkama, bilo je jako lijepo, a i rezultatom sam zadovoljan (zeznuli su mi score u zavr\u0161nim rezultatima :roll:, ali oprostit \u0107u im kad mi po\u0161alju moju zaslu\u017eenu tre\u0107u nagradu :winner_third:)."
}
{
"Tag": [
"number theory unsolved",
"number theory"
],
"Problem": "Let a,b be integer integers such that ab-1 divides a^{2}+b^{2}.Show that:\r\n\\frac{a^{2}+b^{2}}{ab-1}=5",
"Solution_1": "Very very very often posted before.",
"Solution_2": "Very very very well-known method to solve it...",
"Solution_3": "Does anyone try it?\r\nPS:ZetaX,do you know Erich Ludendorff?",
"Solution_4": "very very very very very easy:[url]http://en.wikipedia.org/wiki/Erich_Ludendorff[/url]:wink:",
"Solution_5": "From history lessons, I'm not that old to be able to have lived with him ;)"
}
{
"Tag": [
"inequalities"
],
"Problem": "Let $ x,y > 0$ such that $ x^5 \\plus{} y^5 \\equal{} 2$ . Prove that :\r\n\r\n$ (x^2 \\plus{} y^6)(x^8 \\plus{} y^4) \\plus{} 2\\ge x^{10} \\plus{} y^{10}$",
"Solution_1": "By Cauchy Schwartz inequality \r\n\r\n$ (x^2 \\plus{}y^6)(x^8 \\plus{}y^4) \\geq (x^5 \\plus{}y^5 )^2$\r\n\r\n$ (x^5 \\plus{}y^5 )^2 \\plus{}2 \\geq x^{10} \\plus{}y^{10}$\r\n\r\n$ (x^5 \\plus{}y^5)^2 \\equal{}x^{10} \\plus{}2x^5 y^5 \\plus{}y^{10}$\r\n\r\nThen \r\n\r\n$ (x^5 \\plus{}y^5 )^2 \\plus{}2 \\equal{}x^{10} \\plus{}2x^5 y^5 \\plus{}y^{10} \\plus{}2 \\geq x^{10} \\plus{}y^{10} \\iff 2x^5 y^5 \\plus{}2 > 0$,no equality case \r\n :wink:",
"Solution_2": "Is there something wrong with the problem?\r\n\r\n$ (x^2\\plus{}y^6)(x^8\\plus{}y^4)\\equal{}x^{10}\\plus{}x^2y^4\\plus{}x^8y^6\\plus{}y^{10}>x^{10}\\plus{}y^{10}$\r\n\r\nSo apparently the result is true for any x,y...",
"Solution_3": "[quote=\"Pain rinnegan\"]Let $ x,y > 0$ such that $ x^5 \\plus{} y^5 \\equal{} 2$ . Prove that :\n\n$ (x^2 \\plus{} y^6)(x^8 \\plus{} y^4) \\plus{} 2\\ge x^{10} \\plus{} y^{10}$[/quote]\r\n\r\n\r\n this isn\u00b4t stronger than cauchy, since there is obviously no equality case...",
"Solution_4": "[quote=\"v235711\"][quote=\"Pain rinnegan\"]Let $ x,y > 0$ such that $ x^5 \\plus{} y^5 \\equal{} 2$ . Prove that :\n\n$ (x^2 \\plus{} y^6)(x^8 \\plus{} y^4) \\plus{} 2\\ge x^{10} \\plus{} y^{10}$[/quote]\n\n\n this isn\u00b4t stronger than cauchy, since there is obviously no equality case...[/quote]\r\n\r\nThe correct version\r\n\r\n$ (x^2 \\plus{} y^6)(x^8 \\plus{} y^4) \\ge x^{10}\\plus{}y^{10}\\plus{}2$ with $ x^5\\plus{}y^5\\equal{}2$"
}
{
"Tag": [
"greatest common divisor",
"number theory",
"least common multiple",
"prime factorization",
"relatively prime"
],
"Problem": "Prove that the product of the greatest common factor and lowest common multiple of two numbers is equal to the product of the numbers.\r\n\r\n[hide=\"Hint\"]Prime factorization[/hide]",
"Solution_1": "[hide]Let the numbers be $A$ and $B$. Let $g$ denote the GcF of $A$ and $B$. Then let $A$ and $B$ equal $gx$ and $gy$, respectively, where $x$ and $y$ are relatively prime integers. This means that the LcM of $A$ and $B$ is $gxy$. \n\n$gx\\cdot gy=g\\cdot gxy$, so the statement is true.[/hide]",
"Solution_2": "[hide=\"A formal construction of the proof by prime factorization\"] Let $e_{p}(n)$ denote the exponent of the prime $p$ in the prime factorization of a positive integer $n$. I give some unproven lemmas involving this function.\n\n[b]Lemma:[/b] $e_{p}(ab) = e_{p}(a)+e_{p}(b)$\n\n[b]Lemma:[/b] $e_{p}(a) = e_{p}(b) \\forall p \\in \\mathbb{P}\\Leftrightarrow a = b$\n\n[b]Lemma:[/b] $e_{p}( gcd(a, b) ) = min( e_{p}(a), e_{p}(b) )$\n\n[b]Lemma:[/b] $e_{p}( lcm(a, b) ) = max( e_{p}(a), e_{p}(b) )$\n\nLastly, recall that given two numbers $x, y$,\n\n$min(x, y)+max(x, y) = x+y$\n\nThen the proof is rather simple, and takes the following form:\n\n$e_{p}( gcd(a, b) )+e_{p}( lcm(a, b) ) = e_{p}(a)+e_{p}(b) \\forall p \\in \\mathbb{P}$ [/hide]",
"Solution_3": "For this to work, you must allow for zero exponents.",
"Solution_4": "$(a,b)\\cdot[a,b]=a\\cdot b$\r\n\r\n[hide]Let $(a,b)=k$. Then we have $a=m\\cdot k$, $b=n \\cdot k$ and $[a,b]=p\\cdot k$.\n$k^{2}\\cdot p = k^{2}\\cdot m \\cdot n$. We divide by $k$ so that $p=m \\cdot n$. Replacing, $k^{2}\\cdot m \\cdot n = k^{2}\\cdot m \\cdot n$ which is true and i'm stuck.[/hide]",
"Solution_5": "[quote=\"icx\"]$[a,b]=p\\cdot k$.\n$k^{2}\\cdot p = k^{2}\\cdot m \\cdot n$. [/quote]\r\n\r\nEr, you're using the statement you're trying to prove."
}
{
"Tag": [
"function",
"\\/closed"
],
"Problem": "Can one of you please post an announcement that spans all of the forums telling everyone to use the spoiler button? I understand that the spoiler button either wasnt at mathlinks or wasnt used much, but its appreciated tremendously here.\r\nThanx.",
"Solution_1": "Working on a lot of these little things. In the MathLinks view, users cannot (yet) even put things into spoiler (so don't blame them for not doing it!) without explicitly typing the tags. What we're looking into is making 'reveal all spoilers' an option in the Options panel, so that people who hate having to highlight answers won't have them hidden in the first place. This should make everyone somewhere close enough to happy.",
"Solution_2": "[quote=\"jelyman\"]Can one of you please post an announcement that spans all of the forums telling everyone to use the spoiler button? I understand that the spoiler button either wasnt at mathlinks or wasnt used much, but its appreciated tremendously here.\nThanx.[/quote]Basically people do not want in the advanced section the spolier. It is a general belief that at that level you do not need to hide information in that manner, as you can refrain yourself from seeing if you wish so. \r\nSo what I am asking is that people should post without spoilers in the Advanced Section and College Playground. It would be highly appreciated.\r\n\r\nPS you can call me Valentin, I like that better than Vornicu :)",
"Solution_3": "[quote=\"Valentin Vornicu\"][quote=\"jelyman\"]Can one of you please post an announcement that spans all of the forums telling everyone to use the spoiler button? I understand that the spoiler button either wasnt at mathlinks or wasnt used much, but its appreciated tremendously here.\nThanx.[/quote]Basically people do not want in the advanced section the spolier. It is a general belief that at that level you do not need to hide information in that manner, as you can refrain yourself from seeing if you wish so. \nSo what I am asking is that people should post without spoilers in the Advanced Section and College Playground. It would be highly appreciated.\n\nPS you can call me Valentin, I like that better than Vornicu :)[/quote]\r\n\r\nI'm not sure who are you talking about when you say its a 'general belief', but this is definitely not true of the AoPS people, and as you don't have spoiler at Mathlinks I do not understand what you mean. At AoPS, in every problem forum including the advanced forum, the spoiler function is extremely appreciated and used. \r\nWe would all appreciate it if you continue using the system that has worked so well for us.",
"Solution_4": "[quote=\"rrusczyk\"]Working on a lot of these little things. In the MathLinks view, users cannot (yet) even put things into spoiler (so don't blame them for not doing it!) without explicitly typing the tags. What we're looking into is making 'reveal all spoilers' an option in the Options panel, so that people who hate having to highlight answers won't have them hidden in the first place. This should make everyone somewhere close enough to happy.[/quote]\r\n\r\nHow do people feel about this proposal? This wouldn't change anything for those who don't like having to highlight text to read it - they could set their personal settings to 'reveal all spoilers' and thus they wouldn't have to highlight anything.",
"Solution_5": "[quote=\"rrusczyk\"][quote=\"rrusczyk\"]Working on a lot of these little things. In the MathLinks view, users cannot (yet) even put things into spoiler (so don't blame them for not doing it!) without explicitly typing the tags. What we're looking into is making 'reveal all spoilers' an option in the Options panel, so that people who hate having to highlight answers won't have them hidden in the first place. This should make everyone somewhere close enough to happy.[/quote]\n\nHow do people feel about this proposal? This wouldn't change anything for those who don't like having to highlight text to read it - they could set their personal settings to 'reveal all spoilers' and thus they wouldn't have to highlight anything.[/quote]\r\nI like that a lot, if the people who use this option will still put things in the spoilers (it would be easy to forget to use spoilers, if you never saw them).",
"Solution_6": "[quote=\"rrusczyk\"][quote=\"rrusczyk\"]Working on a lot of these little things. In the MathLinks view, users cannot (yet) even put things into spoiler (so don't blame them for not doing it!) without explicitly typing the tags. What we're looking into is making 'reveal all spoilers' an option in the Options panel, so that people who hate having to highlight answers won't have them hidden in the first place. This should make everyone somewhere close enough to happy.[/quote]\n\nHow do people feel about this proposal? This wouldn't change anything for those who don't like having to highlight text to read it - they could set their personal settings to 'reveal all spoilers' and thus they wouldn't have to highlight anything.[/quote]\r\n\r\nYes, this is a very good idea.",
"Solution_7": "[quote=\"rrusczyk\"][quote=\"rrusczyk\"]Working on a lot of these little things. In the MathLinks view, users cannot (yet) even put things into spoiler (so don't blame them for not doing it!) without explicitly typing the tags. What we're looking into is making 'reveal all spoilers' an option in the Options panel, so that people who hate having to highlight answers won't have them hidden in the first place. This should make everyone somewhere close enough to happy.[/quote]\n\nHow do people feel about this proposal? This wouldn't change anything for those who don't like having to highlight text to read it - they could set their personal settings to 'reveal all spoilers' and thus they wouldn't have to highlight anything.[/quote]\r\nthat'd be awesome",
"Solution_8": "This is implemented now. We will be making a global announcement about this and other items next week. \r\n\r\nAll old AoPS users have been set to have spoilers still hidden, while old ML users have been set to have them revealed by default. Go to Options above to change your default."
}
{
"Tag": [
"geometry",
"geometric transformation"
],
"Problem": "This problem was just shown to me by a friend. I don't know if I can restate it as clearly as possible.\r\n\r\nIn a creek, three structures and paths between them. At one side of a creek is a resthouse and at the other side and at the other side a butterfly dome and a hut. A pond is also to be made beside the creek. Suppose that you want to create a path in this order; resthouse, hut, pond and dome build a single bridge, connecting the three structures, and where will you put the bridge and the pond it so that the total distance of your path is minimum?\r\n\r\nSee skeleton drawing below: (Ignore the asterisk, its for spacing)\r\n\r\n*******[b]Resthouse[/b]\r\n \r\n______________________________________________\r\n Creek\r\n______________________________________________\r\n\r\n************************************[b]Dome[/b]\r\n\r\n[b]Hut[/b]",
"Solution_1": "Well, you should connect the resthouse and the hut with a straight line, and then connect the hut and dome with a straight line. That gives you the shortest distance for those three. Then put the bridge over the creek on the line from the resthouse to the hut, and put the pond anywhere on the line from the hut to the dome, and you have your locations on the shortest possible path...",
"Solution_2": "[quote=\"DanK\"]Well, you should connect the resthouse and the hut with a straight line, and then connect the hut and dome with a straight line. That gives you the shortest distance for those three. Then put the bridge over the creek on the line from the resthouse to the hut, and put the pond anywhere on the line from the hut to the dome, and you have your locations on the shortest possible path...[/quote]\r\n\r\nbut you should put the pond beside the river...",
"Solution_3": "[hide]\nLabel the resthouse $R$, the hut $H$, the creek $c$, and the dome $D$. The shortest path can be found by first finding the shortest distance $RH$ and then finding the shortest path from $H$ to $D$ that touches $c$.\n\nThe first part is easy: the shortest distance is just a straight line, so the bridge should be put at the intersection between $\\overline{RH}$ and $c$.\n\nFor the second part, one will have to touch $c$. Now suppose $c$ is horizontal and $R$ is above $c$. Then one will have to move back down in order to reach $D$. If one moves up instead, the end of the path will be at a point $D^\\prime$ which is found by reflecting $D$ over $c$. So for any path from $H$ to $D$ that touches $c$, there is a path of equal length from $H$ to $D^\\prime$ that touches $c$.\n\nThe minimum distance from $H$ to $D$ touching $c$ is equal to the minimum distance from $H$ to $D^\\prime$ touching $c$. But [i]all[/i] paths from $H$ to $D^\\prime$ touch $c$, so this is just a straight line, like the first part. The pond should be placed at the intersection between $\\overline{HD^\\prime}$ and $c$.\n[/hide]",
"Solution_4": "[quote=\"scorpius119\"][hide]\nLabel the resthouse $R$, the hut $H$, the creek $c$, and the dome $D$. The shortest path can be found by first finding the shortest distance $RH$ and then finding the shortest path from $H$ to $D$ that touches $c$.\n\nThe first part is easy: the shortest distance is just a straight line, so the bridge should be put at the intersection between $\\overline{RH}$ and $c$.\n\nFor the second part, one will have to touch $c$. Now suppose $c$ is horizontal and $R$ is above $c$. Then one will have to move back down in order to reach $D$. If one moves up instead, the end of the path will be at a point $D^\\prime$ which is found by reflecting $D$ over $c$. So for any path from $H$ to $D$ that touches $c$, there is a path of equal length from $H$ to $D^\\prime$ that touches $c$.\n\nThe minimum distance from $H$ to $D$ touching $c$ is equal to the minimum distance from $H$ to $D^\\prime$ touching $c$. But [i]all[/i] paths from $H$ to $D^\\prime$ touch $c$, so this is just a straight line, like the first part. The pond should be placed at the intersection between $\\overline{HD^\\prime}$ and $c$.\n[/hide][/quote]\r\n\r\nThis is a good explanation, i think im getting the point. But the problem is, i made a mistake with my drawing. The hut should be slightly lower than the dome.. How are we going to compute it then? I have redrawn the sketch, please see the problem again.",
"Solution_5": "Um, I think his method should still work...",
"Solution_6": "I got it already, through translation and reflection."
}
{
"Tag": [
"geometry",
"3D geometry",
"sphere",
"calculus",
"calculus computations"
],
"Problem": "What is the solid angle subtended by a cone of semivertical angle $ \\alpha$.In general ho to find the solid angle",
"Solution_1": "Draw a sphere of radius $ 1$ centered at the vertex of the cone. Find the area of the part that lies inside the cone. Divide by the total area, which is $ 4\\pi$."
}
{
"Tag": [
"function",
"quadratics",
"limit",
"algebra unsolved",
"algebra"
],
"Problem": "Find all functions $f: \\mathbb R \\setminus \\{\\pm 1\\} \\to \\mathbb R$ satisfying\n\\[ f(1-2x) = 2 f\\left(\\frac 1x \\right) + 1, \\quad \\forall x \\in \\mathbb R, x \\neq 0,\\pm 1.\\]",
"Solution_1": "Ok, I consider that the question is :\r\nFind all functions $ f$ : $ \\mathbb R\\backslash\\{ - 1, + 1\\}\\to\\mathbb R$ such that $ f(1 - 2x) = 2f(\\frac {1}{x}) + 1$ $ \\forall x\\in\\mathbb R\\backslash\\{ - 1,0, + 1\\}$\r\n\r\n\r\n$ f(1 - 2x) = 2f(\\frac {1}{x}) + 1$ $ \\forall x\\in\\mathbb R\\backslash\\{ - 1,0, + 1\\}$ $ \\Leftrightarrow$ $ f(x) = 2f(\\frac {2}{1 - x}) + 1$ $ \\forall x\\in\\mathbb R\\backslash\\{ - 1, + 1, + 3\\}$\r\n\r\nLet $ g(x) = f(x) + 1$ and $ h(x) = \\frac {2}{1 - x}$\r\n\r\nThe problem is now : Find all functions $ g$ : $ \\mathbb R\\backslash\\{ - 1, + 1\\}\\to\\mathbb R$ such that $ g(x) = 2g(h(x))$ $ \\forall x\\in\\mathbb R\\backslash\\{ - 1, + 1, + 3\\}$\r\n\r\nWe'll have (with good conditions over $ x$) : $ g(x) = 2^ng(h(h(h( ...$n times$ ...(x)))) ... )$. And clearly the $ n^{th}$ composit of $ h(x)$ is an homographic function.\r\n\r\nLet then the sequence $ a_n$ defined $ \\forall n\\in\\mathbb Z$ as :\r\n$ a_0 = 0$\r\n$ a_1 = 2$\r\n$ \\forall n > 2$ : $ a_n = a_{n - 1} - 2a_{n - 2}$\r\n\r\n$ \\forall n < 0$ : $ a_n = \\frac {a_{n + 1} - a_{n + 2}}{2}$\r\n\r\nIt's easy to check that $ a_n = \\frac {\\sqrt 2}{2}(b^n - (\\frac { - 1}{b})^n$ (where $ b = 1 + \\sqrt 2$) $ \\forall n\\in\\mathbb Z$ and that $ a_n\\neq 0$ $ \\forall n\\in\\mathbb Z^*$.\r\n\r\nLet then the family of functions defined for any ${ k\\in\\mathbb Z}$ as : $ h_k(x) = 2\\frac {a_{k - 1}x - a_k}{a_kx - a_{k + 1}}$ ($ h_0$ : $ \\mathbb R\\to \\mathbb R$; $ h_k$ : $ \\mathbb R\\backslash\\{\\frac {a_{k + 1}}{a_k}\\}\\to\\mathbb R$)\r\n\r\nIt's easy to check that $ h_0(x) = x$ and $ h_{k + 1} = h\\circ h_k$ $ \\forall k\\in\\mathbb Z$. So, $ g(x) = 2^kg(h_k(x))$\r\nIt's also easy to check that, $ \\forall k\\neq 0$, $ h_k(x)$ has no fixed point (the equation $ h_k(x) = x$ implies a quadratic whose discriminant is $ - \\frac {7a_k^2}{4} < 0$\r\n\r\nLet's then the relation $ \\sim$ defined as $ x\\sim y$ $ \\iff$ $ \\exists$ a unique $ k\\in \\mathbb Z$ such that $ y = h_k(x)$\r\nThis relation is an equivalence one :\r\n$ x\\sim x$ : $ h_0(x) = x$ and no other $ k$ is available since $ \\forall k\\neq 0$, $ h_k(x)$ has no fixed point.\r\n$ x\\sim y$ $ \\implies$ $ y\\sim x$ : $ y = h_k(x)$ $ \\iff$ $ x = h_{ - k}(x)$ and unicity is obtained thru the no_fixed_points property.\r\n$ x\\sim y$ and $ y\\sim z$ $ \\implies$ $ x\\sim z$ : $ y = h_i(x)$ and $ z = h_j(z)$ imply $ z = h_{i + j}(x)$ and unicity is obtained thru the no_fixed_points property.\r\n\r\nSo we can define a function $ k(x,y)$ defined in each equivalence class of $ \\sim$ as $ y = h_{k(x,y}(x)$\r\nLet $ C(x)$ the equivalence class of $ x$ and $ A = C( - 1)$\r\n\r\nWe have $ - 1 = h(3)$ and $ 1 = h( - 1)$ and so $ \\{ - 1, + 1, + 3\\}\\subset A$\r\n\r\nLet $ r(x)$ any choice function which associates to each real $ x$ a representative of its equivalence class (unique per class).\r\n\r\nThen :\r\n$ \\forall x\\notin A$, we have $ x = h_{k(r(x),x)}(r(x))$ and so $ g(x) = 2^{k(x,r(x))}g(r(x))$ and so we just have to define $ g(r(x))$ as we want.\r\n\r\nThe situation for $ A$ is a little bit different :\r\nWe have $ 0 = h_4(3)$ but we dont have $ g(0) = 2^4g(h_4(3))$ since we dont have $ g(1) = 2g(h(3))$\r\nSo we can choose g(x) freely for $ x = 3$ and for $ x = 0$.\r\n\r\nSo here is the general solution :\r\n================================\r\nLet $ U = \\{r(x)$ $ \\forall x\\notin A = C( - 1)\\}\\cup\\{0,3\\}$\r\nLet $ m(x)$ any function from $ U\\to\\mathbb R$\r\n\r\n$ \\forall x\\notin C( - 1)$ : $ f(x) = 2^{k(x,r(x))}m(r(x)) - 1$\r\n$ \\forall x\\in C( - 1)\\backslash\\{ - 1, + 1\\}$ such that $ k(0,x)\\geq 0$ : $ f(x) = 2^{k(x,0)}m(0) - 1$\r\n$ \\forall x\\in C( - 1)\\backslash\\{ - 1, + 1\\}$ such that $ k(0,x) < 0$ : $ f(x) = 2^{k(x,3)}m(3) - 1$\r\n\r\nAnd obviously this is a general solution.\r\n\r\nIf we want a continuous solution :\r\n================================\r\n\r\nIf we suppose $ g(x)$ continuous in $ \\mathbb R\\backslash\\{ - 1, + 1\\}$, and since $ \\lim_{k\\to - \\infty}h_k(x) = -2b$, we have, using the continuity and $ k\\to - \\infty$, $ g(x) = 0$ $ \\forall x\\in\\mathbb R\\backslash A$. Then, since $ A$ is a countable set, we can always find, in any neighborhood of an element of A, elements of $ \\mathbb R\\backslash A$. And so $ g(x) = 0$ $ \\forall x\\in R\\backslash\\{ - 1, + 1\\}$ (remember $ g(x)$ is not defined in $ \\{ - 1, + 1\\}$ And so the only continuous function solution of the initial problem is $ f(x) = - 1$"
}
{
"Tag": [
"inequalities",
"inequalities proposed"
],
"Problem": "For positive reals $ a,b,c$, prove that\r\n\\[ (a^{2}+ab+b^{2})(b^{2}+bc+c^{2})(c^{2}+ca+a^{2}) \\leq (((b+c)(c+a)(a+b))^{\\frac{2}{3}}-(abc)^{\\frac{2}{3}})^{3}. \\]",
"Solution_1": "Very nice and good inequality! Congratulations, Kim.",
"Solution_2": "[quote=\"Sung-yoon Kim\"]For positive reals $ a,b,c$, prove that\n\\[ (a^{2}\\plus{}ab\\plus{}b^{2})(b^{2}\\plus{}bc\\plus{}c^{2})(c^{2}\\plus{}ca\\plus{}a^{2})\\leq (((b\\plus{}c)(c\\plus{}a)(a\\plus{}b))^{\\frac{2}{3}}\\minus{}(abc)^{\\frac{2}{3}})^{3}.\\]\n[/quote]\r\nRewrite the inequality as\r\n$ \\left( 1\\plus{}\\sqrt[3]{\\frac{(a^{2}\\plus{}ab\\plus{}b^{2})(b^{2}\\plus{}bc\\plus{}c^{2})(c^{2}\\plus{}ca\\plus{}a^{2})}{a^{2}b^{2}c^{2}}}\\right)^{3}\\le\\frac{(a\\plus{}b)^{2}(b\\plus{}c)^{2}(c\\plus{}a)^{2}}{a^{2}b^{2}c^{2}}$\r\nBy Holder inequality,\r\n$ \\left( 1\\plus{}\\sqrt[3]{\\prod_{cycl}\\frac{a^{2}\\plus{}ab\\plus{}b^{2}}{ab}}\\right)^{3}\\le\\prod_{cycl}\\left( 1\\plus{}\\frac{a^{2}\\plus{}ab\\plus{}b^{2}}{ab}\\right) \\equal{}\\prod_{cycl}\\frac{(a\\plus{}b)^{2}}{ab}$",
"Solution_3": "Nice. In fact, my solution uses the similar method.\r\n\r\n$ \\prod (a^{2}\\plus{}ab\\plus{}b^{2}) \\equal{}\\prod ((a\\plus{}b)^{2}\\minus{}ab)\\leq (RHS)$, since $ \\prod (p_{i}\\minus{}q_{i})\\leq ((\\prod p_{i})^{\\frac{1}{3}}\\minus{}(\\prod q_{i})^{\\frac{1}{3}})^{3}$ holds by Holder."
}
{
"Tag": [
"number theory",
"prime factorization",
"number theory unsolved"
],
"Problem": "For each natural $ n$,we define $ f(n)$ as the exponent of number $ 2$ in the prime factorization of $ n!$.Show that for any natural \r\n$ a$, the equation $ n\\minus{}f(n)\\equal{}a$ has infinitely many solutions.",
"Solution_1": "We have $ (2n)!\\equal{}2^n \\cdot n!\\cdot1\\cdot3\\cdot5\\cdot...\\cdot(2n\\minus{}1)$ so $ 2n\\minus{}f(2n)\\equal{}2n\\minus{}(n\\plus{}f(n))\\equal{}n\\minus{}f(n)$.\r\nBut $ 1\\minus{}f(1)\\equal{}1 \\Rightarrow 2^a\\minus{}f(2^a)\\equal{}1$ so $ f(2^a\\minus{}1)\\equal{}(2^a\\minus{}1)\\minus{}a$ and $ 2^k(2^a\\minus{}1)\\minus{}f(2^k(2^a\\minus{}1))\\equal{}a$ for every nonnegative integer $ k$.",
"Solution_2": "[quote=\"mazur89\"]We have $ (2n)! \\equal{} 2^n \\cdot n!\\cdot1\\cdot3\\cdot5\\cdot...\\cdot(2n \\minus{} 1)$ so $ 2n \\minus{} f(2n) \\equal{} 2n \\minus{} (n \\plus{} f(n)) \\equal{} n \\minus{} f(n)$.\nBut $ 1 \\minus{} f(1) \\equal{} 1 \\Rightarrow 2^a \\minus{} f(2^a) \\equal{} 1$ so $ f(2^a \\minus{} 1) \\equal{} (2^a \\minus{} 1) \\minus{} a$ and $ 2^k(2^a \\minus{} 1) \\minus{} f(2^k(2^a \\minus{} 1)) \\equal{} a$ for every nonnegative integer $ k$.[/quote]\r\nNote that $ S_p(n)$ is some digits of n in base p.\r\nThen $ v_p(n)\\equal{}\\frac{n\\minus{}S_p(n)}{p\\minus{}1}$\r\nApply this for $ p\\equal{}2$ then \r\n$ f(n)\\equal{}n\\minus{}S_2(n)$ \r\nSo $ n\\minus{}f(n)\\equal{}S_2(n)$\r\nObvious the equation $ S_2(n)\\equal{}a$ have infinite solution on N."
}
{
"Tag": [
"geometry"
],
"Problem": "Prove that the centroid and orthocenter of an equilateral triangle are concurrent.",
"Solution_1": "[hide]\nThe median of an equilateral triangle is perpendicular to the opposite side, and so is collinear to the altitude. Hence, the intersection of the medians is the same as the intersection of the altitudes, and so the centroid and the orthocenter are concurrent.\n[/hide]"
}
{
"Tag": [
"function",
"abstract algebra",
"complex numbers",
"linear algebra",
"geometry unsolved",
"geometry"
],
"Problem": "how do we prove that an isometry in the complex plane is always a function of the form az+b or a(conjugate of z)+b, where modulus of a = 1, a and b are complex numbers\r\nAlso, anyone pls explain me what is meant by 'fixed points' of an isometry",
"Solution_1": "First question: http://www.math.uconn.edu/~kconrad/blurbs/grouptheory/isometrycpx.pdf\r\n\r\n(knowing the types of isometric transformations and the way to represent them in complex plane makes your question simple, and the process is given in the paper.)\r\n\r\nSecond: If $ f$ is a transformation, point $ z_0$ for which $ f(z_0)\\equal{}z_0$ is called fixed point of the transformation-it's simply the point which remains intact during the transform.",
"Solution_2": "thanks for the help hsiljak. Will u pls tell me what is meant by 'kernel'?",
"Solution_3": "Haven't had much time to study the paper, but I assume that the typical definition of linear operator's kernel and image is what you need.\r\n\r\nAnd this is how it was presented to me in my Linear algebra course:\r\n\r\n[quote]The set of all vectors from vector space $ X$ being mapped in vector space's $ Y$ null-vector is denoted with $ \\text{Ker} A$ and called kernel or zero-subspace of linear operator $ A$.\n\nThe set of all vectors in $ Y$ which are mapping images of elements in $ X$ is denoted with $ \\text{Im} A$ or $ A(X)$ and called image of operator $ A$. \n\nIn other words:\n\n$ \\text{Ker} A : \\equal{} \\{ x \\in X|A(x) \\equal{} 0\\}$\n$ \\text{Im} A : \\equal{} \\{y \\in Y|y \\equal{} A(x) \\text{for some } x \\in X\\}$.[/quote]"
}
{
"Tag": [
"geometry",
"geometry solved"
],
"Problem": "Let $O_1,O_2,O_3$ be the centers of ciclers $(C_1),(C_2),(C_3)$ , respectively.\r\nThe circles $(C_1),(C_2),(C_3)$ intersect at point $O$. Let $A_1,A_2,A_3 $ be point on $(C_1),(C_2),(C_3)$, respectively, such that: $ OA_1||O_2O_3;OA_2||O_3O_1;\r\nOA_3||O_1O_2$.\r\nProve that quadrilateral $OA_1A_2A_3$ ic cyclic. (maybe it was posted before)\r\n\r\n[color=red][Moderator edit: Typo corrected.][/color]",
"Solution_1": "Grobber, that one is for you :D\r\n\r\n darij",
"Solution_2": "You're too kind.. :D:D\r\n\r\nDo you know how hard I tried not to use inversion? I failed :)\r\n\r\nAfter inversion wrt $O$ (any power you want) you get the following problem (I'll leave out the details of how you get to it: it's straightforward): Given a triangle $ABC$ and a point $O$, assume the perpendicular through $O$ to $OA$ cuts $BC$ in $A_1$. In the same way obtain $B_1,C_1$. Show that $A_1,B_1,C_1$ are collinear.\r\n\r\nThis has been discussed before: [url=http://www.artofproblemsolving.com/Forum/viewtopic.php?t=1036&highlight=]here[/url]'s the link.",
"Solution_3": "Indeed, I don't have a non-inversive solution either...\r\n\r\nJust a remark:\r\n\r\n[quote=\"grobber\"]you get the following problem (I'll leave out the details of how you get to it: it's straightforward): Given a triangle $ABC$ and a point $O$, assume the perpendicular through $O$ to $OA$ cuts $BC$ in $A_1$. In the same way obtain $B_1,C_1$. Show that $A_1,B_1,C_1$ are collinear.[/quote]\r\n\r\nThere is a famous solution of this problem in Jacques Hadamard's geometry book. You can find it in the note \"Problem: The ortho-intercepts line\" on [url=http://de.geocities.com/darij_grinberg]my website[/url].\r\n\r\n Darij",
"Solution_4": ":rt5:",
"Solution_5": "[quote=\"pestich\"]In triangle O1O2O3 we have point O and lines thru O parallel \n to the sides. Parallels intersect the 3rd circle in a point symmetrical to O\n wrt perpendicular thru center of 3rd circle. So points are concyclic\n with the center of the resulting circle being orthocenter of O1O2O3.[/quote]\r\n\r\nThis is correct and very beautiful except of the last line, where \"orthocenter\" has to be replaced by \"circumcenter\". Thank you!\r\n\r\n Darij"
}
{
"Tag": [
"inequalities proposed",
"inequalities"
],
"Problem": "Let $ x,y,z>0$ . Show that :\r\n\r\n\\[ \\frac{x^2(y\\plus{}z)}{(x\\plus{}y)(z\\plus{}x)}\\plus{}\\frac{y^2(z\\plus{}x)}{(y\\plus{}z)(x\\plus{}y)}\\plus{}\\frac{z^2(x\\plus{}y)}{(z\\plus{}x)(y\\plus{}z)}\\le \\frac{x\\plus{}y\\plus{}z}{2}\\]",
"Solution_1": "[quote=\"Pain rinnegan\"]Let $ x,y,z > 0$ . Show that :\n\\[ \\frac {x^2(y \\plus{} z)}{(x \\plus{} y)(z \\plus{} x)} \\plus{} \\frac {y^2(z \\plus{} x)}{(y \\plus{} z)(x \\plus{} y)} \\plus{} \\frac {z^2(x \\plus{} y)}{(z \\plus{} x)(y \\plus{} z)}\\le \\frac {x \\plus{} y \\plus{} z}{2}\\]\n[/quote]\n\n[quote=\"Pain rinnegan\"]Let $ x,y,z > 0$ . Show that :\n\\[ \\frac {x^2(y \\plus{} z)}{(x \\plus{} y)(z \\plus{} x)} \\plus{} \\frac {y^2(z \\plus{} x)}{(y \\plus{} z)(x \\plus{} y)} \\plus{} \\frac {z^2(x \\plus{} y)}{(z \\plus{} x)(y \\plus{} z)}\\le \\frac {x \\plus{} y \\plus{} z}{2}\\]\n[/quote]\r\nwithout loss of generality, we may assume that $ x\\ge y\\ge z$.\r\n\\[ \\frac {x^2(y \\plus{} z)}{(x \\plus{} y)(z \\plus{} x)} \\plus{} \\frac {y^2(z \\plus{} x)}{(y \\plus{} z)(x \\plus{} y)} \\plus{} \\frac {z^2(x \\plus{} y)}{(z \\plus{} x)(y \\plus{} z)}\\le \\frac {x \\plus{} y \\plus{} z}{2}\\]\r\n\\[ \\Longleftrightarrow \\sum{\\frac {x (x \\minus{} y) (x \\minus{} z)}{2 (x \\plus{} y) (x \\plus{} z)}} \\ge 0\\]\r\n\\[ \\Longleftrightarrow \\frac{{x\\left( {x \\minus{} y} \\right)^2 }}{{2\\left( {x \\plus{} y} \\right)\\left( {x \\plus{} z} \\right)}} \\plus{} \\frac{{x\\left( {x \\minus{} y} \\right)\\left( {y \\minus{} z} \\right)z}}{{\\left( {x \\plus{} y} \\right)\\left( {y \\plus{} z} \\right)\\left( {z \\plus{} x} \\right)}} \\plus{} \\frac{{z\\left( {y \\minus{} z} \\right)^2 }}{{2\\left( {z \\plus{} x} \\right)\\left( {z \\plus{} y} \\right)}} \\ge 0\\]\r\nClearly."
}
{
"Tag": [
"probability",
"graph theory",
"combinatorics theorems",
"combinatorics"
],
"Problem": "Hello everybody. I'm preparing for mathematical olympiads. I want to learn graph theory. Can you suggest me good graph theory books? As far as I know , graph theory is the most important topic in combinatorics. I found some but i decided to ask you for help. thx.",
"Solution_1": "[b]I have a few recommendations:[/b]\r\n\r\n- Bodendiek, Rainer and Burosch, Gustav: [url=http://www.amazon.de/Streifz%C3%BCge-durch-Kombinatorik-Rainer-Bodendiek/dp/386025393X]Streifz\u00fcge durch die Kombinatorik:[/url] An excellent book in German using many problems from old shorlists and Russia written for the German IMO preparation. [url=http://www.baltic-college.de/kontakt/42-campus-guestrow/18-prof-dr-gustav-burosch-.html]Gustav Burosch[/url] was team leader of the German Democratic Republic from 1970 to 1990 except for 1976, 1971 and 1981 due to political problems. For other political reasons he could not continue his work after the reunification of the two Germanys after 1990. He specifically discusses how many problems can be phrased in the language of graph theory. Burosch was a world expert on combinatorics and graph theory.\r\n\r\n- L\u00e1szl\u00f3 Lov\u00e1sz, [url=http://www.amazon.com/Combinatorial-Problems-Exercises-Chelsea-Publishing/dp/0821842625/ref=sr_1_1?ie=UTF8&s=books&qid=1247754784&sr=8-1]Combinatorial problems and exercises:[/url] IMO math and more advanced research problem collection\r\n\r\n- Ioan Tomescu, R.A. Melter: [url=http://www.amazon.com/Problems-Combinatorics-Interscience-Discrete-Mathematics/dp/0471801550/ref=sr_1_1?ie=UTF8&s=books&qid=1247754876&sr=1-1]Problems in Combinatorics and Graph Theory, 1985:[/url] Romanian IMO Training from the 80s and 90s\r\n\r\n- Diestel, Reinhard: [url=http://www.math.uni-hamburg.de/home/diestel/books/graph.theory/]Graph Theory:[/url] Good reference book and can can be downloaded from the linked URL. \r\n\r\n- Alon, Noga and Spencer, Joel: [url=http://www.amazon.com/Probabilistic-Wiley-Interscience-Discrete-Mathematics-Optimization/dp/0470170204/ref=sr_1_1?ie=UTF8&s=books&qid=1247754925&sr=1-1]The Probabilistic Method:[/url] This book uses graph theory and also teaches a useful technique which can be seen from the book title. A few sections can be downloaded from [url=http://cs.nyu.edu/cs/faculty/spencer/nogabook/nogabook.html]here.[/url]\r\n\r\n- Engel, Konrad: [url=http://opt.math.uni-rostock.de/engel/sperner.html]Sperner Theory:[/url] This book is also quite specialized but teaches you thinking in combinatorics and graph theory",
"Solution_2": "[quote=\"orl\"]\n\n- Diestel, Reinhard: [url=http://www.math.uni-hamburg.de/home/diestel/books/graph.theory/]Graph Theory:[/url] Good reference book and can can be downloaded from the linked URL. \n[/quote]\r\n\r\nThank you, and can you tell me how good is the quoted book because I already have it?",
"Solution_3": "Well, I think it is an excellent book. But it is a book written for university students and not necessarily targeted at olympiad preparing students only.",
"Solution_4": "[quote=\"fallinlovewithmaths\"]Hello everybody. I'm preparing for mathematical olympiads. I want to learn graph theory. Can you suggest me good graph theory books? As far as I know , graph theory is the most important topic in combinatorics. I found some but i decided to ask you for help. thx.[/quote]\r\nIntroduction to Graph Theory By Gary Chartrand\r\n1.\u3000Introduction . \r\n1.1.\u3000Graphs and Graph Models 1 \r\n1.2.\u3000Connected Graphs 9 \r\n1.3.\u3000Common Classes of Graphs 19 \r\n1.4.\u3000Multigraphs and Digraphs 26 \r\n2.\u3000Degrees \r\n2.1.\u3000The Degree of a Vertex 31 \r\n2.2.\u3000Regular Graphs 38 \r\n2.3.\u3000Degree Sequences 43 \r\n2.4.\u3000Excursion:Graphs and Matrices 48 \r\n2.5.\u3000Exploration:Irregular Graphs 50 \r\n3.\u3000Isomorphic Graphs \r\n3.1.\u3000The Definiition of Isomorphism 55 \r\n3.2.\u3000Isomorphism as a Relation 63 \r\n3.3.\u3000Excursion:Graphs and Groups 66 \r\n3.4.\u3000Excursion:Reconstruction and Solvability 76 \r\n4.\u3000Trees \r\n4.1.\u3000Bridges 85 \r\n.\r\n..\r\n..\r\n.",
"Solution_5": "What about \r\n\r\nDouglas B. West's\r\n\r\nIntroduction to Graph Theory\r\n\r\nhttp://www.amazon.com/Introduction-Graph-Theory-Douglas-West/dp/0130144002\r\n\r\nBut it is too expensive in English. Nevertheless, we can download by searching in Google Book.",
"Solution_6": "Also try Andreescu's 102 Combinatorial Problems and A Path to Combinatorics for Undergraduates.. Sure, they are really good books for your training.",
"Solution_7": "There is a new book on graph theory, specifically aimed towards mathematical olympiads. I don't know how much theory is dicussed in this book, but I browsed the first few pages. The problems discussed are very insightful. I haven't read it, so I can't recommend that you buy it, but it is certainly worth a look.\n\nGraph Theory by Xiong Bin and Zheng Zhongyi, vol.3 of the Mathematical Olympiad Series, published by World Scientific. [url=http://www.amazon.com/Graph-Theory-Mathematical-Olympiad-Xiong/dp/9814271128]Here[/url] is a link.",
"Solution_8": "[quote=\"fallinlovewithmaths\"]graph theory is the most important topic in combinatorics. [/quote]\nSurely not!\nAnyway, Douglas West's book is good for theory and [url=http://www.artofproblemsolving.com/Forum/viewtopic.php?f=45&t=414427]here[/url] are a few problems :)",
"Solution_9": "[url=http://www.amazon.com/Introduction-Graph-Theory-Douglas-West/dp/0130144002]Introduction to Graph Theory by Douglas B. West[/url] or [url=http://www.amazon.com/Graph-Theory-Applications-Adrian-Bondy/dp/0444194517]Graph Theory With Applications by John Bondy and U.S.R Murty[/url] are 2 books that I have and am studying for Iran National Olympiad in Informatics.",
"Solution_10": "[url]http://www.amazon.com/Walk-Through-Combinatorics-Miklos-Bona/dp/9810249012[/url]\n\nThis book seems to have some chapters in graph theory.\nAre they suitable for Olympiad problems?",
"Solution_11": "I am searching about graph theory too. You have given really important suggestions about some graph theory books. I just want to know the applications of graph theory. Can you make me get some information about it?",
"Solution_12": "[quote=\"aestik\"][quote=\"fallinlovewithmaths\"]Hello everybody. I'm preparing for mathematical olympiads. I want to learn graph theory. Can you suggest me good graph theory books? As far as I know , graph theory is the most important topic in combinatorics. I found some but i decided to ask you for help. thx.[/quote]\nIntroduction to Graph Theory By Gary Chartrand\n1.\u3000Introduction . \n1.1.\u3000Graphs and Graph Models 1 \n1.2.\u3000Connected Graphs 9 \n1.3.\u3000Common Classes of Graphs 19 \n1.4.\u3000Multigraphs and Digraphs 26 \n2.\u3000Degrees \n2.1.\u3000The Degree of a Vertex 31 \n2.2.\u3000Regular Graphs 38 \n2.3.\u3000Degree Sequences 43 \n2.4.\u3000Excursion:Graphs and Matrices 48 \n2.5.\u3000Exploration:Irregular Graphs 50 \n3.\u3000Isomorphic Graphs \n3.1.\u3000The Definiition of Isomorphism 55 \n3.2.\u3000Isomorphism as a Relation 63 \n3.3.\u3000Excursion:Graphs and Groups 66 \n3.4.\u3000Excursion:Reconstruction and Solvability 76 \n4.\u3000Trees \n4.1.\u3000Bridges 85 \n.\n..\n..\n.[/quote]\n\nWow! This looks like an excellent resource! However, AoPS Intermediate Counting and Probability only uses one chapter to introduce graph theory to the reader. Is there anything that I should do before I start reading this book or should I go dive into it?"
}
{
"Tag": [
"geometry",
"trapezoid",
"Gauss"
],
"Problem": "[b][size=100][color=DarkBlue]\u03a0\u03a1\u039f\u03a4\u0391\u03a3\u0397. - \u0394\u03af\u03b4\u03b5\u03c4\u03b1\u03b9 \u03b9\u03c3\u03bf\u03c3\u03ba\u03b5\u03bb\u03ad\u03c2 \u03c4\u03c1\u03b1\u03c0\u03ad\u03b6\u03b9\u03bf $ ABCD$ \u03bc\u03b5 $ AB\\parallel CD$ \u03ba\u03b1\u03b9 \u03ad\u03c3\u03c4\u03c9 $ P\\equiv AC\\cap BD$ \u03ba\u03b1\u03b9 $ Q\\equiv AD\\cap BC.$ \u039c\u03b5 \u03b4\u03b9\u03b1\u03bc\u03ad\u03c4\u03c1\u03bf\u03c5\u03c2 \u03c4\u03b9\u03c2 \u03b2\u03ac\u03c3\u03b5\u03b9\u03c2 \u03c4\u03bf\u03c5 $ AB,$ $ CD,$ \u03b3\u03c1\u03ac\u03c6\u03bf\u03c5\u03bc\u03b5 \u03c4\u03bf\u03c5\u03c2 \u03ba\u03cd\u03ba\u03bb\u03bf\u03c5\u03c2 $ (O_{1}),$ $ (O_{2})$ \u03b1\u03bd\u03c4\u03b9\u03c3\u03c4\u03bf\u03af\u03c7\u03c9\u03c2, \u03bf\u03b9 \u03bf\u03c0\u03bf\u03af\u03bf\u03b9 \u03c4\u03ad\u03bc\u03bd\u03bf\u03bd\u03c4\u03b1\u03b9 \u03c3\u03c4\u03b1 \u03c3\u03b7\u03bc\u03b5\u03af\u03b1 \u03ad\u03c3\u03c4\u03c9 $ E,$ $ F,$ $ ($ $ E,$ $ B,$ $ C,$ \u03c0\u03c1\u03bf\u03c2 \u03c4\u03bf \u03b1\u03c5\u03c4\u03cc \u03bc\u03ad\u03c1\u03bf\u03c2 \u03c4\u03b7\u03c2 $ O_{1}O_{2}$ $ ).$ \u0391\u03c0\u03bf\u03b4\u03b5\u03af\u03be\u03c4\u03b5 \u03cc\u03c4\u03b9 $ \\angle KEM \\equal{} \\angle NEQ$ \u03ba\u03b1\u03b9 $ PE\\perp EQ,$ \u03cc\u03c0\u03bf\u03c5 $ K\\equiv O_{1}O_{2}\\cap EF$ \u03ba\u03b1\u03b9 $ M\\equiv O_{1}O_{2}\\cap (O_{1})$ \u03ba\u03b1\u03b9 $ N\\equiv O_{1}O_{2}\\cap (O_{2})$ $ ,($ $ M,$ \u03bc\u03b5\u03c4\u03b1\u03be\u03cd \u03c4\u03c9\u03bd $ O_{1},$ $ Q$ \u03ba\u03b1\u03b9 $ N,$ \u03bc\u03b5\u03c4\u03b1\u03be\u03cd \u03c4\u03c9\u03bd $ O_{2},$ $ Q$ $ ).$[/color][/size][/b]\r\n\r\n\u039a\u03ce\u03c3\u03c4\u03b1\u03c2 \u0392\u03ae\u03c4\u03c4\u03b1\u03c2.\r\n\r\n[b][size=100][color=DarkBlue]PROBLEM. - An isosceles trapezium $ ABCD$ is given, with $ AB\\parallel CD$ and let be the points $ P\\equiv AC\\cap BD$ and $ Q\\equiv AD\\cap BC.$ We draw the circles $ (O_{1}),$ $ (O_{2}),$ with diameters its bases $ AB,$ $ CD$ respectively, which intersect each other at points $ E,$ $ F,$ $ ($ $ E,$ $ B,$ $ C,$ in to the same place with respect to the line segment $ O_{1}O_{2}$ $ ).$ Prove that $ \\angle KEM \\equal{} \\angle NEQ$ and $ PE\\perp EQ,$ where $ K\\equiv O_{1}O_{2}\\cap EF$ and $ M\\equiv O_{1}O_{2}\\cap (O_{1})$ and $ N\\equiv O_{1}O_{2}\\cap (O_{2}),$ $ ($ $ M,$ between $ O_{1},$ $ Q$ and $ N,$ between $ O_{2},$ $ Q$ $ ).$[/color][/size][/b]\r\n\r\nKostas Vittas.",
"Solution_1": "\u03a4\u03bf \u03c0\u03c1\u03ce\u03c4\u03bf \u03b5\u03c1\u03ce\u03c4\u03b7\u03bc\u03b1 \u03c6\u03b1\u03af\u03bd\u03b5\u03c4\u03b1\u03b9 \u03b4\u03cd\u03c3\u03ba\u03bf\u03bb\u03bf (\u03b2\u03b1\u03c3\u03b9\u03ba\u03ac \u03b5\u03af\u03bd\u03b1\u03b9 \u03b1\u03c1\u03ba\u03b5\u03c4\u03ac \u03bc\u03c0\u03b5\u03c1\u03b4\u03b5\u03bc\u03ad\u03bd\u03bf \u03c4\u03bf \u03c3\u03c7\u03ae\u03bc\u03b1) \u03b1\u03bb\u03bb\u03ac \u03c4\u03bf \u03b4\u03b5\u03cd\u03c4\u03b5\u03c1\u03bf \u03bd\u03bf\u03bc\u03af\u03b6\u03c9 \u03cc\u03c4\u03b9 \u03b9\u03c3\u03c7\u03cd\u03b5\u03b9 \u03c7\u03c9\u03c1\u03af\u03c2 \u03ba\u03b1\u03bd \u03c4\u03bf $ ABCD$ \u03bd\u03b1 \u03b5\u03af\u03bd\u03b1\u03b9 \u03c4\u03c1\u03b1\u03c0\u03ad\u03b6\u03b9\u03bf! \r\n\u03a3\u03cd\u03bc\u03c6\u03c9\u03bd\u03b1 \u03bc\u03b5 \u03c4\u03bf \u03b8\u03b5\u03ce\u03c1\u03b7\u03bc\u03b1 Gauss-Bodenmiller \u03c3\u03b5 \u03ad\u03bd\u03b1 \u03c0\u03bb\u03ae\u03c1\u03b5\u03c2 \u03c4\u03b5\u03c4\u03c1\u03ac\u03c0\u03bb\u03b5\u03c5\u03c1\u03bf \u03bf\u03b9 \u03ba\u03cd\u03ba\u03bb\u03bf\u03b9 \u03bc\u03b5 \u03b4\u03b9\u03b1\u03bc\u03ad\u03c4\u03c1\u03bf\u03c5\u03c2 \u03c4\u03b9\u03c2 \u03b4\u03b9\u03b1\u03b3\u03c9\u03bd\u03af\u03bf\u03c5\u03c2 \u03c4\u03bf\u03c5 \u03ad\u03c7\u03bf\u03c5\u03bd \u03ba\u03bf\u03b9\u03bd\u03cc \u03c1\u03b9\u03b6\u03b9\u03ba\u03cc \u03ac\u03be\u03bf\u03bd\u03b1 \u03bf \u03bf\u03c0\u03bf\u03af\u03bf\u03c2 \u03bc\u03ac\u03bb\u03b9\u03c3\u03c4\u03b1 \u03b4\u03b9\u03ad\u03c1\u03c7\u03b5\u03c4\u03b1\u03b9 \u03ba\u03b9 \u03b1\u03c0' \u03c4\u03b1 \u03bf\u03c1\u03b8\u03cc\u03ba\u03b5\u03c4\u03c1\u03b1 \u03c4\u03c9\u03bd \u03c4\u03c1\u03b9\u03b3\u03ce\u03bd\u03c9\u03bd \u03c4\u03bf\u03c5 \u03c0\u03bb\u03ae\u03c1\u03bf\u03c5\u03c2 \u03c4\u03b5\u03c4\u03c1\u03b1\u03c0\u03bb\u03b5\u03cd\u03c1\u03bf\u03c5! \u0395\u03b4\u03ce \u03c6\u03b1\u03bd\u03b5\u03c1\u03ac \u03bf\u03b9 \u03b4\u03b9\u03b1\u03b3\u03ce\u03bd\u03b9\u03bf\u03b9 \u03b5\u03af\u03bd\u03b1\u03b9 \u03bf\u03b9 $ AB, CD, PQ$ \u03ba\u03b1\u03b9 \u03bf \u03c1\u03b9\u03b6\u03b9\u03ba\u03cc\u03c2 \u03ac\u03be\u03bf\u03bd\u03b1\u03c2 \u03b7 $ EF$ \u03ac\u03c1\u03b1 \u03bf \u03ba\u03cd\u03ba\u03bb\u03bf\u03c2 \u03bc\u03b5 \u03b4\u03b9\u03ac\u03bc\u03b5\u03c4\u03c1\u03bf $ PQ$ \u03b4\u03b9\u03ad\u03c1\u03c7\u03b5\u03c4\u03b1\u03b9 \u03b1\u03c0' \u03c4\u03b1 $ E,F$ \u03ac\u03c1\u03b1 \u03c4\u03bf \u03b6\u03b7\u03c4\u03bf\u03cd\u03bc\u03b5\u03bd\u03bf :)",
"Solution_2": "\u0393\u03b9\u03b1 \u03c4\u03bf \u03c0\u03c1\u03ce\u03c4\u03bf \u03b5\u03c1\u03ce\u03c4\u03b7\u03bc\u03b1 \u03b1\u03c1\u03ba\u03b5\u03af \u03bd\u03b1 \u03b5\u03af\u03bd\u03b1\u03b9:\r\n$ ENM \\equal{}$ \u03b9\u03c3\u03bf\u03c3\u03ba\u03b5\u03bb\u03ad\u03c2 \u03b3\u03b9\u03b1\u03c4\u03af \u03c4\u03cc\u03c4\u03b5 $ EF // AB$ \u03ac\u03c1\u03b1 $ EF$ \u03ba\u03ac\u03b8\u03b5\u03c4\u03b7 \u03c3\u03c4\u03b7\u03bd $ MN$ \u03ba\u03b1\u03b9 \u03ac\u03c1\u03b1 $ EK$ = \u03cd\u03c8\u03bf\u03c2 $ \\Leftrightarrow EK$ = \u03b4\u03b9\u03c7\u03bf\u03c4\u03cc\u03bc\u03bf\u03c2\r\n\r\n\u03c0\u03c1\u03ac\u03b3\u03bc\u03b1\u03c4\u03b9:\r\n[hide]\u03b1\u03bd $ O, P$ \u03c4\u03b1 \u03bc\u03ad\u03c3\u03b1 \u03c4\u03c9\u03bd $ AB, CD$ \u03b5\u03af\u03bd\u03b1\u03b9 $ K \\equal{}$ \u03bc\u03ad\u03c3\u03bf $ OP$ \u03ac\u03c1\u03b1 $ EOP \\equal{}$ \u03b9\u03c3\u03bf\u03c3\u03ba\u03b5\u03bb\u03ad\u03c2,\n\u03b1\u03c6\u03bf\u03cd \u03b5\u03af\u03bd\u03b1\u03b9 \u03b1\u03ba\u03cc\u03bc\u03b1 $ EK \\equal{}$ \u03ba\u03ac\u03b8\u03b5\u03c4\u03b7 \u03c3\u03c4\u03b7\u03bd $ OP$\n\n\u03ac\u03c1\u03b1 $ \\widehat{KOE} \\equal{} \\widehat{KPE} \\Leftrightarrow$\n(\u03b5\u03c0\u03b5\u03b9\u03b4\u03ae \u03c4\u03b1 \u03c4\u03c1\u03af\u03b3\u03c9\u03bd\u03b1 $ EPN, EOM$ \u03b5\u03af\u03bd\u03b1\u03b9 \u03b9\u03c3\u03bf\u03c3\u03ba\u03b5\u03bb\u03ae \u03b1\u03c6\u03bf\u03cd \u03ad\u03c7\u03bf\u03c5\u03bd \u03b4\u03cd\u03bf \u03c0\u03bb\u03b5\u03c5\u03c1\u03ad\u03c2 \u03c4\u03bf\u03c5\u03c2 \u03af\u03c3\u03b5\u03c2 \u03c9\u03c2 \u03b1\u03ba\u03c4\u03af\u03bd\u03b5\u03c2 \u03c4\u03c9\u03bd \u03ba\u03cd\u03ba\u03bb\u03c9\u03bd $ (O2), (O1)$ \u03b1\u03bd\u03c4\u03af\u03c3\u03c4\u03bf\u03b9\u03c7\u03b1)\n$ \\widehat{OME} \\equal{} \\widehat{PNE}$\n\n\u03ac\u03c1\u03b1 $ ENM \\equal{}$ \u03b9\u03c3\u03bf\u03c3\u03ba\u03b5\u03bb\u03ad\u03c2 :D \n[/hide]\r\n\r\n\r\n\u0388\u03b3\u03c1\u03b1\u03c8\u03b1 \u03c4\u03b9\u03c2 \u03c3\u03ba\u03ad\u03c8\u03b5\u03b9\u03c2 \u03bc\u03bf\u03c5 \u03c0\u03bf\u03bb\u03cd \u03c3\u03c5\u03bc\u03c0\u03b7\u03ba\u03bd\u03c9\u03bc\u03ad\u03bd\u03b5\u03c2 \u03b3\u03b9\u03b1\u03c4\u03af \u03b5\u03af\u03bd\u03b1\u03b9 \u03c3\u03c0\u03ac\u03c3\u03b9\u03bc\u03bf \u03bd\u03b1 \u03c4\u03b9\u03c2 \u03b3\u03c1\u03ac\u03c6\u03c9 \u03c3\u03c9\u03c3\u03c4\u03ac. :blush: \u039a\u03b1\u03b9 \u03c0\u03b1\u03af\u03c1\u03bd\u03bf\u03bd\u03c4\u03b1\u03c2 \u03c5\u03c0\u03cc\u03c8\u03b9\u03bd \u03cc\u03c4\u03b9 \u03b4\u03b5\u03bd \u03ad\u03ba\u03b1\u03bd\u03b1 \u03ba\u03b1\u03bd \u03c3\u03c7\u03ae\u03bc\u03b1 \u03af\u03c3\u03c9\u03c2 \u03bd\u03b1 \u03ad\u03c7\u03c9 \u03bb\u03ac\u03b8\u03b7. \u03a0\u03b5\u03af\u03c4\u03b5 \u03b1\u03bd \u03b2\u03c1\u03b5\u03af\u03c4\u03b5 \u03ba\u03ac\u03c0\u03bf\u03b9\u03bf.",
"Solution_3": "\u0394\u03c5\u03c3\u03c4\u03c5\u03c7\u03ce\u03c2, \u03be\u03b1\u03bd\u03b1\u03b2\u03bb\u03ad\u03c0\u03bf\u03bd\u03c4\u03b1\u03c2 \u03c4\u03b7\u03bd \u03ac\u03c3\u03ba\u03b7\u03c3\u03b7, \u03c3\u03c5\u03bc\u03c0\u03b5\u03c1\u03b1\u03af\u03bd\u03c9 \u03cc\u03c4\u03b9 \u03c4\u03bf \u03c0\u03c1\u03ce\u03c4\u03bf \u03c5\u03c0\u03bf\u03b5\u03c1\u03ce\u03c4\u03b7\u03bc\u03b1 \u03b4\u03b5\u03bd \u03b9\u03c3\u03c7\u03cd\u03b5\u03b9 \u03b3\u03b9\u03b1\u03c4\u03af\r\n\u03c3\u03b5 \u03cc\u03bb\u03b5\u03c2 \u03c4\u03b9\u03c2 \u03c3\u03c7\u03ad\u03c3\u03b5\u03b9\u03c2 \u03c0\u03bf\u03c5 \u03c7\u03c1\u03b7\u03c3\u03b9\u03bc\u03bf\u03c0\u03bf\u03af\u03b7\u03c3\u03b1 \u03b9\u03c3\u03c7\u03cd\u03b5\u03b9 \u03b4\u03b9\u03c0\u03bb\u03ae \u03c3\u03c5\u03bd\u03b5\u03c0\u03b1\u03b3\u03c9\u03b3\u03ae \u03ba\u03b1\u03b9\r\n\u03b3\u03b9\u03b1 \u03bd\u03b1 \u03b5\u03af\u03bd\u03b1\u03b9 $ K \\equal{}$ \u03bc\u03ad\u03c3\u03bf \u03c4\u03bf\u03c5 $ OP$ \u03c0\u03c1\u03ad\u03c0\u03b5\u03b9 $ EO \\equal{} EP$ \u03b1\u03c6\u03bf\u03cd $ EK \\equal{}$ \u03ba\u03ac\u03b8\u03b5\u03c4\u03bf \u03c3\u03c4\u03bf $ OP$\r\n\u03b4\u03b7\u03bb\u03b1\u03b4\u03ae \u03c0\u03c1\u03ad\u03c0\u03b5\u03b9 \u03bf\u03b9 \u03b1\u03ba\u03c4\u03af\u03bd\u03b5\u03c2 \u03c4\u03c9\u03bd \u03ba\u03cd\u03ba\u03bb\u03c9\u03bd \u03bd\u03b1 \u03b5\u03af\u03bd\u03b1\u03b9 \u03af\u03c3\u03b5\u03c2, \u03b4\u03b7\u03bb\u03b1\u03b4\u03ae \u03bf\u03b9 \u03b2\u03ac\u03c3\u03b5\u03b9\u03c2 \u03bd\u03b1 \u03b5\u03af\u03bd\u03b1\u03b9 \u03af\u03c3\u03b5\u03c2, \u03b4\u03b7\u03bb\u03b1\u03b4\u03ae \u03bd\u03b1 \u03ad\u03c7\u03bf\u03c5\u03bc\u03b5 \u03bf\u03c1\u03b8\u03bf\u03b3\u03ce\u03bd\u03b9\u03bf \u03c0\u03b1\u03c1\u03b1\u03bb\u03bb\u03b7\u03bb\u03cc\u03b3\u03c1\u03b1\u03bc\u03bc\u03bf, \u03bf\u03c0\u03cc\u03c4\u03b5 \u03b4\u03b5\u03bd \u03c4\u03ad\u03bc\u03bd\u03bf\u03bd\u03c4\u03b1\u03b9 \u03bf\u03b9 \u03c0\u03bb\u03b5\u03c5\u03c1\u03ad\u03c2\r\n\u03b1\u03ba\u03cc\u03bc\u03b1, \u03c0\u03c1\u03ad\u03c0\u03b5\u03b9 \u03bd\u03b1 \u03b4\u03bf\u03b8\u03b5\u03af \u03b7 \u03b4\u03b9\u03b5\u03c5\u03ba\u03c1\u03af\u03bd\u03b7\u03c3\u03b7 \u03cc\u03c4\u03b9 \u03c4\u03bf \u03cd\u03c8\u03bf\u03c2 \u03c0\u03c1\u03ad\u03c0\u03b5\u03b9 \u03bd\u03b1 \u03b5\u03af\u03bd\u03b1\u03b9 \u03bc\u03b9\u03ba\u03c1\u03cc\u03c4\u03b5\u03c1\u03bf \u03c4\u03bf\u03c5 \u03b7\u03bc\u03b9\u03b1\u03b8\u03c1\u03bf\u03af\u03c3\u03bc\u03b1\u03c4\u03bf\u03c2 \u03c4\u03c9\u03bd \u03b2\u03ac\u03c3\u03b5\u03c9\u03bd \u03b3\u03b9\u03b1 \u03bd\u03b1 \u03c4\u03ad\u03bc\u03bd\u03bf\u03bd\u03c4\u03b1\u03b9 \u03bf\u03b9 \u03ba\u03cd\u03ba\u03bb\u03bf\u03b9,"
}
{
"Tag": [
"inequalities",
"geometry",
"search",
"inequalities unsolved"
],
"Problem": "Prove or disprove: $a, b, c > 0 \\rightarrow a^4 + b^4 + c^4 + d^4 + 8abcd \\geq a^2bc + ab^2c + abc^2 + a^2bd + ab^2d + abd^2 + a^2cd + ac^2d + acd^2 + b^2cd + bc^2d + bcd^2$\r\n\r\nAre there any other inequalities with a weak $abcd$ term on the wrong side (which is no schurable)? Edit: I added this line just as harazi replied.",
"Solution_1": "For positive real numbers, take a look at a more general problem in the open questions area, caller \"Schur for n numbers\", posted by Vasc. I still remember the nice solution I gave to it. Your inequality is the case n=4.",
"Solution_2": "To answer your second question, the strongest inequality I know with the product in the suffering part of the inequality is Suranyi's inequality.",
"Solution_3": "Can you write that inequality for me ? Ok.\r\n\r\n\r\n---------------\r\nReyes.",
"Solution_4": "Just use search button and not only you'll find it, but alsoi its proof.",
"Solution_5": "Thanks! http://www.mathlinks.ro/Forum/viewtopic.php?highlight=schur&t=14906",
"Solution_6": "I have read it .It's hard and beautiful.Thanks!"
}
{
"Tag": [],
"Problem": "As the subject says, the problem below may seem ugly at first, but there is an excellent solution to it.\r\n\r\nFind the largest integer N such that $N < (\\sqrt{33+ \\sqrt{128}} + \\sqrt{2} - 8)^{-1}$",
"Solution_1": "Considering that $\\sqrt{128}=8.\\sqrt{2}$:\r\n\r\n$(\\sqrt{33+ \\sqrt{128}} + \\sqrt{2} - 8)^{-1}=({\\sqrt{\\frac{(\\sqrt{ 2}+8)^2}{2}}+\\sqrt{2} - 8})^{-1}=({\\frac{\\sqrt{2}+8}{\\sqrt{2}}+\\sqrt{2} - 8})^{-1}=({5.\\sqrt{2}-7})^{-1}$ \r\nthus:\r\n\r\n${}(5.\\sqrt{2}-7)^{-1}=\\frac{1.(5.\\sqrt{2}+7)}{(5.\\sqrt{2}-7).(5.\\sqrt{2}+7)}=5.\\sqrt{2}+7$\r\n\r\nKnowing that the approached value of $\\sqrt{2}$ is $1,41$ then the approach of $5.\\sqrt{2}+7$ is $14,07$ and the nearest number $N$ of the approach is 14",
"Solution_2": "FWIW - A \"fun\" way is to notice, as you have shown $5\\sqrt2 - 7$ is less than 1(being reciprocal of $5\\sqrt 2 +7 )$) and we see \r\n$(5\\sqrt 2 +7 ) - (5 \\sqrt 2 - 7 )$ is $14$ (so the requred answer is 14 without sort of pluging in the value of $\\sqrt 2$ :)"
}
{
"Tag": [
"function",
"real analysis",
"real analysis unsolved"
],
"Problem": "Prove or supply a counterexample: If the function $f$ from $\\mathbb{R}$ to $\\mathbb{R}$ has both a left limit and a right limit at each point of $\\mathbb{R}$, then the set of discontinuities of $f$ is,at most,countable.",
"Solution_1": "It's a very well-known problem, and I think it's been discussed before (I remember Myth posting very similar one, used in a math contest from Russia this year).\r\n\r\nThe set $A_n$ consisting of points $x$ for which $|f_+(x)-f_-(x)|\\ge\\frac 1n$ is discrete and thus countable ($f_+,f_-$ are the right and left limits, respectively), and since the set of discontinuities is the union of the $A_n$'s, that's that.",
"Solution_2": "[quote=\"grobber\"]The set $A_n$ consisting of points $x$ for which $|f_+(x)-f_-(x)|\\ge\\frac 1n$ is discrete and thus countable ($f_+,f_-$ are the right and left limits, respectively), and since the set of discontinuities is the union of the $A_n$'s, that's that.[/quote]\r\nIt doesn't include all the cases, not $|f_+(x)-f_-(x)|\\ge\\frac 1n$. Use $\\max\\{|f_+(x)-f(x)|,|f_-(x)-f(x)|\\}\\ge\\frac 1n$ instead.",
"Solution_3": "Right. Sorry, I was thinking about monotonic functions :).",
"Solution_4": "For monotonic functions, we don't need to do it in that way..."
}
{
"Tag": [
"geometry",
"3D geometry"
],
"Problem": "What is the sum of all the perfect cubes under 100?\r\n\r\n\r\n\r\n\r\nHow do you solve this? Please help. :?:",
"Solution_1": "This seems too easy so maybe I'm wrong: $ (1)^{3}\\plus{}2^{3}\\plus{}3^{3}\\plus{}4^{3}\\equal{}1\\plus{}8\\plus{}27\\plus{}64\\equal{}100$.",
"Solution_2": "Well, we know that\r\n\r\n$ 1^3\\plus{}2^3\\plus{}3^3\\plus{}...\\plus{}n^3\\equal{}(1\\plus{}2\\plus{}3\\plus{}...\\plus{}n)^2$. So, since the biggest\r\n\r\nperfect cube less then $ 100$ is $ 64\\equal{}4^3$, $ n\\equal{}4$. \r\n\r\nSo, $ (1\\plus{}2\\plus{}3\\plus{}4)^2\\equal{}10^2\\equal{}100$."
}
{
"Tag": [
"geometry",
"rectangle",
"induction"
],
"Problem": "Jean is paving a 2 by 10 rectangle with 1 by 2 paving stones. In how many different ways can she do this?",
"Solution_1": "lol. Did you put this up because you like the question or the pun?",
"Solution_2": "[hide]Let f(n) be the number of ways to pave a 2xn rectangle. By induction, we found that f(n)=f(n-1)+f(n-2), where f(1)=1, f(2)=2. So f(10)=89.[/hide]",
"Solution_3": "super008 wrote:[hide]Let f(n) be the number of ways to pave a 2xn rectangle. By induction, we found that f(n)=f(n-1)+f(n-2), where f(1)=1, f(2)=2. So f(10)=89.[/hide]\n\n\n\nsuper, to find your pattern did you calculate some of the numbers and notice the pattern or do you have a reason why it would follow that recurrence? If you have a reason, you should probably explain it.",
"Solution_4": "I have done this problem before. I first saw the pattern then did the induction.",
"Solution_5": "[quote=\"zabelman\"]lol. Did you put this up because you like the question or the pun?[/quote]\r\n\r\nJust the pun, sorry.\r\n\r\nThough Super008 should show his induction.",
"Solution_6": "What I meant is that you should explain why the pattern is true so other people can follow your solution.",
"Solution_7": "[hide]We observed that f(1)=1, f(2)=2, f(3)=3, f(4)=5, f(5)=8. And they look like consecutive Fibonacci numbers. \n\n\n\nClaim: f(n) = F_(n-1), where F_n is the nth Fibonacci number. \n\nProof: We will prove the claim by induction on n.\n\n\n\nWe have verify that it is true for n=1. \n\nAssume that f(n)=F_(n-1) is true for n=k, we need to prove that it is also true for n=k+1.\n\n\n\nNow in order to pave a 2x(k+1) rectangle, we could either put a 2x1 rectangle down like \"|\", then pave the rest, which is a 2xk rectangle; or we could put down two rectangles across like \"=\", then pave the rest, which is a 2x(k-1) rectangle. Since thses are the only starting position to pave the rectangle, we have f(k+1) = f(k) + f(k-1). \n\n\n\nBy assumption, f(k) = F_(k-1) and f(k-1) = F_(k-2). Substitute them in, we have f(k+1) = F_(k-1) + F_(k-2) = F_k. Thus we have proved that the assumption is true for n=k+1. By induction, we know that f(n)=F_(n-1) is true for positive integer n.[/hide]",
"Solution_8": "That should actually be F_(n + 1), but good job otherwise."
}
{
"Tag": [
"inequalities",
"inequalities unsolved"
],
"Problem": "suppose a,b,c are side length of a triangle, proof\r\n(1)$ a^3\\plus{}b^3\\plus{}c^3\\plus{}3abc\\minus{}2b^2a\\minus{}2c^2b\\minus{}2a^2c\\ge0$\r\n(2)$ 3a^2b\\plus{}3b^2c\\plus{}3c^2a\\minus{}3abc\\minus{}2b^2a\\minus{}2c^2b\\minus{}2a^2c\\ge0$ :D",
"Solution_1": "I've also been wondering whether I was really the first one to find them. None of them is hard anyway.\r\nhttp://www.mathlinks.ro/Forum/viewtopic.php?t=35295\r\n\r\n darij",
"Solution_2": "Easy :?: .. not at all to me\r\n\r\nwhy $ x^{3} \\plus{} 2xy^{2} \\plus{} y^{3} \\plus{} 2yz^{2} \\plus{} z^{3} \\plus{} 2zx^{2}\\geq 3(x^{2}y \\plus{} y^{2}z \\plus{} z^{2}x)$? (not necessary sides of triangle any more)",
"Solution_3": "[quote=\"greentreeroad\"]Easy :?: .. not at all to me\n\nwhy $ x^{3} \\plus{} 2xy^{2} \\plus{} y^{3} \\plus{} 2yz^{2} \\plus{} z^{3} \\plus{} 2zx^{2}\\geq 3(x^{2}y \\plus{} y^{2}z \\plus{} z^{2}x)$? (not necessary sides of triangle any more)[/quote]\r\n its easy by EMV theorem :)",
"Solution_4": "[quote=\"greentreeroad\"]Easy :?: .. not at all to me\n\nwhy $ x^{3} \\plus{} 2xy^{2} \\plus{} y^{3} \\plus{} 2yz^{2} \\plus{} z^{3} \\plus{} 2zx^{2}\\geq 3(x^{2}y \\plus{} y^{2}z \\plus{} z^{2}x)$? (not necessary sides of triangle any more)[/quote]\r\nIf $ x\\equal{}\\min\\{x,y,z\\},$ $ y\\equal{}x\\plus{}u$ and $ z\\equal{}x\\plus{}v$ then $ \\sum_{cyc}(x^3\\minus{}3x^2y\\plus{}2xy^2)\\geq0\\Leftrightarrow$\r\n$ \\Leftrightarrow2(u^2\\minus{}uv\\plus{}v^2)x\\plus{}u^3\\minus{}3u^2v\\plus{}2uv^2\\plus{}v^3\\geq0,$ which obviously true.",
"Solution_5": "[quote=\"M.A\"]\n its easy by EMV theorem :)[/quote]\r\n\r\nWhat is EMV... :blush:",
"Solution_6": "[quote=\"greentreeroad\"][quote=\"M.A\"]\n its easy by EMV theorem :)[/quote]\n\nWhat is EMV... :blush:[/quote]\r\n its entirely mixing variables method :wink:",
"Solution_7": "[quote=\"M.A\"][quote=\"greentreeroad\"][quote=\"M.A\"]\n its easy by EMV theorem :)[/quote]\n\nWhat is EMV... :blush:[/quote]\n its entirely mixing variables method :wink:[/quote]\r\nwhat is entirly mixing variables please post that theory",
"Solution_8": "[quote=\"Javkhaa\"][/quote][quote=\"M.A\"][quote=\"greentreeroad\"][quote=\"M.A\"]\n its easy by EMV theorem :)[/quote]\n\nWhat is EMV... :blush:[/quote]\n its entirely mixing variables method :wink:[/quote][quote=\"Javkhaa\"]\nwhat is entirly mixing variables please post that theory[/quote]\r\nDownload file: http://reflections.awesomemath.org/2006_5/2006_5_entirelymixing.pdf",
"Solution_9": "[quote=\"greentreeroad\"]suppose a,b,c are side length of a triangle, proof\n(1)$ a^3 \\plus{} b^3 \\plus{} c^3 \\plus{} 3abc \\minus{} 2b^2a \\minus{} 2c^2b \\minus{} 2a^2c\\ge0$\n[/quote]\r\nSuppose that $ c\\equal{}min\\{a;b;c\\}$\r\nThis ineq can rewrite:\r\n$ (a\\plus{}b\\minus{}c)(a\\minus{}b)^2\\plus{}(a\\plus{}c\\minus{}b)(a\\minus{}c)(b\\minus{}c) \\geq 0$\r\nNote that $ a,b,c$ are side length of a triangle,so we have $ a\\plus{}b\\minus{}c>0;a\\plus{}c\\minus{}b>0$\r\nDone!",
"Solution_10": "[quote=\"greentreeroad\"]suppose a,b,c are side length of a triangle, proof\n(2)$ 3a^2b \\plus{} 3b^2c \\plus{} 3c^2a \\minus{} 3abc \\minus{} 2b^2a \\minus{} 2c^2b \\minus{} 2a^2c\\ge0$ :D[/quote]\r\nSuppose that $ c\\equal{}min\\{a;b;c\\}$\r\nWe can rewrite this ineq:\r\n$ c(a\\minus{}b)^2\\plus{}(2a\\minus{}b)(a\\minus{}c)(b\\minus{}c) \\geq 0$\r\nWe have $ 2a\\minus{}b \\geq a\\plus{}c\\minus{}b >0$\r\nDone!",
"Solution_11": "Is this SOS-Schur?"
}
{
"Tag": [
"Ramsey Theory",
"combinatorics unsolved",
"combinatorics"
],
"Problem": "Find the minimum number of point in plane in which three arbitrary point aren't collinear satisfy for all ways of painting all segment over two point always exit a triangle in which three sides are painted by a same colour!",
"Solution_1": "Well...it depends on the number of colours... :wink: \r\nFor $ n$ colours, the desired result is the Ramsey number $ R(3,\\cdots,3)$.\r\nIn particular, for two colours, the desired minimum is $ 6$.\r\n\r\nPierre."
}
{
"Tag": [
"Columbia",
"function",
"integration",
"algebra",
"polynomial",
"real analysis",
"real analysis unsolved"
],
"Problem": "Let $ n$ a positive integer, and $ f: [0,1]\\to\\mathbb{R}$ a continuos function such that \\[ \\int_{0}^{1}x^{k}f(x) dx\\equal{}1\\] for $ k\\equal{}0,1,2,\\ldots,n\\minus{}1$. Prove that \\[ \\int_{0}^{1}f^{2}(x) dx\\geq n^{2}\\]",
"Solution_1": "See http://www.mathlinks.ro/Forum/viewtopic.php?&t=146373 or http://www.mathlinks.ro/viewtopic.php?p=443980#443980\r\n(The approaches are similar, but I don't understand why $ \\int_{0}^{1}\\left( f(x)\\minus{}q(x)\\right) q(x)\\, dx \\equal{} 0$ in jmerry's solution.)",
"Solution_2": "It's about orthogonality. Let $ p_{0},p_{1},\\dots,p_{n\\minus{}1}$ be an orthogonal basis for polynomials of degree $ \\le n\\minus{}1$; the hypothesis is equivalent to the system $ \\langle f,p_{i}\\rangle\\equal{}c_{i}$ for some fixed constants $ c_{i}$. When we subtract the solutions, we get $ \\langle f\\minus{}q,p_{i}\\rangle\\equal{}0$ for each $ i$. Since the $ p_{i}$ span the polynomials of degree $ \\le n\\minus{}1$, $ f\\minus{}q$ is orthogonal to all polynomials of degree $ n\\minus{}1$, including $ q$ in particular."
}
{
"Tag": [
"probability"
],
"Problem": "I had breadsticks and Marinara.(breakfast, not lunch.)",
"Solution_1": "wtf is up wit tis\r\n\r\nwhatev, ill do it\r\n\r\ni had poha\r\n\r\nguz wat tat is",
"Solution_2": "I had banana bread, 2 bagels, 1 sports drink, 1 water, and two small pieces of sandwich...lunch time!",
"Solution_3": "I haven't had breakfast yet...",
"Solution_4": "Right now I'm having lunch....chocolate and bananas :wink:",
"Solution_5": "lunch=grilled cheese and fries",
"Solution_6": "a bowl of cinnamon toast crunch with 1% milk.",
"Solution_7": "Nothing, I don't eat breakfast.",
"Solution_8": "Rice with butter and a fried egg - I leave for college after that.",
"Solution_9": "i never eat breakfast(seriously)",
"Solution_10": "poha is good!\r\n\r\ni had puri aloo kudu and hulwa, because its janmashtmi",
"Solution_11": "I never eat breakfast either.",
"Solution_12": "[quote=\"star99\"]poha is good!\n\ni had puri aloo kudu and hulwa, because its janmashtmi[/quote]\r\n\r\nGARBA",
"Solution_13": "I had a chili dog and a pizza pretzel",
"Solution_14": "I'm eating waffles!!!!! Ego my Lego! Or wtf.\r\nI always thought that meant I have my I say b/c of greek.",
"Solution_15": "[quote=\"funcia\"]I'm eating waffles!!!!! Ego my Lego! Or wtf.\nI always thought that meant I have my I say b/c of greek.[/quote]\r\nLol! :rotfl: \r\n\r\nSorry, but I find this very funny.",
"Solution_16": "The first thing I do when I wake up is eat. I've tried to wait until I'm out of my room to eat since I think it bothers my roommate when I rummage through my cabinet while he's still trying to sleep :P (I mean, I try really hard to be quiet :( ).\r\n\r\nAnyway, today I had an energy bar, a couple pieces of bread with peanut butter, and a banana.\r\n\r\nMy two favorite things for breakfast are muffins and breakfast burritos (from Anna's Taqueria)...mmm.",
"Solution_17": "I had a chocolate chip cookie dough poptart and strawberries. Now I get to have a bowl of cereal as an almost-1am snack. :jump: :rotfl:",
"Solution_18": "I usually have something random: The probability that I'll have oatmeal is definately greater tha 1/2 :(",
"Solution_19": "Ah, I need to update this with what I ate today:\r\n\r\ntwo bagels (with nutella), 2 pieces of french toast, 1 pancake, a handful of fruits+nuts, 2 glasses of smoothies, 1 slice of ham, 1/2 nutri-grain bar\r\n\r\nSo full....",
"Solution_20": "Two fried eggs, a pickle, and green tea. :D",
"Solution_21": "Two boiled eggs, a piece of toast with peanut butter, a bowl of cereal, three glasses of orange juice, a bagel with cream cheese, and half a blueberry muffin.",
"Solution_22": "NOTHING AT ALL. I have to fast today :(",
"Solution_23": "Me too, that's why I ate so much yesterday."
}
{
"Tag": [],
"Problem": "Go [url=http://www.artofproblemsolving.com/Forum/viewtopic.php?t=209860]here[/url] for the rules\r\n\r\nOne difference: there will be no quests.\r\n\r\n14 people to start. (1 king, 1 queen, 2 known knights, 1 unknown knight, 9 citizens)\r\n\r\nSignups:\r\n\r\n1. meewhee009\r\n2. Scamper\r\n3.\r\n4.\r\n5.\r\n6.\r\n7.\r\n8.\r\n9.\r\n10.\r\n11.\r\n12.\r\n13.\r\n14.",
"Solution_1": "1. meewhee\r\n2.\r\n3.\r\n4.\r\n5.\r\n6.\r\n7.\r\n8.\r\n9.\r\n10.\r\n11.\r\n12.\r\n13.\r\n14.\r\n\r\nOh yeah. /in :)",
"Solution_2": "Me joins as well.",
"Solution_3": "1. meewhee009\r\n2. Scamper\r\n3. jjfun1\r\n4.\r\n5.\r\n6.\r\n7.\r\n8.\r\n9.\r\n10.\r\n11.\r\n12.\r\n13.\r\n14.\r\n\r\nSounds fun.\r\n[b]Join.[/b]\r\nW00t, I got my lucky number. :)",
"Solution_4": "Then why aren't you jjfun3? This has been a public shaming of jjfun1. Thank you. [yet another revival]\r\n\r\n1. meewhee009 \r\n2. Scamper \r\n3. jjfun1 \r\n4. ernie\r\n5. \r\n6. \r\n7. \r\n8. \r\n9. \r\n10. \r\n11. \r\n12. \r\n13. \r\n14.",
"Solution_5": "1. meewhee009 \r\n2. Scamper \r\n3. jjfun1 \r\n4. ernie \r\n5. 15!!!\r\n6. \r\n7. \r\n8. \r\n9. \r\n10. \r\n11. \r\n12. \r\n13. \r\n14.",
"Solution_6": "[quote=\"ernie\"]Then why aren't you jjfun3? This has been a public shaming of jjfun1.[/quote]\r\nMaybe in ernie-land, three and fun rhyme. But not in jjfun1-land. :P",
"Solution_7": "1. meewhee009 \r\n2. Scamper \r\n3. jjfun1 \r\n4. ernie \r\n5. 15!!! \r\n6. westiepaw\r\n7. \r\n8. \r\n9. \r\n10. \r\n11. \r\n12. \r\n13. \r\n14.",
"Solution_8": "/in :D :D :D",
"Solution_9": "1. meewhee009\r\n2. Scamper\r\n3. jjfun1\r\n4. ernie\r\n5. 15!!!\r\n6. westiepaw\r\n7. EggyLv.999\r\n8.\r\n9.\r\n10.\r\n11.\r\n12.\r\n13.\r\n14.",
"Solution_10": "\\me is in.",
"Solution_11": "1. meewhee009\r\n2. Scamper\r\n3. jjfun1\r\n4. ernie\r\n5. 15!!!\r\n6. westiepaw\r\n7. EggyLv.999\r\n8. spaceguy524\r\n9.\r\n10.\r\n11.\r\n12.\r\n13.\r\n14.",
"Solution_12": "1. meewhee009\r\n2. Scamper\r\n3. jjfun1\r\n4. ernie\r\n5. 15!!!\r\n6. westiepaw\r\n7. EggyLv.999\r\n8. spaceguy524\r\n9. FlyAgaric\r\n10.\r\n11.\r\n12.\r\n13.\r\n14.",
"Solution_13": "Sure, I'll play.\r\n\r\n1. meewhee009\r\n2. Scamper\r\n3. jjfun1\r\n4. ernie\r\n5. 15!!!\r\n6. westiepaw\r\n7. EggyLv.999\r\n8. spaceguy524\r\n9. FlyAgaric\r\n10. cf249\r\n11.\r\n12.\r\n13.\r\n14.",
"Solution_14": "join!\r\n :nhl: \r\nI will beat everyone I want to beat using this hockey stick.\r\nAlso, this reminds me of \"The once and future king\"!\r\n*Read, Read, Journal, Read...*",
"Solution_15": "Mwahahaha :).\r\n\r\nmeewhee009-20 pts\r\nScamper-21 pts\r\njjfun1-9 pts\r\nwestiepaw-5 pts\r\nEggyLv.999-10 pts\r\nspaceguy524-10 pts\r\nFlyAgaric-10 pts\r\nBramblestar-7 pts\r\nmathling235-18 pts \r\n[b]Scamper for 1[/b]",
"Solution_16": "When are execution days again?",
"Solution_17": "[b]Teh king for 1[/b]",
"Solution_18": "Every third day is execution day.",
"Solution_19": "When is the next execution day?",
"Solution_20": "Day 9 is next execution day.",
"Solution_21": "meewhee009-19 pts \r\nScamper-21 pts \r\njjfun1-9 pts \r\nwestiepaw-5 pts \r\nEggyLv.999-9 pts \r\nspaceguy524-10 pts \r\nFlyAgaric-10 pts \r\nBramblestar-7 pts \r\nmathling235-18 pts \r\n\r\nDay 8 ends.\r\n\r\nDay 9 starts. Everyone gets their salaries:\r\n\r\nmeewhee009-22 pts \r\nScamper-23 pts \r\njjfun1-10 pts \r\nwestiepaw-6 pts \r\nEggyLv.999-10 pts \r\nspaceguy524-11 pts \r\nFlyAgaric-11 pts \r\nBramblestar-8 pts \r\nmathling235-19 pts \r\n\r\nRandom Event: King Execution Day\r\n\r\nThe king may pay 15 points to instantly kill any one person. Under the terms of the new law, EggyLv.999 can be executed for free.",
"Solution_22": "If we attack the king today, can the king execute us for free or does she have to wait for the next execution day? Or is the law no longer being imposed?\r\n\r\nEggyLv.999 should attack the king.",
"Solution_23": "meewhee009-22 pts \r\nScamper-23 pts \r\njjfun1-10 pts \r\nwestiepaw-6 pts \r\n[b]EggyLv.999 haz been killedzzz![/b]\r\nspaceguy524-11 pts \r\nFlyAgaric-11 pts \r\nBramblestar-8 pts \r\nmathling235-19 pts \r\n :P \r\nI have a new aim...\r\nhehehehe...",
"Solution_24": "The king now has to wait until day 12 until another execution can be done.",
"Solution_25": "meewhee009-22 pts \r\nScamper-23 pts \r\njjfun1-10 pts \r\nwestiepaw-6 pts \r\nspaceguy524-11 pts \r\nFlyAgaric-11 pts \r\nBramblestar-8 pts \r\nmathling235-19 pts \r\n\r\nDay 9 ends.\r\n\r\nDay 10 starts. Everyone gets their salaries\r\n\r\nmeewhee009-25 pts \r\nScamper-25 pts \r\njjfun1-11 pts \r\nwestiepaw-7 pts \r\nspaceguy524-12 pts \r\nFlyAgaric-12 pts \r\nBramblestar-9 pts \r\nmathling235-20 pts\r\n\r\nRandom Event: Small Money Flow\r\n\r\nSo money is becoming scarce in the country, and very little is getting to the king.\r\n\r\n[b]Starting tomorrow, the king only gets 2 points a day in salaries.[/b]",
"Solution_26": "darnit\r\n\r\ni gots executed",
"Solution_27": "more activity please",
"Solution_28": "Does the law passed on Day 9 still apply?",
"Solution_29": "It does, but meewhee can only execute one of you at a time, and there are 7 of you left..."
}
{
"Tag": [
"Pythagorean Theorem",
"geometry"
],
"Problem": "How do I go about doing this?\r\n\r\nI know how to prove the distance formula by using the pythagorean theorem, however, I don't know how to prove the mid-point formula by using the distance formula.\r\n\r\nI can prove the mid-point formula by using a graph and relating it to the distance formula, but how do I do this by solely using the distance formula?\r\n\r\nPlease enlighten me.\r\n\r\nRemember:\r\n\r\nD = sqrt((Xsub2 - Xsub1)squared + (Ysub2 - Ysub1)squared) \r\n\r\nand\r\n\r\nM = ((Xsub2 + Xsub1)/2 , (Ysub2 + Ysub1)/2)\r\n\r\nThanks,\r\ndfunk",
"Solution_1": "Nobody?",
"Solution_2": "So we have the points $A \\ (x_1,y_1)$ and $B \\ (x_2,y_2)$ and want to prove that $M \\ \\left(\\frac{x_1+x_2}{2},\\frac{y_1+y_2}{2}\\right)$ is the midpoint. It is sufficient to show that the distance $AM=BM=\\frac 12 AB$. So we have:\r\n\r\n\\begin{eqnarray*}\r\nAB &=& \\sqrt{(x_1-x_2)^2+(y_1-y_2)^2}\\\\\r\nAM &=& \\sqrt{\\left(x_1-\\frac{x_1+x_2}{2}\\right)^2+\\left(y_1-\\frac{y_1+y_2}{2}\\right)^2}\\\\\r\nBM &=& \\sqrt{\\left(x_2-\\frac{x_1+x_2}{2}\\right)^2+\\left(y_2-\\frac{y_1+y_2}{2}\\right)^2}\r\n\\end{eqnarray*}\r\n\r\nSo $AM=\\sqrt{\\left(\\frac{x_1-x_2}{2}\\right)^2+\\left(\\frac{y_1-y_2}{2}\\right)^2}$. Doing the algebra we easiliy see this is equivalent to $BM$. Now we only need to prove that this is $\\frac 12 AB$ as follows:\r\n\r\n\\begin{eqnarray*}\r\nAM &=& \\sqrt{\\left(\\frac{x_1-x_2}{2}\\right)^2+\\left(\\frac{y_1-y_2}{2}\\right)^2}\\\\\r\n &=& \\sqrt{\\frac 14\\left(\\left(\\frac{x_1-x_2}{2}\\right)^2+\\left(\\frac{y_1-y_2}{2}\\right)^2\\right)}\\\\\r\n &=&\\frac 12 \\sqrt{(x_1-x_2)^2+(y_1-y_2)^2}\\\\\r\n &=& \\frac 12 AB\r\n\\end{eqnarray*}",
"Solution_3": "Thanks a bunch."
}
{
"Tag": [
"geometry",
"calculus",
"topology",
"analytic geometry",
"graphing lines",
"slope",
"geometric transformation"
],
"Problem": "What sorts of topics and questions belong on this forum? \r\n\r\nTo help you decide what the use of this forum is, I have compiled some example questions that might be asked here. These are not strict guidelines by any means. Generally, you can post anything here that is math related but does not fall into one of the three other math forums. If you are unsure, please use the other posts on this forum and the example questions below as a guide.\r\n\r\n(1) Resources for problems and problem solving.\r\n\ufffd\tWhere can I find a book full of past AMC 12-level geometry problems?\r\n\ufffd\tWhat is a good textbook on Number Theory?\r\n\ufffd\tWould you recommend Mathematical Reasoning: Writing and Proof by Ted Sundstrom for beginners learning how to write proofs?\r\n\r\n(2) Problems from topics that are not generally tested on high school mathematics competitions, like a difficult problem from a textbook. \r\n\ufffd\tI don't understand this calculus question; can you give me a hint?\r\n\ufffd\tI would like to suggest this math-oriented Physics/Chemistry problem as a challenge.\r\n\ufffd\tCan someone help me with this beginning topology question?\r\n\r\n(3) Questions about theory (not a problem).\r\n\ufffd\tI do not understand mathematical induction. Can you explain the difference between inductive and deductive reasoning?\r\n\ufffd\tWhat is the proof for the Binomial Theorem?\r\n\ufffd\tHow many *Fundamental Theorem of XYZ* are there and what are they?\r\n\r\n(4) Discussion of current events in mathematics. \r\n\r\n(5) Methods of teaching mathematics.\r\n\r\n(6) Studying mathematics. These should be questions about studying mathematics that do not belong in the \ufffdCollege\ufffd forum.\r\n\ufffd\tWhat is the calculus sequence you learn in college?\r\n\ufffd\tWhat are the other types of mathematics you can learn in college that are not part of the calculus sequence, such as combinatorics, statistics, number theory, and discrete math?\r\n\r\n(7) Discussion of other topics in mathematics which are similar to threads on the \ufffdRound Table\ufffd but are mathematics-oriented.\r\n\ufffd\tShould I take AP Calculus AB, AP Calculus BC, or AP Statistics?\r\n\ufffd\tWhy is the letter m used to represent the slope of a line?\r\n\r\nPlease remember that these are not strict guidelines.\r\n\r\nThanks, and please use the forum responsibly!",
"Solution_1": "That's a great list of topics, and well reflects how this forum has been used over the last school year. I would only add that if the topic has to do with math, it really belongs [b]here[/b], on Other Problem Solving Topics, and [b]not[/b] on the Round Table, which at its best is more for issue-oriented political such as several of the highest-volume threads posted there recently. \r\n\r\nI'm glad to see that you've come on board as a co-moderator here.",
"Solution_2": "Yes, I think that that is a wonderful guide. Welcome to the elite corps of moderators! ;)",
"Solution_3": "Thanks! I feel elite already :D ! I will edit it w/Tokenadult's suggestion."
}
{
"Tag": [
"ceiling function",
"logarithms",
"number theory",
"greatest common divisor",
"relatively prime",
"number theory proposed"
],
"Problem": "Given $a_0 > 1$, the sequence $a_0, a_1, a_2, ...$ is such that for all $k > 0$, $a_k$ is the smallest integer greater than $a_{k-1}$ which is relatively prime to all the earlier terms in the sequence.\r\nFind all $a_0$ for which all terms of the sequence are primes or prime powers.",
"Solution_1": "No odd numbers larger than 3 can be included unless the number following is a power of two, and the number following that is a power of three. 2 and 3 work in this set, as 4 is a power of a prime, 2, and all following numbers will be prime (as they are relatively prime to 3 and 4, and every number before that relatively prime to 3 and 4 etc.). 7 also works, because directly above 7 is a power of 2, and above that is a power of 3, and a factor of 5 will be included at 25, also a power of a prime.\r\n\r\nI don't think any other numbers exist, as there it must go in the order (prime, power of 2, power of 3, power of 5, etc) all the way up to a power of the prime below the starting prime, which can be shown (I think) cannot happen.",
"Solution_2": "A hint: show that if n is sufficiently large then $a_n$ is prime.",
"Solution_3": "Does anyone have a full solution to the problem?",
"Solution_4": "All solutins are $a_{0}=2,3,4,7,8.$ It is easy to check that if $a_{0}<10$ only these solutions. For prove, that it had not another solutions sufficiently consider equations $2^{k}\\pm1=3^{l}$, $2^{k}\\pm 1=5^{l}$ (k>3) and $3^{k}\\pm 2=5^{l}$(k>1). This equations had not solutions. It give if $a_{0}>9$ had not another solutions.",
"Solution_5": "Suppose $a_1,a_2,...,$ is a sequence such that the $a_i$ are all prime powers. (I'm starting at $a_1$ because starting with $a_0$ is annoying)\n\n[b]Lemma 1:[/b] Every prime divides a term in the sequence.\n[b]Proof:[/b] Take any prime $p$. Suppose $k$ is the least integer such that $a_1 \\le p^k$. Now let $z$ be the least such integer such that $a_z \\ge p^k$. If $a_z \\neq p^k$, then we have $p$ divides one of $a_1,a_2,...,a_{z-1}$. But then we're done.\n\n[b]Corollary:[/b] If $p$ is prime, then $p^{\\lceil \\log_p{a_1} \\rceil}$ is one of the $a_i$.\nIt immediately follows for sufficiently large $n$, we have the $a_n$ are just a bunch of consecutive primes. Note that by the corollary we have that if $a_n > a_0^2$ then $a_n$ is prime.\n\nI claim we can pick two primes $p,q$ such that $pq \\in [a_1,a_1^2]$ but $p^{\\lceil \\log_p{a_1} \\rceil},q^{\\lceil \\log_q{a_1} \\rceil} > pq$\n\nBy Bertrand's Postulate we can find a prime $p$ such that $\\frac{\\sqrt{a_1}}{2} - 1 < p < \\sqrt{a_1}$ when $a_1 \\ge 16$. Let $\\frac{\\sqrt{a_1}}{p} = z$. Then we can also find a prime $q$ such that $z\\sqrt{a_1} < q < 2z\\sqrt{a_1}$.\nNow, clearly $pq > a_1$. However, $p^{\\lceil \\log_p{a_1} \\rceil} \\ge p^3 > pq$. We also have $q^{\\lceil \\log_q{a_1} \\rceil} = q^2 > pq$. Thus it easily follows $pq$ must be one of the $a_i$, contradiction and thus we have $a_1 < 16$. Checking which of these work is just stupid casework.\n\nEDIT : So I just realized $16$ is not big enough as then in strange cases we can have $q \\ge p^2$. This method actually yields something like $a_1 \\le 256$ which is pretty hard to check up to... though working with [b]proglote[/b] a bit on this we deduced $a_0$ must be of the form $2^{2^n}$ except in some exceptional cases so I guess its not too bad.",
"Solution_6": "Here's another solution with some casework:\n\nIf $a_0$ is odd, the next terms in the sequence are clearly $a_0 + 1$ and $a_0 + 2$. Since $a_0 +1$ is even, it must be a power of two, and either $a_0$ or $a_0 + 2$ is a power of three. So it remains to solve $2^{a} \\pm 1 = 3^{b}$ which is easy, giving $a = 2,3 \\implies a_0 \\in \\{3,7\\}.$\nIf $a_0$ is even, then the next terms in the sequence are clearly $a_0 +1, a_0 +3, a_0 +5$ (though we can't say anything further since $a_0 + 7$ and $a_0 + 1$ may both be multiples of 3). Also note that $a_0 = 2^a$ for some positive integer $a$.\nIf $a$ is 1 mod 3, then $7|2^{a} +5$, i.e. $a_0 + 5$ is a power of $7.$ So $2^a + 5= 7^b$, which implies $a=1$, so $a_0 = 2.$\nIf $a$ is 2 mod 3, then similarly it remains to solve $2^a + 3 = 7^b$, and the only solution is clearly $a = 2$, so $a_0 = 4.$\nIf $3|a$, then $a_0 + 1= 2^{3a'} + 1$ for some positive integer $a'.$ Note that $x^3 + 1 = (x+1)(x^2 - x + 1)$ and $(x+1, x^2 - x + 1) = (x+1, 3).$ Since $2^{3a'} + 1$ is a prime power, it cannot be factored into two coprime integers greater than 1, so the gcd of the two factors is 3 and we get $2^a + 1 = 3^b$ for some $b.$ So the only solution is $a=3$, i.e. $a_0 = 8.$\nTherefore, $a_0 \\in \\{2,3,4,7,8\\}.$",
"Solution_7": "[quote=proglote]Here's another solution with some casework:\n\nIf $a_0$ is odd, the next terms in the sequence are clearly $a_0 + 1$ and $a_0 + 2$. Since $a_0 +1$ is even, it must be a power of two, and either $a_0$ or $a_0 + 2$ is a power of three. So it remains to solve $2^{a} \\pm 1 = 3^{b}$ which is easy, giving $a = 2,3 \\implies a_0 \\in \\{3,7\\}.$\nIf $a_0$ is even, then the next terms in the sequence are clearly $a_0 +1, a_0 +3, a_0 +5$ (though we can't say anything further since $a_0 + 7$ and $a_0 + 1$ may both be multiples of 3). Also note that $a_0 = 2^a$ for some positive integer $a$.\nIf $a$ is 1 mod 3, then $7|2^{a} +5$, i.e. $a_0 + 5$ is a power of $7.$ So $2^a + 5= 7^b$, which implies $a=1$, so $a_0 = 2.$\nIf $a$ is 2 mod 3, then similarly it remains to solve $2^a + 3 = 7^b$, and the only solution is clearly $a = 2$, so $a_0 = 4.$\nIf $3|a$, then $a_0 + 1= 2^{3a'} + 1$ for some positive integer $a'.$ Note that $x^3 + 1 = (x+1)(x^2 - x + 1)$ and $(x+1, x^2 - x + 1) = (x+1, 3).$ Since $2^{3a'} + 1$ is a prime power, it cannot be factored into two coprime integers greater than 1, so the gcd of the two factors is 3 and we get $2^a + 1 = 3^b$ for some $b.$ So the only solution is $a=3$, i.e. $a_0 = 8.$\nTherefore, $a_0 \\in \\{2,3,4,7,8\\}.$[/quote]\n\nmacetei"
}
{
"Tag": [
"geometry",
"perimeter",
"geometry solved"
],
"Problem": "Could anybody prove the following theorem\r\n\r\n[color=green][b]Theorem.[/b] Let P and Q be two convex polygons such that P is on the non-externally region of Q, then prove that perimeter(P)=< perimeter(Q).[/color]\r\n\r\nUsing this lema (I proved this lema)\r\n\r\n[color=green][b]Lema. [/b]Let Q be a convex polygons and let P be a triangle such that P is on the non-externally region of Q, then prove that perimeter(P)=< perimeter(Q).[/color]\r\n\r\nThanks a lot..\r\n\r\nJos\u00e9 Carlos",
"Solution_1": "In [url=http://www.mathlinks.ro/Forum/viewtopic.php?p=197437#p197437]http://www.mathlinks.ro/Forum/viewtopic.php?t=31419 post #3[/url], this was proven for the case when the external figure Q is a circular disc rather than a polygon, but for a polygon the proof is exactly the same.\r\n\r\n Darij"
}
{
"Tag": [
"limit",
"trigonometry",
"inequalities",
"real analysis",
"ratio",
"quadratics",
"continued fraction"
],
"Problem": "Is it true that\r\n\\begin{eqnarray*} \\lim_{n \\rightarrow \\infty} \\frac{1}{n \\sin n} & = & 0 \\end{eqnarray*}\r\nWhat about\r\n\\begin{eqnarray*} \\underset{n \\longrightarrow \\infty}{\\lim \\sup} \\frac{1}{n \\sin n} & = & ? \\end{eqnarray*}",
"Solution_1": "I should think not. We can find some $m$ (infinitely many in fact) so that $|\\pi - \\frac{n}{m}| < \\frac{1}{m^2}$. Then $\\frac{1}{n\\sin(n)}$ is about $\\frac{1}{n(\\frac{1}{m^2})} \\approx \\frac{m}{\\pi}$, which is arbitrarily large. \r\n\r\n(for rigor you should probably take two terms of the taylor series for sine so that the \"about\" is an inequality in the direction we want, and then bound from each side by a constant times $m$)\r\n\r\nI bet it does converge in some weaker sense, like Cesaro.",
"Solution_2": "The first fact is a corollary of Hurwitz theorem, as far as I know.\r\nOtherwise, I don't think you really need to mess up with Taylor series. Some basic inequalities should suffice.",
"Solution_3": "For any $c$, $\\limsup \\left|\\frac1{n\\sin cn}\\ge\\frac1{\\pi}\\right|$. This limsup is $\\infty$ when the continued fraction for $\\frac c{\\pi}$ has arbitrarily large quotients or $\\frac c{\\pi}$ is rational.\r\n\r\nFor any integer $m$, $|\\sin cn|=|\\sin(cn-m\\pi)|< |cn-m\\pi|=n\\pi\\left|\\frac{c}{\\pi}-\\frac mn\\right|$. If $\\frac pq$ is a convergent (in lowest terms) of the continued fraction for $\\frac c{\\pi}$, $\\left| \\frac{c}{\\pi}-\\frac pq\\right|<\\frac1{q^2}$. There are infinitely many convergents $\\frac pq$; choosing these as our $m,n$, we have $n\\pi\\left|\\frac{c}{\\pi}-\\frac mn\\right|<\\frac{\\pi}n$. Taking the reciprocal and dividing by $n$, we have $|\\frac1{n\\sin cn}|> \\frac1{\\pi}$.\r\n\r\nXevarion's argument misses one of these factors of $n$; whether the $\\limsup$ is finite in any particular case is an interesting question. It is always nonzero.\r\n\r\nIf we drop the absolute value signs, we can show that $\\limsup \\frac1{n\\sin cn}\\ge\\frac1{4\\pi}$ and $\\liminf \\frac1{n\\sin cn}\\le-\\frac1{4\\pi}$. The convergents $\\frac pq$ alternate between being too large and too small; if we then let $n=2q$, $\\sin 2cn$ will be positive when $\\frac pq$ is low and negative when $\\frac pq$ is high. In both cases, we lose a factor of 4 by doubling.",
"Solution_4": ":blush: \r\nThanks for correcting my error jmerry. Can you please give some more detail on the case where the continued fraction for $\\frac{c}{\\pi}$ has arbitrarily large quotients? I'm afraid I don't see immediately why it's so ... :oops:",
"Solution_5": "If the quotient after the convergent $\\frac pq$ is $r$, we have $\\left|\\frac c{\\pi}-\\frac pq\\right|\\le\\frac1{rq^2}$.\r\n\r\nI think so, anyway; it's been a while since I worked out the details. The two quantities are comparable in any case.\r\nAs a corollary, large quotients indicate unusually good rational approximations. As a famous example, here's the first few terms of the continued fraction for $\\pi$:\r\n3;7,15,1,292\r\nIf we cut off just before the 292, we get $\\frac{355}{113}$ as a convergent.",
"Solution_6": "Oh, I get it, that's really cool! Thanks :D \r\n\r\nAre there general methods for knowing whether the continued fraction of some irrational has arbitrarily large quotients (other than actually knowing the general term, like in case of the golden ratio)?",
"Solution_7": "[quote=\"Xevarion\"]... like in case of the golden ratio)?[/quote]\r\nTheorem: the quotients of the continued fraction of a number are periodic if and only if the number is a quadratic irrational - that is an irrational number that can be written in the form $\\frac{a+\\sqrt{b}}c$ or $\\frac{a-\\sqrt{b}}c$ for integers $a,b,c.$ (If the number is rational, then the continued fraction representation terminates.) Of course, the golden ratio is a quadratic irrational.\r\n\r\nSo quadratic irrationals cannot have arbitrarily large quotients, because the quotients are periodic. But we can't say anything about the converse."
}
{
"Tag": [
"inequalities proposed",
"inequalities"
],
"Problem": "If $ a,b,c>0$ such that $ a\\plus{}b\\plus{}c\\equal{}3$, then: \r\n\r\n$ (a\\plus{}\\frac1{b^2\\plus{}c^2})(b\\plus{}\\frac1{a^2\\plus{}c^2})(c\\plus{}\\frac1{b^2\\plus{}a^2})\\le\\frac{27}8$",
"Solution_1": "[quote=\"akai\"]If $ a,b,c > 0$ such that $ a \\plus{} b \\plus{} c \\equal{} 3$, then: \n\n$ (a \\plus{} \\frac1{b^2 \\plus{} c^2})(b \\plus{} \\frac1{a^2 \\plus{} c^2})(c \\plus{} \\frac1{b^2 \\plus{} a^2})\\le\\frac {27}8$[/quote]\r\nTry $ a\\equal{}b\\equal{}0.001,c\\equal{}2.998$.\r\nIf you meant that\r\n\\[ (a \\plus{} \\frac1{b^2 \\plus{} c^2})(b \\plus{} \\frac1{a^2 \\plus{} c^2})(c \\plus{} \\frac1{b^2 \\plus{} a^2})\\ge\\frac {27}8\\]\r\nThen try $ a\\equal{}b\\equal{}\\frac{1}{2},c\\equal{}2$."
}
{
"Tag": [],
"Problem": "if f(z)=z+1/z-1, then find f to the 1991 power (2+i).",
"Solution_1": "[hide]\nTry the first two values to find that f^2(2+i) = 2+i, so f^1991(2+i) = f^1(2+i) = (3+i)/(1+i).[/hide]"
}
{
"Tag": [
"geometry",
"incenter",
"geometry unsolved"
],
"Problem": "$ O$ is incircle of a $ \\triangle ABC$. An another circle $ O'$ tangent to rays $ AB,AC$.If center of $ O'$ is lie circle $ O$, prove that $ \\triangle ABC$ is isosceles. \r\n\r\n H.\u0130BRAH\u0130M AYANA",
"Solution_1": "i don't see why this needs to be true:\r\n[asy]size(200); pair A,B,C,D,X,Y,Z; C=origin; B=(50,20); A=(30,70); draw(A--B--C--cycle); draw(incircle(A,B,C)); D=incenter(A,B,C); dot(D); X=(13.5,32); Y=(43,37); draw(incircle(A,X,Y)); Z=incenter(A,X,Y); dot(Z); label(\"A\",A,N); label(\"B\",B,E); label(\"C\",C,W); label(\"O\",D,S); label(\"O'\",Z,N);[/asy]"
}
{
"Tag": [
"MATHCOUNTS",
"modular arithmetic"
],
"Problem": "1. (AoPS) Factor 141, 1441, and 14441.\r\n\r\n2. (AoPS) Find the base 8, 9, and 16 representations of $47_{10}$.\r\n\r\n3. (AoPS) Find the base 10 equivalents of $BEE_{16}$, $DEF_{16}$, and $A1_{16}$.\r\n\r\n4. (MATHCOUNTS 1986) Find the value of digit $A$ if the five-digit number 12A3B is divisible by both 4 and 9, and $A\\ne B$.\r\n\r\n5. (MATHCOUNTS 1988) In how many ways can a debt of 69 dollars be paid exactly using only 5 dollar bills and 2 dollar bills?\r\n\r\n6. (MATHCOUNTS 1988) Find the sum of all four digit natural numbers of the form $4AB8$ which are divisible by 2, 3, 4, 6, 8, and 9.\r\n\r\n7. (Mu Alpha Theta 1990) If $a\\ne 1$ and $\\sqrt[a]{10000_{a}}= 10_{a}$, find $a$.\r\n\r\nGood luck!",
"Solution_1": "[quote=\"chess64\"]1. (AoPS) Factor 141, 1441, and 14441.\n\n2. (AoPS) Find the base 8, 9, and 16 representations of $47_{10}$.\n\n3. (AoPS) Find the base 10 equivalents of $BEE_{16}$, $DEF_{16}$, and $A1_{16}$.\n\n4. (MATHCOUNTS 1986) Find the value of digit $A$ if the five-digit number 12A3B is divisible by both 4 and 9, and $A\\ne B$.\n\n5. (MATHCOUNTS 1988) In how many ways can a debt of 69 dollars be paid exactly using only 5 dollar bills and 2 dollar bills?\n\n6. (MATHCOUNTS 1988) Find the sum of all four digit natural numbers of the form $4AB8$ which are divisible by 2, 3, 4, 6, 8, and 9.\n\n7. (Mu Alpha Theta 1990) If $a\\ne 1$ and $\\sqrt[a]{10000_{a}}= 10_{a}$, find $a$.\n\nGood luck![/quote]\r\n\r\n[hide=\"1.\"]\n$141 = 47 \\cdot 3$\n$1441 = 11 \\cdot 131$ \n$14441 = 7 \\cdot 2063$\n[/hide]\n\n[hide=\"2.\"]\n$48_{10}= 57_{8}$\n\n$48_{10}= 52_{9}$\n\n$48_{10}= 2F_{16}$\n\ni want my ranking to be fixed...\n[/hide]",
"Solution_2": "About time somebody did them...\r\n\r\nBoth correct. :)",
"Solution_3": "I'm not in middle school but I'll try these anyway. :) \r\n\r\n[hide=\"#4\"]\nTo be divisible by 4, the last two digits must form a number divisible by 4, so B=2 or B=6. To be divisible by 9, the sum of the digits must be a multiple of 9. 1+2+3+6=12, so if B=6, A=6, which is not allowed. 1+2+3+2=8, so if B=2, A=1. This is the only solution, so A=1.\n[/hide]",
"Solution_4": "uhh.. I hope you don't have to be from Maryland to answer these but here's #7.\r\n[hide=\"7\"]Changinge everything to base 10, we get $\\sqrt[a]{a^{4}}=a\\rightarrow a^{4}=a^{a}\\rightarrow a=4$[/hide]",
"Solution_5": "well actually you have to be from cockeysville middle school, but since they aren't answering them you can :lol:",
"Solution_6": "Hmm I understand the divisibility stuff but like number 5 how can you use those rules and apply them to real life based problems?",
"Solution_7": "[hide=\"5\"]\nWe have $5a+2b=69$ for some positive integers $a,b$. Modulo 5, the equation becomes $2b\\equiv 4\\pmod{5}$ so $b\\equiv 2\\pmod{5}$. Therefore, the smallest possible value of $b$ is 2, and when $b=2$, $a=13$. Notice that decreasing $a$ by 2 and increasing $b$ by 5 will give us another solution. Therefore, the solutions are (13, 2), (11, 7), (9, 12), (7, 17), (5, 22), (3, 27), (1, 32). So the answer is $\\boxed{7}$.\n[/hide]"
}
{
"Tag": [
"calculus",
"integration",
"function",
"real analysis",
"calculus computations"
],
"Problem": "How can one prove that\r\n\\[ \\int f(x) \\, \\mbox{sgn} \\left( f(x) \\right) \\, \\mbox{d}x \\equal{} \\mbox{sgn} \\left( f(x) \\right) \\int f(x) \\, \\mbox{d}x\\]\r\nIt is trival, but I don't have slightest idea how to prove it in a nice way :(\r\n\r\nIs it enough when we consider intervals when f>0 and f<0 :?:",
"Solution_1": "What is sgn? Does it stand for the sign (+ or - symbol) of a function? And if I'm interpreting that correctly, the proof is indeed trivial, write the LHS as a Riemann sum and factor out the sign of the function.",
"Solution_2": "sgn stands for sign function.\r\nOk, thanks.",
"Solution_3": "If you look carefully the equation doesn't make sense (at least without some conditions). How do you 'factor out' the sign function while keeping the variable the same?",
"Solution_4": "$ f sgn(f) \\equal{} |f|$"
}
{
"Tag": [
"AoPSwiki",
"AMC",
"AIME",
"LaTeX",
"articles",
"USA(J)MO",
"USAMO"
],
"Problem": "If you haven't checked out the [url=http://www.artofproblemsolving.com/Wiki/]AoPSWiki[/url] yet, you really ought to!\r\n\r\nOne of the ongoing projects in the AoPSWiki is the development of the [url=http://www.artofproblemsolving.com/Wiki/index.php/AMC_Problems_and_Solutions]AMC Problems/Solutions[/url]. This is not a small task, so we will need a lot of help. Anyone can help. You just use your AoPS account for the AoPSWiki.\r\n\r\nI've already started with the [url=http://www.artofproblemsolving.com/Wiki/index.php/2006_AIME_I]2006 AIME I[/url]. If you look around, you should get a feel for it.\r\n\r\nI haven't put the solutions in for the 2006 AIME I. Also, if you want to put links to message board posts with the problems, that would be a good idea. Just make sure to put the links in the ''See also'' section of the article.\r\n\r\nThe biggest advantage of the wiki over the contest directory is that anyone can edit it at any time.\r\n\r\nThanks.",
"Solution_1": "Another note for entering problems into the wiki:\r\n\r\nLook at the [url=http://www.artofproblemsolving.com/Wiki/index.php/2006_AIME_I_Problems]2006 AIME 1 Problems[/url]. It has all the problems listed as their own section. After the problem statement it has a solution link which brings you to a page dedicated solely for that problem. When making these solution pages you MUST name them like the following: 2006 AIME I Problems/Problem 1. What this does is it creates a subarticle for that problem (instead of making a whole new article).\r\n\r\nThanks.",
"Solution_2": "COOL! :coolspeak:",
"Solution_3": "An additional note:\r\n\r\nLook at the way I entered the [url=http://www.artofproblemsolving.com/Wiki/index.php/University_of_South_Carolina_High_School_Math_Contest/1993_Exam]USC exam[/url] into the wiki. I think the AMC's should follow this format (it's open to improvement though if you have suggestions).\r\n\r\nAlso, note that on the individual problem pages ( e.g. http://www.artofproblemsolving.com/Wiki/index.php/University_of_South_Carolina_High_School_Math_Contest/1993_Exam/Problem_2 ) I have links to the previous problem, next problem, and back the exam page. I also added the [[Category:...] tags for all of them.",
"Solution_4": "Sorry to bump....\r\n\r\nbut the AIME's certainly still need more work!!!\r\n\r\nToday I fixed around a couple things about the AIME's (preliminaries for like 3 or 4 tests, one completely added)\r\nFor many, the Latex is up but it just need to have